You are on page 1of 164

The Hail Mary: Sales Outline: Gaffney Fall 2022

CHAPTER 1: INTRODUCTION TO THE UNIFORM COMMERCIAL CODE AND SALES LAW


I. History of the UCC and the Law of Sales
 Ancient Sources of Commercial Law: Code of Hammurabi (1772 B.C.): One half of the known Babylonian Code deals with matters of
commercial law, such as the wages to be paid to an oxen driver or the liability of a builder for a house that collapses. Roman Law:
contract law and commercial transactions was not a separate branch of the law but was embedded in the civil law.
– Code of Justinian
 Lex Mercatoria (“Law Merchant”) or Ius Mercatorum: Between 500 and 1500 A.D. the medieval merchants themselves developed
rules of law governing commercial transactions. These cases were heard at medieval fairs, where panels or juries composed of fellow
merchants resolved disputes. This kept feudal lords out of mercantile disputes and avoided traditional methods of trial such as trial
by ordeal and trial by combat.
– Much of this law still contributed to the laws under the U.C.C. and is used today.
 British Common Law: In the 1600s under the leadership of Lord Chief Justice Sir Edward Coke, the British courts established
independence from the crown, and with that independence came the power to interpret the law. The fundamental law was no
longer the will of the monarch but was rather the common law – the customs and traditions of the British people. Contract law, and
specifically the law governing the sale of goods, developed in the British and American courts as part of the common law. The British
courts based much of contract law on the “Law Merchant” that had developed in the Middle Ages.
 Swift v. Tyson (1842) and Federal Common Law: In Swift v. Tyson (1842) Justice Joseph Story of the United States Supreme Court
ruled that in diversity cases if no state statute controlled, the federal courts should apply federal common law instead of state
common law. The reason for the decision was to create a uniform national law governing commercial transactions. Swift v. Tyson
was overruled in 1938 in Erie R.R. v. Tompkins, where the Court ruled that state substantive law applies in diversity cases in federal
court. Because federal common law no longer applied in commercial cases, and because there were no federal statutes on the
subject, this left the United States without a comprehensive uniform code to govern commercial transactions.
 British Sale of Goods Act 1893: There had been earlier efforts to codify the common law governing commercial transactions. In 1893
the British Parliament adopted the Sale of Goods Act. This comprehensive statute codified much of the common law of Sales.
 The Uniform Sales Act: Following the adoption of the British Sale of Goods Act, in the United States the Uniform Sales Act was
drafted in 1906 by Samuel Williston, the author of the highly influential Treatise on the Law of Contracts. The Uniform Sales Act was
adopted in 34 states – mainly the states with highly developed manufacturing and transportation networks.
 Drafting of the Uniform Commercial Code: In 1942, four years after the Supreme Court’s decision in Erie R.R. v. Tomkins, the
American Law Institute and the National Conference of Commissioners on Uniform State Laws (Uniform Law Commission) began
work drafting the Uniform Commercial Code. The first version of the U.C.C. was published in 1952. Several revisions of the U.C.C.
have been drafted and adopted since then. The leading drafters of the UCC were Soia Mentschikoff and Karl Llewellyn.
 The U.C.C. Is a Model Act
o The Uniform Commercial Code is a model act. It is not law until it is adopted by a state legislature.
o All 50 states have adopted the U.C.C., but only 49 states have adopted Article 2, the exception being Louisiana.
 Constant Updating and Revision of the Uniform Commercial Code
o The U.C.C. is constantly under revision under the oversight of the Permanent Editorial Board of the Uniform Law Commission.
 Canvas Quiz Questions
o Commercial Law has ancient roots, dating back to the Code of Hammurabi, Roman Law, and the medieval Lex Mercatoria
(Law Merchant).
 True - Commercial law does date back to those antecedents.
o The law of sales developed in the courts of Great Britain and the United States as part of the common law of contracts.
 True
o The Uniform Commercial Code was originally drafted in the 1990s.
 False - The first draft of the U.C.C. was approved in 1952.
o Which of the following are accurate statements about the U.C.C.?
 Answer: The leading drafters of Article 2 of the U.C.C. were Soia Mentschikoff and Karl Llewellyn.
 The U.C.C. has been adopted in identical form in every state; there are no variations among the states. - different states
have adopted varying provisions of the Uniform Commercial Code.
 The U.C.C. has not been modified since its initial adoption in 1952; there are no ongoing efforts to update and revise the
U.C.C. - the U.C.C has been continuously updated and modified since its introduction in 1952.
 The U.C.C. becomes law as soon as it is approved by the National Conference of Commissioners on Uniform State Laws and
the American Law Institute; legislative action is not necessary. - The U.C.C. is a model act that does not become law until
the state legislatures enact it.
II. UCC and Consumer Protection
 The U.C.C. Has Relatively Few Protections for Consumers

1
o In its original form the Uniform Commercial Code was essentially oriented towards creating a uniform law of commercial
transactions for the benefit of merchants, manufacturers, and banks, and it was not concerned with protecting the interests of
consumers.
o That’s one of the reasons that the Code was proposed as state law rather than federal law. Business interests did not want
Congress to enact a federal commercial code, because Congress would have been much more protective of consumers than the
state legislatures were.
 Abusive Practices Permitted by the U.C.C.
o Due to the U.C.C.’s lack of consumer protection it allows many abusive practices, such as:
 Article 2 permits manufacturers and stores to disclaim warranties and to sell goods “as is.”
 Article 3 retains the Holder in Due Course rule which normally works against consumers and small businesses by requiring
them to pay a negotiable instrument (a check or a promissory note) even if they have a valid defense to payment.
 Article 9 permits creditors to repossess property without prior notice or a judicial hearing (“self-help” repossession).
 State Consumer Protection Laws
o As a consequence of the fact that the U.C.C. permitted a number of abusive practices, state legislatures have adopted laws
specifically designed to protect consumers from abusive practices, such as laws prohibiting “deceptive acts and practices.”
These laws supplement or supplant provisions of the U.C.C.
 Federal Consumer Protection Laws
o The federal government has also adopted many consumer protections laws to supplement or preempt portions of the U.C.C.
One recent federal consumer protection law is the Dodd-Frank Act regulating the financial services industry. An example of a
federal consumer protection law affecting the law of sales is the Magnuson-Moss Warranty Act.
 Amended Article 2
o In 2002 and 2003 Amended Article 2 was approved by the National Conference of Commissioners on Uniform State Laws and
the American Law Institute. The final draft of Amended Article 2 contained several additional protections for consumers relating
to warranties.
o Due to industry opposition Amended Article 2 was adopted in only two states (Virginia and Maryland) and efforts to promote it
have been abandoned by the NCCUSL and the ALI.
 Canvas Quiz Questions
o The U.C.C. was designed and intended as a consumer protection law.
 False - The U.C.C. was drafted with very few protections for consumers.
o The U.C.C. expressly permits the following anti-consumer practices:
 Disclaimers of implied warranties ("as is" sales) - The U.C.C. expressly permits sellers to disclaim the warranty of
merchantability.
 The "holder in due course" rule - The U.C.C. retains the holder in due course rule.
 "Self-help" repossession by debtors - The U.C.C. and permits self-help repossession.
 No-Fraudulent commercial practices.
o In light of the fact that the U.C.C. contains few protections for consumers,
 the federal government has enacted a number of consumer protection laws that preempt provisions of the U.C.C. such as
the Magnuson-Moss Warranty Act.
 the states have enacted other statutes protecting consumers that supplement the U.C.C. such as laws prohibiting
deceptive acts and practices.
 the drafters of the U.C.C. created Amended Article 2 which has been adopted in every state. - The NCCUSL and the ALI
proposed Amended Article 2, it was adopted in only two states.
 no other laws protecting consumers have been adopted at the state or federal level. - There have been other consumer
protection laws adopted by the states and the federal government that supplement or supplant provisions of the U.C.C.
III. Structure of the U.C.C.: Articles, Parts, and Sections
 The Uniform Commercial Code is divided into articles, parts, and sections. A proper citation to the Code includes all three
designations.
The Articles of the U.C.C. Parts of Article 2
 Article 1 – General Provisions  Part 1. Short Title, General Construction, and Subject
 Article 2 – Sales Matter
 Article 2A – Leases  Part 2. Form, Formation, and Readjustment of Contracts
 Article 3 – Negotiable Instruments  Part 3. General Obligation and Construction of Contracts
 Article 4 – Bank Deposits and Collections  Part 4. Title, Creditors, and Good Faith Purchasers
 Article 4A – Funds Transfers  Part 5. Performance
 Article 5 – Letters of Credit  Part 6. Breach, Repudiation, or Excuse
 Article 6 – Bulk Transfers and Bulk Sales  Part 7. Remedies
 Article 7 – Warehouse Receipts, Bills of Lading and
other Documents of Title
 Article 8 – Investment Securities
2
 Article 9 – Secured Transactions
 Official Comments
o The Official Comments to the Uniform Commercial Code are an important source of information about the intent of the drafters
of the Code.
 Example: Section 2-314
o Section 2-314 is contained in Article 2, Part 3. Article 2 governs the sale of goods. Part 3 of Article 2 (the 2-300s) contains a
number of provisions governing the obligations of the parties to a contract for the sale of goods. For example, Section 2-314
creates an “implied warranty of merchantability,” meaning that the seller of goods is deemed to have promised that the goods
“are fit for the ordinary purposes for which such goods are used.”
 Canvas Quiz Questions
o Article 1 of the U.C.C. applies generally to all commercial transactions that are covered by the other Articles of the U.C.C.
 True
o The scope of Articles 2 and 2A of the U.C.C. is as follows:
 Answer: Article 2 applies to sales of goods and Article 2A applies to leases of goods.
 Article 2 applies to leases of goods and Article 2A applies to sales of goods.
 Article 2 applies to Negotiable Instruments and Article 2A applies to Secured Transactions.
 Article 2 applies to contracts and Article 2A applies to torts.
o Section 2-207 of the Uniform Commercial Code
 is in Article 2 of the U.C.C.
 is in Part 7 of Article 2. - Section 2-207 is in Part 2 of Article 2
 is entitled "Battle of the Forms." - It is entitled "Offer and Acceptance in Formation of Contract"; and it does deal with the
"battle of the forms."
 deals with the problem referred to as "the battle of the forms."
IV. Section 1-103: The Purposes of the U.C.C. and Its Relation to the Common Law
 Article 1
o Article 1 of the U.C.C. contains definitions and other provisions that apply to all commercial transactions that are governed by
the Code.
 The Purposes of the U.C.C. and the Window to the Common Law
o Section 1-103(a) of the U.C.C. sets forth the general purposes of the U.C.C.
o Section 1-103(b) describes the relation of the Code to the common law and to important equitable principles such as fraud,
duress, misrepresentation, estoppel, etc.
 Section 1-103(a): General Purposes of the U.C.C.
The Uniform Commercial Code] must be liberally construed and applied to promote its underlying purposes and policies, which are:
(1) to simplify, clarify, and modernize the law governing commercial transactions; (2) to permit the continued expansion of
commercial practices through custom, usage, and agreement of the parties; and (3) to make uniform the law among the various
jurisdictions.
 Simplify, Clarify, and Modernize the Law
o One of the principal purposes of the U.C.C. is to permit both merchants and consumers to understand the law and to apply it
without the assistance of attorneys.
o This is, of course, not always possible.
 Commercial Practice
o When interpreting or applying the U.C.C., it is important to remember that Section 1-103(a) requires that the Code be “liberally
construed” – that the Code must allow the expansion of commercial practice, not constrain it to existing forms.
 Make Uniform the Law
o Theoretically the law governing commercial transactions should be the same in every state, because all states have adopted the
U.C.C.
o However, different states have adopted different variations of specific provisions of the Uniform Commercial Code.
o And even in states that have enacted precisely the same provision of the U.C.C., state courts have often interpreted those
provisions differently. Ideally the states would give weight to the interpretations of the U.C.C. in other states so that the law can
be uniform among the several states – but this does not always happen.
 The Relation Between the Common Law and the U.C.C.
o Normally when a statute is enacted it replaces the common law, and the U.C.C. is no exception; to the extent that the U.C.C. is
inconsistent with the common law, the U.C.C. controls. However, there are many broad principles of the common law that still
“supplement” the U.C.C.
o The relation between the common law and the U.C.C. is governed by Section 1-103(b) and Official Comment 2 to that section.
 Section 1-103(b): Relation of the Code to the Common Law and Important Equitable Principles

3
Unless displaced by the particular provisions of [the Uniform Commercial Code], the principles of law and equity, including the law
merchant and the law relative to capacity to contract, principal and agent, estoppel, fraud, misrepresentation, duress, coercion,
mistake, bankruptcy, and other validating or invalidating cause supplement its provisions.
Official Comment 2 to Section 1-103: Relation Between the Common Law and the U.C.C.
Applicability of supplemental principles of law. Subsection (b) states the basic relationship of the Uniform Commercial Code to
supplemental bodies of law. The Uniform Commercial Code was drafted against the backdrop of existing bodies of law, including the
common law and equity, and relies on those bodies of law to supplement its provisions in many important ways. At the same time,
the Uniform Commercial Code is the primary source of commercial law rules in areas that it governs, and its rules represent choices
made by its drafters and the enacting legislatures about the appropriate policies to be furthered in the transactions it covers.
Therefore, while principles of common law and equity may supplement provisions of the Uniform Commercial Code, they may not
be used to supplant its provisions, or the purposes and policies those provisions reflect, unless a specific provision of the Uniform
Commercial Code provides otherwise. In the absence of such a provision, the Uniform Commercial Code preempts principles of
common law and equity that are inconsistent with either its provisions or its purposes and policies.
 The Tension Between the Text of Section 1-103(b) and Official Comment 2 to Section 1-103
o On the one hand, Section 1-103(b) identifies certain principles of common law and equity that supplement the U.C.C. unless
“displaced” by particular provisions of the U.C.C.
o On the other hand, Comment 2 to Section 1-103 states that the U.C.C. “preempts” principles of common law and equity that are
inconsistent with its provisions or its purposes and policies.
 The Differences Between the Rules of the U.C.C. and the Principles of Law and Equity
o The common law and the principles of equity primarily reflect notions of fairness – broad standards of right and wrong.
o The Uniform Commercial Code primarily reflects the goal of economic efficiency – creating clear rules that will make commerce
work.
o There is thus a tension between the specific provisions of the U.C.C. and the broad principles of equity and the common law.
Does the common law “supplement” the U.C.C. or is it “displaced” by it? Different courts in different circumstances have come
to different results. All we can say is – it depends on the court’s judgment about the relative clarity of the specific provision of
the U.C.C. and the need for economic efficiency versus the imperative of fairness that the principle of common law or equity
stands for.
 How does Article 2 differ from the Common Law of Contracts?
o To give some examples:
 Firm offers are irrevocable even with no consideration, so long as if a merchant makes the offer and it is in writing.
• Under the common law if there is no consideration then the offer is void.
 Contract formation is done easily through words or conduct, not requiring a matching offer and acceptance
• Under the common law offer and acceptance must mirror each other.
 Mercantile concepts (course of dealing, usage of trade, and course of performance) may be used to add non-express terms
to the contract
• The common law requires certainty and specificity regarding offer and acceptance.
 The statute of frauds is easier to meet, and parol evidence easier to admit
 A seller automatically makes an implied warranty of merchantability
 Article 2 provides many gap-filler terms (such as price, allocation of risks, exclusive dealing, output contract), as opposed to
declining to write a contract for the parties
o Article 2 also sets out rules tailored to the sales contract between a buyer and a seller (such as warranties, delivery, risk of loss
of the goods, remedies keyed to market prices), which may differ from common law of contracts.
 For example, a seller must make a “perfect tender” (subject to exceptions) in order to be entitled to the price, as opposed
to substantial performance of the contract.
 Canvas Quiz Questions
o Article 1 of the U.C.C. contains definitions and other provisions that apply generally to all commercial transactions that are
governed by the Code.
 True
o Section 1-103 of the U.C.C.:
 sets forth the general purposes of the U.C.C.
 describes the relation of the U.C.C. to the common law and to important equitable principles such as fraud, duress,
misrepresentation, estoppel, etc.
 establishes the jurisdiction of the courts in commercial disputes.
 governs conflicts of laws issues in commercial cases.
o The purposes of the Uniform Commercial Code that are set forth in Section 1-103 include the following:
 to simplify, clarify, and modernize the law of commercial transactions.
 to permit the continued expansion of commercial practices through custom, usage, and agreement of the parties.
 to make uniform the law among the various jurisdictions.
 to protect consumers from abusive commercial practices.

4
o Which of the following statements most accurately describes the relation between the Uniform Commercial Code and the
common law?
 Answer: The Uniform Commercial Code preempts principles of common law that are inconsistent with either its provisions
or its purposes and policies; however, certain principles of the common law such as the law relative to capacity to contract,
principal and agent, estoppel, fraud, misrepresentation, duress, coercion, and mistake may supplement the provisions of
the U.C.C.
 The U.C.C. utterly displaces the common law in all respects.
 Where there is a conflict between the common law and the U.C.C., the common law controls.
 The U.C.C. and the common law never conflict because the U.C.C. embodies the common law in all respects.
V. Interpretation of the U.C.C.
 Sources of Interpretation
o There are five sources that lawyers, and judges use to interpret statutes. These are:
1. The text of the statute itself
2. The intent of the persons who drafted or adopted the statute
3. Judicial precedent interpreting the statute
4. The traditional ways that people had acted in accordance with the law
5. The underlying purposes and policies that the law is supposed to serve
 The Text of the U.C.C.
o The starting point for the interpretation of the Uniform Commercial Code is the text of the law itself. In this course we shall
study the U.C.C. as proposed by the N.C.C.U.S.L. and the A.L.I. and as enacted in the State of Pennsylvania. In practice you will
use the text of the U.C.C. as adopted in the jurisdiction whose law governs the transaction.
o Section 1-107 of the U.C.C. provides that section captions (the titles or headings) are part of the U.C.C. Accordingly it is
appropriate to consider those headings or captions when interpreting the meaning of a provision of the U.C.C.
 Pennsylvania Commercial Code § 1107
Notwithstanding 1 Pa.C.S. § 1924 (relating to construction of titles, preambles, provisos, exceptions and headings), section captions
are part of this title.
 The Intent of the Drafters
o In determining the intent of the persons who drafted or adopted a statute it is common to consult the legislative history of the
legislation. In the case of the U.C.C. there is rarely any legislative history from the states that can be used as a guide for
interpreting the law.
o However, every provision of the U.C.C. contains Official Comments from the N.C.C.U.S.L. and the A.L.I. about the meaning of
that provision. Be sure to consult the Official Comments in determining the meaning of any particular provision of the U.C.C.
The Official Comments are highly persuasive authority about the meaning of the law.
 Judicial Precedent
o As with any other state statute, the state supreme court has the final say as to the meaning of that statute. Be sure to consult
the case law in your jurisdiction as to how the courts have interpreted and applied a particular provision of the U.C.C.
Remember that the U.C.C. is a state law, and that the federal courts must defer to the states in their interpretations of the
U.C.C.
o Also, when interpreting the U.C.C., the courts pay more attention to judicial interpretations from other jurisdictions than they
do in the interpretation of other state laws. Remember that one of the principal purposes of the U.C.C. is to create a UNIFORM
system of commercial law throughout the United States.
 Commercial Tradition
o In the commercial context, “tradition” means how parties to commercial transactions have conducted business over decades
or even centuries. Section 1-103 states that the U.C.C. is to be interpreted in accordance with commercial practice – including
emerging forms of commercial practice!
Policy Considerations
o The general purposes of the UCC are set forth in Section 1-103(a), and the U.C.C. should be interpreted with those purposes in
mind.
o Furthermore, every provision of the U.C.C. serves one or more specific purpose or policy. In every case it is necessary to identify
what those purposes and policies are and to attempt to interpret the law in order to advance those purposes and policies.
 Influential Learned Treatises
o There are many learned treatises in the field of Commercial Law. One extremely influential treatise is White & Summers,
Commercial Law. This treatise is considered to be highly persuasive authority as to the proper meaning and application of the
Uniform Commercial Code.
 Interpreting Contracts: Express Terms, Course of Performance, Course of Dealing, and Trade Usage
o “Text” and “tradition” also affect how specific contracts should be interpreted. Section 1-303(e) establishes the following
hierarchy governing how peoples’ behavior affects the interpretation of contracts:
o Express Terms of the Contract
 Course of Performance (by the parties under the contract in question)
5
 Course of Dealing (under previous contracts between the same parties)
 Trade Usage (among other parties in the same commercial setting)
 Section 1-303(e): Hierarchy of Behaviors that Affects the Interpretation of Contracts
Except as otherwise provided in subsection (f), the express terms of an agreement and any applicable course of performance,
course of dealing, or usage of trade must be construed whenever reasonable as consistent with each other. If such a construction
is unreasonable: (1) express terms prevail over course of performance, course of dealing, and usage of trade; (2) course of
performance prevails over course of dealing and usage of trade; and (3) course of dealing prevails over usage of trade.
 Canvas Quiz Questions
o To interpret a statute the courts, look to different sources for guidance. To interpret the meaning and proper application of
the Uniform Commercial Code the courts would likely consider:
 the text of the U.C.C.
 the legislative history of its enactment, if any.
 the headings or captions in the U.C.C.
 the Official Comments of the U.C.C.
 judicial precedent interpreting the U.C.C.
 commercial practices including usages of trade.
 the purposes and policies that the U.C.C. was adopted to serve.
 respected treatises such as White & Summers on Commercial Law.
o Judicial interpretations of the Uniform Commercial Code by the courts of other states are not binding on a court but are given
great weight because one of the purposes of the U.C.C. is to create a uniform law of commercial transactions.
 True - Uniformity in the law of commercial transactions is an important goal of the U.C.C.
o When interpreting the meaning of a specific contract, the U.C.C. establishes the following hierarchy:
1. Express terms of the contract.
2. The parties’ course of performance under this contract.
3. The parties’ course of dealings under previous contracts with each other.
4. Trade usage in that commercial setting.
o In interpreting a particular provision of the Uniform Commercial Code, it is appropriate to consider both the general policies
set forth in Section 1-103(a) as well as the specific purposes and policies that the particular provision of the U.C.C. were
intended to achieve.
 True - It is appropriate to consider both the general policies of the U.C.C. set forth in Section 1-103(a) as well as the specific
purposes and policies of the particular provision of the U.C.C. under consideration.
Review/Assessment Questions
 Who developed the law merchant and why?
o Medieval merchants themselves developed rules of law governing commercial transactions to keep feudal lords out of
mercantile disputes and avoided traditional methods of trial such as trial by ordeal and trial by combat.
 How did the law merchant affect the development of commercial law in the courts?
o Real courts became aware that they were being bypassed by merchants and began to incorporate the law merchant into their
own rules. During the time of the American Revolution, the English court system had as its Chief Justice the great jurist Lord
Mansfield. He believed in pragmatism in his courtroom, and during his tenure the law merchant was given much influence in
English decisions and played a large rule in the development of the common law.
 Who drafted the Uniform Commercial Code and why?
o The American Law Institute and the National Conference of Commissioners on Uniform State Laws (Uniform Law Commission)
sought to create a uniform law of commercial transactions for the benefit of merchants, manufacturers, and bank. The leading
drafters of the UCC were Soia Mentschikoff and Karl Llewellyn
 How does the UCC become law?
o By being adopted through the state legislature.
 How does the UCC define “good faith”?
o The definition of good faith is “honesty in fact and the observance of reasonable commercial standards of fair dealing.” §1-
201(b)(20).

CHAPTER 2: BASIC CONCEPTS


 Contract
o Was there a “contract” between the parties? For the most part this issue is governed by the common law of contracts. There are
some rules in Article 2 regarding contract formation that displace the common law. Section 2-207 of the Code – that deals with
the “battle of the forms” – is unique to Article 2. The Statute of Frauds under Article 2 is also very different from the statute of
frauds under common law.
o Section 2-106 states that the U.C.C. applies to both contracts for the sale of present goods and to contracts for the sale of
future goods.

6
o Finally, Section 2-106 draws a distinction between an agreement and a contract. What is the difference between an agreement
and a contract? See §1-201(b)(3), (12).
 Under the Restatement (Second) of Contracts § 1: A Contract is a promise or a set of promises for the breach of which the
law gives a remedy, or the performance of which the law in some way recognizes as a duty, whereas Under § 3 an
Agreement is a manifestation of mutual assent on the part of two or more persons. However, under §1-201(b)(3), (12) of
the U. C.C., an Agreement means the bargain of the parties in fact, as found in their language or inferred from other
circumstances, including course of performance, course of dealing, or usage of trade as provided in Section 1-303.
I. Scope of Article 2 and Article 2A
 The Scope of Article 2
o Article 2 of the Uniform Commercial Code applies to “transactions in goods” but it does not apply to gifts, leases, or secured
transactions (the use of goods as collateral).
A. “Transactions in Goods”
 Not “Transactions in Goods”: But “Sales of Goods” and “Contracts for the Sale of Goods”
o Section 2-102 states that Article 2 applies to “transactions in goods.” The drafters were only joking when they said that.
Transactions in goods might include sales, leases, security interests, or gifts – but Article 2 applies only to “sales of goods” and
“contracts for the sale of goods.” Other types of transactions in goods are governed by other laws:
 Leases of goods – Article 2A
 Security interests in goods – Article 9
 Gifts of goods – Common law of property
 Merchants, Non-Merchants, and Consumers
o Article 2 applies to all persons who are engaged in the sale of goods: merchants, non-merchants, and consumers.
o However, there are several provisions that impose obligations only on merchants (Section 2-314(1), the implied warranty of
merchantability) and a few other provisions that provide protections relating to consumer goods (Section 2-719(3)), no
limitation for consequential damages for injury to the person resulting from consumer goods).
 Definition of a “Sale of Goods”
o A “sale of goods” is the passing of title to goods for a price. (Section 2-106(1))
 A “price” can be anything. “The price can be made payable in money or otherwise. If it is payable in whole or in part in
goods each party is a seller of the goods which he is to transfer.” (Section 2-304(1))
 Example: Barter
o Mary trades her ’57 Chevy to Mark in return for his coin collection. This transaction is covered by Article 2.
o Mary is the seller and Mark is the buyer of the auto. Mark is the seller and Mary is the buyer of the coins.
 “Contract for Sale”
o Article 2 applies to a “contract for sale of goods.”
o Under Section 2-106(1) a “contract for sale of goods” includes both a present sale of goods and a contract to sell goods at a
future time.
 Definition of “Goods”
o “Goods” are things that are movable at the time of identification to the contract.
 Section 2-105(1): Definition of “Goods”
“Goods” means all things (including specially manufactured goods) which are movable at the time of identification to the
contract for sale other than the money in which the price is to be paid, investment securities (Article 8) and things in action.
“Goods” also includes the unborn young of animals and growing crops and other identified things attached to realty as
described in the section on goods to be severed from realty (Section 2-107).
 “Present Goods” and “Future Goods”
o Pursuant to Section 2-105(2), “present goods” are goods that are both existing and identified to the contract; “future goods”
are goods that are not both existing and identified to the contract.
 Section 2-501(1): Identification to the Contract
The buyer obtains a special property and an insurable interest in goods by identification of existing goods as goods to which the
contract refers even though the goods so identified are non-conforming and he has an option to return or reject them. Such
identification can be made at any time and in any manner explicitly agreed to by the parties. In the absence of explicit
agreement identification occurs
(a) when the contract is made if it is for the sale of goods already existing and identified;
(b) if the contract is for the sale of future goods other than those described in paragraph (c), when goods are shipped, marked or
otherwise designated by the seller as goods to which the contract refers;
(c) when the crops are planted or otherwise become growing crops, or the young are conceived if the contract is for the sale of
unborn young to be born within twelve months after contracting or for the sale of crops to be harvested within twelve
months or the next normal harvest season after contracting whichever is longer.
 Example: Identification to the Contract, Present Goods, and Future Goods
o If I go into a store and pick up a stuffed animal and take it to the register and hand it to the cashier, the item has been
“identified to the contract.”
7
o However, if I were to order a type of stuffed animal from a store online, it would not be “identified to the contract” until it was
selected from inventory by the seller and either marked or packaged to be shipped to me. Until then the plush toy would be
“future goods.”
 Definition of “Consumer Goods”
o Consumer goods are goods that are used or bought for use for personal, family, or household purposes. (Section 9-102(23))
 Example: The First Home Computer
o It is 1975. There is no such thing as a “home computer.” Computers are used at businesses, but no-one has ever had one at
home.
o That is, until Suzanne purchases a computer to use at home to keep track of household expenses. This item, even though at that
time had been exclusively used for business purposes, was “used” or “bought for use” by Suzanne primarily for personal, family,
or household purposes” and thus qualified as “consumer goods.”
 Goods or Real Estate?: The Sale of Fixtures, Crops, and Timber
o Article 2 applies to the sale of goods. What about items that are attached to real estate?
o Crops and timber are considered to be “goods” under the U.C.C., and Article 2 applies to the sale of crops or timber.
o Article 2 does not use the term “fixtures,” but it does refer to the concept. “Fixtures” are things that can be severed from realty
without material harm thereto. In this respect they are similar to crops or timber. The sale of fixtures is governed by Article 2.
 Section 2-107(2): “Fixtures”
A contract for the sale apart from the land of growing crops or other things attached to realty and capable of severance without
material harm thereto but not described in subsection (1) or of timber to be cut is a contract for the sale of goods within this
Article whether the subject matter is to be severed by the buyer or by the seller even though it forms part of the realty at the
time of contracting, and the parties can by identification effect a present sale before severance.
 Example: A Furnace
o Ralph enters into a contract with the local historical society to sell his home’s coal-fired furnace.
o If the furnace can be removed from Ralph’s home without materially harming the building then the furnace is “goods” and this
transaction is governed by Article 2, regardless of whether the furnace is going to be severed by Ralph or the historical society.
 Example: Sale of Timber
o Edward owns land in Beaver County that has a stand of black walnut trees. He enters into a contract to sell the trees to
Georgina, who owns a lumbermill.
o Timber qualifies as goods, and this transaction is governed by Article 2 regardless of whether the trees are to be cut down by
Edward or Georgina.
 Sale of Minerals, Buildings, and Building Materials
o Minerals, structures, and structural materials are “goods” and subject to Article 2 only if they are to be severed by the seller.
 Section 2-107(1): The Sale of Minerals and Buildings
A contract for the sale of minerals or the like (including oil and gas) or a structure or its materials to be removed from realty is a
contract for the sale of goods within this Article if they are to be severed by the seller but until severance a purported present sale
thereof which is not effective as a transfer of an interest in land is effective only as a contract to sell.
 Example: Sale of Coal
o Samuel owns a small coal mine in Fayette County. Thomas, a coal distributor, wishes to purchase a ton of coal from Samuel.
o If Samuel, the seller, is going to mine the coal then this transaction is governed by Article 2. If Thomas, the buyer, is going to
mine the coal then this transaction is governed by the common law.
 Example: Sale of Bricks
o Joshua’s old brick pump house is being demolished by Stanford Wrecking Company. The contract specifies that as payment for
the demolition Stanford will obtain title to the bricks.
o The bricks are not a fixture but rather are building materials. The sale of the bricks is not governed by Article 2 because the
building materials are going to be severed by Stanford, the buyer.
 Goods or Services?
o A recurrent problem in Commercial Law is whether Article 2 or the common law of contracts should apply to the transaction. In
particular, it is often difficult to determine whether a contract calls for the sale of goods or for the sale of services.
 Example: Goods or Services?
o Frank purchased an above-ground pool from Family Fun, Inc., for $10,000. The contract price was divided between the cost of
materials and the cost of assembly and installation.
o Is this transaction a sale of goods governed by Article 2 or a sale of services governed by the common law?
 Casebook Problem PAGE 8
o Whether Article 2 applies may determine the outcome of a case. If Article 2 applies, its rules may favor one party (such as the
implied warranty of merchantability that automatically is given to many buyers by §2-314). Does Article 2 of the Code apply to
the following matters?
(a) The sale of an insurance policy? See Call v. Czaplicki.
 No, this is a service and is in action which is excluded from Section 2-105(1).

8
 Call v. Czaplicki- held that the insurance component of plaintiff’s life insurance contract will be separated from its
investment component for the purposes of applying the New Jersey Consumer Fraud Act.
(b) The sale of real property? What about the sale of a house apart from the realty? See §2-107.
 The sale of real property is not a good; however, under section 2-107(1), a structure to be removed from realty is a
contract for the sale of goods within this Article if they are to be severed by the seller. So, if the house can be removed
from the real property Article 2 applies.
(c) The sale of building materials as part of a construction project? See Audio Visual Artistry v. Tanzer.
 Courts are split, and it depends on the situation. The predominate factor test could be used to determine if they can be
applied under Article 2.
(d) The sale of standing timber? Crops? See §2-107.
 Yes, under §2-107(1) and (2) which provide that they are capable of severance.
(e) A defective spinal plate given a patient in a hospital operating room? See Brandt v. Boston Scientific Corp. The preparation of
false teeth by a dentist? See Carroll v. Grabavoy. The injection of a drug (for which the patient was separately billed) into a
patient’s eye as part of an operation? See Providence Hospital v. Truly.
 These are mixed questions of goods and services for which courts are split.
 Usually, these cases come down to statute of limitations for cause of action.
 The separate billing can help with finding it a good, thus allowing Article 2 to apply.
(f) The sale of membership in a health spa?
 No, this is a service.
(g) The sale of electricity? See GFI Wisconsin, Inc. v. Reedsbury Utility Commn.
 Electricity is moveable through wires, thus is a good and Article 2 applies.
(h) Devices signed a distribution agreement giving Detail Retail the exclusive right to sell Gooseberry phones in the Midwest. Is
this an Article 2 transaction? Same result if Detail Retail also agreed to buy 1 million Gooseberry phones directly from
Devices? See MidAmerican Distribution, Inc. v. Clarification Technology, Inc., Franklin Publications, Inc. v. General Nutrition Corp.
 The agreement is not a good, it is for franchising,
 The devices are goods.
(i) Specially manufacturing and supplying a yacht? See §2-105(1); Fed. Ins. Co. v. Mathews Bros.
 This is a mixed question as there is a good and there is a service involved, but specially manufactured indicates that this will
be looked at as a service.
(j) When a customer paid for groceries at the counter, she was given a plastic bag to put them in. The bag proved defective such
that the groceries spilled, and she suffered serious injuries from broken glass in the bag. Is there a sale of goods as to the plastic
grocery bag by the grocery store? See Fuller v. Winn-Dixie Montgomery.
 The bag is a good, but not identified to the contract. Thus, not covered by Article 2.
 However, if you have to pay for the bag, it becomes a part of the contract.
(k) You go online to Amazon.com and buy a product there. Is Amazon the seller who will be responsible if the product is
defective? See Milo & Gabby, LLC v. Amazon.com, Inc.
 The case here said no, as under sections 2-103 and 2-106 - Amazon never had title.
“Predominant Factor Test” and the “Gravamen Test”
o The principal standard that is used for determining whether a transaction involves the sale of goods or the sale of services is the
“predominant factor test.” In our example this standard would ask whether the sale of the pool was “predominantly” a sale of
goods or a sale of services.
o Another standard that is sometimes used is the “gravamen test.” This standard considers the nature of the plaintiff’s legal
claim against the defendant. Under our example this standard would ask whether the homeowner was claiming that the
materials were defective (sale of goods covered by Article 2) or whether it was improperly installed (sale of services governed by
the common law.)
 Why Does It Matter Whether Article 2 Applies?
o Article 2 is different from the common law of contracts in many respects. Here are some examples of provisions of Article 2 that
differ from the common law of contracts or the common law of torts:
 Warranty of Merchantability – Section 2-314 includes an implied promise that the goods are fit for ordinary purposes.
Under the common law, a higher standard such as negligence would apply.
 Statute of Frauds – Section 2-201 requires contracts for the sale of goods for $500 or more to be evidenced by a writing; at
common law the Statute of Frauds is different;
 Battle of the Forms – Section 2-207 establishes different rules than the common law does for determining the terms of a
contract where the contract was performed but the offer and the acceptance did not perfectly match;
 “Firm Offer” Rule – Section 2-205 provides that a “firm offer” may be enforced, unlike the common law of contracts;
 “Perfect Tender” Rule as a Standard of Performance – Section 2-601 requires a party to perform precisely in accordance
with the contract (unlike the “material breach” rule of the common law);
 Strict Liability for Breach of Warranty – Section 2-714 does not require proof of negligence for breach of warranty as does
the common law of tort;

9
 Privity Rules for Breach of Warranty – Section 2-318 relaxes the common law requirement of “privity” for breach of
warranty;
 Statute of Limitations – Section 2-725 establishes a four-year statute of limitations; at common law the period is typically
longer for breach of contract and shorter for torts.
 Audio Visual Artistry v. Tanzer [PREDOMINATE PURPOSE]
o In this case the seller of a home entertainment system that didn’t work as promised contended that the UCC didn’t apply
because the contract was primarily for the sale of services in setting up the system. The buyer was suing under breach of the
implied warranty of merchantability. The Tennessee court used the “predominate purpose” test, and then worked through its
various elements:
1. the language of the contract;
• The contractual language was important here, the use of “buyer” and “seller”.
2. the nature of the business of the supplier of goods and services;
• The seller’s business was primarily the sale of the goods, not design of the goods themselves.
3. the reason the parties entered into the contract, and
• The reason for the contract was to buy equipment that would produce excellent home entertainment.
4. the amounts paid for the rendition of the services and goods, respectively.
• The court looked at invoice and the price largely consisted of the cost of the goods not the service of installing them
into buyer’s home.
o The court noted that none of these factors alone is dispositive.
o The court held that this was predominately a sale of goods, and Article 2 and its provisions therefore governed the transaction.
 Anthony Pools v. Sheehan
o Anthony Pools shows another approach, the “gravamen” test, first suggested by Professor Hawkland in 1982, which looks at the
complaint and asks it is focusing on the product or how it was installed. This is a hybrid test: “A number of commentators have
advocated a more policy- oriented approach to determining whether warranties of quality and fitness are implied with respect
to goods sold as part of a hybrid transaction in which service predominates. . .To support their position, these commentators in
general emphasize loss shifting, risk distribution, consumer reliance and difficulties in the proof of negligence.”
o So, in this case the sale of a diving board was covered by Article 2 (and its implied warranty of merchantability), even where the
contract as a whole was for construction services. Sheehan could use the implied warranty of merchantability provided by §2-
314, after he was injured falling from the new diving board which allegedly had insufficient skid resistant material on its edges. It
was the diving board itself that was at fault here, not its installation, and that was the “gravamen” of the complaint.
o Subsequent cases, even in Maryland, have not adopted this gravamen test, and it has not been influential in other jurisdictions.
Thus, this is not really good law.
o White & Summers, at §10-2, see Anthony Pools as an example of a case where a court may be influenced by a sympathetic
plaintiff in determining that Article 2 does apply.
 Applying Article 2 by Analogy
o Even if Article 2 does not apply to a transaction, it is possible that the courts might apply Article 2 by analogy instead of using
the common law of contract.
 The Special Case of Computer Software
o Computer software may be sold on a disk, but as everyone knows the purchaser is not buying a disk but rather the information
stored on the disk. Nevertheless, the courts have ruled that computer software that is mass-manufactured is “goods,” and that
Article 2 applies to the transaction. Software that is specially designed for a specific customer constitutes “services,” and Article
2 does not apply to the transaction.
o A model statute – the Uniform Computer Information Transactions Act (UCITA) – has been proposed to govern the sale of
software, but it has been adopted in only two states (Maryland and Virginia).
o In addition, the ALI has published the “Principles of the Law of Software Contracts” and courts may look to that document for
guidance as well.
 Simulados Software, Ltd. v. Photon Infotech Private, Ltd.
o Here the court struggles with the issue of Article 2’s application to the creations of software. It’s clear that the sale of packages
of software is the sale of goods in most jurisdictions, as is the sale of downloadable software, but the issue gets murkier with
software designed especially for a particular purpose, as here. Here, the court concludes that the contract was to custom design
existing software so as to change it to meet the plaintiff’s particular needs, and that this therefore was the sale of services: the
talent needed to change the existing software, not the software itself. White & Summers, at §2-1, note that courts tend to find
a sale of goods where pre-existing software is sold with customizing or upgrades, and services where there is entirely new
software made from scratch. Such courts view software development contract as predominantly one for services, because the
buyer relies on the seller’s application of knowledge and expertise. But note that there is similar reliance in a contract for
specially manufactured goods, which Article 2 specifically includes within the definition of goods. §2-105(1).
 Casebook Problem PAGE 43
o Boswell Consultants purchased a Hawking 2020 printer from Ganges Devices. For the contract price, Ganges Devices agreed to
deliver the internet-enabled printer along with its software, to store printed files in Ganges’s cloud, to provide remote

10
diagnostic and maintenance, and to provide software updates tailored to Boswell Consultants’ pattern of use. Is the transaction
covered by Article 2? See §2-105(1)
 The sale of software as part of a sale of goods connected with the software is somewhat easier than the question of
whether software sold alone is the sale of goods, as to which the courts have reached differing results. Is software a “thing”
or merely a “process”? If the courts decide the latter then the UCC does not apply and the lawsuit will be resolved under
the murkier rules of the common law (typically contract law, but negligence and other theories can creep in also). But here
there is a physical object being sold and the software is part of helping the buyer use and maintain it, so the courts, using
that “predominant purpose” test, are likelier to find that a sale of goods carries the day, though it’s not a sure thing. There
was a big movement at the turn of the century to amend Article 2 to include software in its coverage, but that effort died
aborning, and we are left with the usual unpredictability on this issue.
 Sale of Goods or Lease of Goods?
o Sales of goods are governed by Article 2 of the U.C.C. Leases of goods are governed by Article 2A. It does not matter whether
the transaction is called a sale or a lease. What matters is whether the only reasonable outcome of the transaction is that the
buyer/lessee will retain the goods for the economic life of the goods; if so, the transaction is a sale of goods.
o The standards for determining whether a transaction is a sale, or a lease are set forth in Section 1-203.
Section 1-203: WHETHER A TRANSACTION IS A SALE OR A LEASE
(a) Whether a transaction in the form of a lease creates a lease or security interest is determined by the facts of each case.
(b) A transaction in the form of a lease creates a security interest if the consideration that the lessee is to pay the lessor for
the right to possession and use of the goods is an obligation for the term of the lease and is not subject to termination by the
lessee, and: (1) the original term of the lease is equal to or greater than the remaining economic life of the goods; (2) the
lessee is bound to renew the lease for the remaining economic life of the goods or is bound to become the owner of the
goods; (3) the lessee has an option to renew the lease for the remaining economic life of the goods for no additional
consideration or for nominal additional consideration upon compliance with the lease agreement; or (4) the lessee has an
option to become the owner of the goods for no additional consideration or for nominal additional consideration upon
compliance with the lease agreement.
(c) A transaction in the form of a lease does not create a security interest merely because: (1) the present value of the
consideration the lessee is obligated to pay the lessor for the right to possession and use of the goods is substantially equal to or
is greater than the fair market value of the goods at the time the lease is entered into; (2) the lessee assumes risk of loss of the
goods; (3) the lessee agrees to pay, with respect to the goods, taxes, insurance, filing, recording, or registration fees, or service
or maintenance costs; (4) the lessee has an option to renew the lease or to become the owner of the goods; (5) the lessee has
an option to renew the lease for a fixed rent that is equal to or greater than the reasonably predictable fair market rent for the
use of the goods for the term of the renewal at the time the option is to be performed; or (6) the lessee has an option to
become the owner of the goods for a fixed price that is equal to or greater than the reasonably predictable fair market value of
the goods at the time the option is to be performed.
(d) Additional consideration is nominal if it is less than the lessee's reasonably predictable cost of performing under the
lease agreement if the option is not exercised. Additional consideration is not nominal if: (1) when the option to renew the lease
is granted to the lessee, the rent is stated to be the fair market rent for the use of the goods for the term of the renewal
determined at the time the option is to be performed; or (2) when the option to become the owner of the goods is granted to
the lessee, the price is stated to be the fair market value of the goods determined at the time the option is to be performed.
(e) The "remaining economic life of the goods" and "reasonably predictable" fair market rent, fair market value, or cost of
performing under the lease agreement must be determined with reference to the facts and circumstances at the time the
transaction is entered into.
 Summary of Section 1-203(b)
o A transaction is a lease if the lessee may cancel the lease before the expiration of the term of the lease.
o A transaction is a sale if the required term of the lease is longer than economic life of the goods; if the lessee is bound to renew
the lease for the remaining economic life of the goods; or if the lease gives the lessee the option to become the owner or renew
the lease for the remaining economic life of the goods for no or nominal consideration.
 Example of a “Lease” That Is Really a “Sale”
o Frederick leases a large power washer from A-1 Tools for $80 per month for five years. When the “lease” was entered into the
expected life of the power washer was five years; it was expected that the washer would be worth nothing at the end of that
period. Under the lease Frederick does not have the power to cancel the lease; he is required to make the lease payments for
the full five-year period.
o This transaction is actually a “sale” governed by Article 2.
 Another Example of a “Lease” That Is Actually a “Sale”
o Sally leased a large trench digger from Rogers Equipment Company for four years, paying $320 per month. When the lease was
entered into it was expected that the trench digger would have an economic life of six years and that at the end of the four-year
lease term the trench digger would still be worth $5000. The lease provided that Sally could purchase the trench digger for $1 at
the end of the four-year lease term. Under the lease Sally does not have the power to cancel the lease during its four-year term.
o This transaction is actually a “sale” governed by Article 2.

11
 Scope of Article 2A
o Article 2A largely takes the Article 2 rules for sales and makes them applicable to leases—which makes sense, because acquiring
goods through leases has many relevant similarities to buying them. The big distinction between 2 and 2A is the finance lease—
where the lessor is really more like a lender.
 Example of A Finance Lease
o Business is interested in acquiring a machine from Supplier. Rather than buy the machine on credit, Business arranges for
Finance Co. to buy the machine and lease it to Business. Because this is a finance lease (1) Finance Co. will make no warranties—
Business should get them from Supplier; and (2) Business will be bound to pay all the lease payments to Financer even if the
machine is defective (and should then recover from Supplier). This issue will be expanded upon in detail later in the course.
 Lease Disguised as a Sale on Credit.
o Sometimes a transaction is called a “lease” when it is in reality a sale of goods. This is done for various reasons: the bookkeeping
and tax considerations mentioned above, plus the desire to escape from the necessity of complying with either Article 2 or the
secured transaction rules of Article 9.
o If a so-called lease does not pass the tests in Section 1-203, Article 2A is not triggered. Instead, the “lease” will be treated as a
disguised sale on credit, with the “lessor’s” interest in reality being nothing more than the “reservation of a security interest.”
o Sales on credit are qualified as what Article 9 of the UCC calls a “secured transaction,” meaning, the seller has the right to
repossess the goods if payment is not made by using its “security interest” (i.e., a lien on the goods) to reclaim them (and being
also required to follow the formidable rules of Article 9).
o If the “lease” is really a “sale,” and the seller did not follow the Article 9 rules, it runs the substantial risk of losing the ability to
repossess the goods and/or having junior priority in them to other creditors (or to the buyer’s bankruptcy trustee). It is
therefore very important for the so-called lessor to prevail in this argument, or to take the precautionary step of complying with
the rules of Article 9 in case it has to battle other Article 9 claimants to the goods; see §9-505 (allowing a filing without it
necessarily being an admission of a disguised sale).
 Casebook Problem PAGE 52
o BIG Machines, Inc., leased a computer to Helen’s Flower Shoppe for a five-year period. The machine was new and had cost BIG
Machines $10,000. Helen’s Flower Shoppe promised to pay $225 a month as rent. Is this a lease or a disguised sale? Is your
answer affected by the following considerations?
 Since Helen’s Flower Shoppe will have paid $13,500 as rent for the computer over the life of the lease, it has certainly more
than compensated the lessor for the value of the machine, though this does not answer the key question of whether the
computer will have any remaining economic life at the end of the lease term. In fact, §1-203(c)(1) expressly decries use of
the fair market value of the goods as a deciding factor. Without more, it is therefore impossible to say whether this is a true
lease or a disguised sale.
(a) The lease provided that the lessee could terminate the lease at any time and return the computer to the lessor.
 If the lessee has a right to terminate at any time, a true lease has been affected. An escape clause is not an attribute of a
typical sale. Under the statutory language this result is reached by referring to the part of the definition excepting leases
“not subject to termination by the lessee.”
(b) Assume there was no such option as described in (a), but the goods had no value at the end of the five-year period.
 If the computer has no value at the end of the term, its economic life has been exhausted and, under the first (a) of the
definition, a disguised sale has occurred.
(c) Assume instead that the rental amount is only $150 a month and the computer will be worth $3,500 at the end of the five-year
period. The lease has a clause giving Helen’s Flower Shoppe the option to purchase the computer for that amount at that time.
Is this a true lease? What if the lease requires the lessee to renew this lease at the end of the five-year period for another five
years?
 Here the computer has economic life left at the end of the leased term, and the option to renew is neither required nor is it
obvious that the lessee will want to renew. Under all the tests in the section, this is a true lease. If the lease requires ten
years of payments (120 x $150 = $18,000), the fair market value of the computer will have been paid to the lessor, and it is
hard to imagine that the computer will have any economic life left at the end of the lease, so, under the language of §1-
203(b)(2), a disguised security interest is indicated.
 Canvas Quiz Questions
o Which body of law primarily applies to the following kinds of transactions in goods?
 Sales of goods - Article 2 of the U.C.C.
 Leases of goods - Article 2A of the U.C.C.
 Gifts of goods - The common law of
property
 Security interests in - Article 9 of the U.C.C.
goods
o What is the definition of a "sale" of goods under the U.C.C.?
 A "sale" occurs when a merchant advertises goods at less than their regular cost.
 A "sale" is the same as a lease or a security interest.

12
 Answer: A "sale" is "the passing of title from the seller to the buyer for a price."
 A "sale" of goods includes a gift of goods.
o Goods are things that are movable at the time of identification to the contract.
 True
o Please match the type of goods to its definition
 Consumer goods - Section 9-102(23) -- "goods that are used or bought for use primarily for
personal, family, or household purposes"
 Goods that are known as "fixtures" - Section 2-107(2) -- "other things attached to realty and capable of severance
(Article 2 does not use that term) without material harm thereto but not described in subsection (1) [subsection
1 refers to minerals, structures, and structural materials]
 Future goods - Section 2-105(2) -- Goods that "are not both existing and identified"
 Minerals, structures, and structural - Section 2-107(1) -- minerals or the like (including oil and gas) or a structure or
materials its materials
o Article 2 applies only if the goods have been identified to the contract. A contract for the future sale of goods is governed by the
common law.
 False - Article 2 applies to both a present sale of goods and a contract for the sale of goods that have not yet been identified
to the contract.
o Does Article 2 apply to a sale of the following items if the item is severed from realty only by the buyer, only by the seller, or
by either the buyer or seller?
 Sale of a building - Covered by Article 2 only if severed by the seller
 Sale of minerals currently under the - Covered by Article 2 only if severed by the seller
land such as oil and gas
 Sale of a fixture in a building - Covered by Article 2 if severed either by buyer or seller
 Sale of standing timber - Covered by Article 2 if severed either by buyer or seller
 Sale of standing crops - Covered by Article 2 if severed either by buyer or seller
o The key difference between a sale and a lease is
 that sales of goods are covered by Article 2A while leases of goods are covered by Article 2.
 what the parties call the transaction. If the parties call the transaction a sale, then Article 2 applies. If they call it a lease,
then Article 2A applies. – It does not matter whether the parties themselves refer to the transaction as a lease or a sale.
 Answer: whether the transaction is structured such that the "buyer/lessee" will end up with the goods for the entire
economic life of the goods. If so, then the transaction is a "sale" within the meaning of Article 2.
 whether the "seller/lessor" is a merchant in the business of dealing in goods of the kind. If so, the transaction is a "sale"
within the meaning of Article 2.
o What legal standards are used for distinguishing a contract for the sale of goods from a contract for the sale of services?
 The "predominant factor test" - whether the contract was predominantly for the sale of goods or predominantly for the sale
of services.
 The "gravamen test" -- whether the plaintiff's injury was caused by a defect in the goods or because the services that were
rendered were negligent.
 The "labeling test" -- whether the parties themselves described the contract as a sale of goods or a sale of services.
 The "choice of law" test -- whether the parties themselves stated in the contract that any disputes would be resolved under
Article 2 or the common law of contract.
o Nathan owns a house he intends to tear down. Before demolition he intends to remove and sell a number of items from the
house: a washing machine, the central air conditioner, the furnace ducts, and some copper piping. The washing machine will
just unplug from the wall; the central air conditioner and the furnace ducts are firmly attached but can be removed with
minimal damage to the house; and the copper piping can't be removed without opening large holes in the walls, floors, and
ceilings. What category of goods does each item belong to?
 Washing machine - Goods
 Furnace ducts - Fixture -- considered "goods" whether severed by the seller or the buyer
 Copper piping - Building materials -- not "goods" unless severed by the seller
 Central air - Fixture -- considered "goods" whether severed by the seller or the buyer
conditioner
o Carol rented a large, high quality industrial grade washer/dryer for her house from Zippy Appliances. The lease is for three years
and the payments are $60 per month. The washer and dryer are expected to last about 10 years. Carol is obligated to make the
lease payments for the entire three-year lease term, but at the end of that time she may purchase the appliance for $20. (It's
expected that the equipment would be worth about $500 at that time.) This transaction will be governed by the following
areas of law:
 Answer: Article 2 and Article 9, because functionally this is a sale of goods and the retention of a security interest in the
seller. - The only rational choice that Carol would make would be to exercise the option to purchase the goods at the end of
the three-year lease term.

13
 The common law, because the Uniform Commercial Code does not apply.
 Article 2A, because it is a lease of goods.
 The common law, because the law of real property governs appliances.
o Frank purchased an above-ground pool from Family Fun, Inc. The entire price of the pool was $10,000. The contract allocated
$2,000 to the cost of the materials and $8,000 for the cost of assembly and installation. After Family Fun set up the pool it did
not work properly because the pump and the water filter were mechanically defective. What law governs this transaction?
 Under the "Predominant Factor Test" Article 2 governs this transaction because it was predominantly a sale of goods, not a
sale of services.
 Under the "Gravamen Test" Article 2 governs this transaction because the gravamen of Frank's claim is that the goods that
were sold were defective.
 Under the "Predominant Factor Test" the common law governs this transaction because it was predominantly a sale of
services, not a sale of goods.
 Under the "Gravamen Test" the common law governs this transaction because the gravamen of Frank's claim is that the
assembly and installation of the pool was faulty.
o If Article 2 does not apply according to its terms, then the courts will not use it. The courts never apply Article 2 by analogy.
 False - The courts often apply the rules of Article 2 by analogy, even if the transaction is not a "sale of goods."
II. Merchants
 “Merchants” – Why is this important?
o Article 2 applies to contracts for the sale of goods regardless of who the parties are. It applies to contracts for the sale of goods
between merchants, contracts between merchants and non-merchants, and contracts for the sale of goods between two non-
merchants.
o Nevertheless, there are provisions of Article 2 that apply only to merchants and other provisions that apply only to consumers,
so it is important to distinguish merchants from non-merchants. The term “merchant” is defined in Section 2-104.
o Examples of Article 2’s rules that apply only to merchants: a merchant automatically makes an implied warranty of
merchantability, firm offers by merchants are irrevocable, special rules govern merchants with respect to contract formation.
 Section 2-104(1): Definition of “Merchant”
“Merchant” means a person who deals in goods of the kind or otherwise by his occupation holds himself out as having
knowledge or skill peculiar to the practices or goods involved in the transaction or to whom such knowledge or skill may be
attributed by his employment of an agent or broker or other intermediary who by his occupation holds himself out as having such
knowledge or skill.
 Section 2-104(3): Definition of “Between Merchants”
"Between Merchants" means in any transaction with respect to which both parties are chargeable with the knowledge or skill of
a merchant.
 “Merchants”
o Rooted in the “law merchant” concept of a professional in business.
o Based on specialized knowledge as to the goods, specialized knowledge as to business practices, or specialized knowledge as to
both.
o Almost every person in business would meet the definition of a “merchant” so long as that person is acting within his/her
mercantile capacity.
o But note that certain sections, e.g., §2-314 (warranty of merchantability) apply only if the “seller is a merchant with respect to
goods of that kind.” See also §§2-402(2), 2-403(2).
o The language “…or to whom such knowledge or skill may be attributed by his employment of an agent or broker…” means that
even persons such as universities can come within the definition, for example, if they have purchasing groups or business
personnel who are familiar with business practices.
 Casebook Problem PAGE 43
o Portia Moot, a third-year law student, sold her car to a fellow student. Does Article 2 of the UCC apply to this transaction?
Would §2-314 apply to the sale?
 Article 2 does apply. The point here is that Article 2 applies to all transactions in goods taking place within the enacting
jurisdiction. NOTE: Article 2 does not only apply to transactions involving merchants or only to transactions in goods for
more than $500).
 That said, the warranty of merchantability is not made unless the seller is a merchant as to goods of the kind sold, which
Portia is not. Please read carefully the sections of the Code to determine which apply only to merchants and which apply to
any sale of goods.
Frix v. Integrity Medical Systems, Inc.
o This is a “battle of the forms” case, which is one of the subjects of our next module, but the issue that should attract your
attention is whether a doctor ordering medical equipment is a “merchant” for Code purposes.
o Certainly, here the doctor was not a seller of the goods, but was he a merchant as to “practices”? Would it be fair to hold him to
mercantile standards in this situation because his occupation sometimes includes the purchase of such goods?

14
 The court decides the answer is no, but it was a close call. Doctors order a lot of medical equipment and frequently must do
some installation.
Casebook Problem PAGE 49
o Are the following persons merchants?
(a) Amanda, who quit her teaching job on Friday and on Monday opened a clothing store?
 Amanda would clearly fit within the explicit language of §2-104(1), which appears to give no grace period for the acquisition
of the skills of a merchant. This makes some sense: if someone goes into business, the public who deals with this person will
expect compliance with mercantile standards and not to be the victims of a learning curve.
(b) Tom Tiller, a farmer selling his produce to a wholesaler? Compare Loeb & Co. v. Schreiner (“Although a farmer might sell his
cotton every year, we do not think that this should take him out of the category of a ‘casual seller’ and place him in the category
with ‘professionals.’ “) with Continental Grain Co. v. Brown (“A sale of 75,000 bushels of corn for a total price in excess of
$212,000 is not a ‘casual’ sale.”). See also Vince v. Broome (“Some farming operations are worth millions of dollars.... It would
stretch the imagination to conclude that all these operations were exempt from coverage under the Commercial Code.”); Ohio
Grain Co. v. Swisshelm, (the modern farmer is more than “a simple tiller of the soil, unaccustomed to the affairs of business and
the marketplace.... Only an agribusinessman may hope to survive.”); Harvest States Cooperatives v. Anderson (man who had
some cows and made occasional sales of corn not a merchant).
 Farmers are or are not merchants depending on how the court reads Official Comments 1 and 2 of §2-104 (“casual or
inexperienced seller,” “professional in business”) and Official Comment 3 to §2-314 (Merchantability), which says that one
making an isolated sale is not a “merchant.”
 The courts are really split on the farmer qua merchant. Most courts apply Article 2 if the farming operation has any degree
of sophistication so that it’s fair to hold the farmer to the rules of business practices
III. International Sale of Goods
 The CISG, the UNIDROIT Principles, and the Incoterms
o Three principal sources of international commercial law are the CISG, the UNIDROIT principles, and the incoterms.
 International Transactions
o In international transactions the United Nations Convention for the International Sale of Goods (CISG) presumptively applies
unless the parties opt out of it and choose to be regulated by some other law. Despite the fact that the CISG presumptively
applies to international sales American companies almost uniformly choose the U.C.C. over the CISG. The U.C.C. is more precise
and more detailed and in any event much of the CISG was modeled after Article 2 of the U.C.C. People who are conducting
business need for the law to be clear in order to minimize the transaction costs of resolving any disputes that arise.
 The United Nations Convention on the International Sale of Goods (CISG)
o The United Nations Convention on Contracts for the International Sale of Goods (CISG) is a multilateral treaty that establishes a
uniform legal regime for contracts for the cross-border sale of goods.
o According to its drafter, the United Nations Commission on International Trade Law (UNCITRAL), such sales contracts “operate
as the backbone of international trade, and uniform rules for sales contracts provide traders with an efficient, modern and
comprehensive legal framework to support their commercial activities.”
o Its uniform rules, specifically designed to meet the needs of cross border commerce, increase certainty in commercial
transactions and decrease their costs.
o The United Nations Commission on International Trade Law (UNCITRAL) has since 1968 drafted a number of treaties governing
international commerce, including the United Nations Convention on the International Sale of Goods – the CISG.
o As of 2020, the CISG has been adopted by 94 States from all legal traditions and levels of economic development and that
together represent over two-thirds of the global economy. Two notable exceptions that have not signed the CISG are the United
Kingdom and India.
o Certain countries have adopted the CISG subject to authorized declarations. There are also several instances of States
accompanying their acceptances with interpretive comments (a procedure not authorized by the CISG).
o The United States reservation: "Pursuant to article 95 the United States will not be bound by subparagraph (1) (b) of Article 1”
(Havana Charter).
 Key Provisions of the CISG
o The CISG governs contracts for the international sales of goods between private businesses, excluding sales to consumers and
sales of services, as well as sales of certain specified types of goods.
o It applies to contracts for sale of goods between parties whose places of business are in different Contracting States, or when
the rules of private international law led to the application of the law of a Contracting State.
o It may also apply by virtue of the parties' choice.
o The second part of the CISG deals with the formation of the contract, which is concluded by the exchange of offer and
acceptance.
o The third part of the CISG deals with the obligations of the parties to the contract.
o Obligations of the sellers include delivering goods in conformity with the quantity and quality stipulated in the contract, as well
as related documents, and transferring the property in the goods.
o Obligations of the buyer include payment of the price and taking delivery of the goods.
15
o In addition, this part provides common rules regarding remedies for breach of the contract. The aggrieved party may require
performance, claim damages or avoid the contract in case of fundamental breach.
o Additional rules regulate passing of risk, anticipatory breach of contract, damages, and exemption from performance of the
contract.
o Finally, while the CISG allows for freedom of form of the contract, States may lodge a declaration requiring the written form.
o The CLOUT (Case Law on UNCITRAL Texts) system contains numerous cases relating to the application of the CISG.
 Certain matters relating to the international sales of goods, for instance, the validity of the contract and the effect of
the contract on the property in the goods sold, fall outside the Convention's scope
 Opting Out of the CISG
o If the contracting parties do not want the CISG to apply it is advisable for the contract to expressly so provide. For example:
o “The rights and obligations of the parties arising under this contract shall be determined in accordance with the law of the State
of ___ including the Uniform Commercial Code as enacted in the State of ___. The rights and obligations of the parties arising
under this contract shall not be determined by the United Nations Convention on the International Sale of Goods.”
 The CISG is Binding Law
o The CISG was ratified by the United States and has been our law since January 1, 1988. It is a self-executing multilateral treaty
between the United States and other signatories; thus, it is binding, by default, upon private international “contracts for the sale
of goods between parties whose places of business are in different States … when the States are contracting States.”
o UCC Article 2 deals with Sales and is often referred to as the “domestic analog of the CISG.” There are many similarities between
the two codes (UCC and CISG); however, there are important differences.
o U.S. manufacturers who use their domestic sales documents for international transactions may be in for a big surprise!
 Key Points
o A choice of the law of a CISG Contracting State includes the choice of the CISG. (The Draft Hague Principles on Choice of Law in
International Commercial Contracts, July 2014)
o The parties to a contract may agree to apply a signatory’s domestic law, but only by affirmatively opting-out of the CISG. (CISG
Article 6)
o Therefore, a choice of law provision, to be effective, must not only select the law that will apply, but affirmatively state that the
CISG will not apply to the contract.
 The Scope of the CISG
o The CISG governs contract formation, performance, and remedies.
The CISG does not govern the validity of the contract (illegality, capacity, duress, etc.), or any property rights other than those arising
out of the contract.
 Similarities Between the CISG and the U.C.C.
o The CISG and Article 2 of the U.C.C. are similar in many respects, including the following:
1. In determining whether the contract is a contract for the sale of goods or a contract for the sales of services;
2. Interpretation of contracts;
3. Implied warranties of merchantability and fitness;
4. The right of the seller to cure;
5. Revocation of acceptance;
6. Adequate assurances; and
7. Measure of monetary damages.
 Sale of Goods versus Sale of Services
o In determining whether the contract is a contract for the sale of goods or a contract for the sales of services, most U.S.
jurisdictions use the “predominant purpose” test. Similarly, Article 3 of the CISG excludes transactions where the “preponderant
part” of the seller’s obligations “consists in the supply of labour or other services.”
 The Interpretation of Contracts
o Both Section 1-303 of the U.C.C. and Article 9 of the CISG seek to interpret the intent of the parties by looking to the express
words of the contract, the previous conduct of the parties, and prevailing commercial practice.
 Implied Warranties
o The implied warranty of merchantability and the implied warranty of fitness for a particular use are nearly identical under the
CISG and U.C.C. Compare Sections 2-314 and 2-315 of the U.C.C. to Article 35 of the CISG.
 Seller’s Right to Cure
o 2-508 of the U.C.C. on its face seems to give the seller an absolute right to cure any non-conformities in the goods. However, it
has been interpreted to mean that if there are repeated attempts to cure then there may come a time when “enough is
enough” and the buyer is entitled to revoke acceptance of the goods. Article 34 of the CISG expressly provides that a seller has a
right to cure if it can do so without unreasonable delay or by causing the buyer unreasonable inconvenience.
 Revocation of Acceptance
o Under Section 2-608 of the U.C.C. a buyer may revoke acceptance of the goods if the non-conformity “substantially impairs” the
value of the goods. Under Article 46 of the CISG a party may avoid the contract if the breach is “fundamental” – which is defined

16
in Article 25 as a breach that “results in such detriment to the other party as substantially to deprive [it] of what [it] is entitled to
expect under the contract.”
 Adequate Assurances
o Under both Section 2-609 of the U.C.C. and Article 71 of the CISG, if circumstances reasonably call into question a party’s ability
or intention to perform, the other party is entitled to demand adequate assurances of performance. If a party fails to respond to
the request or fails to give adequate assurances of performance, the party who demanded the assurances may cancel the
contract.
 Measure of Monetary Damages
o The CISG, like the U.C.C., requires the breaching party to compensate the non-breaching party for losses that are caused by the
breach, including losses that are reasonably foreseeable consequences of the breach. Accordingly, the monetary remedies for
restitution, expectancy, incidental and consequential damages under the CISG are similar to the monetary remedies that are
available under the U.C.C. For example, under Section 2-712 of the U.C.C. and Article 75 of the CISG if the buyer covers
(purchases replacement goods) after a breach by the seller, the buyer is entitled to recover the difference between the contract
price and the cover price.
 Differences Between the CISG and the U.C.C.
o The CISG and the U.C.C. are also different in certain respects:
1. Application to consumers. The U.C.C. applies both to sales to merchants and sales to consumers. Article 2 of the CISG
provides that the treaty does not apply to sales of goods where the buyer intends to use the goods for personal, family, or
household purposes. As a consequence, the CISG applies only to transactions between commercial parties.
2. CISG Article 11: A contract of sale need not be concluded in or evidenced by writing and is not subject to any other
requirement as to form. It may be proved by any means, including witnesses.
3. Statute of Frauds. Section 2-201 of the U.C.C. provides that contracts for the sale of goods over $500 must be in writing to
be enforceable. The CISG does not have a Statute of Frauds; oral contracts are enforceable.
4. CISG Article 8 of the CISG permits the court to consider the parties’ negotiations, course of dealing, and subsequent conduct
to determine the parties’ intent when interpreting the contract.
• Note: No parol evidence rule like UCC §2-202!
5. CISG Article 19: A reply to an offer which purports to be an acceptance but contains additions, limitations or other
modifications is a rejection of the offer and constitutes a counteroffer.
• Note: this is more like the common law “mirror image rule” and less like UCC §2-207.
– However, a reply to an offer, which purports to be an acceptance, but contains additional or different terms which
do not materially alter the terms of the offer, constitutes an acceptance, unless the offeror, without undue delay,
objects orally to the discrepancy or dispatches a notice to that effect. If he does not so object, the terms of the
contract are the terms of the offer with the modifications contained in the acceptance. Additional or different terms
relating, among other things, to the price; payment; quality and quantity of the goods; place and time of delivery;
extent of one party’s th to the other; or the settlement of disputes are considered to alter the terms of the offer
materially.
6. Level of detail. The U.C.C. is far longer and more detailed than the CISG.
 Practice Pointer
o Your client’s standard terms and conditions may expressly provide that the agreement will be governed by the law of your state
and may require the seller to consent to the exclusive jurisdiction of your federal court.
o However, if it does not affirmatively state that the CISG does not apply to this contract, you may end up defending the breach of
contract claim under CISG (and not under the UCC) in your federal court and not in state court.
o Note: if a contract states that it is governed by UNIDROIT principles or terms and conditions available for inspection on the
internet at a specified URL (e.g., incoterms) – or states in German that the exclusive jurisdiction and venue is Berlin, Germany
– the fact that you did not review the T&Cs online and that you cannot sprechen sie deutsch will not prevent your federal court
from dismissing your claim and sending you packing for Berlin!
– Auf wiedersehen!
 Sample Clause
o This Agreement and all Addenda and other related documents including all exhibits attached hereto, and all matters arising out
of or relating to this Agreement, whether sounding in contract, tort, or statute are governed by, and construed in accordance
with, the laws of the Commonwealth of Pennsylvania, United States of America without giving effect to any choice or conflict of
law provision or rule whether of the Commonwealth of Pennsylvania or any other jurisdiction that would cause the application
of laws of any jurisdiction other than those of the Commonwealth of Pennsylvania. The parties expressly agree that the United
Nations Convention on Contracts for the International Sale of Goods does not apply to this Agreement.
 The UNIDROIT Principles
o The International Institute for the Unification of Private Law has drafted a set of model principles of contract law applicable to
commercial parties. The UNIDROIT principles are not law in and of themselves.
o Instead, the principles are used mainly in arbitration. It is common for international contracts to require disputes to be resolved
in arbitration and to require the arbitrators to apply the UNIDROIT principles.

17
o The Principles of International Commercial Contracts 2016 (most frequently referred to as UNIDROIT Principles and often also
referred to as PICC) is a set of 211 rules for international contracts. They have been drawn up since 1984 by an international
working group of the inter-governmental organization UNIDROIT, and they were ratified by its Council representing 64
governments of member states.
o These principles help harmonize international commercial contract law by providing rules supplementing international
instruments like the CISG and even national laws.
o Most importantly in private practice, they offer a neutral contractual regime which the parties can choose, either by
incorporation into their contracts (in whole or in parts), or by a straightforward choice of the UNIDROIT Principles (e.g., “This
contract is governed by the UNIDROIT Principles of International Commercial Contracts 2016”; in practice such a clause is often
combined with an arbitration clause).
o As noted above, if two international parties do not want the CISG to apply, it is advisable for the contract to expressly provide
that it does not apply.
 The Equitable Doctrines of the UNIDROIT Principles
o The UNIDRIOT principles incorporate a number of equitable doctrines, including principles relating to good faith, reasonable
expectations, unconscionability, and specific performance.
o For example, under the U.C.C. the principle of good faith is applicable to the “performance and enforcement” of contracts. The
UNIDROIT principles extend the obligation of good faith to the negotiation of contracts as well. The principles also contain
express provisions that protect against “surprising terms” that are inconsistent with the reasonable expectations of a party or
terms that unjustifiably give one party to the contract an excessive advantage, particularly where there is a gross disparity in
the bargaining power of the parties. Finally, as under the CISG, the UNIDROIT principles prefer the remedy of specific
performance over the award of monetary damages.
 Incoterms
o International contracts recognize a set of shipping terms (“incoterms”) that are different from the abbreviations that are
recognized by the U.C.C. Incoterms are three-letter trade terms that determine whether the contract is a shipment or a
destination contract, who will bear the expense of shipment, who will bear the risk of loss, and several other factors.
o “Incoterms®” is an acronym standing for international commercial terms. “Incoterms®” is a trademark of International Chamber
of Commerce, registered in several countries.
o The Incoterms® rules feature abbreviations for terms, like FOB (“Free on Board”), DAP (“Delivered at Place”) EXW (“Ex Works”),
CIP (“Carriage and Insurance Paid To”), which all have very precise meanings for the sale of goods around the world. E.g., FCA
are the initials used for “Free Carrier,” or the seller’s obligation to deliver the goods to the carrier nominated by the buyer at
the seller’s premises or another named place.
o Be careful! Some of the incoterms are identical to the delivery abbreviations used by the U.C.C. but they have different
meanings under international law. For example, in international usage the term “F.O.B.” is used only for non-container
shipment by water.
Review/Assessment Questions
 What difference does it make if a transaction is a transaction in goods?
o This determines whether Article 2 applies, as it only applies to “goods” identified by the contract.
 What law governs transactions in services?
o The common law applies to services.
 How do courts differentiate between transactions in goods versus services?
o The predominate factor test, gravamen test, or applying by analogy.
 What difference does it make if someone is a merchant, for purposes of Article 2 of the UCC?
o Article 2 applies to the sale of goods; however, certain sections, e.g., §2-314 (warranty of merchantability) apply only if the
“seller is a merchant with respect to goods of that kind.” See also §§2-402(2), 2-403(2).
 How do courts distinguish between a sale of goods and a lease? Why?
o Leases of goods are governed by Article 2A. It does not matter whether the transaction is called a sale or a lease. What matters
is whether the only reasonable outcome of the transaction is that the buyer/lessee will retain the goods for the economic life of
the goods; if so, the transaction is a sale of goods.
 The standards for determining whether a transaction is a sale, or a lease are set forth in Section 1-203.
 When does the CISG apply to a transaction?
o CISG presumptively applies in the sale of international goods, unless the parties opt out of it.
 Which of the following cases would most likely be governed by Article 2?
o Gately Construction provided detailed specifications for inventory control software. Van Dyne Apps agreed to design, develop,
and test software to meet those specifications. - As the cases in the chapter indicate, courts are somewhat split on whether a
contract to design, develop, and implement software is a contract for services or for goods.
o Van Dyne Apps agreed to provide inventory control software, update the software periodically, make any adaptations necessary
to run it on Gately Construction’s system, and train Gately Construction’s employees to use the software. - The contract not

18
only requires the design and coding of software, but continued services to update and adapt the software, not to mention
training employees. This would be predominantly services, not subject to Article 2.
o Answer: Van Dyne Apps sold Gately Construction a standard software package, identical to the package Van Dyne Apps sold to
many small businesses. - Courts routinely hold that the sale of standard software is a transaction in goods, subject to Article 2.
o All of the above.
 Incandenza liked a Marconi sedan offered at Pemulis Auto for $14,000 but wished to keep options open. Incandenza and Pemulis
Auto signed a “Sales Contract,” under which Incandenza would pay $400 per month to Pemulis Auto to use the car. Incandenza
could at any time return the car and be obliged to make no more payments. Incandenza may also keep the car forever, as long as
he makes the monthly payment. Does Article 2 apply to the transaction?
o Yes. The contract clearly states that it is a sales contract. - The fact that the parties call it a sale has little weight.
o Answer: No. The transaction is governed by UCC Article 2A, Leases. - The parties called it a sale, but this is a lease: Incandenza
may keep the car only as long as he pays for it, may terminate at any time, and never becomes the owner.
o Article 2 will apply until Incandenza returns the car.
o Yes. The subject of the contract is a car, which falls into the category of goods, not services. - Both leases and sales may apply to
goods.
 Avril agreed to design and build a custom wheelchair for Mario. The contract specified capabilities and dimensions of the chair
and provided for Avril to determine the design and build the chair. Upon completion and delivery, Avril would be entitled to a
lump-sum payment based on her time and other specified costs. Does Article 2 apply to the transaction?
o Answer: Yes, because Article 2 applies to specially made goods. - The contract calls for some services, but all geared toward
providing the wheelchair, a specially made item of goods. All goods require services to be made, so any transaction in goods
necessarily will require performance of services. This contract ultimately requires Avril to deliver goods, not to perform
associated services such as training or maintenance.
o No, because the predominant purpose of the contract is services in designing and building the chair.
o No, because Avril is not a merchant.
o Yes, because Avril is a merchant.
 Steeply runs an art gallery in Baldwinsville, New York. She signs a contract to purchase a security system from Marathe
Prevention of Montreal, Quebec. The contract provides that it is governed by Article 2 of the UCC. Is the contract governed by
Article 2 of the UCC or by the Convention on Contracts for the International Sale of Goods (CISG)?
o CISG, because the parties live in two countries that are party to the CISG.
o Answer: UCC Article 2, because the CISG allows the parties to choose the law to govern their transaction.
 The CISG would apply to the contract, because the parties are residents of different CISG states. But Article 6 of the CISG
allows parties to choose governing law. They chose Article 2, so Article 2 will provide the rules that govern their duties and
rights under the contract.
o Conduct in Quebec is governed by the CISG and conduct in the United States by UCC Article 2.
o The parties must comply with all provisions of both the UCC and the CISG. Citizens do not choose the law.
 Van Dyne Apps sold Gately Construction some software that would run its inventory control systems. What difference would it
make if the transaction is governed by Article 2 of the UCC, rather than the common law of contracts?
o Article 2 provides an implied warranty of merchantability, even if Van Dyne Apps did not make any specific promises about the
software. - In the cases on the scope of Article 2, one party may be arguing that Article 2 applies so that the party gets an
implied warranty of merchantability.
o Article 2’s rules on contract formation are more flexible, finding an enforceable contract even where there is no matching offer
and acceptance. - One party may be arguing that there will be an enforceable contract where the parties’ forms or conduct show
an agreement, even though there is no offer and acceptance.
o Article 2’s perfect tender rule may require the seller to deliver exactly the software agreed upon to be entitled to the price.
Common law contract may only require substantial performance.
o Article 2 will readily fill in gaps that the parties did not address, such as price or time for performance. - One party may be
arguing that to use some other gap-filler in Article 2, such as finding a price where the parties did not specifically agree on price
o Answer: All of the above.

CHAPTER 3: CONTRACT FORMATION


I. The Statute of Frauds (MY LEGS)
Analytical Framework:
(1) Does it fall within the statute?
(2) Does an exception apply?
(3) If an exception doesn’t apply, is the statute satisfied?
 The Difference Between the Statute of Frauds and the Parol Evidence Rule

19
o The Statute of Frauds determines whether or not a contract is enforceable. Certain contracts must be evidenced by a writing
or they are not enforceable. The purpose of the Statute of Frauds is to guard against someone falsely claiming that a contract
was entered into.
o The Parol Evidence Rule applies only if the parties have exchanged confirmatory memoranda agreeing on certain terms or if
there is a writing that is intended by the parties as a final expression of the terms contained therein. The Parol Evidence Rule
prohibits other evidence outside the writing from being introduced to prove that the contract consists of terms different from or
additional to what is in the writing.
 Requirements of the Statute of Frauds
1. The “writing” need not be the contract itself; it must simply a document that “indicates that a contract for sale has been
made”;
2. The writing need not state all of the terms of the contract nor must it correctly state all of the terms, but it must be “sufficient”
to indicate that the contract was entered into;
3. The writing must state the “quantity” of the goods to be sold and the contract is not enforceable beyond the quantity stated;
4. The writing must be “signed” by the party against whom enforcement is sought or by their authorized agent.
 Section 2-201(1): Requirement of a Signed Writing for Contracts Where the Price is $500 or More
Except as otherwise provided in this section a contract for the sale of goods for the price of $500 or more is not enforceable by way
of action or defense unless there is some writing sufficient to indicate that a contract for sale has been made between the parties
and signed by the party against whom enforcement is sought or by his authorized agent or broker. A writing is not insufficient
because it omits or incorrectly states a term agreed upon, but the contract is not enforceable under this paragraph beyond the
quantity of goods shown in such writing.
 Section 1-201(43): Definition of “Writing”
"Writing" includes printing, typewriting, or any other intentional reduction to tangible form. "Written" has a corresponding
meaning.
o Includes any tangible form of expression. It may be a typed or handwritten document, an email message, or clay tablets.
 Section 1-201(b)(37): Definition of “Signed”
"Signed" includes using any symbol executed or adopted with present intention to adopt or accept a writing.
o For a writing to be “signed” the U.C.C. does not require a handwritten signature. It could be a letterhead, a wax seal, or any
symbol that the person uses with the intent to “adopt or accept a writing.”
 The Purpose of the Statute of Frauds
o The purpose of the statute of frauds is to prevent fraud by a person falsely claiming that the parties had entered into an oral
agreement.
o But remember that a person may also falsely deny the existence of an oral agreement! Oral agreements are enforceable if the
Statute of Frauds either does not apply, or if an exception to the Statute of Frauds applies.
EXCEPTIONS

 Sale for LESS THAN $500


 Section 2-201(2): Exception: CONFIRMATION AMONG MERCHANTS (Read your mail rule)
Instead of a writing signed by the party against whom enforcement is sought, as between merchants a contract for the sale of
goods for a price of $500 or more is enforceable if the party seeking to enforce the contract sent a written and signed
“confirmation” of the oral agreement and the other party did not object within ten days.
Between merchants if within a reasonable time a writing in confirmation of the contract and sufficient against the sender is
received and the party receiving it has reason to know its contents, it satisfies the requirements of subsection (1) against such party
unless written notice of objection to its contents is given within 10 days after it is received.
 Section 2-201(3)(a): Exception: SPECIALLY MANUFACTURED GOODS
Despite the lack of a signed writing evidencing the contract the contract is enforceable:
if the goods are to be specially manufactured for the buyer and are not suitable for sale to others in the ordinary course of the
seller's business and the seller, before notice of repudiation is received and under circumstances which reasonably indicate that the
goods are for the buyer, has made either a substantial beginning of their manufacture or commitments for their procurement ….
 Section 2-201(3)(b): Exception: JUDICIAL ADMISSION
Despite the lack of a signed writing evidencing the contract the contract is enforceable:
if the party against whom enforcement is sought admits in his pleading, testimony or otherwise in court that a contract for sale
was made, but the contract is not enforceable under this provision beyond the quantity of goods admitted ….
 Section 2-201(3)(c): Exception: PERFORMANCE OF THE CONTRACT
Despite the lack of a signed writing evidencing the contract the contract is enforceable:
with respect to goods for which payment has been made and accepted or which have been received and accepted (2-606).
 Example 1: The $499 Sale
o Madeline offers to sell a set of wicker furniture to Joshua for $499, and Joshua accepts. Joshua then changes his mind and tells
Madeline that he no longer wishes to purchase the furniture. Can Madeline enforce the contract against Joshua?

20
 ANSWER: Yes! It is a sale of goods for less than $500, so Section 2-201 does not apply.
 Example 2: The Letter
o Assume that Madeline and Joshua entered into an oral agreement for the sale of the wicker furniture for $600. Madeline then
writes a letter to Joshua stating, “I have found a better price for the wicker furniture. I am sorry to break my word to you.” She
does not sign the letter, but it does have a letterhead that has her name on it. Can Joshua enforce the oral agreement against
Madeline?
 ANSWER: Yes! There is now a writing evidencing the agreement signed by the person against whom enforcement is sought.
Joshua can enforce the agreement against Madeline.
 Example 3: The Oral Admission
o Assume that Madeline and Joshua entered into an oral agreement for the sale of the wicker furniture for $600. Madeline then
backed out of the agreement. In front of several other witnesses on several occasions Madeline admitted that she had agreed to
sell Joshua the furniture at that price. Can Joshua enforce the oral agreement against Madeline?
 ANSWER: No! The admission was not in a signed writing nor was it made as part of a judicial proceeding.
 Example 4: The Confirmation
o Assume that Madeline and Joshua entered into an oral agreement for the sale of the wicker furniture for $600. Assume further
that Madeline is a merchant but that Joshua is not. Finally, assume that two days after they entered into the oral agreement
Joshua sent Madeline an email confirming the sale, and that Madeline did not respond to Joshua’s email message. Can Joshua
enforce the agreement against Madeline?
 ANSWER: No. The “confirmation” exception to the Statute of Frauds applies “as between merchants,” and Joshua is not a
merchant.

Becton, Dickinson and Co. v. Biomedomics, Inc. (Specially manufactured goods exception)
o Manufacturer alleged that a contract for the sale of Covid-19 anti-body tests fits into the specially manufactured goods
exception to UCC Article II statute of frauds.
o The court rejected that argument because there was no showing that the tests were not suitable for sale to others in the
ordinary courtse of business
o Rather, the rests had already been developed before the buyer made its requests and the tests were not components or
parts of other products sold only by the buyer.
Elling v. Molina
o Are oral promises enforceable? – No.
 In this case a married couple discussed buying a ring from a jeweler, where the jeweler made a guarantee that they could
return it within one year and he would buy it back for 10% of the purchase price. This was never put in writing. The ring was
purchased for $600,000 and the couple decided to return the ring within one year. The jeweler sent a letter, which he
signed to the couple, that confirmed the purchase and guarantee but added the requirement that he needed to resell the
ring before he could pay the couple. The couple brought suit because of the added requirement.
 The court determined that there was a sufficient indication of the contract to satisfy §2-201, primarily the July 11, 2012
“Dear Brother” letter signed by the defendant proved most damning when combined with the invoices and wire transfers to
establish the price and other details. (price >$500, letter was signed)
 All of this was enough to meet the minimum requirements of the Code, and the plaintiffs prevail.
 Casebook Problem P.65 # 8 (Oral agreement, look for SOF) ( 1) Does SOF apply? 2) is there an exception? 3) is the rule satisfied?)
On December 10, James Ross, president of Ross Ice Cream Shoppes, Inc., phoned Robert Scott, president of Amundsen Ice
Company, and negotiated the purchase of two tons of ice from Amundsen at $256/ton. As they talked on the phone, Scott
picked up a memo pad inscribed “Amundsen Ice Company from the Desk of the President,” wrote on it “2 tons Ross Co.,” and
then scribbled his initials on it. When the parties hung up the phones, Scott placed the memo on a spindle marked “Orders.”
Ross wrote Scott a letter beginning “Dear Bob: This is to confirm our ice purchase deal . . .,” which described their transaction
completely. Scott received the letter on December 14. On January 17, Scott phoned Ross and denied the existence of the
contract detailed in the Ross letter. Answer these questions:

(a) Does the memo pad note satisfy §2-201(1)? See §1-201(b)(37) and Official Comment 37.
 The memo pad should satisfy §2-201. It contains a quantity, indicates that a sale had been made to Ross Company, and
contains a sufficient signature in the initials. If the initials had been left off, query whether the letterhead had been
“adopted” by the handwriting on it, so that it might itself serve as a sufficient signature  Probably yes.

(b) What legal effect did the December 14 letter have? Same result if Ross’s letter failed to mention the quantity? Even if the
letter satisfies the Statute of Frauds, is it conclusive as to the existence and terms of the contract? See Hinson-Barr, Inc. v.
Pinckard
 This question is meant to raise the “merchants must read their mail” rule of §2-201(2). If quantity is not mentioned, the
letter is not “sufficient against the sender” so subsection (2) does not apply, and the Statute of Frauds would be a problem.
Even if the Statute is satisfied, a binding obligation does not necessarily result; the parties may still litigate the issue of the

21
correctness of the letter, with the sender having the burden of proving that the letter accurately reflected the terms of the
oral deal, which is what the cited South Carolina case holds. As White & Summers, put it at §3-4, “the plaintiff must still
persuade the trier of fact that the parties did make an oral contract and that its terms were thus and so.” All that §2-201(2)
accomplishes is to take the Statute of Frauds out of the case, but then the plaintiff must still establish the terms of the
contract by sufficient evidence.
(c) Did Scott’s denial of the terms contained in Ross’s letter avoid the operation of §2-201(2)? Suppose Scott had immediately
written Ross a letter stating, “You haven’t stated the terms correctly. We only agreed to sell you 1 ton.” Would that letter be
sufficient notice of objection? See Simmons Oil Corp. v. Bulk Sales Corp.
 No, because the objections were too late and not in writing. The Simmons case raises the possibility that the suggested
reply even if timely may not object enough. By objecting to only one term, the letter is impliedly agreeing that the sender’s
letter is correct to other details. In effect, this admits the existence of a contract instead of denying it, with only the
disputed term still at issue.
 Would an unsigned receipt for artworks satisfy §2-201? Beard v. Chase. In the cited case the court found that the creation
of a painting is not primarily a contract for the sale of services, and that a receipt alone was not sufficient to satisfy the
statute of frauds where there was not signed and was unclear as to which works of art were being sold. The lesson is to get
a signed contract with enough details that the court can believe an agreement was made, and don’t rely on ambiguous
receipts. A lot of money was lost because of the sloppiness of the dealings here.
St. Ansgar Mills, Inc. v. Streit
o What is a “reasonable time”? – usually a jury determination.
 St. Ansgar bought corn from grain farmers and sold it to livestock farmers. St. Ansgar took a phone order to sell to a farmer,
Streit, and held the confirmation slip for 40 days for the farmer to stop by the office. As the price fell, the farmer did not
stop by. Finally, St. Ansgar delivered a confirmation 40 days after the phone call. Streit did not make a timely objection, but
later refused delivery, having bought at a better price on the open market.
 The court found that, only in rare cases evidence so one-sided that the court can determine the factual question of whether written
confirmation of an oral agreement was received within a reasonable time. To determine whether the confirmation was received
within a reasonable time, the UCC provides that all relevant circumstances, including the course of conduct (performance and
dealings) of the parties, must be considered.
• In this case factors of volatile market conditions, large sale price, custom and practice of the parties in the delivery of
confirmations, long-time amicable business relationship, and defendant hog farrowing operation's practice of regularly
coming to Ansgar grain dealer’s business revealed a genuine dispute for the jury over the reasonableness of delivering
the written confirmation of the sale as an exception to the statute of frauds.
Casebook Problem # 9 p. 74
The city manager of Thebes, Utah, which is world-famous for its beautiful desert golf course, orally (implicates SOF) ordered a huge
water tank to be made in the shape of a golf ball on a tee from Tanks of America, Inc. The price was agreed to be $30,000, and the city
sent Tanks a down payment check for $3,000, signed by the city comptroller and marked “Tank” on the memo line. Tanks of America built
the tank and was in the process of painting “City of Thebes” on the side when a representative of a newly elected city administration
called and said that the new administration considered the contract unenforceable. Answer these questions:

(a) Does the check satisfy §2-201(1)? Where is the quantity? See Beautiful Jewellers Private, Ltd. v. Tiffany & Co.,
 The cited case held that a partial payment on an indivisible item took the entire contract out of the Statute of Frauds. A
quantity of “one” can be inferred.

(b) What legal argument can Tanks of America make based on §§2-201(3)(a) and 2-201(3)(c)? Does the City of Thebes have a good
response to the §2-201(3)(c) argument? See W.I. Snyder Corp. v. Caracciolo.
 Tanks of America will argue that this tank is just the sort of “white elephant” item that excuses the necessity of a writing
under §2-201(3)(a), since it is obviously meant for this particular buyer.
 Alternatively, Tanks of America can argue that the down payment is enough payment to trigger §2-201(3)(c) and excuse the
need for a writing.
 To this the City of Thebes might reply that it is only a partial payment, but, as the cited Pennsylvania case holds and as
mentioned in (a) above, the courts have held that the partial payment on a non-divisible item (like the tank) indicates a
sufficient contract for the sale of the whole item.

(c) If the city had promised to sign a written contract but had never gotten around to doing so, can promissory estoppel or
equitable estoppel be used to circumvent §2-201? Reread §1-103(b) and see Allen M. Campbell Co. v. Virginia Metal Indus.,
Inc., Amber Chemical, Inc. v. Reilly Industries, Inc.
 Some courts have been willing to use promissory estoppel in this situation and other not; both of the cited cases allowed
promissory estoppel to bypass §2-201, but not all courts would agree; see White & Summers §3-6. Estoppel enters, of
course, through §1-103(b).

22
(d) Would it make a difference if the city had a Vender Buying Agreement posted on its website, which disclaimed the
effectiveness of any oral agreements with the city? See Grandoe Corp. v. Gander Mt. Co.
 In the cited case the buyer’s attorney decided that oral contracts for the purchase of gloves were too messy and created a
clause for buyer’s website stating that oral agreements were not binding until reduced to writing. Buyer then sent a notice
to all sellers with whom it had worked in the past informing them they should read this notice on the website.
 One seller did not do so, nor did it reply in any way, but it did accept big oral orders for the special manufacture of gloves
and then created said gloves. When buyer then decided it only wanted a portion of the order, it took comfort in the
language of its website. But the court held that seller had never agreed to the proposal described on that website, so it was
not part of the resulting oral contract. In that case the special manufacturing of the gloves would satisfy the statute of
frauds, and the seller prevailed.
Casebook Problem #10
Tomorrow, Inc. (a computer software company), and Systems Unlimited (a company specializing in advising other companies how to
maximize their computer operations) entered into a written joint venture contract by which Tomorrow, Inc., promised to design and sell
to Systems Unlimited software that would enable the latter’s customers to receive engineering drawings by phone. The parties agreed
that their arrangement was “non-exclusive” (meaning either was allowed to deal with other buyers and sellers of the same product). The
contract described the obligations of the parties in some detail and stated that the contract would terminate after two years unless
renewed. In fact, after working with Tomorrow, Inc., for only six months, Systems Unlimited decided it could develop its own software
more cheaply than buying it from Tomorrow, Inc., so it faxed a letter to the latter stating that their contract was at an end. Systems
Unlimited declined to purchase any further software. Tomorrow, Inc., which had incurred substantial startup costs in developing the
software for this contract, was astounded and promptly filed suit. Systems Unlimited sought refuge in the Statute of Frauds, arguing that
the contract signed by the parties stated no quantity.

Does §2-201(1) always require a specific quantity? Look at the statutory language very carefully. Compare §2-306. See also Advent Sys. Ltd.
v. Unisys Corp.
 This is not a requirement contract like those addressed in §2-306 because the parties have no agreement of exclusive
dealing (“all my requirements” or “all our output”).
 The Problem is based on the cited case, which traced the White & Summers argument that a careful reading of §2-201(1)
does not require that the contract state the quantity. Instead, the last sentence thereof simply says that a contract is not
enforceable beyond any quantity that is stated (not that one is required to be stated).
 Indeed, the described transaction, similar to a requirements contract, cannot state a quantity when the parties do not yet
know what it will be. Surely, however, that is no reason for allowing the technicalities of the Statute of Frauds to invalidate
a perfectly reasonable commercial arrangement, and the court refused to require a stated quantity in the writing.
Casebook Problem #11
Sued by a trusting seller after refusing delivery, Renege Retail’s president calmly testifies that he had agreed to buy the goods, but the
parties never put anything in writing.

Without a signed contract, memorandum, confirmation or any other writing, is Renege Retail off the hook? See §2-201(3)(b).
 Under §2-201(3)(b), a contract may be enforced without a signed writing if the other party admits in litigation that there
was a contract.
 So, it should only be necessary to put Retail’s president under oath, at a deposition or at trial. See Comment 7.
 Article 2 does not allow a party to simply rely on the Statute of Frauds as a pure technicality if the party admits there was a
contract.
 J.B.B. Investment Partners, Ltd. v. Fair
o UETA Case
o This is one of the few cases exploring electronic signatures and triggering the two major statutes, federal and state, on point.
Since California had adopted the Uniform Electronic Transactions Act [UETA], federal law [E-Sign: The Electric Signatures in
Global and National Commerce Act] defers to it.
o Here the electronic signature was in an email, and you’d think that would be good enough, but the court says that UETA
requires more formality: the person signing must have done so “with an intent to sign the electronic record.” ‘
o It was not clear that the email was the contract, and without that UETA did not apply.
o This decision bothered a lot of attorneys. The thrust of UETA was to make electronic agreements easier, and this case
complicates them by requiring some formal statement that by sending the email and signing it the parties intend to sign an
electronic record.
o One suspects that the drafters of the statute would have disapproved of this result, but in California at least such boilerplate
language may be necessary before UETA saves the transaction from legal attack for lack of a valid signature.
Canvas Quiz Questions
o Under the Statute of Frauds of Article 2 of the Uniform Commercial Code, contracts for the sale of goods where the price is
more than $500 must be evidenced by a writing signed by the party against whom enforcement is sought.
 True

23
o The differences between the Statute of Frauds and the Parol Evidence Rule are that:
 The Statute of Frauds governs whether or not a contract is enforceable; the Parol Evidence Rule governs whether certain
terms are or are not a part of the contract.
 The Statute of Frauds requires that there be a writing evidencing the contract. The Parol Evidence Rule applies only if there
are confirmatory memoranda or a writing intended by the parties as a final expression of their agreement with respect to
the terms contained therein.
 The Statute of Frauds is governed by Section 2-201. The Parol Evidence Rule is governed by Section 2-202.
 The Statute of Frauds has no exceptions, while the Parol Evidence Rule is easily circumvented.
o To satisfy the Statute of Frauds contained in Section 2-201, the writing evidencing the contract
 must constitute the entire contract between the parties.
 must be signed by both parties.
 must be sufficient to indicate that a contract was entered into.
 must state a term as to the quantity of the goods sold.
o Even if there is no writing evidencing the contract, a contract for the sale of goods for more than $500 is enforceable if:
 the goods are to be specially manufactured.
 as between merchants, the party seeking to enforce the contract sent a written confirmation satisfying the Statute of
Frauds to the other party, and the other party did not give a written notice of objection within 10 days of receipt.
 if the party against whom enforcement is sought admits in a telephone call that the contract was entered into.
 insofar as goods have been received and accepted with or with respect to goods for which payment has been made and
accepted.
o Angus McDuffer, a trader in sports memorabilia, entered into an oral agreement with Ralph Cramdon, another trader, to
purchase the golf ball famously used by Bobby Jones at the 1925 U.S. Open. The price they agreed upon was $250,000. Two
days later called Angus and told him although he admitted that he had entered into the oral contract that he had received a
better offer and that "the price has moved" to $400,000. Angus recorded the conversation; please assume that no privacy law or
rule of evidence would bar the introduction of this recording into evidence. Nevertheless, is this oral contract enforceable
under the Statute of Frauds?
 Yes, because if the recording is played in court that will constitute a judicial admission that the contract was entered into
within the meaning of Section 2-201(3)(b). - The "admission" was not made as part of a judicial proceeding.
 Yes, because this is a unique item -- one of a kind! Accordingly, Angus is entitled to specific performance of the contract
within the meaning of Section 2-716(1). - This item, although unique, was not "specially manufactured.
 Answer: No. - This is a contract for the sale of goods with a price of more than $500, and none of the exceptions to the
Statute of Frauds applies. The contract is not enforceable.
 Yes, because the phone call constitutes "confirmation between merchants" of the oral agreement within the meaning of
Section 2-201(2). - The "confirmation" was not in writing.
II. Parol Evidence Rule
 What Is “Parol Evidence”?
o Literally, the word “parol” means “by word of mouth” or “oral.” In the law, however, the term “parol evidence” has come to
mean the same thing as “extrinsic evidence” – that is, any evidence, whether oral or written, that is external to the words of
the contract. s
o The “Parol Evidence Rule,” which in certain circumstances bars evidence of both oral and written agreements that are extrinsic
to a written contract, is itself a term of art that doesn’t mean what it literally says.
 The Scope and Purpose of the Parol Evidence Rule
o The Parol Evidence Rule bars evidence of oral and written agreements that the parties may have entered into that contradict
or that are in addition to a written contract.
o The purpose of the Parol Evidence Rule is to permit the parties to a contract to invalidate or override other agreements that
they may have reached during the negotiations leading up to the final, written contract. It gives both parties the certainty and
security that the written contract they have executed is either the final expression of the terms that are in the writing or the
complete and exclusive statement of all the terms of the contract.
 Ambiguity and the Parol Evidence Rule
o The Parol Evidence Rule does not change what you learned in the law of Contracts about the admissibility of extrinsic evidence
to interpret the meaning of words, phrases, or terms of a contract that are ambiguous. If the language of a contract is
ambiguous, the courts of necessity must consider extrinsic evidence in order to interpret and enforce the contract.
o The parol evidence rule is concerned not with ambiguity, but rather with whether the parties entered into any agreements
other than those contained in a written contract.
 Course of Performance, Course of Dealing, Trade Usage, and the Parol Evidence Rule
o Nor does the Parol Evidence Rule change what you learned in Contracts about the admissibility of the parties’ course of
performance, course of dealing, and trade usage. The Parol Evidence Rule does not bar these types of evidence; course of
performance, course of dealing, and trade usage are always admissible to prove the terms of a contract.
 The Difference Between the Parol Evidence Rule and the Statute of Frauds

24
o The Parol Evidence Rule is often confused with the Statute of Frauds.
 The Statute of Frauds is concerned with whether a contract is enforceable.
 The Parol Evidence Rule is concerned with what the terms of a contract are.
 The Statute of Frauds makes certain contracts unenforceable unless there is written evidence that a contract was entered
into.
 The Parol Evidence Rule bars evidence of certain different or additional terms other than what was contained in the
writings.
 The Relation Between the Parol Evidence Rule and the Statute of Frauds
o A signed writing that satisfies the Statute of Frauds because it “evidences” an agreement does not have to be a “final
expression” of the terms of the agreement. In fact, under Article 2 of the U.C.C. dealing with contracts for the sale of goods, a
writing that satisfies the statute of frauds may omit essential terms of the agreement, and in certain circumstances may even
be contradicted by extrinsic evidence regarding the terms of the agreement.
 The Parol Evidence Rule Under the Common Law and Under U.C.C. Section 2-202
o The Parol Evidence Rule under the common law is the same as under Section 2-202 of the Uniform Commercial Code, which
applies to contracts for the sale of goods. The only difference between the common law and Section 2-202 is in the
terminology.
 “Final Expression” and “Complete and Exclusive Statement”
o U.C.C. Section 2-202 recognizes two types of written contracts that the Parol Evidence Rule applies to: “final expressions” and
“complete and exclusive statements.”
 Final Expression of the Terms Contained in the Writing
o Under the Parol Evidence Rule, if a written contract is intended to be the final expression of the terms that are contained in the
writing, then evidence of prior contradictory agreements is barred, but evidence of consistent additional terms is admissible.
 Complete and Exclusive Statement of the Terms of the Agreement
o If a written contract is intended by the parties to be the complete and exclusive statement of all of the terms of the
agreement, then evidence of both contradictory agreements and consistent additional terms are barred.
 Section 2-202: Complete and Exclusive Statement of the Terms of the Agreement
Terms with respect to which the confirmatory memoranda of the parties agree or which are otherwise set forth in a writing intended
by the parties as a final expression of their agreement with respect to such terms as are included therein may not be contradicted by
evidence of any prior agreement or of a contemporaneous oral agreement but may be explained or supplemented
(a) by course of performance, course of dealing, or usage of trade (Section 1-303); and
(b) by evidence of consistent additional terms unless the court finds the writing to have been intended also as a complete and
exclusive statement of the terms of the agreement.
 “Partial Integration” and “Complete Integration”
o It was noted above that although the Parol Evidence Rule is the same under the common law and the Uniform Commercial
Code, the terminology differs.
o Under the common law, a “final expression” of the terms contained in the writing is referred to as a “partially integrated
agreement” and a “complete and exclusive statement” of the terms of an agreement is called a “fully integrated agreement.”
 Merger Clauses: Final Expression of the Terms or Complete and Exclusive Statement of the Terms
o The parties may, if they wish, include contractual language that expressly brings the Parol Evidence Rule into play. A contract
may expressly provide that it constitutes the “final expression” of the terms set forth therein, or the contract may go further
and state that it constitutes the “complete and exclusive statement” of the agreement between the parties.
o This type of language is called a “merger clause” or “integration clause.”
 Note: the parol evidence rule does not bar evidence of later modifications of the agreement, whether written or oral!
 Comment 3 to U.C.C. § 2-202: Consistent Additional Term
Under paragraph (b) consistent additional terms, not reduced to writing, may be proved unless the court finds that the writing was
intended by both parties as a complete and exclusive statement of all the terms. If the additional terms are such that, if agreed
upon, they would certainly have been included in the document in the view of the court, then evidence of their alleged making
must be kept from the trier of fact.
 Is Extrinsic Evidence Admissible to Prove Whether a Contract Is Integrated?
o Is parol evidence admissible as to the applicability of the parol evidence rule? In other words, is extrinsic evidence admissible
on the question of whether a writing is either a “final expression” of the terms that the writing refers to or a “complete and
exclusive statement” of all of the terms of the agreement?
o On that question, the courts are split, just as they are split on the question of whether extrinsic evidence is admissible to prove
whether the language of a contract is “ambiguous.”
 Sample Merger (Integration) Clause
o The Parties hereto intend that this Agreement is the full, final, complete and exclusive agreement of the Parties. No
modification of the terms of this Agreement can be effective unless in a writing signed by the Party against whom enforcement
is sought.
The Parol Evidence Rule Does NOT Bar Extrinsic Evidence of the Following …
25
1. Matters relating to whether a valid contract was formed (capacity, duress, illegality, forgery, etc.);
2. Whether there was a condition precedent to the contract going into effect;
3. To interpret a term of the contract that is ambiguous;
4. To prove course of performance, course of dealing, or trade usage;
5. To prove the existence of a separate contract supported by separate consideration.
6. To prove that the contract was later modified or that a right was waived (modifications to contracts are governed by U.C.C.
Section 2-209 or Restatement (Second) Section 89);
 Example 1: The Gym Equipment and the Consistent Additional Term
o Excelsior Gym Equipment and Ted exchange confirmatory memoranda stating that Excelsior has agreed to sell Ted a home gym
for the amount of $3,100, and that the gym will be delivered on August 1. The memoranda do not state whether Excelsior will
assemble the gym. Ted contends that Excelsior orally agreed to assemble the gym. Would Ted be permitted to testify as to that
alleged promise?
 ANSWER: Yes, the evidence is admissible. The confirmatory memoranda appear to be the “final expression” of the terms
contained in the writings, but not an “exclusive statement” of the terms of the contract. Accordingly, evidence of
“consistent additional terms” would be admissible.
 Example 2: The Gym Equipment and the Different Term
o Excelsior Gym Equipment and Ted exchange confirmatory memoranda stating that Excelsior has agreed to sell Ted a home gym
for the amount of $3,100, and that the gym will be delivered on August 1. Ted contends that during negotiations Excelsior
orally agreed that the price would only be $2,500. Would Ted be permitted to testify as to that alleged promise?
 ANSWER: No, the evidence is not admissible. If the confirmatory memoranda constitute a “final expression” of the terms
agreed to in the writings this evidence would be inadmissible. This is not a “consistent additional term” but rather a
different term than what the writings indicated the parties had agreed to and is therefore barred by the parol evidence
rule.
 Example 3: The Gym Equipment and the Oral Modification
o Excelsior Gym Equipment and Ted exchange confirmatory memoranda stating that Excelsior has agreed to sell Ted a home gym
for the amount of $3,100, and that the gym will be delivered on August 1. Ted contends that after the memoranda were
exchanged Excelsior orally agreed to reduce the price to $2,500. Would Ted be permitted to testify as to that alleged
modification?
 ANSWER: Yes, Ted may testify about this later agreement because the parol evidence rule does not bar evidence of
subsequent modifications of the contract. It is possible for the contract to provide that oral modifications of the contract
are invalid, but that is governed by a separate rule in U.C.C. Section 2-209. Section 2-202, the Parol Evidence Rule, does
not apply to modifications of a written contract.
 Example 4: The Gym Equipment and Complete Integration
o Excelsior and Ted both signed a written contract providing that Excelsior would sell, and Ted would buy the home gym for
$3,100 and that Excelsior would deliver the gym to Ted’s home. The written contract is silent regarding who will assemble the
gym. The contract contains a merger clause stating that the written contract constitutes the complete and exclusive statement
of the terms of the agreement. Ted contends that during negotiations Excelsior orally agreed to assemble the gym in Ted’s
home as part of the transaction. Will Ted be permitted to testify as to that alleged promise?
 ANSWER: No, Ted would not be allowed to testify that Excelsior had earlier agreed to assemble the gym. This time the
writing is not simply a “final expression” of the terms contained in the writing but is also the “complete and exclusive
statement” of all the terms of the agreement. Ted’s testimony regarding evidence of this consistent additional term is
inadmissible.
 Casebook Problem
o Lawyers for Swinging Singles Magazine negotiated for an entire year with Space Age Aircraft Co. to obtain a contract for the
construction of a special Swinging Singles airplane. (The plane was to be black and silver, with the Swinging Singles emblem
painted on the tail; it was to contain a living room, bed chamber, swimming pool, hot tub, and a dance floor.) The resulting 30-
page contract also contained a merger clause, stating that all prior negotiations were merged into the written contract that
contained all the terms of the agreement. The contract was signed by both parties. Does §2-202 bar the introduction of
evidence of the following?
(a) An alleged precontract agreement that Space Age would provide free flying lessons to Hi Handsome, president of Swinging
Singles, Inc.? The contract says nothing about this. See the test found in Official Comment 3 to §2-202.
 It seems that the court could find that the alleged agreement was a “consistent additional term”.
(b) An alleged precontract agreement that Swinging Singles could use the plane for two months, and if they did not like it, they
could return it for a full refund?
 This matter is the sort of thing that, if true, “would certainly have been included” in the written contract (the test from
Official Comment 3) and should be barred by the parol evidence rule.
 Columbia Nitrogen Corp. v. Royster Co.
o Columbus Nitrogen allowed evidence about course of dealing and usage of trade to show that price terms in a three-year
contract were subject to adjustment.

26
o The case shows that Article 2 may allow a lot of extrinsic evidence in.
o As long as the terms shown by usage of trade and course of dealing were not directly contradictory to terms in the agreement,
they were admissible.
o The merger clause did not keep terms out because such a clause limits only “consistent additional terms,” not course of
dealing, course of performance, or usage of trade.
o So even where the contract stated price and quantity in plain terms, it could be explained or supplemented by showing a
course of dealing between the parties or usage of trade to vary from the terms in the contract.
o This case demonstrates the most liberal treatment of the parol evidence rule.
o A Better Merger Clause?
 I believe that a disclaimer of usage of trade, etc., is something of a useless gesture unless the specific practice being
disclaimed is mentioned with particularity.
 For instance, part of the usage of trade is that the parties are standing on the planet Earth. Try and disclaim that.
 P.s. [The cited case did not allow in usage of trade to show that a stated quantity was merely an estimate in spite of the
lack of a merger clause.]
 Canvas Quiz Questions
o Under the Parol Evidence Rule of Section 2-202 what is the difference between a written contract that is a "final expression"
of its terms and one that is a "complete and exclusive statement of the terms of the agreement"?
 Answer: If a written contract is the "final expression" of its terms, then the Parol Evidence Rule only bars evidence of
terms that are different from those terms. If a written contract is the "complete and exclusive statement" of the terms of
the contract, then evidence of consistent additional terms is also barred.
 There is no difference between a written contract that is a "final expression" of its terms and one that is a "complete and
exclusive statement" of the terms of the agreement.
 If a written contract is the "complete and exclusive statement" of the terms of an agreement, then the Parol Evidence Rule
only bars evidence of terms that are different from those terms. If a written contract is the "final expression" of its terms,
then evidence of consistent additional terms is also barred.
 If a written contract is the "final expression" of its terms, then the Parol Evidence Rule permits evidence that the contract
was later modified. If a written contract is the "complete and exclusive statement" of the terms of the contract, then
evidence that the contract was later modified is also barred.
o Rebecca, who resides at Sunnybrook Farm, Pennsylvania, entered into a contact to sell her handwritten collection of stories
and poems to the Wiggen Foundation. The written contract, signed by both parties, contains an integration clause stating: "This
contract is the complete and exclusive statement of the terms of this agreement. No evidence of different or additional terms is
admissible to alter or supplement the terms of this agreement." However, it turns out that Rebecca was only 15 years old when
she signed the agreement. Is evidence of her age admissible to prove that she lacked the capacity to enter into a binding
contract?
 Answer: Yes, evidence of her age is admissible despite the integration clause.
 No, evidence of her age is barred by the Parol Evidence Rule.
 Yes, evidence of her age is admissible because integration clauses are unconscionable.
 No, evidence of her age is inadmissible because she signed the contract.
o Madeline, who owns a construction company, entered into a contract with Paris Equipment Co. of Erie, PA to purchase 40
double plumb bobs used for making two straight lines. The contract stated that the plumb bobs would be delivered on
September 1, and further stated that the contract was "the complete and exclusive statement of the terms of the agreement."
Madeline, however, now seeks to cancel the agreement because the project she intended to use the plumb bobs for -- a replica
of the Louvre that would have been located in Sharon, Pennsylvania -- was cancelled by the county authorities. Madeline
contends that Paris Equipment at all times understood that the order for the plumb bobs was contingent upon the approval of
the museum project. True or False? -- The Parol Evidence Rule and the contract's integration clause make Madeline's
testimony regarding the condition precedent inadmissible.
 False - The Parol Evidence Rule and an integration clause may bar extrinsic evidence regarding the terms of a contract, but
they do not operate to bar evidence regarding a condition precedent to the contract going into effect.
o Larry Wittgenstein ordered 75 "drones" for his firecracker delivery business from Wicked Witch Drone Company. The contract
between Larry and Wicked contained a full integration clause stating that "This agreement represents the complete and
exclusive statement of the terms of the agreement -- all previous and contemporaneous agreements are merged into this
agreement and are of no force or effect." The problem is that Wicked sells two kinds of drones - eight blade and sixteen blade -
and the contract does not specify which kind of drone Larry was purchasing. True or false? - The parties may offer evidence of
conversations that the parties had during negotiations that will bear upon which type of drone was the subject of the
contract.
 True - Despite the Parol Evidence Rule and the full integration clause the parties may offer evidence of conversations that
the parties had during negotiations that will shed light upon the meaning of an ambiguous term of the contract.
III. Offer and Acceptance
A. General Rules

27
 Section 2-204: Formation in General
(1) A contract for sale of goods may be made in any manner sufficient to show agreement, including conduct by both parties which
recognizes the existence of such a contract.
(2) An agreement sufficient to constitute a contract for sale may be found even though the moment of its making is
undetermined.
(3) Even though one or more terms are left open a contract for sale does not fail for indefiniteness if the parties have intended to
make a contract and there is a reasonably certain basis for giving an appropriate remedy.
 Section 2-206 (1): Offer and Acceptance in Formation of Contract
Unless otherwise unambiguously indicated by the language or circumstances
(a) an offer to make a contract shall be construed as inviting acceptance in any manner and by any medium reasonable in the
circumstances;
(b) an order or other offer to buy goods for prompt or current shipment shall be construed as inviting acceptance either by a
prompt promise to ship or by the prompt or current shipment of conforming or non-conforming goods, but such a shipment
of non-conforming goods does not constitute an acceptance if the seller seasonably notifies the buyer that the shipment is
offered only as an accommodation to the buyer.
 Section 2-206 (2) Offer and Acceptance in Formation of Contract
Where the beginning of a requested performance is a reasonable mode of acceptance an offeror who is not notified of acceptance
within a reasonable time may treat the offer as having lapsed before acceptance.
 Casebook Problem
o Mastervoice TV ordered 20,000 fuses from the Generous Electric Company, the order stating, “Reply by return mail.” Instead of
a formal reply, Generous immediately shipped the fuses. When the fuses arrived, they were found to be defective.
Mastervoice, which had to procure substitute goods elsewhere to meet its production schedule, sued Generous Electric for
breach of warranty. Answer these questions:
(a) At what moment was the contract formed?
 At the moment of shipment; §2-206(1)(b).
 Some of you will undoubtedly argue that §2-206(1)(b) does not apply because Mastervoice “otherwise unambiguously
indicated.”
 Others might disagree and would argue that Mastervoice would have to say “This offer must be accepted in writing” or
“reply by return mail only” to work such a result.
 Usage of trade §1-303(c) also could be utilized to defend an answer either way.
(b) Can Generous make this defense: “There was never any contract since our alleged act of acceptance (the shipment of
defective goods) did not comply with requirements of Mastervoice’s offer (which contemplated only shipment of good
fuses)”?
 This defense (the sort of thing that only a lawyer would think up) will not work; §2-206(1)(b). See Official Comment 4. To
avoid the breach Generous should have announced prior to shipment that the fuses were tendered only as an
accommodation, which, in effect, makes their tender a counteroffer.
(c) Instead of the above, assume that when Generous received the order it discovered that it no longer manufactured the type of
fuses Mastervoice wanted, but that it did carry a very similar type of fuse that it believed would suit Mastervoice’s needs. The
shipping manager for Generous was unable to get through to the relevant people at Mastervoice, so in the end Generous
shipped the slightly different fuses along with a cover note saying, “These are similar to the fuses you ordered, but may not be
right for you. If they are not suitable, we will gladly take them back without charge.” Is Generous now in breach because it
shipped nonconforming goods? See §2-206(1)(b).
 Here the seller has been careful to make a counteroffer and not an acceptance. The substituted fuses are offered only as
an accommodation to the buyer, and the seller is not in breach. The buyer could, of course, refuse to take them.
 Section 2-205: Firm Offers
An offer by a merchant to buy or sell goods in a signed writing which by its terms gives assurance that it will be held open is not
revocable, for lack of consideration, during the time stated or if no time is stated for a reasonable time, but in no event may such
period of irrevocability exceed three months; but any such term of assurance on a form supplied by the offeree must be separately
signed by the offeror.
 Casebook Problem
o For years P. Dreamer had wanted a Rolls Royce Silver Shadow with burgundy-colored trim (he had seen one in an ad and
salivated). He saw one on the lot of Posh Motors. After Dreamer had dickered loud and long over the terms with Paula Posh,
president of Posh Motors, they finally agreed on a price. Dreamer said he wanted to clear the deal with his wife before signing
anything, so Posh promised she would hold the car for the Dreamers until the next day at noon. When Mr. and Mrs. Dreamer
arrived at the dealership the next morning, the car was gone. Posh had made a better deal with another buyer. Do the
Dreamers have a good cause of action? See §2-205.
 The firm offer rule, §2-205, does not apply since a written offer is required. Promissory estoppel is sometimes being used
in this type of situation to aid people in the position of the Dreamers.

28
 Bar Exam Pointer: The firm offer rule is a favorite of bar examiners since if you change any of its predicates (no sale of
goods, no writing, no statement the offer will be left open, or no merchant offeror) the common law rule permitting
revocation still obtains.
B.Battle of the Forms
 Grant Gilmore on Section 2-207
o Grant Gilmore, who was one of America’s leading experts on contract law and the Uniform Commercial Code, called Section 2-
207 a “miserable, bungled, patched-up job” and “arguably the greatest statutory mess of all time.”
 The Problems That Section 2-207 Addresses
o Section 2-207 of the U.C.C. deals with some difficult problems. First, if an offer and an acceptance exchanged by the parties do
not precisely agree about the terms of the contract, is a valid contract formed?
o Second, if a valid contract is formed by the exchange of an offer and an acceptance that do not agree as to all of the terms of
the contract, what are the terms of that contract?
o Third, if a valid contract was not formed by the writings but both parties then engaged in conduct sufficient to establish a
contract, what are the terms of that contract?
 The Common Law: The Mirror Image Rule and the Acceptance as “Counteroffer”
o At common law if the “acceptance” was not the “mirror image” of the “offer” then the exchange of writings did not form a
contract. This created uncertainty in commercial transactions.
o And at common law if the parties went ahead and performed the contract after an exchange of writings that did not match,
then the second writing was treated as a “counter-offer” that the first party “accepted” by conduct … and all of the second
party’s terms were the terms of the contract! This encouraged parties to characterize their communication as a “counter-offer”
so that their terms would be binding.
o Section 2-207 changes both of these results.
 Issue 1: Was a Binding Contract Formed?
o The first issue that Section 2-207 addresses is whether a binding contract has been formed.
 Mirror Image Rule Replaced
o At common law, if the parties did not precisely agree as to every single term of the contract, then no binding contract was
formed.
o Under Section 2-207(1), a “definite and seasonable expression of acceptance operates as an acceptance” even though it states
different or additional terms.
o There are two situations where an expression of acceptance does not “operate as an exception.” First, where the parties do not
agree as to the “core” or “essential” terms of the contract. And second, where one or both of the parties has included a
“proviso clause” in their offer or acceptance.
 Section 2-207(1): Mirror Image Rule Replaced
A definite and seasonable expression of acceptance or a written confirmation which is sent within a reasonable time operates as
an acceptance even though it states terms additional to or different from those offered or agreed upon, unless acceptance is
expressly made conditional on assent to the additional or different terms.
 Exception A: No Agreement as to Core Terms
o Under Section 2-207, a definite and seasonable expression of acceptance operates as an acceptance even though it states
different or additional terms. But what if the “acceptance” states a different price or a different quantity or proposes to ship a
different product than the offer asked for?
o In response to an offer, a communication that changes a “core” term of the offer does not operate as an acceptance.
 Example: The Stepladder (1)
o Taylor, a housepainter, sent a text message to the Primo Ladder Company ordering Model No. 35E, a 35-foot aluminum
extension ladder with safety harness, for $325. Primo Ladder replied by text stating, “We accept your offer. Model 35E is no
longer available. We have shipped Model 40E to you at no extra cost, payment due on delivery.” Taylor texts back, “I do not
want the 40E – too long for my truck.” Primo responds, “Product has already shipped, we demand payment on delivery.” Taylor
refused to accept delivery of the ladder, and Primo sued Taylor for breach of contract. Under Section 2-207, was a binding
contract formed by Primo’s “acceptance”? Is Taylor liable to Primo under the law of Sales?
 ANSWER: No, Taylor is not liable to Primo because no contract was formed. It is true that Section 2-207 provides that a
communication may operate as an acceptance even though it states different or additional terms. However, Primo’s
purported acceptance did not operate as an acceptance because it did not agree with the offer on one of the core or
essential terms of the contract – the item that was being purchased.
 Example: The Stepladder (2)
o What if Primo’s initial text message had stated: “We accept. Model 35E has been shipped to your address, payment due upon
delivery. Any and all disputes must be resolved by arbitration.” Would this communication “operate as an acceptance” under
Section 2-207(1)? Was a contract formed by the exchange of these communications?
 ANSWER: Yes, this communication would operate as an acceptance, even though it contains an additional term (the
arbitration clause) which was not in the offer. A binding contract was formed between the parties.

29
 As explained below, the arbitration clause would not become a part of the contract if it materially alters the contract; but
Primo’s communication nevertheless operated as an acceptance and Primo would be bound to the contract even though
the arbitration clause was not effective.
 Exception B: The Proviso Clause
o Under Section 2-207(1), if the acceptance is expressly made conditional on assent to different or additional terms, then the
acceptance does not “operate as an acceptance,” and no contract is formed.
 Example: The Stepladder (3)
o Assume that Primo’s communication to Taylor was as follows: “We accept. Model 35E will be shipped to your address, payment
due on delivery. All disputes must be submitted to arbitration. Our acceptance of your offer is conditional on your assent to
submit all disputes to arbitration.” Was a binding contract formed by the exchange of these communications?
 ANSWER: No. Primo’s acceptance contained a “proviso clause” that expressly made their acceptance conditional upon
Taylor’s assent to the arbitration clause. No contract was formed.
 Issue 2: If a Contract Was Formed, What Are the Terms of the Contract?
o If the words or the conduct of the parties formed a contract, then it is necessary to determine what the terms of the contract
are. Section 2-207 also addresses this issue.
 Three Ways to Form a Contract: Oral Agreement, by Exchange of Writings, and by Conduct
o If every contract that was entered into was written in a single document that was signed by both of the parties, it would be
easy to determine the terms of the contract.
o However, under Section 2-207 people can form binding contracts in at least three other ways:
A. By means of an oral agreement, often followed by confirmatory memoranda;
B. By means of an exchange of writings which constitute an offer and acceptance; or
C. By their conduct, such as by delivering and accepting goods, even in the absence of an agreement.
 A. Oral Agreement Supplemented by Written Confirmation
o Parties often negotiate a contract orally with the understanding that the contract will not be binding until it is reduced to
writing. In that case none of the terms that were merely discussed but not agreed to during the negotiations are considered
part of the contract.
o But another common scenario is where the parties enter into an oral agreement and then exchange written memoranda
afterwards summarizing the agreement.
o With this second situation, it is important to remember that the oral agreement is the contract! The exchange of writings
afterwards may contain terms that are different from or additional to what was agreed to in the oral agreement. Section 2-207
governs whether those different or additional terms become part of the contract.
 Additional Terms Are Considered Proposals for Addition to the Contract
o Subsection 2-207(2) provides that in all cases the additional terms are considered proposals for addition to the contract.
Accordingly, the additional terms do not become a part of the contract unless the other party expressly agrees to the additional
terms.
o However, if both parties are merchants, the additional terms automatically become part of the contract if they do not
materially alter the contract and if the other party does not object to the additional terms.
 Section 2-207(2): Additional Terms Are Considered Proposals for Addition to the Contract
The additional terms are to be construed as proposals for addition to the contract. Between merchants such terms become part of the
contract unless:
(a) the offer expressly limits acceptance to the terms of the offer;
(b) they materially alter it; or
(c) notification of objection to them has already been given or is given within a reasonable time after notice of them is received.
 Example: The Solar Panels
o On Monday June 1 Noel, the President of Excelsior Solar, negotiated an oral agreement with Parker, the President of Parker
Homes, to sell 1,400 solar roof panels to Parker for $850,000. The panels were to be delivered on August 1 to the site of a new
development, Yorkshire Estates. After reaching an agreement they shook hands. Noel said, “It’s a deal,” and Parker responded,
“It’s a good deal for both of us.” On Tuesday Noel and Parker exchanged confirmatory memoranda regarding the agreement.
Both memoranda stated that Excelsior would deliver the panels to Parker’s construction site on August 1, but Parker’s
memorandum stated that Excelsior had the duty to maintain insurance on the panels until they were installed; Noel’s
memorandum was silent on the question, and Noel did not object to the additional term contained in Parker’s memorandum.
Under the law of Sales, in the absence of any agreement the risk of loss shifts to the buyer after delivery of the goods, and the
buyer would be wise to insure the goods against loss. A tornado whipped through the construction site on August 2 destroying
the panels, which were not insured. Did Excelsior breach the contract by not insuring the panels?
 ANSWER: It depends!
 Section 2-207(2) governs this situation. Excelsior Solar and Parker Homes entered into an oral agreement and then
exchanged confirmatory memoranda. Both Excelsior Solar and Parker Homes are merchants. And Parker’s confirmatory
memorandum contained a term that was not in the oral agreement.
 If Parker’s term relating to insurance is found to be a “different term,” then it did not become part of the contract.

30
 If Parker’s term relating to insurance is found to be an “additional term,” then it became part of the contract unless it
“materially altered” the contract.
 B. Contract Formed by the Writings: The Stepladder (2) Example
o In the Stepladder (2) example above, the parties entered into a contract by the exchange of writings; Primo’s acceptance
operated as an acceptance even though it contained an additional term that was not contained in the offer: an arbitration
clause. Both Taylor and Primo Ladders are merchants, so Section 2-207(2) governs this situation as well. Is the arbitration clause
part of the contract?
 ANSWER: As with the solar panels, it depends! If the arbitration clause is considered to be a “different” term, then it is not a
part of the contract. If it is an “additional” term, then it became a part of the contract unless it materially alters the
contract. In most situations, the addition of an arbitration clause would materially alter a contract, so the term would not
become part of the contract.
 In summary, Section 2-207(2) provides that if a contract was formed by the parties’ communications (orally or in writing),
and if both parties are merchants, and if the confirmatory memoranda or the acceptance contains additional terms that do
not materially modify the contract, then the additional terms become part of the contract.
 Different Terms in the Offer and Acceptance
o Suppose that the writings exchanged by the parties contain contradictory terms on relatively minor matters because the
acceptance contained a “different term” than the offer. Which party’s term controls, if either?
o The courts have interpreted Section 2-207 in two different ways in this situation. Some courts take a “literal approach” to
Section 2-207 and hold that the words “operates as an acceptance” mean that the party who wrote the acceptance in effect
agreed to the contradictory term in the offer. Other courts (perhaps the majority) apply the “knock-out” approach; both of the
contradictory terms drop out of the contract. See, e.g., Dvorske and West, Effect of Different or additional terms in acceptance
or confirmation of contract for sale of goods – Application of “knockout rule” in case of conflicting additional terms, PAJUR
Commercial §10:10
o Section 2-207(3) expressly requires the knock-out approach when contracts are formed by performance.
o Section § 2207 of the Pennsylvania Commercial Code does not directly address the treatment of different terms in an
acceptance. Accordingly, several approaches to the problem have been developed by the courts and scholars.
o The approach adopted by a majority of courts, including the Pennsylvania courts, is the "knockout rule," which recognizes that
merchants are frequently willing to proceed with a transaction even though all terms have not been assented to and that it
would be inequitable to lend greater force to one party's preferred terms than the others. Under the knockout rule, the
conflicting terms in the offer and acceptance cancel one another, that is, are "knocked out."17 Summ. Pa. Jur. 2d Commercial
Law § 10:10 (2d ed.)
o Illustration: An arbitration clause in a purchaser's offer to a manufacturer of gear drive assemblies was canceled out under the
knockout rule pursuant to the Pennsylvania Commercial Code, where the manufacturer accepted the essential terms of the
purchaser's offer, but the acceptance contained a provision requiring resolution of the parties' disagreements in state or federal
courts in Chicago. Flender Corp. v. Tippins Intern., Inc.,
o Observation: The minority approach, in contrast, permits the terms of the offer to control, reasoning that there should be no
distinction between additional terms and different terms and that offerors have more reason to expect that the terms of their
offer will be enforced than the recipient of an offer can hope that its inserted terms will be effective. The last approach
considers any expression of acceptance with differing terms as actual rejection and counteroffer. This view has been widely
discredited as a revival of the common-law rule. Reilly Foam Corp. v. Rubbermaid Corp.
 C. Contract Not Formed Orally or by the Writings, But Rather by Performance
o The third way that a contract might be formed is by the conduct of the parties; that is, by delivering and accepting the goods.
o This situation is governed by Section 2-207(3).
 What Are the Terms of a Contract Formed by Performance?
o Under the common law “mirror image” rule, if the parties did not agree on every single point, then no contract was formed.
However, if the parties went ahead and performed the contract anyway, under the common law the last writing before
performance occurred was considered to be a counteroffer which was deemed to have been accepted when the contract was
performed.
o Section 2-207(3) is more neutral. Under Section 2-207(3) a contract formed by the performance of both parties consists of the
terms agreed to in the writings of the parties; terms determined by course of performance, course of dealing, and trade usage;
plus, the “gap fillers” supplied by the U.C.C. All different or additional terms are “knocked-out” of the contract.
 Section 2-207(3): Terms of a Contract Formed by Performance
Conduct by both parties which recognizes the existence of a contract is sufficient to establish a contract for sale although the writings of
the parties do not otherwise establish a contract. In such case the terms of the particular contract consist of those terms on which the
writings of the parties agree, together with any supplementary terms incorporated under any other provisions of this Act.
 Example: Arcane and Samuel: Contract Formed by Performance
o Arcane Objects and Samuel Umbridge enter into a contract for the sale of an ancient Chinese puzzle that Samuel intends to
display at his art gallery. The parties exchange a written offer and acceptance that agree as to all terms except the price: Arcane
offered to sell Samuel the puzzle for $85,000 and Samuel offered to purchase it for $75,000. Nevertheless, Arcane delivers the

31
puzzle and Samuel accepts it. Did Arcane and Samuel form a contract by the exchange of writings? Did they form a contract by
performance?
 ANSWER: The writings of the parties did not create a contract because they differed as to an essential term. (Note that
under Section 2-305 if the parties had intended, they could have agreed to enter into a contract with the price term open –
but that was not their intent in this example.) However, under Section 2-207(3) a contract was formed by the performance
of the parties.
o What are the terms of the contract formed by performance of Arcane and Samuel?
 ANSWER: Under the common law the contract price would have been $75,000 – Arcane would be considered to have
accepted Samuel’s “counteroffer” by shipping the goods. But Section 2-207(3) has a “knock-out” rule; all of the differing and
additional terms are knocked out, and any terms that are missing are supplied by course of performance, course of dealing,
trade usage, or the “gap-filler” provisions of the U.C.C. In this case, the missing term is price, and under Section 2-305 the
contract price is “a reasonable price at the time for delivery.”
 A Word of Caution
o Section 2-207 substantially changes the common law. Moreover, it is complex and ambiguous in key respects. As a result, there
is disagreement among the courts regarding how it should be interpreted. For example, many courts have applied the “knock-
out” rule in cases where a contract was formed by the writings, but the writings contain different terms, even though Section 2-
207 expressly applies that approach only to contracts formed by performance.
o Obviously, these differing interpretations of Section 2-207 make it difficult to have a “uniform” commercial code.
 Casebook Problem
o The Magic Carpet Co. had a long and profitable business relationship with Alibaba Carpet Manufacturers of Bagdad, Illinois.
Fifty-five times Alibaba had sold carpets to Magic Carpet. Each sale was carried out in the following manner. A partner of Magic
Carpet telephoned Alibaba’s order department and ordered a certain quantity of carpet at the price listed in Alibaba’s catalog.
After each oral order was placed, the credit department was consulted to determine if Magic Carpet was paid up. Then, if the
credit was okay, the order department of Alibaba typed the information from the order on one of its printed acknowledgment
forms, each of which had the following paragraph printed conspicuously on its face:
The acceptance of your order is subject to all of the terms and conditions on the face and reverse side hereof, all of which are
accepted by buyer; it supersedes buyer’s order form, if any. It shall become a contract either (a) when signed and delivered by
buyer to seller and accepted in writing by seller, or (b) at seller’s option, when buyer shall have given to seller specification of
assortments, delivery dates, shipping instructions, or instructions to bill and hold as to all or any part of the merchandise herein
described, or when buyer has received delivery of the whole or any part thereof, or when buyer has otherwise assented to the
terms and conditions hereof.
The provisions on the reverse side of the form provided, among other things, that the seller disclaimed all warranties, express or
implied. Each acknowledgment form was signed by an employee in Alibaba’s order department and mailed to the Magic Carpet
Co. Shortly thereafter, the carpet was shipped. Magic Carpet always received the acknowledgment form before p. 90the carpet.
They placed each form in a file, accepted delivery of the carpet, and paid for it promptly. On the 56th sale, the accepted and
paid-for carpet proved to be nonconforming. Magic Carpet sued Alibaba for breach of warranty. Alibaba replied that its form
disclaimed all warranties.
(a) Was a contract formed between Magic Carpet and Alibaba? See §2-207(1).
 Yes; see Dorton v. Collins & Aikman Corp. Alibaba has failed to use the language required by the proviso in §2-207(1). In the
actual case, the court said that while the words “subject to” in the acknowledgment form “suggests that the acceptances
were conditional to some extent, we do not believe the acceptances were “expressly made conditional on [the buyer’s]
assent to additional or different terms,” as specifically required under the Subsection 2-207(1) proviso. In order to fall
within this proviso, it is not enough that an acceptance is expressly conditional on additional or different terms; rather, an
acceptance must be expressly conditional on the offeror’s assent to those terms. . .. [W]e believe that [the proviso] was
intended to apply only to an acceptance which clearly reveals that the offeree is unwilling to proceed with the transaction
unless he is assured of the offeror’s assent to the additional or different terms therein.” (emphasis added)
 Therefore, Alibaba’s response is an acceptance under §2-207(1) and a contract comes into existence.
(b) Was the disclaimer of warranties part of that contract? See §2-207(2).
 No. Official Comment 4 to §2-207 tells us that a disclaimer of warranties is a material alteration of the contract and is
therefore excised by §2-207(2)(b).
 The rest of the acknowledgment form is still an acceptance, however. Finding a material alteration only jettisons that
particular term.
 The Code is saying to someone in the seller’s position, “Don’t try and add material things in boilerplate language—if it’s
important either negotiate for it openly or we will strike it.”
 Diamond Fruit Growers, Inc. v. Krack Corp.
o In this case, the court walks through the §2-207 Battle of the Forms. Metal-Matic accepted Krack’s offer to buy tubing, but
expressly conditioned on assent to Metal-Mattic's disclaimer of liability. Krack accepted delivery of the goods, without
affirmatively accepting in words.

32
o An acceptance with additional or different terms is effective, but not if (as here) it is expressly conditioned on agreement to
additional or different terms. So §2-207(1) and (2) do not apply.
o Because the parties acted as though they had a contract, they have a contract, under §2-207(3). The contract terms are the ones
the parties’ writings agree on—which does not include Metal-Mattic’s disclaimer.
 Casebook Problem
o Would the following clause in the seller’s acknowledgment to the buyer’s order form be a material alteration under §2-207(2)
(b): “Any disputes concerning this contract shall be subject to binding arbitration”? See ICC Chemical Corp. v. Vitol, Inc.,
(arbitration clause not a material alteration where sometimes used in chemical industry sales); In re Cotton Yarn Antitrust
Litigation, (not a material alteration where arbitration was the usage of trade in the textile industry). How about a forum-
selection clause providing that all disputes must be litigated in the seller’s state? See Ridgelawn Cemetery Assn. v. Grantie
Resources Corp.
 Most courts considering the issue have held that an arbitration clause is a material alteration under §2-207(2)(b), and hence
does not become part of the contract. Some courts have reached the opposite result where arbitration is very common in
the industry, so that it, if effect, is the usage of trade. The Duro court held that a forum selection clause was a material
alteration.
o If these clauses were in an acknowledgment of an international sale of goods to which CISG applies, would it be valid? See
Article 19; Filanto, S.p.A. v. Chilewich Intl. Corp., appeal dismissed; Comment, The U.N. Convention on the International Sale of
Goods and the “Battle of the Forms,”
 Under CISG Article 19(3) a clause regulating the “settlement of disputes” is a material alteration and does not become part
of the resulting contract. The CISG battle of the forms provisions seem to make two major changes from §2-207: (1) an
objection to a non-material alteration voids the entire response as an acceptance, and (2) a response with a material
alteration (defined so broadly in Article 19(3) that it includes about everything one could think of) is a counteroffer (and
presumably performance without objection by the original offeror would be an acceptance thereof, the common law
result).
 Bayway Refining Co. v. Oxygenated Marketing and Trading A.G.
o Bayway addresses a common issue. Where the acceptance with additional terms is effective (i.e., it is not conditioned on
acceptance to those terms), its terms become part of the contract, unless the offer was limited to its terms, the terms materially
alter the contract, or the other party makes a timely objection.
o When is a term a material alteration? The court quotes the Official Comments, to the effect that a material alteration is one
that would “result in surprise or hardship if incorporated without express awareness by the other party.” The clause requiring
the buyer to pay excise tax was not, because it was a clause common in the industry and buyer could avoid the tax by
registering.
o “The cry of hardship rings hollow…” Ouch!
 Casebook Problem
o The purchase order of the buyer ordered a tugboat, and the fine print demanded that the tugboat be warranted for a two-year
period. The tugboat seller’s acknowledgment form contained a statement that disclaimed all warranties. Neither party read the
other’s form, so the tugboat was shipped, accepted, and then had major problems remaining afloat. You are the attorney for
the buyer. Advise your client whether the contract includes a warranty.
 The courts have not definitely settled the matter of differing terms, such as the warranty disagreement here, but most
strike both parties’ terms to the extent they differ (the “knockout rule,” see the next case), and use the gap-filling provision
of the Uniform Commercial Code to reach the fairest result.
 Here, that would mean that the buyer prevails since the Code adds an implied warranty of merchantability, but not a
disclaimer. However, the implied warranty of merchantability only lasts for a reasonable time, not necessarily the two years
that buyer had originally demanded.
 Northrop Corporation, v. Litronic Industries
o In this case, Judge Posner explores an interpretation issue in §2-207. An acceptance with additional or different terms is
effective, and the additional terms become part of the contract, unless they are material alterations.
o What about the different terms? The majority view is that they drop out (the “knockout rule”), because a party should not be
bound by a different term than it proposed. The minority view is that they are treated the same as additional terms if they
consist of non-material variations from the other’s form.
o The Official Comments 2 and 3 are unclear. Judge Posner prefers the minority view, but reasons that Illinois would adopt the
majority view and so follows the majority view. Federal courts apply the UCC in diversity cases, but, of course, are bound by the
relevant state’s law. Note that it is not always clear whether a term is additional or different: a disclaimer of warranty could be
viewed as either.
 Casebook Problem
o On April 25, Plastic Furniture Mart sent a purchase order for 100 tables to the Ersatz Manufacturing Company. In addition to the
usual boilerplate language, the purchase order also stated: “BUYER OBJECTS IN ADVANCE TO ANY TERMS PROPOSED BY SELLER
THAT DIFFER IN ANY WAY FROM THE TERMS OF THIS PURCHASE ORDER.” Ersatz received the order, and on May 3 it sent back
its own acknowledgment form, which disclaimed all warranties and contained this clause: “THIS IS NOT AN ACCEPTANCE UNLESS

33
BUYER ASSENTS TO ALL CHANGES MADE BY THIS ACKNOWLEDGMENT FORM.” Neither party read the details of the other’s
form. On May 6, Ersatz shipped the tables. Is there a contract? See §§2-207(3), 2-206(1)(b); see Building Materials Corporation
of America v. Henkel Corporation. Did Ersatz make a warranty as to the condition of the tables? See §2-314. On May 3 was there
a contract?
 This is an example of what one court called the “irresistible force colliding with the immovable object,” see In re Doughboy
Industries.
 Once Ersatz did ship the tables on May 6th, §2-207(3) creates a contract that the conflicting forms had previously kept from
arising. This contract consists of the matching written terms plus the Code’s supplementary provisions, which would
certainly include the §2-314 warranty of merchantability.
 On May 3rd, prior to performance, there was no contract because the boilerplate language clearly avoids any responsibility
under both §2-207(1) and §2-207(2)(a). On that date either party could have backed out with impunity.
 Question
o Assume that you are an associate at a law firm that represents a seller who frequently receives purchase orders triggering §2-
207’s rules. Your client wants to disclaim warranties, and the senior partner asks you to come up with a way of doing this. What
do you advise? How comfortable will be you be sleeping that night?
 There is no obvious practical answer to this question; how in the world can the mechanisms of §2-207 be used to allow a
seller to control the warranties?
 There are a number of unattractive choices. Sellers could reject all purchase orders and then send out their own similar
offers to the buyers, with the warranty disclaimers as part of those offers. (i.e., Seller becomes the offeror)
 Or sellers could respond to the purchase orders with a form using the language of the proviso and be careful not to ship the
goods until there was agreement on the warranty disclaimer (it is hard to write this sentence with a straight face—those in
business don’t behave like that, much as their lawyers wish they would).
 Sellers might make the buyers sign a form specifically discussing the warranty disclaimers before shipment of the goods,
another tactic that delays the sale and will not prove popular with the seller-client.
 What about having the seller offer the buyer two different prices: one with warranty and one without. That might be a neat
way of resolving the difficulty.
 It may be that the internet will solve the problem by requiring all buyers in cyberspace to consent to the seller’s website
terms in order to purchase the desired product. Indeed, there have been predictions that §2-207 will cease to be of much
importance as more and more sellers require all contracts to be formed on their websites with terms favorable to the seller,
and buyers can only place an order by clicking on “I Agree” icons.
 Casebook Problem
o Through a series of phone calls, the parties reached a complete oral agreement on the terms of a sales contract. Seller then sent
buyer an e-mail saying “I’m delighted we’ve made this deal. The contract is attached.” The attachment was seller’s usual
Acknowledgement Form, and it first disclaimed warranties (which had not been discussed in the oral communications), and then
clearly tracked the language of §2-207(1)’s proviso, by stating that this was not an acceptance unless buyer expressly agreed to
seller’s new terms. Buyer responded by sending payment, and seller promptly shipped the goods. When the goods proved to be
unmerchantable, buyer sued, and seller pointed to the disclaimer of warranties in its Acknowledgement Form. Can seller use
the proviso in this situation? See Air Prods. & Chem., Inc. v. Fairbanks Morse, Inc.
 If the parties have reached an oral agreement, the proviso’s use is a nullity; there is no way to avoid the already existing
contract. The contract then consists of the oral terms and the seller’s “written acknowledgement” (a phrase used in
§207(1), only triggers the rules of subsection (2) to possibly add non-material additions. The plain wording of 2-207 would
seem to allow the writings to change the oral agreement. But not all courts agree. The cited Wisconsin Supreme Court
decisions said at one point: “All of the language following the comma in subsection (1) simply preserves for the offeree his
right to make a counter-offer if he does so expressly. This phrase cannot possibly affect the deal between parties that have
reached an agreement and then exchanged confirmations. In that situation it is too late for a counter-offer and subsection
(2) must be applied to determine what becomes of the non-matching terms of the confirmations.”
 If the oral contract had not addressed the issue of warranties, then subsection (3) would supply the implied warranty of
merchantability, and (as usual) the seller’s attempt to avoid it would fail.
 Klocek v. Gateway, Inc.
o Klocek is one of a number of cases on the shrinkwrap issue, where the seller includes a form that buyer sees only after opening
the shrinkwrapped package. The 7th Circuit, in Hill and ProCD, held that §2-207 did not apply, there being no battle of the forms
where only one form is involved. But Klocek holds that §2-207 is not so limited, as its Official Comment 1 makes clear. Because
the buyer is not a merchant, §2-207(2) is not applicable and the additional terms do not become part of the contract.
o As noted above, the shrinkwrap issue is becoming less pressing, as merchants become more skilled at getting affirmative
consent to terms (“clickwrap”). The trend among courts is to enforce those agreements, even against consumers, unless the
terms are unconscionable (a flexible standard).
 Canvas Quiz Questions
o The two issues that Section 2-207 addresses are:
 Whether a contract is formed when two parties exchange an offer and an acceptance that do not precisely match.

34
 What the terms of a contract are where the writings do not form a contract but both parties performed the contract.
 When two parties exchange writings proposing terms, which one is the "offer" and which is the "acceptance."
 Whether a writing evidencing the existence of a contract is required in order for a contract to be enforceable.
o At common law, in order to form a valid contract, there had to be a perfect "meeting of the minds" -- the offer and the
acceptance had to be the "mirror image" of each other or a valid contract was not formed.
 True
o Section 2-207 abolishes the mirror image rule and permits the formation of a contract even though the acceptance states
terms that are different from or additional to the offer.
 True
o In order for a non-matching offer and acceptance to create a valid contract, the parties must agree as to the "core" or
"essential" terms. A valid contract can still be formed if the "boilerplate" terms do not match.
 True
o A party purporting to accept an offer with an acceptance that contains different or additional terms may not make the
formation of a valid contract conditional upon acceptance of all of the terms set forth in that party's acceptance.
 False - Section 2-207 expressly permits a party to make the formation of a valid contract conditional upon acceptance of all
of the terms set forth in that party's acceptance.
o If a contract is formed by the exchange of an offer and an acceptance that do not perfectly match because the acceptance
contains different and additional terms, then
 any different or additional terms in the acceptance are to be construed as proposals for addition to the contract.
 any additional terms in the acceptance are to be construed as proposals for addition to the contract.
 the additional terms in the acceptance automatically become part of the contract.
 as between merchants, any additional terms contained in the acceptance become part of the contract unless the offer
expressly limits acceptance to the terms of the offer; the additional terms materially alter the contract; or the offeror
notifies the acceptor of objection to those additional terms within a reasonable time after notice of them is received.
 the terms of the offer are deemed to have been "accepted" by the acceptance.
o If the parties to a contract do not form a contract through the exchange of an offer and acceptance, then it is impossible to
form a contract through performance (i.e., delivery and acceptance of goods).
 False - Even if the writings of the parties do not form a valid contract, the parties may create a contract through their
conduct, for example by shipping and accepting goods.
o Where the parties to a contract exchanged writings, but the writings were insufficient to form a contract, but the parties
nevertheless formed a contract by mutual performance (for example by shipping and accepting the goods), then the terms of
the contract consist of:
 the terms on which the writings of the parties agree.
 terms supplied by course of performance, course of dealing, and usage of trade.
 terms supplied by the "gap filler" provisions of Article 2.
 the terms contained in the last document that was exchanged prior to performance, because that document is considered
to be a "counteroffer" that was accepted by the performance of the other party.
o Festus Pet Stuff offered to sell 2000 pounds of dog food to Fido's Spa, a kennel that cares for dogs whose owners are on
vacation. Festus sends an offer sheet proposing to deliver to Fido's Spa 200 pounds of "Type ABY" dog food per week for 10
weeks at a price of $1 per pound. Festus's offer sheet states that deliveries will occur before noon on Thursday of each week.
The owner of Fido's Spa immediately sends back a document "accepting" Festus's offer and repeating all the essential terms but
stating that delivery should occur before noon on Wednesday of each week. Neither the offer nor the acceptance prohibits the
formation of a contract except upon agreement to all of the terms of each document. Please answer the following questions
about the legal effect of this exchange of writings.
 The exchange of writings was sufficient to form a contract between Festus and Fido's because Fido's response was a
"definite and seasonable expression of acceptance" that agreed with all of the essential terms of Festus's offer.
 The contract will require Festus to deliver the dog food on Thursday of each week because Fido's proposed term for
Wednesday deliveries was different from the term proposed by Festus.
 The contract will require Festus to deliver the dog food on Wednesday of each week because this was an additional term
proposed by Fido's which, as between merchants, became a part of the contract.
 The exchange of writings was not sufficient to form a contract between the parties because the "acceptance" was not the
"mirror image" of the offer.
o Agnes, the interior designer for Skylark Hotels, offered to purchase a chandelier from Premium Lighting. Skylark offered to pay
$3,900 for the "Dance Hall" Chandelier if the item could be delivered before August 15 to its hotel in Fargo, North Dakota.
Payment would be due by August 31. Premium wrote back to Skylark stating, "We accept your offer, and will deliver the "Dance
Hall" Chandelier to the Skylark Hotel in Fargo, ND, before August 15, payment due by August 31. However, the price of the
chandelier will be $4,300." Four days after emailing this "acceptance" Premium shipped the chandelier to the hotel in Fargo.
The hotel installed the chandelier. Was a contract formed by the writings? Was a contract formed by performance? And what
is the contract price for the chandelier?

35
 A contract was formed by the written offer and acceptance because the writing from Premium was a "definite and
seasonable expression of acceptance."
 A contract was not formed by the exchange of writings because they did not agree as to the "essential terms" of the
contract.
 The contract price is $3,900.
 The contract price is $4,300.
 The contract price is a "reasonable price" at the time and place for delivery.
 A contract was formed by the conduct of the parties in shipping and accepting the chandelier.
 Review/Assessment Questions
 What are the exceptions to the UCC Article 2 Statute of Frauds?
o 2-201(2)(a): Exception: Confirmation Among Merchants-Instead of a writing signed by the party against whom enforcement is
sought, as between merchants a contract for the sale of goods for a price of $500 or more is enforceable if the party seeking to
enforce the contract sent a written and signed “confirmation” of the oral agreement and the other party did not object within
ten days. Between merchants if within a reasonable time a writing in confirmation of the contract and sufficient against the
sender is received and the party receiving it has reason to know its contents, it satisfies the requirements of subsection (1)
against such party unless written notice of objection to its contents is given within 10 days after it is received.
o 2-201(3)(a): Exception: Specially Manufactured Goods- Despite the lack of a signed writing evidencing the contract the contract
is enforceable: if the goods are to be specially manufactured for the buyer and are not suitable for sale to others in the ordinary
course of the seller's business and the seller, before notice of repudiation is received and under circumstances which reasonably
indicate that the goods are for the buyer, has made either a substantial beginning of their manufacture or commitments for
their procurement
o 2-201(3)(b): Exception: Judicial Admission- Despite the lack of a signed writing evidencing the contract the contract is
enforceable: if the party against whom enforcement is sought admits in his pleading, testimony or otherwise in court that a
contract for sale was made, but the contract is not enforceable under this provision beyond the quantity of goods admitted
o 2-201(3)(c): Exception: Performance of the Contract -Despite the lack of a signed writing evidencing the contract the contract is
enforceable: with respect to goods for which payment has been made and accepted or which have been received and accepted
(2-606).
 What sorts of parol evidence are admissible and subject to what conditions?
o Evidence of consistent additional terms is admissible so long as the written contract is intended to be the final expression and
not a Complete and Exclusive Statement of the Terms of the Agreement;
o Matters relating to whether a valid contract was formed (capacity, duress, illegality, forgery, etc.);
o Whether there was a condition precedent to the contract going into effect;
o To interpret a term of the contract that is ambiguous;
o To prove course of performance, course of dealing, or trade usage;
o To prove the existence of a separate contract supported by separate consideration.
o To prove that the contract was later modified or that a right was waived (modifications to contracts are governed by 2-209 or
Restatement (Second) Section 89)
 What are the requirements for an effective electronic signature?
o An intent to sign the electronic record.
 What different ways may an offer be accepted under UCC Article 2?
o Any manner sufficient to show agreement, including conduct by both parties which recognizes the existence of such a contract.
 What is the structure of the Battle of the Forms rule?
1. The “writing” need not be the contract itself; it must simply a document that “indicates that a contract for sale has been
made”;
2. The writing need not state all of the terms of the contract nor must it correctly state all of the terms, but it must be “sufficient”
to indicate that the contract was entered into;
3. The writing must state the “quantity” of the goods to be sold and the contract is not enforceable beyond the quantity stated;
4. The writing must be “signed” by the party against whom enforcement is sought or by their authorized agent.
 Avril and Mario had a long conversation, ending with them agreeing that Avril would build Mario a custom wheelchair for an
agreed price. When the wheelchair was almost finished, Avril contacted Mario to arrange delivery and payment. Mario denied
that they had an enforceable contract, because he had not signed a contract and she had not sent him a confirmation. Is there an
enforceable contract?
o No, for lack of a signed writing. - Section 2-201 provides that a signed contract is generally required, but then provides several
generous exceptions. One applies here. No writing is required if the goods are specially manufactured for the buyer, not suitable
for sale to others, and seller has made a “substantial beginning of their manufacture.” §2-201(3)(a).
o Answer: Yes, because an oral agreement for specially manufactured goods is enforceable where the seller has made a
substantial beginning.
o Yes, because no writing is required where the parties are not both merchants. - Merchant status is not a factor in the rule
o No, because a writing is required to enforce a contract against a nonmerchant. - Merchant status is not a factor in the rule
36
 Steeply agreed over the phone with Orin to purchase a painting by Joelle for $7,000. Not long after, Joelle disappeared in a
volcanic explosion, which drove the price of her artwork up. Orin refused to deliver the painting. In court, Orin testified, “We had
an agreement, but sorry to say that it cannot be enforced without a writing.” Can Steeply enforce the contract?
o No, for lack of a signed writing. - One of the biggest exceptions to the writing requirement is that no writing is necessary if the
defendant admits in court that there was a contract.
o Yes, because the goods are unique. - There is an exception for specially manufactured goods, but not unique goods.
o Answer: Yes, because Orin admitted they had a contract.
o No, because Steeply could have sent a confirmation. - No confirmation is required to be sent.
 Pemulis Auto signs a contract with Lyle for the sale of a 2007 Pluto automobile. Which of the following could be introduced as
evidence of terms that were not included in the writing?
o A common practice in the used car business that sellers would replace any parts that failed for the first year.
o A practice between Pemulis Auto and Lyle, in their previous transactions, that Lyle could return the car within 30 days and get
his money back.
o An oral agreement between the two that Pemulis Auto would provide free service for the first two years.
o Answer: All of the above. - This question reminds us that the written contract may be explained or supplemented by usage of
trade, course of dealing, and consistent additional terms, where writing is not shown to be the complete and exclusive statement
of the parties’ terms.
 Gately Construction faxes an order to Van Dyne Apps for 50 tablet computers. Van Dyne faxes back an acceptance, with the
following provision: “THERE ARE NO WARRANTIES EXPRESS OR IMPLIED, INCLUDING NO WARRANTY OF MERCHANTABILITY FOR
THE TABLET COMPUTERS.” The tablet computers are duly shipped, accepted, and paid for. Not long after, Gately Construction
finds the performance of the computers to be sub-par. Did Van Dyne’s provision excluding warranties become part of the parties’
contract?
o Yes, because Gately Construction did not object to the provision. - Does not natter that Gately Construction did not object.
o Yes, because Gately Construction’s offer did not expressly limit acceptance to the terms of the offer. - Does not matter that
Gately Construction limited its original offer to its terms.
o Answer: No, because the provision would materially alter the contract. -The acceptance is effective, under the Battle of the
Forms provision, §2-207(1). Additional terms become part of the contract only if they are not excluded by §2-207(2). The
exclusion of all warranties would be excluded by §2-207(2)(b), because it is a material alteration.
o No, because an acceptance that is different from the offer is not effective to form a contract.
 Unlike a stricter view under some common law cases, the UCC allows an acceptance to be effective even if it does not match
the offer.
 Enfield Academy sent Microcenter an order for 25 Yushituyo Media Players. The order provided that “[a]n acceptance of this offer
is not effective if the acceptance changes any of the terms of the offer.” Microcenter quickly sent back an acceptance letter, which
contained an arbitration provision. Enfield Academy paid for the media players on delivery but was dissatisfied to learn they lack
the capacity to show tennis videos shot with a special slow-motion feature. Will Enfield Academy be subject to the arbitration
clause, if there is a dispute?
o No, because the parties never formed a contract. -Conduct may form a contract, under §2-207(3).
o Answer: No, because the arbitration clause would not be part of the contract. -Microcenter’s acceptance is not effective to form
a contract. Under §2-207(1), if the offer limits acceptance to the terms of the offer, an acceptance on different terms is not
effective. So even if the arbitration clause is not a material alteration, the parties’ writings do not form a contract. The terms of
that contract would be the agreed terms, which would not include the arbitration clause.
o Yes. The arbitration clause would be part of the contract because it was not a material alteration.
o Yes, the arbitration clause would be part of the contract because Enfield Academy did not object to the clause.

CHAPTER 4: WARRANTIES
I. Warranty of Title
 Sellers Warrant Good Title Free of Liens
o Under Section 2-312(1), sellers – whether they are merchants or non-merchants – warrant to their buyers that they are
conveying good title to the goods and that the goods are free of all liens or security interests.
o For example, Ed sells a television set to Mark at a garage sale. Ed warrants to Mark that Mark will acquire good title to the TV
free of all liens that Mark is not aware of.
 Section 2-312(1): Sellers Warrant Good Title Free of Liens
Subject to subsection (2) there is in a contract for sale a warranty by the seller that
(a) the title conveyed shall be good, and its transfer rightful; and
(b) the goods shall be delivered free from any security interest or other lien or encumbrance of which the buyer at the time of
contracting has no knowledge.
 Exclusion or Modification of the Warranty of Title

37
o Under Section 2-312(2) the warranty of title can be excluded or modified by specific language or by circumstances that give the
buyer reason to know that the seller either does not have good title to the goods, or that the seller is not claiming to have good
title to the goods.
o For example, if I purchase a new refrigerator from an individual selling it off the back of a pickup truck for $100 there is
probably no warranty of title.
 Section 2-312(2): Exclusion or Modification of the Warranty of Title
A warranty under subsection (1) will be excluded or modified only by specific language or by circumstances which give the buyer
reason to know that the person selling does not claim title in himself or that he is purporting to sell only such right or title as he or a
third person may have.
 Warranty Against Infringement
o Under Section 2-312(3) merchants who deal in goods of the kind also warrant that the goods are free of any rightful claim of
infringement by a third party.
o For example, if Dobbs Camera sells Bob a device manufactured by Higgens Electronics that transmits pictures over the internet,
but the device stops working because Higgens infringed a patent and a judge issues an injunction against the operation of the
device, Bob would have a claim against Dobbs Camera for violation of the warranty against infringement.
 Section 2-312(3): Warranty Against Infringement
Unless otherwise agreed a seller, who is a merchant regularly dealing in goods of the kind warrants that the goods shall be
delivered free of the rightful claim of any third person by way of infringement or the like but a buyer who furnishes specifications to
the seller must hold the seller harmless against any such claim which arises out of compliance with the specifications.
 Casebook Problem
o Fast Eddie stole Mabel Stanley’s car from a shopping center in Phoenix. He drove it to Las Vegas, where he sold it for $500 to
Sealed Lips Used Cars. This firm somehow obtained a Nevada certificate of title for the car, which showed clean title in Sealed
Lips. The car was then sold for $2,000 to a bona fide purchaser, Frederick Duty. Duty drove it for a month until a bad run at the
roulette wheel forced him to sell it in order to finance further recreational activity. Another bona fide purchaser, Samuel Pirate,
bought the car from Duty for $1,900. He drove it for only one week before the Nevada State Police impounded it and returned it
to Mabel in Phoenix. Pirate sued Duty for breach of the warranty of good title.
(a) Duty argues that he thought he had good title, and since he was not negligent or in any way at fault in so believing, the
warranty was not breached. Does this defense succeed? See Brokke v. Williams.
 The warrantor’s state of mind and/or freedom from negligence are irrelevant to liability for breach of warranty, as the
cited case held. Warranty creates an absolute obligation which the warrantor either meets or does not.
(b) Duty argues that he did have good title, and hence the warranty was not breached. Did he? See §2-403(1); Inmi-Etti v. Aluisi,
(Nemo dat qui non habet — “He who hath not cannot give.”).
 Courts hold that §2-403 gives Duty no better title than his transferor had, i.e., none. Note the Latin phrase, quoted in the
cited Maryland case. The more Latin the better. It always makes a lawyer sound learned.
 UCC §2-403(1) A purchaser of goods acquires all title which his transferor had or had power to transfer …
 Nemo dat qui non habet
(c) Would your answer to (b) change if Fast Eddie had bought the car from Mabel with a bad check? See §2-403; Allan Nott
Enters., Inc. v. Nicholas Starr Auto, L.L.C.
 Yes, see §2-403(1)(b). “Voidable title” generally means that the person who resold the goods did not steal them, but was
given them voluntarily by the original owner, who now has the ability to rescind the transaction (for fraud, bad checks, or
any number of other reasons), but who takes subject to the rights of an innocent purchaser. The Ohio Supreme Court
acknowledged this rule in the cited case but held that the Ohio Certificate of Motor Vehicle Act reached the opposite
result and allowed the cancellation of the new title even in the hands of a §2-403 bona fide purchaser, reasoning that the
UCC provision applies to the sale of all goods but the Motor Vehicle Act applied to cars, and, when in conflict, the more
specific statute governs over a broader one. Since this case involved cars the latter statute prevailed.
 UCC §2-403 When goods have been delivered under a transaction of purchase the purchaser has such power even though
… (b) the delivery was in exchange for a check which is later dishonored…
(d) Does the law give any relief to Duty? See §2-607(5)(a) (describing a procedure known as vouching in).
 Duty can give a vouching-in notice to his seller and the latter would be bound by any common factual determination if
Sealed Lips does not defend the action. The courts have required that the vouching-in notice more or less track the
language of §2-607(5)(a) [see Bendix- Westinghouse Automotive Air Brake Co. v. Swan Rubber Co.
 UCC §2-607(5) Where the buyer is sued for breach of a warranty or other obligation for which his seller is answerable over
(a) he may give his seller written notice of the litigation. If the notice states that the seller may come in and defend and that
if the seller does not do so he will be bound in any action against him by his buyer by any determination of fact common to
the two litigations, then unless the seller after seasonable receipt of the notice does come in and defend he is so bound.
 Casebook Problem
o Before the Nevada State Police found Mabel’s car, discussed in the last Problem, they accidentally impounded a very similar car
owned by P.T. Boss. Boss had recently purchased the car from Croupier Motors. In order to convince the police that they had
made a mistake and should release the car to him, Boss had to hire an attorney, Arnold Sunglasses. The latter did retrieve the

38
car and then sent Boss a bill for $4,100. Boss forwarded the bill to Croupier Motors, along with a cover letter to the effect that
title problems were the seller’s headache per §2-312. Should the car dealership pay Sunglasses’s bill? See Official Comment 1
to §2-312; White & Summers §10-15(b); Saber v. Dan Angelone Chevrolet. See also §2-607(5).
 No. One useful test is whether the third party’s claim is spurious or colorable, with the warranty being breached only in the
latter case. Here the title claim of Mabel Stanley to Boss’s car is clearly spurious, and Croupier Motors should be advised not
to pay Sunglasses’ bill. We are all at risk of having to defend a baseless lawsuit. However, in the cited case the police
impounded the vehicle, and the court held that even though eventually it was decided that there was valid title; the
impoundment and other facts were a sufficient cloud on the title to breach the §2-312 warranty. Also, in that case the court
did not give the plaintiff consequential damages in the form of attorney’s fees.
 Casebook Problem
o Sellers can sometimes evade or disclaim the warranty of title. Determine if the warranty is present in the following situations:
(a) The sales contract has this clause: “The product is sold ‘As Is’ and seller makes no warranties, express or implied, as part of
this sale.” See Official Comment 6 to §2-312.
 The cited Official Comments should answer the questions. See also White & Summers §10-17. (“There are no warranties,
express or implied”) are understood by everyone to refer to quality problems and will not get rid of the warranty of title.
 Moore v. Pro Team Corvette Sales, Inc. imposes a high standard for express disclaimers of the warranty of title, as well it
should. Avoiding the warranty of title is an extraordinary thing to do.
(b) Repossession Motors sold a car on credit to a customer who returned it after failing to make the first payment. It then
conducted an Article 9-type resale. (See §9-610(d).) Does the resale buyer get the benefit of a §2-312 warranty? See Official
Comment 5.
 As to the resale, the law here has changed with the adoption of the 1999 version of Article 9. At common law, resales by
secured parties (and judicial sales by sheriffs, etc.) were traditionally been caveat emptor as to title problems (one of the
reasons they bring so little), so the warranty of good title was not triggered unless this is a sale in the ordinary course of
Repossession’s business, in which case the warranty would attach. The chief question would be whether the buyer
appreciated that this was a foreclosure sale.
 However, under the current version of Article 9, a warranty of title is made at a foreclosure sale conducted by a
repossessing secured party unless clearly disclaimed; §9-610. The Official Comment to §2-312 had to be rewritten to reflect
this change in the law, which occurred in 1999.
 Moore v. Pro Team Corvette Sales, Inc.
o Pro Team unknowingly sold a stolen 1974 Chevrolet Corvette to Moore. The car was confiscated by state troopers and Moore
sued Pro Team.
o They argued that the warranty of title had been excluded by this clause in the contract: “All warranties pursuant to U.C.C.2-312
are hereby excluded from this transaction.”
o Section 2-312 permits an exclusion of the warranty of title but requires “specific language.”
o Courts apply a demanding standard of specificity. When buyers see the word “warranty,” they will think of promises about the
qualities of the goods themselves, not about whether buyer will own the goods buyer just paid for. So, courts will require words
that will convey the risk to buyer, such as specific reference to “ownership.”
o The reference to §2-312 was insufficient, because it would tell only the rare buyer (a lawyer familiar with warranty law) that
seller was making no promise the goods were not stolen.
o Note that the court reached the right result, but its reasoning was little vague. The court states at one point that it interpreted
the language as a limitation on remedy, as opposed to a disclaimer of warranty.
o But it then seemed to state the sound reason for its result: the disclaimer of the warranty of title which merely referred to §2-
312 did not have the required specificity.
o How would a buyer, even that hypothetical lawyer, know what the statutory reference meant without more of an explanation?
o How would you, as a lawyer, draft a clause that would effectively disclaim the warranty of title?
o Note the questions and comments following the case.
 Casebook Problem
o Yankees fan Billy Slider bought a baseball at an auction that was supposedly signed by both Babe Ruth and Lou Gehrig. He was
worried it wasn’t authentic, so he took it to Sports Confidence, a company that specialized in examining the bona fides of sports
memorabilia. He asked them to look into its history and authenticity. The owner of the company immediately recognized it as a
major find and decided to steal it. He sold it a man named Alex Stewart and told Billy Slider that the baseball was a fake. On
learning the truth, can Slider recover the baseball from Stewart? Do you need to know anything more about Stewart? See §2-
403(2) and (3); Zaretsky v. William Goldberg Diamond Corp.; Galin v. Hamada.
 Billy Slider entrusted the baseball to an expert on sports memorabilia, but it is not clear that the expert also was a merchant
who sold such items routinely. If not, then this is simply conversion and the sale is invalid, thus allowing Billy Slider to
recover the baseball.
 If the expert were also a merchant who routinely sold rare baseballs, the entrustment rule does apply, arguably giving the
expert power (“voidable title”) to effectively transfer ownership of the baseball to a buyer in the ordinary course.

39
 Much will depend on whether Alex Stewart, the purchaser, so qualifies. What did Stewart know about the provenance of
the baseball at the time he purchased it? How suspicious was the sale to him? Was he acting in good faith? If Stewart is not
a buyer in the ordinary course, his cause of action is against the expert for breach of the warranty of title.
 The first cited case, Zaretsky, involves a basic application of the entrustment rule, with a diamond being consigned to a
diamond merchant who then sold it in violation of an agreement not to do so without the permission of the owner. The
buyers prevailed since they qualified as purchasing in the ordinary course.
 The second case, Galin,, is similar, involving a painting entrusted to an art dealer who then sold it in the ordinary course to a
buyer.
 Lindholm v. Brant
o §2-403(2) (the entrustment doctrine) where the entrusting party grants a person who deals in goods of the kind the power to
convey good title to a Buyer in the Ordinary Course.
o In the “Red Elvis” case, Lindholm entrusted her painting to Malmberg to have it shown at art galleries. Without her permission
he sold it to Brant for $2.9 million.
o The Court found that Brant as a buyer in the ordinary course. Brant followed usual or customary practices and observed
reasonable commercial standards of fair dealing in the art industry in his dealings with Malmberg; Brant took the unusual step
of hiring an attorney and checking to make sure that Lindholm owned the painting. He did not contact Lindholm to make sure
that Malmburg had authority to sell it.
o Rule: a person purchasing art is only required to obtain documentary evidence that the seller has good title only where there is
“severe doubt” that the owner has such title. Confidentiality is the norm in the art world, and the court stated that it is not up to
the courts to determine what commercial standards a market should customarily follow.
 Casebook Problem
o Going on a family vacation, Kahr stowed sterling silver and credit cards in a bag in the attic. Not long after returning, Kahr took
several bags of clothing to donate to Goodwill — and unknowingly brought along the bag of valuables. Kahr handed the bags
over to Goodwill, and signed a form describing the donation as “used clothing.” After sorting through the bags, a Goodwill
employee set the credit cards aside, but put the silver out for sale. A bargain hunter soon snapped it up. Can Kahr recover the
silver? See §2-403(2) and (3); Kahr v. Markland.
 The cited case held that Kahr could recover the silver, because Kahr had not “entrusted” it to a merchant, but rather had
delivered it unknowingly. Close call, but good (will)result.
 Canvas Quiz Questions
o When goods are sold, the seller warrants to the buyer that the buyer will receive "good title" to the goods.
 True
o Only sellers who are merchants warrant that the buyer will receive good title to the goods.
 False - Both merchants and non-merchants who sell goods warrant to their buyers that the buyers will receive good title to
the goods.
o A scruffy individual stopped me on the sidewalk, pulled me to the side, and whispered, "Would you like to buy a Rolex for
$50?" If the watch turns out to be stolen, this individual is liable to me for breach of Warranty of Title.
 False - The circumstances clearly reason to know that the seller does not claim to have good title.
o Only sellers who are merchants in goods of the kind warrant against infringement.
 True
II. Warranty of Quality
A. Express Warranties
 Form, Intent, and Reliance on an Express Warranty
o Express warranties may be oral or written.
o They may also result from the seller showing the buyer samples or brochures.
o It is not necessary for the seller to use the words “warranty” or “guarantee.”
o It is not necessary to prove that the seller intended to create an express warranty; it is sufficient that the seller or its
representative made the statement or engaged in the conduct that constitutes the warranty.
o It is not necessary to prove that the buyer relied upon the warranty; it is sufficient if it is proven that the warranty was “part of
the basis of the bargain.”
 Puffing versus Promises
o The most difficult aspect of the law of express warranties is to distinguish between statements of fact and statements of
opinion. For example, imprecise statements of opinion like “This suit makes you look like a movie star” or statements of value
like “You won’t find a better deal anywhere, anytime, anyhow” are not express warranties.
 Four Ways to Create an Express Warranty
o To be an express warranty the seller or its agent may:
1. Make a statement of fact about the goods;
2. Make a promise about the goods;
3. Give a description of the goods; or
4. Show the buyer a sample or model of the good
40
 Section 2-313 (1) and (2): Express Warranties
(1) Express warranties by the seller are created as follows:
(a) Any affirmation of fact or promise made by the seller to the buyer which relates to the goods and becomes part of the
basis of the bargain creates an express warranty that the goods shall conform to the affirmation or promise.
(b) Any description of the goods which is made part of the basis of the bargain creates an express warranty that the goods
shall conform to the description.
(c) Any sample or model which is made part of the basis of the bargain creates an express warranty that the whole of the
goods shall conform to the sample or model.
(2) It is not necessary to the creation of an express warranty that the seller use formal words such as “warrant” or “guarantee” or
that he have a specific intention to make a warranty, but an affirmation merely of the value of the goods or a statement
purporting to be merely the seller's opinion or commendation of the goods does not create a warranty.
 Examples: Affirmations of Fact and Promises
o The following statements are express warranties even though they do not use the term “warranty” or “guarantee”:
 “This airplane is FAA flyable.”
 “This vacuum cleaner is rated at 12 amps.”
 “This truck will get you 25 miles per gallon.”
 Examples: Samples and Models
o A customer in a jewelry store points out a watch in a display case and says “I’ll take that one.” The salesperson, outside the
vision of the customer, selects, wraps, and sells the customer a different type of watch. This is a breach of an express warranty.
o A customer in a pet store wishes to purchase bird seed and asks to see a sample of “Mix No. 40.” The salesperson shows the
customer a sample that is mostly sunflower seed. However, when the customer arrives home and opens the bag of “Mix No.
40” it is mostly grain. This is a breach of an express warranty.
 “Part of the Basis of the Bargain”
o Another requirement about express warranties is that the statement of fact, promise, description, or sample must form “part
of the basis of the bargain.”
o For example, most courts have held that a statement or promise that is made after the contract is entered into is not “part of
the basis of the bargain.”
o And many courts have held that a statement of fact or a promise that the buyer did not learn about until after the purchase is
not “part of the basis of the bargain.”
 Sophistication and Inspection
o In some cases, the courts have held that a buyer who is particularly sophisticated or one who carefully inspects the goods prior
to purchase may not be able to sue for breach of an express warranty because the statement or promise was not “part of the
basis of the bargain.”
o For example, suppose the seller describes a horse as “sound.” The buyer, an experienced owner and trainer of racehorses, has
doubts and has the horse inspected by an expert. The inspection reveals that the horse is probably not “sound.” In light of the
inspection the buyer negotiates a lower price for the horse. In this instance a court found that the buyer could not sue the
seller for breach of an express warranty because the original affirmation of fact was not “part of the basis of the bargain.”
 Fraud vs. Warranty
o Even though a statement by a seller may not constitute an express warranty, it may constitute “fraud” – and under Section 1-
103 the law of fraud supplements the U.C.C.
o For example, the statement “There are plenty of stores that sell replacement parts for this lawn tractor” may not be an express
warranty if the court finds that the statement does not relate to the goods – but it could form the basis for a claim of fraud.
o However, it is more difficult for a buyer to recover for “fraud” than it is to recover for “breach of warranty.” To prevail in a
lawsuit for fraud the buyer would have to prove that the seller knew that the statement was false and intended for the buyer
to rely upon the false statement. The elements of “knowledge,” “intent” and “reliance” do not have to be proven to recover
for breach of an express warranty – only that the statement was false and that it was part of the basis of the bargain.
 Casebook Problem
(a) The salesman at the lot of Smiles Pre-Owned Vehicles told the woman buying the car that it was in “A-1 shape.” She bought the
car, but it broke down the next day, stranding her in the country. Was this oral statement mere puffing? See Wat Henry Pontiac
Co. v. Bradley; White & Summers §10-4. Is it an easier case if the seller tells the buyer that the used car is in “mint condition”?
See Taylor v. Alfama; Nigro v. Lee, (no express warranty where eBay seller described Mercedes Benz automobile as “gorgeous”
with “minor blemishes,” but sold “as is”).
 The cited White & Summers discussion involves the Wat Henry case and points out that the courts have not been very
consistent in these sorts of problems. The tension is between protecting free speech (and, some would say, the capitalistic
system) and protecting buyer expectations (with the reliance interest, as always, at the heart of contract law).
 In the actual case, the plaintiff prevailed on her theory that “A-1 shape” was a warranty, but she was a World War II wife
buying a used car to get from Oklahoma to visit her serviceman husband, traveling with a seven-month-old child, stranded
in Arkansas. Give us equities like that and we’d bet we could win any legal dispute that comes up.

41
 As for the “mint condition” statement, the cited opinion found an express warranty, but the issue is clouded because the
seller further warranted that the car had a rebuilt engine.
(b) When the farmer looked over the young chickens, he was contemplating purchasing from the poultry company, he complained
that they looked pretty scruffy. The salesman explained that that was because they were on half-feed and that when they were
placed on full feed, they would “bloom out, straighten up, and fly right,” and they would “do a good job in your chicken house.”
The farmer purchased the chickens, and two months later they started dying in droves. The farmer sued, claiming breach of an
express warranty. Is he right? Is this a question of law or of fact for the jury? See Woodruff v. Clark County Farm Bureau Coop.
 Chickens will “bloom out, straighten up, and fly right.”
 The Woodruff case found these statements express warranties, though the issue was one for the jury since the statements
were oral. If the statement were in writing, the issue is one for the judge (a rule left over from the days in which jurors
could not read!).
(c) Portia Moot, a third-year law student, had taken the course in Sales, so when she went to buy a used car, she listened very
carefully to the sales pitch. The smarmy salesman was quite friendly, but he only made two statements about the car she
bought: “This is a great car!” and “You’re going to love it!” In fact, the car broke down a great deal, and Portia quickly grew to
hate it. Does she have a cause of action here?
 “This is a great car. You’re going to love it.”
 No, Portia is out of luck in our opinion. These statements are the purest puff— having no reality referent. Modern American
advertising is typically phrased this way for the precise reason that these and similar statements mean absolutely nothing
more than the seller’s commendation of the goods, exaggerations tolerated by §2-313(2).
(d) Assume that the car salesman told Portia that the used car she was contemplating purchasing had been thoroughly inspected by
the car dealership’s crack repair department and was “mechanically in perfect condition.” Is that an express warranty? See
Bobholz v. Banaszak. Portia was suspicious about the reliability of the car and before she bought it, she took it to her own
favorite mechanic for an inspection. She didn’t buy the car until her mechanic cleared it as fine. When the car broke down a few
days later, she decided to bring suit on the express warranty. What defense will the car dealership raise?
 Car is “mechanically in perfect condition.”
 But she was suspicious and had her own mechanic inspect it, and he said it was fine.
 Portia is going have trouble here. She quite obviously did not rely on the seller’s representation, so it did not go to the basis
of the bargain. I once had a law student/client who did this very thing, and the trial court refused to find an express
warranty due to the buyer’s own investigation. We argued in vain that the subsequent inspection was not in lieu of the
warranty, but an additional assurance on top of it, but the trial court would have none of that. Thus, we have here a rare
situation where the seller can prove non-reliance on the alleged warranty, and thereby escape liability for its breach.
 Basically, if seller can prove non-reliance on the statement, then it is not an express warranty!
(e) David Vargo purchased only very healthy foods. A vegetable oil he bought stated on the label that it was “All Natural” but then
was found to contain genetically modified organisms. Has an express warranty been breached? See Ault v. J.M. Smucker Co.
 Adrianna Ault sued Smucker’s because it advertised Crisco Oil as “All Natural” when in fact it was produced with genetically
modified ingredients. Smucker’s moved to dismiss the complaint. In 2014, the District Court refused to dismiss the
complaint. The court stated:
 First, Defendant's labeling of Crisco Oil as "All Natural" is an actionable warranty. Under New York Uniform Commercial
Code Law § 2-313(I)(a), "[a]ny affirmation of fact or promise made by the seller to the buyer which relates to the goods and
becomes part of the basis of the bargain creates an express warranty." Generalized statements by the defendant, however,
do not support an express warranty claim if they are "such that a reasonable consumer would not interpret the statement
as a factual claim upon which he or she could rely." But what a reasonable consumer's interpretation might be is a matter of
fact which is not appropriate for decision on a motion to dismiss…”
 In 2015 the District Court ruled that Ault could not maintain a class action because the proposed class is not ascertainable.
Also, Smucker’s has now removed the “All Natural” label from its brands.
(f) Should a court accept the argument that a “Manufacturer’s Suggested Retail Price” is an express warranty of product quality,
because “status-conscious consumers tend to use a price cue also as a surrogate indicator of prestige”? Marino v. Coach, Inc.
 The court did not buy this argument. In the case, products were sold at a discount from the listed “Manufacturer’s
Suggested Retail Price” but in actuality no products were sold at that price. That might be deceptive (as common law fraud,
for example), but the implication of a superior product as an express warranty didn’t fly. There was nothing “express” about
it.
 In re Toshiba America HD DVD Marketing and Sales Practices Litigation
o Toshiba sold DVD players using the HD DVD format. In extensive marketing, Toshiba used the slogan “For Today, Tomorrow and
Beyond.” A rival format, Blu-Ray, became the industry standard. Toshiba exited the market, leaving buyers with DVD players
that worked perfectly well, but with the few movies on the market in HD DVD format. The court held that “For Today,
Tomorrow and Beyond” was no express warranty, but rather mere puffery. Even if there were an express warranty, the court
noted, the DVD players still worked as promised, to play discs in HD DVD format. The problem was that no one was selling
movies in that format anymore, something that Toshiba could not be understood to warrant, though you may disagree.
 Casebook Problem

42
o Upon graduation from law school, Andrew Loner hung out his shingle and waited. Mr. and Mrs. Consumer were his first clients,
and they told him the following story. Two weeks earlier they had visited a wallpaper store, Paper & Paste, Inc., and inquired
about vinyl wallpaper for their dining room. The salesman told them that the “finest” wallpaper in the store was Expenso-Paper,
a vinyl wallpaper selling at $75 a roll. When he learned that the Consumers had never before put-up wallpaper, the salesman
assured them that Expenso-Paper “goes up easily, can be put on with any paste, and dries immediately.” He said that it “would
look wonderful” and, moreover, that Expenso-Paper “was used by Mary Magic,” the famous movie star, in her dining room. He
showed them a sample book, and they picked out a pattern they liked and ordered ten rolls. When the paper arrived the next
week, it proved to be very stiff and hard to work with. It tore easily and refused to stay flat on the wall (it either bubbled or, due
to its heavy weight, fell down on drying). In addition, it was dyed a darker color than the version of the pattern in the sample
book. The final result was that the Consumers’ dining room looked terrible. To top it all off, the Consumers discovered that Mary
Magic did not own a home (she lived exclusively in hotels).
Upon complaining to Paper & Paste, the Consumers were told by the manager that Expenso-Paper needs a special brand of
paste, to wit, Expenso-Paste. They were also told that Expenso-Paper was an inferior brand and that next time they should buy
Super Wall, a better product that the store carried.
The Consumers told Loner that they signed the contract without reading it and that the statement about Mary Magic’s dining
room was made after they signed the agreement. Loner (and you) have to answer these questions:
(a) Which of the salesman’s representations amount to express warranties?
(1) finest? (2) goes up easily? (3) can be put on with any paste? (4) dries immediately? (5) would look wonderful? (6) was
used by Mary Magic?
Do you see any other express warranties?
 The only real argument will be whether “finest wallpaper” is “puffing,” §2-313(2), or has substance to it (as it may if
Expenso-Paper is truly an inferior brand).
 The other statements (with the possible exception of “would look wonderful”) would appear to meet the §2-313 tests.
 The sample is also an express warranty: §2-313(1)(c).
 The testimonial implied by Mary Magic’s supposed use of the product is also an express warranty since it might have played
a part in the buyer’s decision to buy (ask any parent with a cupboard full of cereal supposedly eaten by the children’s
favorite sports figure).
(b) Is the Mary Magic statement part of the basis of the bargain, arising as it did after the contract was signed? See §2-209(1);
Official Comment 7 to §2-313.
 Even though the cited sections and comment authorize the post-transaction warranty, some courts have shown no
enthusiasm for imposing liability on the seller where buyer’s reliance is not obvious. But others have been willing to go this
far; see Daughtrey v. Ashe, (statement made in insurance application after sale was part of basis of the bargain).
 Casebook Problem
o Balding Paul bought a wig from Hair, Inc. He became annoyed when the wig changed colors slightly from season to season. He
did not do anything about it until one day, while thumbing through a newspaper, he noticed an ad for Hair, Inc., that claimed
that their wigs did not shrink or change color. On checking back, he discovered that Hair, Inc., had run an identical ad during the
week prior to his purchase of the wig. He sues. On the witness stand Paul confesses that he never saw the ad until a year after
his purchase of the wig. Is this admission fatal to his recovery on a theory of express warranty? See Winston Indus., Inc. v.
Stuyvesant Ins. Co. (buyer held protected by manufacturer’s warranty on mobile home even though he never received a copy).
 Here we have the clash of ideas of basic justice (the seller should live up to its deliberately undertaken promises) and a legal
technicality (the buyer never saw the representation). Courts are reluctant to let the seller escape in this situation, but it is
difficult to get around the “basis of the bargain” issue. State consumer laws regulating deceptive advertising might provide
some sort of relief.
 One theory (that I have never seen tried) is to argue that the seller made this express warranty to those who did see the
original ad and who purchased on that basis, and that Paul is a third-party beneficiary of the warranty promise made to
such purchasers, who would certainly expect that anyone injured by a breach of the warranty should be able to sue.
 Canvas Quiz Questions
o Express warranties are made by:
 Answer: A seller who makes an affirmation of fact that relates to the goods, makes a promise that relates to the goods,
gives a description of the goods, or shows a sample or model of the goods, if that affirmation, promise, description, sample,
or model becomes part of the basis of the bargain.
 Merchants who deal in goods of the kind automatically as a matter of law.
 A seller who at the time of contracting has reason to know any particular purpose for which the goods are required, and
that the buyer is relying to the seller's skill or judgment to select or furnish the goods.
o Frances was shopping at Hollywood Hilarities, an upscale boutique selling novelty items in Beverly Hills, California, and saw a
ventriloquist's dummy for sale for $2,000. "Why so much?" he wondered. His curiosity piqued, he sought out Virginia, the
store's assistant manager who assured him that this puppet was the original Charlie McCarthy, the same one that Edgar Bergen
had created as a young man and used in his radio programs during the 1930s, 40s, and 50s. Impressed, Frances purchased the
puppet. Sadly, the real Charlie McCarthy puppet is in the Smithsonian, and the puppet that Frances purchased was just a very

43
high-quality replica, down to the little baby shoes. Virginia hadn't lied - she was just misinformed. The receipt the store gave
Frances simply states, "Charlie McCarthy Puppet - $2,000." Is the store liable to Frances for breach of an express warranty?
 Answer: Yes. - Virginia made an affirmation of fact that related to the goods and became part of the basis of the bargain.
The store breached an express warranty.
 No. Virginia acted in good faith and did not knowingly mislead Frances.
 No. Frances should have realized that it simply isn't possible that there was a radio show built around a ventriloquist act.
 No. Frances should have realized that the price was vastly lower than an historical artifact of this magnitude would
command; at a minimum he should have demanded a certificate of authenticity or conducted some research or had the
puppet inspected.
B. Implied Warranties
II. Implied Warranty of Merchantability
 Merchants Who Deal in Goods of the Kind Make the Promise of Merchantability
o Under Section 2-314(1) merchants who sell goods of the kind warrant to their buyers that the goods are “merchantable.”
 Section 2-314(1): Merchants Who Deal in Goods of the Kind Make the Promise of Merchantability
Unless excluded or modified (Section 2-316), a warranty that the goods shall be merchantable is implied in a contract for their sale if
the seller is a merchant with respect to goods of that kind. Under this section the serving for value of food or drink to be consumed
either on the premises or elsewhere is a sale.
 What Makes Goods Merchantable?
o Under Section 2-314(2) merchantability consists of six separate promises:
1. That the goods are what the merchant says they are;
2. That if fungible the goods are of “fair average quality”;
3. That the goods are fit for ordinary purposes for which such goods are used;
4. That the goods run within any variations permitted by the agreement;
5. That the goods are adequately contained, packaged, and labeled; and
6. That the goods conform to the promises or affirmations of fact made on the container or label.
 Section 2-314(2): Merchantability
Goods to be merchantable must be at least such as
(a) pass without objection in the trade under the contract description; and
(b) in the case of fungible goods, are of fair average quality within the description; and
(c) are fit for the ordinary purposes for which such goods are used; and
(d) run, within the variations permitted by the agreement, of even kind, quality and quantity within each unit and among all units
involved; and
(e) are adequately contained, packaged, and labeled as the agreement may require; and
(f) conform to the promise or affirmations of fact made on the container or label if any.
 Example 1: The Television at the Garage Sale
o Ed sells a television to Mark at a garage sale. Does Ed make an implied warranty that the television will work?
 ANSWER: No, Ed did not make an implied warranty of merchantability by selling a television at a garage sale. The warranty
of merchantability is only made by merchants dealing in goods of the kind.
 Example 2: The Used Air Conditioner
o Shannon purchases a reconditioned air conditioner from a local store called Alphonse’s Reconditioned Air Conditioners. It stops
working after a year. Is the store liable to Shannon for breach of the implied warranty of merchantability?
 ANSWER: The store is a merchant dealing in goods of the kind, so it did make an implied warranty of merchantability to
Shannon. The question is, how long should a reconditioned air conditioner ordinarily work? Was this air conditioner of “fair
average quality” and “fit for the ordinary purposes for which such goods are used”? If the product did not rise to the level of
fair average quality for a used air conditioner, then Shannon has a claim against the store for violation of the implied
warranty of merchantability.
 Shaffer v. Victoria Station, Inc.
o Section 2-314 provides many a plaintiff with a cause of action. A merchant selling goods automatically makes, among other
things, the broad warranty that the goods “(c) are fit for the ordinary purposes for which such goods are used; and… (e) are
adequately contained, packaged, and labeled as the agreement may require.”
o The issue in Shaffer was whether the warranty applied to the wine glass at a restaurant that broke and injured a customer.
Section 2-314 expressly provides that “Under this section the serving for value of food or drink to be consumed either on the
premises or elsewhere is a sale.” The court construed this, sensibly, to apply to the container in which the wine was served.
Without a container the wine could not be served. The plaintiff’s lawyer was very clever in reading the definition of
merchantability carefully so as to argue that if the container was dangerous, the warranty was breached.
o Comment (h) to 402A states that the “where the container is itself dangerous, the product is sold in a defective condition.”
o QUESTIONS following:
1. In a warranty suit, negligence vel non is irrelevant. The only issues are whether the warranty was made and breached; it is a
form of strict liability.
44
2. 2. The cited case decided that a sufficient sale was going on because the gamblers were, in effect, paying for their drinks by
the act of gambling (and how).
 Casebook Problem
(a) Are cigarettes that cause lung cancer merchantable if used over a period of years? See Haglund v. Philip Morris, Inc.; Franklin
E. Crawford, Fit for Its Ordinary Purpose? Tobacco, Fast Food, and the Implied Warranty of Merchantability; White & Summers
§10-12; See also Hoyte v. Yum! Brands, Inc., (KFC did not breach implied warranty of merchantability, although its products
contained trans fats); Bodie v. Purdue Pharma Co. (addictive nature of painkiller OxyContin did not breach warranty of
merchantability). If the seller’s advertisements stated that the cigarettes were “mild,” would that create an express warranty?
 The leading case on cigarette liability is Cipollone v. Liggett Group, Inc. The court concluded that (state law claims of)
express warranties are not preempted by federal law. See also Grisham v. Philip Morris, Inc., (“Low-tar” not an express
warranty about health and safety for cigarettes).
 As to implied warranties and cigarettes, many courts feel, reasonably or not, that they have no jurisdiction to wipe out the
tobacco industry by judicial decision. The legislative concerns are very great here, and the courts show distaste for the
ultimate step. Nonetheless there have been a spate of recent decisions going against the tobacco companies, so things may
be changing.
 In the cited Massachusetts case, the court found that the warranty of merchantability did protect users of cigarettes unless
they knew of the danger and continued to smoke after discovery of health risks exacerbated by inhaling tobacco.
(b) To treat menopausal symptoms, plaintiff took medicines manufactured by the defendant that combined the drug progestin and
estrogen. As a consequence, she developed breast cancer. Contending that the warranty of merchantability was breached when
the defendant failed to warn her of this possibility, she sued. Does that warranty contain a “failure to warn” component? See
Hines v. Wyeth.
 Failure to warn may breach §2-314(e)’s requirement that the goods be adequately labeled, and the cited case so held.
(c) Following a recipe in the Exotic Cookbook, Martha prepared pufferfish fillet for Thanksgiving. The book, however, neglected to
advise chefs to remove the pufferfish’s poisonous organs. After a lengthy recuperation, Martha sued the bookstore and
publisher for breach of the implied warranty of merchantability. What result? See Cardozo v. True. Cardozo emphasized one
major consideration: “It is unthinkable that standards imposed on the quality of goods sold by a merchant would p. 146require
that merchant, who is a book seller, to evaluate the thought processes of the many authors and publishers of the hundreds and
often thousands of books which the merchant offers for sale. One can readily imagine the extent of potential litigation. Is the
newsdealer, or for that matter the neighborhood news carrier, liable if the local paper’s recipes call for inedible ingredients? We
think not.” Do you agree?
 Courts, like the cited case, have routinely declined to hold that faulty information in publication breaches the UCC implied
warranty of merchantability or subjects the publisher and sellers to strict products liability.
(d) Officer Krupke, a New York policeman by profession, sold his family car to his next-door neighbor, Maria, telling her it was a
“good car.” In fact, it was falling apart and blew up the first time she drove it. Has Krupke breached the implied warranty of
merchantability? See §2-104(1); Official Comment 3 to §2-314; §1-304. Should §2-314 be extended so that the warranty is made
by all sellers? See Ingrid Hillinger, The Merchant of §2-314: Who Needs Him?
 Krupke is not a merchant, so he makes no warranty of merchantability. “Good car” does not appear to us to have enough
substance to be more than puffing; §2-313(2).
(e) If the dashboard touchscreen, which controlled the radio, climate control, and navigation systems, was defective, would that
make the car in (d) unmerchantable? See Rasnic v. FCA US LLC.
 Assuming a merchant sold the car, i.e., a car dealer, these defects would arguably breach the implied warranty of
merchantability. The cited case so held, with the court stating that there is more to an automobile than mere ability to
move from place to place: “plaintiffs allege that an optional feature with an itemized price did not work on a three-year-old
car.
 The malfunction allegedly affected the car’s heating, air conditioning and navigation systems. Further, plaintiffs allege that
the flashing touchscreen affected their ability to drive safely because it created a “severe distraction” that diverted their
attention from the road.” These things were enough to allege the car was non-merchantable.
 Webster v. Blue Ship Tea Room, Inc.
o Is fish chowder with bones in it “fit for the ordinary purposes for which such goods are used,” as §2-314(c) requires?
o It is in Massachusetts, according to this famous case. The court reasons that chowder consumers must beware of bones, so that
sellers are not reduced to selling pureed fish.
o QUESTIONS following
1. Is the court saying that a natural substance does not breach the warranty or that the plaintiff’s reasonable expectation
should have included the bone? See Phillips v. Town of West Springfield.
 The latter, according to the case cited. In Webster, the court notes that plaintiff’s New England background is “fact of some
consequence” and in the last major paragraph states that the bone is “to be anticipated,” and is “so well known to anyone
who eats [chowder].”
• But where in §2-314 is the test the “reasonable expectation of the buyer”?

45
• Should the court have used the “trade usage” disclaimer language from 2-316? Why not use the language of the UCC,
not the doctrines of the common law?
2. Would the plaintiff have recovered if she had been born and reared in South Dakota and was on her first visit to New
England?
 Apparently, this would make a big difference under this court’s reasoning. Should it?
3. Would the result be the same if the plaintiff had purchased a can of fish chowder and encountered the bone?
 An argument can be made that stricter standards are required of a canned version of the chowder simply because there is
more time for inspection and the manufacturer is more likely to realize that the product may reach South Dakota-type
buyers. Restatement of Torts Second §402A would seem to mandate liability in a strict product liability lawsuit, so the
courts should be less hesitant in imposing it in a warranty action.
 Casebook Problem
o Natty Bumpo was driving through upstate New York when a deer ran in front of his car. He swerved to avoid it and ran into a
tree. His major injuries came from his sudden contact with the steering wheel and the inside of the driver’s door, where he
smashed up against sharp points on the dashboard’s GPS system. Natty sued the car manufacturer, the Mohican Motor
Company, for breach of the warranty of merchantability. His theory was that the manufacturer should have designed a much
safer dashboard. The manufacturer’s defense was that the car was fit for its ordinary purpose and that Natty had misused it.
How should this come out? See Larsen v. General Motors Corp.; Comment, Intended Use and the Unsafe Automobile:
Manufacturers’ Liability for Negligent Design.
 The Larsen case has sparked a great deal of scholarly attention; see the series of articles on design defect litigation in 61
Corn. L. Rev. 495 and 541. See also L. Frumer & M. Friedman, Products Liability, Ch. 11, Manufacturing and Design Defects,
for a discussion and a collection of the cases.
 The courts have found liability for design defects only as to obviously foreseeable consequences, but those certainly include
the possibility that the car will be involved in an accident. Thus, the interior of the car should not further injure the
passengers in what is called the “second” crash.
 Casebook Problem
o Carry Nation, on the advice of her beautician, Parker Pillsbury, bought a hair dye named “Intoxicating Fragrance” and proceeded
to use it in accordance with the instructions on the package. Unfortunately, the product contained alcohol, to which Ms. Nation
was allergic, and she suffered considerable burn damage to her scalp and ears. When she sued the manufacturer, Harper’s Hair
Products, Inc., the basic defense was that only 0.5 percent of the population had this allergic reaction. Is this a good defense?
 Allergic reaction problems are typically handled as strict product liability matters, and the suit is rarely resolved purely
under warranty theories. Merchantability focuses on fitness for ordinary purposes, but does this mean only ordinary people
get the benefit of protection, or does it extend to those with extraordinary sensitivity?
 Further, how out of the ordinary must the sensitivity be before the law will withhold protection? If 50% of the users will
suffer injury, it seems clear the warranty of merchantability is breached, but what about 20%? 15%? 5%?
 It gets easier to answer these questions in favor of liability if the manufacturer knew of the risk and failed to warn against it
(or where the manufacturer did not perform adequate testing).
 In re Carrier IQ, Inc.
o A very modern problem: invasive software on mobile devices as breaching implied warranties! The plaintiffs urged that this
defect violated a number of statutes, federal and state, but the portion of the opinion that we focus on is the merchantability
claim. Here the defendant argued that the “ordinary purpose” of mobile devices is communication, and the products did that
well. But even if the major function is not impeded, this court and others have allowed the warranty claim to go forward where
the alleged defect is not one which the buyers would have agreed to if they’d have known it was present and thus this defect is
“core” so as to be fundamental to the bargain—and the court gives a number of examples. Here the software violated the
plaintiffs’ privacy expectations, and the court allowed the merchantability claim to go forward.
 Canvas Quiz Questions
o The implied warranty of merchantability is made by
 Answer: A merchant with respect to goods of that kind.
 Any merchant.
 Any person who sells a thing in a transaction that is covered by Article 2.
 A seller who at the time of contracting has reason to know any particular purpose for which the goods are required and that
the buyer is relying to the seller's skill or judgment to select or furnish the goods.
o Carl owns a machine shop. He purchased 300 sets of polycrystalline diamond drill bits from the manufacturer, Tungsten Carbide
Company. Unfortunately, due to a defect in the manufacturing process the layer of diamond particles was not properly bonded
to the drill bits, and the drill bits quickly became worn and would not cut. The contract is silent as to any warranties, express or
implied. Which of the following statements is accurate about the legal consequences of this situation?
 To recover for breach of warranty Carl will have to prove that Tungsten Carbide expressly warranted the quality of the drill
bits. - It is not necessary for Carl to prove that Tungsten made an express warranty.
 To recover for breach of warranty Carl will have to prove that Tungsten Carbide committed negligence in the production of
the drill bits. - It is not necessary for Carl to prove that Tungsten acted negligently.

46
 To recover for breach of warranty Carl will have to prove that Tungsten Carbide deliberately engaged in fraud in selling and
delivering these drill bits. – It is not necessary for Carl to prove that Tungsten acted fraudulently
 To recover for breach of warranty Carl will have to prove that Tungsten Carbide was aware of the particular purpose that
Carl had for the drill bits.
 To recover for breach of warranty Carl will simply have to prove that the drill bits were not "merchantable," that is, that the
drill bits were not of "fair average quality" or not "fit for the ordinary purposes for which such goods are used."
 To recover for breach of warranty it is not necessary for Carl to prove that Tungsten acted negligently or fraudulently nor is
it necessary for him to prove that Tungsten made an express warranty.
1. Implied Warranty of Fitness for a Particular Use
III. Implied Warranty of Fitness
o Under Section 2-315 a seller makes an implied warranty about the goods if the seller has “reason to know” that the buyer needs
the goods for a particular purpose and that the buyer is relying on the seller’s skill or judgment to select or furnish those goods.
o The seller does not have to be a merchant and the warranty does not have to be in writing.
 Section 2-315: Implied Warranty of Fitness
Where the seller at the time of contracting has reason to know any particular purpose for which the goods are required and that
the buyer is relying on the seller's skill or judgment to select or furnish suitable goods, there is unless excluded or modified under
the next section an implied warranty that the goods shall be fit for such purpose.
 Example: The Climbing Equipment
o Logan tells Hunter, who is not a merchant but whom Logan knows is an experienced rock climber, that Logan wants to climb
Zeus, an eighty-foot rock wall in the local state park and asks whether Hunter has any used climbing equipment that Logan could
purchase. Hunter sells Logan a set of used hexes, nuts, and cams that are in good shape, but doesn’t bother telling Logan
something Hunter knows that Logan is unaware of – that all climbing equipment is prohibited in the park. Did Hunter violate
the implied warranty of merchantability? Did Hunter violate the implied warranty of fitness for a particular use?
 ANSWER: Hunter did not make an implied warranty of merchantability because Hunter is not a merchant dealing in goods
of the kind; even if Hunter were a merchant, the warranty of merchantability was not breached because the goods Hunter
sold Logan are fit for the ordinary purpose of rock climbing.
 Hunter did violate the warranty of fitness for a particular use. Hunter had reason to know that Logan was purchasing
equipment to climb Zeus, and Hunter had reason to know that Logan was relying on Hunter’s skill and judgment to select or
furnish suitable equipment.
 UCC §2-315 Comment 1
Whether or not this warranty arises in any individual case is basically a question of fact to be determined by the circumstances of the
contracting. Under this section the buyer need not bring home to the seller actual knowledge of the particular purpose for which
the goods are intended or of his reliance on the seller's skill and judgment, if the circumstances are such that the seller has reason
to realize the purpose intended or that the reliance exists.
The buyer, of course, must actually be relying on the seller.
Cf. UCC §1-102(b) "Knowledge" means actual knowledge. "Knows" has a corresponding meaning
 UCC §2-315 Comment 2
A "particular purpose" differs from the ordinary purpose for which the goods are used in that it envisages a specific use by the buyer
which is peculiar to the nature of his business whereas the ordinary purposes for which goods are used are those envisaged in the
concept of merchantability and go to uses which are customarily made of the goods in question. For example, shoes are generally
used for the purpose of walking upon ordinary ground, but a seller may know that a particular pair was selected to be used for
climbing mountains.
A contract may of course include both a warranty of merchantability and one of fitness for a particular purpose.
The provisions of this Article on the cumulation and conflict of express and implied warranties must be considered on the question
of inconsistency between or among warranties. In such a case any question of fact as to which warranty was intended by the parties
to apply must be resolved in favor of the warranty of fitness for particular purpose as against all other warranties except where the
buyer has taken upon himself the responsibility of furnishing the technical specifications.
 Casebook Problem
o When Christopher Wren finished building a recreation room in his basement, he wanted a heater for it. He saw an ad for the A-1
Hotblast Heater, which seemed to be what he needed. A good friend of Wren’s named Inigo Jones ran a nearby appliance store.
Wren went there and told Jones that he wanted the A-1 Hotblast Heater for the new room. Jones knew the room well; he had
helped build it. When the heater arrived, it worked perfectly, but it simply did not have the capacity to heat the room. May
Wren sue Jones for breach of either §2-314 or §2-315? See Comment 5 to §2-315; Englebreacht v. W.D. Brannan & Sons, Inc.
 The heater worked, so §2-314 is not breached.
 Since Wren specified the model, he wanted there is an argument (see the cited case) that there was no reliance on the
seller’s skill or judgment, ergo no §2-315 breach.
 The counter argument is that Jones was a good friend who knew the basement well and Wren was expecting him to speak
up if the model ordered would not work. If Jones had reason to know of Wren’s reliance on his judgment, the §2-315
warranty arises and is breached.

47
 UCC §2-315 Comment 5 states, in part, “If the buyer himself is insisting on a particular brand he is not relying on the seller's
skill and judgment and so no warranty results.”
 Englebrecht v. W.D. Brannan & Sons, Inc.
• In Englebrecht, the case cited in your text, the buyer specified the product the seller should use in spraying the buyer’s
farm crop, and that undercut the argument for §2-315 being breached (also, apparently the product worked, so no §2-
314 breach), holding, “… his designation of Aatrex as the chemical to be applied prima facie excludes the application of
the statutory warranty (of fitness).”
• UCC §2-315 Comment 5 states, in part, “If the buyer himself is insisting on a particular brand he is not relying on the
seller's skill and judgment and so no warranty results.”
 Casebook Problem
o Lee Matheson went to the Easy Paint Store and bought a can of green paint, which the store mixed on the premises from
various pigments. Lee used the paint on his dining room walls, but due to a miscalculation on his part, he ran out when he was
half finished. He took the empty paint can back to the store. He told the clerk that he was only half done with the job and
needed another can, which the clerk promptly mixed and sold him. Lee finished the painting and then noticed two things: (1)
the dried paint gave off an offensive odor, and (2) the paint from the second can did not match the first. What cause(s) of action
does he have?
 The smelly paint would breach §2-314(2)(c); the non-matching color would breach §2-315.
 UCC §2-314(2)(c) (“goods to be merchantable must be at least such as … (c) are fit for the ordinary purposes for which
such goods are used.”)
 UCC §2-315 (“Where the seller at the time of contracting has reason to know any particular purpose for which the goods
are required and that the buyer is relying on the seller's skill or judgment to select or furnish suitable goods, there is unless
excluded or modified under the next section an implied warranty that the goods shall be fit for such purpose.”)
 The Problem is designed to illustrate the difference between the two implied warranties.
 Casebook Problem
o Donald Souse ordered a martini at the Tired Executives Club. When he bit into the olive, he cracked his new $2,000 dentures on
a pit. Is there a cause of action under either §2-314 or §2-315? See Hochberg v. O’Donnell’s Restaurant, Inc., 272 A.2d 846 (D.C.
1971).
Courts faced with this last Problem (harmful substances in food) have split into two camps: those that deny liability if the object
is a natural substance, as opposed to a foreign object, and those that permit recovery even where the consumer is injured by a
natural substance as long as the biter’s “reasonable expectation” is that it would have been removed. The problem reoccurs in
the cases as gourmands encounter stones in cherry pies, pits in olives, or, as in the Webster case above, bones in fish. See
Annot., 7 A.L.R.2d 1027.
 The issue involves harmful substances ion food – courts are split.
1. Natural substance (no liability) versus foreign object (liability) cases.
2. “Reasonable expectation” that even natural substances would be removed cases.
 This Problem is based on the cited case which opted for a reasonable expectations test; the case was settled prior to retrial.
 My guess is that martini drinkers would have to give expert testimony on the usage of trade: are martini olives routinely de-
pitted prior to being served?
 Hochberg v. O’Donnell’s Restaurant, 272 A.2d 846 (D.C. App. 1971)… the Hochberg court applied the reasonable
expectation test, explaining, “In our view it is unrealistic to deny recovery as a matter of law if, for example, a person is
injured from a chicken bone while eating a sliced chicken sandwich in a restaurant, simply because the bone is natural to
chicken. The exposure to injury is not much different than if a sliver of glass were there. ‘Naturalness of the substance to
any ingredients in the food served is important only in determining whether the consumer may reasonably expect to find
such substance in the particular type of dish or style of food served.’ [Citation omitted.] Because a substance is natural to
a product in one stage of preparation does not mean necessarily that it will be reasonably anticipated by the consumer in
the final product served. [Citations omitted.] It is a different matter if one is injured by a bone while eating a chicken leg or a
steak or a whole baked fish. There, it may well be held as a matter of law that the consumer should reasonably expect to
find a bone.” (Id. at p. 849.)
 Canvas Quiz Questions
o The implied warranty of fitness for a particular purpose is made by
 A merchant with respect to goods of that kind.
 Any merchant.
 Any person who sells a thing in a transaction that is covered by Article 2.
 Answer: A seller who at the time of contracting has reason to know any particular purpose for which the goods are required
and that the buyer is relying on the seller's skill or judgment to select or furnish the goods.
o Carolyn owns a six-year-old Shetland Pony named Grant. Carolyn has owned Grant for more than five years. George wants to
buy Grant, and George knows that Carolyn has owned the horse almost all of its life. Grant is healthy. Grant is beautiful. Grant
has won many competitions as a racehorse. There is only one problem. Grant is as mean an animal as you would ever want to
meet - stamping, biting, and if you can believe it, growling! Carolyn is not a merchant - this is the only horse she has ever owned.

48
Anyway, George wants to surprise his daughter Daisy with a pony of her own that she can brush and ride and play with and that
will be her best friend in all the world. He tells Carolyn at length all of his lovely daydreams of Daisy and her loving pony and
offers $15,000 for the beast -- twice what it is worth even as a racehorse! So Carolyn sells Grant to George without mentioning
anything about Grant's temperment. There is no written contract - just the exchange of money for the horse. Which of the
following is true about the legal consequences flowing from this transaction?
 Answer: Carolyn has violated the implied warranty of fitness for a particular purpose. - Carolyn had a legal duty to let
George know that this horse was not suitable for the particular purpose he had in mind.
 Carolyn didn't violate the implied warranty of fitness for a particular purpose because she isn't a merchant.
 Carolyn didn't violate the implied warranty of fitness for a particular purpose because she just remained silent and didn't
interrupt George as he fantasized about the horse.
 Carolyn didn't violate the implied warranty of fitness for a particular purpose because this horse is at least of "fair average
quality" and "fit for the ordinary purposes" to which a Shetland Pony is put.
C. Warranty Disclaimers and Limitations
I. Disclaiming Express Warranties
 Two Ways for a Seller to Avoid Liability for an Express Warranty
o If a seller wants to avoid liability on an express warranty there are two strategies that the seller may adopt:
1. Don’t make any affirmations of fact or promises about the goods, describe the goods, or show the prospective buyer any
samples or models – in short, don’t make any express warranties in the first place (Section 2-313, governing Express
Warranties); or
2. Use the Parol Evidence Rule to exclude terms that are different from or additional to the written contract (Section 2-202,
the Parol Evidence Rule).
 A Seller Can’t Make an Express Warranty and Then Disclaim It
o Section 2-316(1) states that if it isn’t reasonable to construe a warranty and a disclaimer as consistent with each other then the
disclaimer is “inoperative” and the warranty is effective. If the written contract includes an express warranty then no disclaimer
of that particular warranty can possibly be effective.
 Section 2-316(1): Exclusion or Modification of Warranties
Words or conduct relevant to the creation of an express warranty and words or conduct tending to negate or limit warranty shall be
construed wherever reasonable as consistent with each other; but subject to the provisions of this Article on parol or extrinsic
evidence (Section 2-202) negation or limitation is inoperative to the extent that such construction is unreasonable.
o Section 2-313 Official Comment 4.
In view of the principle that the whole purpose of the law of warranty is to determine what it is that the seller has in essence
agreed to sell, the policy is adopted of those cases which refuse except in unusual circumstances to recognize a material
deletion of the seller's obligation. Thus, a contract is normally a contract for a sale of something describable and described. A
clause generally disclaiming "all warranties, express or implied" cannot reduce the seller's obligation with respect to such
description and therefore cannot be given literal effect under Section 2-316.
o Note: This section is designed principally to deal with those frequent clauses in sales contracts which seek to exclude "all
warranties, express or implied." It seeks to protect a buyer from unexpected and unbargained language of disclaimer by denying
effect to such language when inconsistent with language of express warranty.
 Example: A Disclaimer That Does Not Work
o Morgan, the manager of a marina, sold Carter an outboard motor. The sales contract states, “The parts and workmanship of this
product are guaranteed to be free of material defect for a period of one year” but the contract also states “Seller makes no
express warranties with respect to this product.”
o The warranty is effective and the disclaimer is inoperative.
 It Is Possible to Avoid Liability on an Express Warranty by Using the Parol Evidence Rule
o If there are no express warranties in the final written agreement and if the written agreement is a “final expression” or a
“complete and exclusive statement” of the terms of the agreement under the Parol Evidence Rule, then it is possible to disclaim
all express warranties with language such as “Seller makes no express warranties with respect to this product.”
 Example: A Disclaimer that Does Work
o Morgan, the manager of the marina, orally promised Carter that the outboard motor came with a one-year warranty covering
parts and labor. However, the contract that Carter signed conspicuously states that “This contract is the final expression of the
terms of this agreement” and that “Seller makes no express warranties with respect to this product.”
o Through the operation of the parol evidence rule the written contract effectively disclaims the prior oral warranty. Morgan and
the marina might be liable to Carter for fraud, but not for breach of warranty.
 Legal Rights That Cannot Be Disclaimed
o Under Section 1-103(b) the common law of fraud and misrepresentation supplement the provisions of the U.C.C. These rights
cannot be disclaimed.
o Under Section 1-304 the principle of good faith applies to every transaction under the U.C.C., and under Section 1-302 the
obligation of good faith cannot be disclaimed.
o Under Section 2-302 contractual terms that are unconscionable may not be enforced.
49
o Federal laws such as Magnuson-Moss Warranty Act and other state laws such as state Deceptive Sales Practices Acts protect
consumers and preempt the provisions of the U.C.C.
 Example: Rights That Cannot Be Disclaimed
o Assume that a seller adds the following clause to its standard sales agreement:
o “Seller disclaims any liability for bad faith. This agreement is fully enforceable despite any illegality, lack of capacity, fraud,
duress, unconscionability, or violation of any statute or common law principle. In particular the Magnuson-Moss Warranty Act,
the Pennsylvania Unfair Trade Practices and Consumer Protection Law, and all regulations issued by the Federal Trade
Commission are inapplicable to this transaction.”
o This clause would have no legal effect.
 Note 1
o Most contracts for the sale of goods include various warranties of quality. A warranty is a guarantee or promise by the seller to
the buyer that specific facts about the goods are true.
o Warranties tend to become an issue of contention when the buyer becomes dissatisfied with the goods after purchase. Article 2
recognizes both express warranties and implied warranties. Within limits, the UCC permits sellers to exclude or modify implied
warranties on the goods they sell. Although the implied warranties of merchantability and fitness for a particular purpose are
imposed by operation of law, the UCC provides specific requirements by which the seller can disclaim either of these warranties.
 Note 2
o Unlike implied warranties, an express warranty is the result of a negotiated exchange between the buyer and seller. When it
comes to disclaiming express warranties, however, the UCC is not so explicit.
 Note 3 – Review UCC §2-313
o A seller may create an express warranty by an affirmative statement of fact or promise regarding the goods being sold, through
a description of the goods, or by providing to the buyer a sample or model of the goods. An express warranty will arise from any
of these, provided it becomes part of the basis of the bargain of the contract, and the goods must conform to the description,
promise, or model or sample. By making a promise or statement of fact about the goods, or providing a description of the goods
that are part of the basis of the bargain of the contract, the seller creates an express warranty that the goods will conform to
the description. Similarly, the seller is providing the buyer with a sample or model that becomes part of the basis of the bargain
between the parties. This creates an express warranty that “the whole of the goods” will conform to the sample or model.
 UCC §2-316(1) Note 4
o In broad terms, the UCC provides that endeavors to disclaim warranties should be construed reasonably and enforced unless
doing so would be unreasonable under the circumstances. To allow a seller to disclaim an express warranty that the seller freely
promised would appear to be illogical.
o UCC §2-313 Official Comment states: “‘Express warranties rest on ‘dickered’ aspects of the individual bargain, and go so clearly
to the essence of that bargain that words of disclaimer in a form are repugnant to the basic dickered terms.” The conclusion that
an express warranty, once made, cannot be disclaimed would appear to be indisputable.
 UCC §2-316(1) Note 5
o Nevertheless, the UCC permits a seller to disclaim an express warranty through a confusing provision. Section 2-316 provides
that wherever reasonable, an express warranty and a disclaimer of any express warranty are to be construed as consistent with
each other. However, subject to the provisions of the UCC on parol or extrinsic evidence, a disclaimer of an express warranty “is
inoperative to the extent that such construction is unreasonable.” Section 2-316 is designed to protect both the buyer and the
seller.
 UCC §2-316(1) Note 6 Scenario A
o The first situation involves an express warranty stated in the written agreement that also contains a general disclaimer, such as
“seller disclaims all warranties” or “seller makes no warranties, either express or implied, with respect to these goods.”
o In this instance, the general disclaimer is inoperative. This result relies on a general rule of contract construction that when
there is a conflict between specific and general provisions in the written contract, the specific provisions prevail. Giving effect to
the general written disclaimer provision over the specific express written warranty is unreasonable and therefore the general
disclaimer is inoperative.
 UCC §2-316(1) Note 6 Scenario B
o The second situation involves an express warranty stated in the written agreement that also contains a specific disclaimer.
o In this instance, the general rule of contract construction whereby a specific provision of the agreement prevails over a general
provision does not apply because there is a head-to-head meeting of two specific provisions. However, the inclusion in the
written agreement of an express warranty and a specific disclaimer cannot be construed as reasonably consistent with one
another. In this case, section 2- 316(1) explicitly states “negation or limitation is inoperative to the extent that such construction
is unreasonable.” The result in both the first and second situations is the same in that the disclaimer is inoperative.
 UCC §2-316(1) Note 6 Scenario C
o The third situation involves an express warranty not appearing in a written agreement that contains a specific disclaimer of all
oral express warranties.
o Generally, in a situation such as this, the express warranty is oral. In this instance, the seller’s specific disclaimer will prevail.
The seller will be able to rely upon the parol evidence rule contained in UCC section 2-202. The parol evidence rule would
50
prevent the buyer from proving and relying on the alleged oral express warranty. Thus, the oral express warranty would be
inconsistent with the disclaimer in a written agreement intended by the parties as the final expression of their written
agreement. The disclaimer prevails.
o §2-202 Terms with respect to which the confirmatory memoranda of the parties agree or which are otherwise set forth in a
writing intended by the parties as a final expression of their agreement with respect to such terms as are included therein may
not be contradicted by evidence of any prior agreement or of a contemporaneous oral agreement …
 UCC §2-316(1) Note 6 Scenario D
o Finally, the last situation involves an express warranty not appearing in a written agreement that contains a general disclaimer
of all oral express warranties. As in the third situation, the seller’s general disclaimer will prevail. Again, the seller will be able to
rely upon the UCC’s parol evidence rule to prevent the buyer from introducing parol or extrinsic evidence of any oral express
warranties.
o §2-202 Terms with respect to which the confirmatory memoranda of the parties agree or which are otherwise set forth in a
writing intended by the parties as a final expression of their agreement with respect to such terms as are included therein may
not be contradicted by evidence of any prior agreement or of a contemporaneous oral agreement …
 Bell Sports, Inc. v. Yarusso
o Bell Sports illustrates the point made in the text before the case.
o Language creating express warranties (such as affirmations about the qualities of the goods) will trump language trying to
disclaim express warranties.
o So where the manual made representations about how the helmet would reduce head injuries, language elsewhere purporting
to exclude express warranties was ineffective.
 Casebook Problem
o When Portia Moot went to buy a new car, she asked the salesman how many miles to the gallon it would get. He replied that it
would get “between 30 and 35 MPG in the city and 40 to 45 on the highway.” Delighted, she bought the car. The very best the
car ever did, even in highway driving, was 27 MPG, and Portia was upset. When she threatened a lawsuit, the dealership
pointed out the following three clauses in the contract she had signed that it relied on to avoid liability. This contract said
nothing about miles per gallon of gas. In your opinion is there any way around these clauses?
 The parol evidence rule is the problem; in fact, §2-316(1) codifies it as a problem. Nonetheless, some courts have simply
ignored the parol evidence rule language in §2-316(1) and permitted recovery on the warranty; others have deemed the
disclaimer fraudulent and therefore ineffective; others have held it unconscionable; still others have found that both parties
did not intend for the writing to be a “final expression of their agreement” (§2-202) and admitted the warranty.
 Of course, some courts have barred the evidence and enforced the merger clause, though such cases are rare where
consumer goods are the subject of the sale. In the cited federal case from Iowa, the court enforced an integration clause
that stated no other representations were made, but this was a business-to-business sale. The court stated that any oral
warranties that were made did not become part of the basis of the bargain because the integration clause said so. Hence
the oral statements were, technically, not disclaimed, but by failing to be part of the basis of the bargain never rose to the
status of a warranty at all.
• §2-316(1) Words or conduct relevant to the creation of an express warranty and words or conduct tending to negate or
limit warranty shall be construed wherever reasonable as consistent with each other; but subject to the provisions of
this Article on parol or extrinsic evidence (Section 2-202) negation or limitation is inoperative to the extent that such
construction is unreasonable.
• §2-202 Terms with respect to which the confirmatory memoranda of the parties agree or which are otherwise set forth
in a writing intended by the parties as a final expression of their agreement with respect to such terms as are included
therein may not be contradicted by evidence of any prior agreement or of a contemporaneous oral agreement …
(1) “This is the entire contract, and there are no other matters agreed to by the parties that are not contained herein.” Would this
clause be more effective in a business contract to exclude oral statements made during negotiations on the theory that the
written disclaimer keeps such oral matters from being part of the “basis of the bargain”? See Lincoln Sav. Bank v. Open
Solutions, Inc., 2013 WL 997894 (N.D. Iowa 2013).
 If Portia can find a way around the parol evidence rule, the express warranties survive the attempted disclaimer.
(2) “There are no other express or implied warranties except those contained herein.”
 The implied warranty of merchantability was not properly disclaimed because merchantability was not mentioned by name.
Even if it had been, query whether this disclaimer is conspicuous?
• §2-316(2) Subject to division (3) of this section, to exclude or modify the implied warranty of merchantability or any part
of it the language must mention merchantability and in case of a writing must be conspicuous, and to exclude or modify
any implied warranty of fitness the exclusion must be by a writing and conspicuous. Language to exclude all implied
warranties of fitness is sufficient if it states for example, that "There are no warranties which extend beyond the
description on the face hereof."
(3) “No salesperson has the authority to give express warranties other than those contained herein.” (As to this last clause, see
White & Summers §13-4, and the brief reference to the matter in Official Comment 2 to §2-316.)

51
 Finally, Portia might have major trouble with the “lack of authority” clause, which White & Summers say should protect the
seller if the clause is conspicuous.
 I have my doubts about this: courts should be slow to say that meaningful statements (about the gas mileage) are
meaningless simply because a clause in the contract says to ignore the salesperson’s representations. Particularly where
consumers are being misled is such a clause of dubious legal value.
 Canvas Quiz Questions
o In general, it is very difficult to disclaim an express warranty because the U.C.C. provides that if a warranty and a disclaimer
cannot be reasonably construed as consistent with each other, the disclaimer is inoperative.
 True
o In general, there are only two ways for a seller to avoid liability for an express warranty. First, don't make an express warranty.
Second, use the Parol Evidence Rule: make sure that the written contract is the final expression of the terms of the contract,
don't include any express warranties in the written contract, and conspicuously disclaim all express warranties.
 True
o It is possible for a seller to disclaim liability for bad faith, fraud, unconscionability, or other similar principles of law and equity.
 False - No it is not possible for a seller to disclaim liability for violation of such principles.
II. Disclaiming Implied Warranties
 The Disclaimer of Implied Warranties Is Subject to Two Subsections of the U.C.C.
o The implied warranties of merchantability and fitness may be disclaimed either under Section 2-316(2) or under Section 2-
316(3).
o The basic idea of §2-316, is that sellers are able to exclude or disclaim implied warranties BUT
1. to exclude or modify the implied warranty of merchantability or any part of it the language must mention merchantability
and in case of a writing must be conspicuous;
2. to exclude or modify any implied warranty of fitness the exclusion must be by a writing and conspicuous;
3. an implied warranty can also be excluded or modified by course of dealing or course of performance or usage of trade;
4. Remedies for breach of warranty can also be limited.
• And remember that a warranty of title cannot be excluded or disclaimed under §2-316 but only under §2-312(2) by
specific language or by circumstances which give the buyer reason to know that the person selling does not claim title
in himself or that he is purporting to sell only such right or title as he or a third person may have.
 Subsection 2-316(2) on Disclaiming the Implied Warranties of Merchantability and Fitness
o Under Section 2-316(2) a disclaimer of the warranty of merchantability need not be in writing. However, the disclaimer must
mention the term “merchantability” and if it is in writing it must be conspicuous.
o A disclaimer of the warranty of fitness must be in writing and must be conspicuous. The warranty of fitness may be disclaimed
with general language such as “There are no warranties which extend beyond the description on the face hereof.”
 Section 2-316(2): Disclaiming the Implied Warranties of Merchantability and Fitness
Subject to subsection (3), to exclude or modify the implied warranty of merchantability or any part of it the language must mention
merchantability and in case of a writing must be conspicuous, and to exclude or modify any implied warranty of fitness the exclusion
must be by a writing and conspicuous. Language to exclude all implied warranties of fitness is sufficient if it states, for example, that
“There are no warranties which extend beyond the description on the face hereof.”
o Language to exclude all implied warranties of fitness is sufficient if it states, for example, that "There are no warranties which
extend beyond the description on the face hereof."
 Subsection 2-316(3) on Disclaiming the Implied Warranties of Merchantability and Fitness
o Under Section 2-316(3) the implied warranties of merchantability and fitness may be disclaimed:
1. By using the words “as is,” “with all faults,” or similar commonly understood terms; or
2. If an examination of the goods ought to have revealed the defect and before entering the contract the buyer examined the
goods as fully as the buyer desired or the seller required, the buyer to examine the goods; or
3. By course of performance, course of dealing or usage of trade.
 Section 2-316(3): Disclaiming the Implied Warranties of Merchantability and Fitness
Notwithstanding subsection (2)
(a) unless the circumstances indicate otherwise, all implied warranties are excluded by expressions like “as is”, “with all faults” or
other language which in common understanding calls the buyer's attention to the exclusion of warranties and makes plain that
there is no implied warranty; and
(b) when the buyer before entering into the contract has examined the goods or the sample or model as fully as he desired or has
refused to examine the goods there is no implied warranty with regard to defects which an examination ought in the
circumstances to have revealed to him; and
(c) an implied warranty can also be excluded or modified by course of dealing or course of performance or usage of trade.
 Comment to 2-316 on “Refusal to Examine the Goods”
In order to bring the transaction within the scope of "refused to examine" in paragraph (b), it is not sufficient that the goods are
available for inspection. There must in addition be a demand by the seller that the buyer examine the goods fully. The seller by the

52
demand puts the buyer on notice that he [the buyer] is assuming the risk of defects which the examination ought to reveal. The
language "refused to examine" in this paragraph is intended to make clear the necessity for such demand.
 Notes on UCC §2-316
o The seller is protected under §2-316(1) against false allegations of oral warranties by its provisions on parol and extrinsic
evidence and against unauthorized representations by the customary "lack of authority" clauses.
o UCC §2-316(4) treats the limitation or avoidance of consequential damages as a matter of limiting remedies for breach, separate
from the matter of creation of liability under a warranty.
o If no warranty exists, there is of course no problem of limiting remedies for breach of warranty. Under subsection (4) the
question of limitation of remedy is governed by the sections referred to rather than by this section.
 Notes on UCC §2-316(2)
o Disclaimer of the implied warranty of merchantability is permitted under subsection (2), but with the safeguard that such
disclaimers must mention merchantability and in case of a writing must be conspicuous.
o Unlike the implied warranty of merchantability, implied warranties of fitness for a particular purpose may be excluded by
general language, but only if it is in writing and conspicuous.
o Subsection (2) presupposes that the implied warranty in question exists unless excluded or modified. Whether or not language
of disclaimer satisfies the requirements of this section, such language may be relevant under other sections to the question
whether the warranty was ever in fact created.
o Thus, unless the provisions of this Article on parol and extrinsic evidence prevent, oral language of disclaimer may raise issues of
fact as to whether reliance by the buyer occurred and whether the seller had "reason to know" under the section on implied
warranty of fitness for a particular purpose.
 Notes on UCC §2-316(3)
o The exceptions to the general rule set forth in paragraphs (a), (b) and (c) of subsection (3) are common factual situations in
which the circumstances surrounding the transaction are in themselves sufficient to call the buyer's attention to the fact that no
implied warranties are made or that a certain implied warranty is being excluded.
o Paragraph (a) deals with general terms such as "as is," "as they stand," "with all faults," and the like. Such terms in ordinary
commercial usage are understood to mean that the buyer takes the entire risk as to the quality of the goods involved.
o Under paragraph (b) of subsection (3) warranties may be excluded or modified by the circumstances where the buyer examines
the goods or a sample or model of them before entering into the contract. "Examination" as used in this paragraph is not
synonymous with inspection before acceptance or at any other time after the contract has been made. It goes rather to the
nature of the responsibility assumed by the seller at the time of the making of the contract.
o Under paragraph (b) of subsection (3) warranties may be excluded or modified by the circumstances where the buyer examines
the goods or a sample or model of them before entering into the contract. "Examination" as used in this paragraph is not
synonymous with inspection before acceptance or at any other time after the contract has been made. It goes rather to the
nature of the responsibility assumed by the seller at the time of the making of the contract.
o In order to bring the transaction within the scope of "refused to examine" in paragraph (b), it is not sufficient that the goods are
available for inspection. There must in addition be a demand by the seller that the buyer examine the goods fully. The seller by
the demand puts the buyer on notice that he is assuming the risk of defects which the examination ought to reveal. The
language "refused to examine" in this paragraph is intended to make clear the necessity for such demand.
o The common law doctrine of "caveat emptor" in all cases where the buyer examines the goods is rejected. The particular
buyer's skill and the normal method of examining goods in the circumstances determine what defects are excluded by the
examination. But a failure to notice defects which are obvious cannot excuse the buyer.
o The situation in which the buyer gives precise and complete specifications to the seller is not explicitly covered in this section,
but this is a frequent circumstance by which the implied warranties may be excluded.
o The warranty of fitness for a particular purpose would not normally arise since in such a situation there is usually no reliance on
the seller by the buyer.
o The warranty of merchantability in such a transaction, however, must be considered in connection with UCC §2-317 on the
cumulation and conflict of warranties. See next slide.
o Under §2-317(c), in case of such an inconsistency the implied warranty of merchantability is displaced by the express warranty
that the goods will comply with the specifications. Thus, where the buyer gives detailed specifications as to the goods, neither of
the implied warranties as to quality will normally apply to the transaction unless consistent with the specifications.
 UCC §2-317
Warranties whether express or implied shall be construed as consistent with each other and as cumulative, but if such construction
is unreasonable the intention of the parties shall determine which warranty is dominant. In ascertaining that intention the following
rules apply:
(a) Exact or technical specifications displace an inconsistent sample or model or general language of description.
(b) A sample from an existing bulk displaces inconsistent general language of description.
(c) Express warranties displace inconsistent implied warranties other than an implied warranty of fitness for a particular
purpose.
 Cate v. Dover Corp.
53
o The section boldly entitled WARRANTY contained an unobtrusive exclusion of warranty—which the court held ineffective,
because exclusions of the implied warranty of merchantability must be CONSPICUOUS.
o The case also hits on a couple of other issues. The majority holds that an inconspicuous disclaimer may be effective if the buyer
has actual knowledge of it, while some see that as contrary to both the letter and policy of §2-316.
o Another concurrence argues that warranty disclaimers should be made ineffective by the Texas legislature, because buyers
generally do not understand the disclaimer or have no other choice. Massachusetts has adopted this position for sales to
consumers in its version of the UCC, as have a couple of other states (but not Pennsylvania).
 Casebook Problem
(a) A statement buried in the fine print of a used car purchase agreement states that “There are no express or implied warranties
that are part of this sale.” See §§2-316(2), 1-201(b)(10).
(1) Are the implied warranties effectively disclaimed?
 Fine print disclaimers are of course not conspicuous and therefore are ineffective. A conspicuous disclaimer would not get
rid of express warranties (assuming a way can be found around the parol evidence rule); §2-316(1). Under the Magnuson-
Moss Act the implied warranty disclaimer would also be ineffective
(2) If the car dealership asks you to redraft this clause so as to comply with the Code, what changes would you make in the
language? See the extended discussion, particularly in the dissent, in Hicks v. Superior Court, 8 Cal. Rptr. 3d 703 (Cal. App.
2004).
(3) What changes would you make in the physical appearance of the clause in the contract? Is it all right to put the disclaimer in a
clause labeled WARRANTY? See Benedict v. Hankook Tire Co. Ltd., 295 F. Supp. 3d 632 (E.D. Va. 2018); Agropur, Inc. v. Scoular
Company, 2017 WL 3411944 (D. Minn. 2017) (disclaimer in contract section titled “CONTAMINATION & GUARANTEE”).
 The clause should be made conspicuous and labeled as a warranty disclaimer. The South Carolina case held that a clause
labeled “Warranty” is deceptive if it really is a warranty disclaimer. If the Magnuson-Moss Act applies, the implied warranty
may not be disclaimed at all, and certain other mandatory disclosures must be made; 16 C.F.R. §701.3, a matter to be
discussed later. The cited California case is well worth reading for its broad policy discussions in both the majority opinion
and the dissent about allowing implied warranties to be disclaimed by boilerplate language. The California Supreme Court
originally granted review of this case, but dropped it when the parties reached a settlement.
(4) Can the car dealer win the legal dispute by arguing that usage of trade (§1-303(c)) permits the burial of warranty disclaimers
in the fine print? (For an annotation collecting the automobile warranty disclaimer cases, see Annot., 54 A.L.R.3d 1217.)
 The usage of trade argument is not convincing in my opinion, particularly since the usage of trade is changing as a result of
various state and federal statutes and/or regulations. It’s hardly right to have a usage of trade that says the consumer is
used to being screwed by standard boilerplate language. I doubt if fine print burial of contract terms is ever going to be
justified as usage of trade when the Code has express provisions designed to outlaw (or at least penalize) this practice. The
parties may not adopt a usage of trade that is contrary to legal requirements.
(b) The words AS IS are written with soap in large letters across the front windshield of the used car. See §2-316(3)(a). Is this
effective to disclaim implied warranties? Express warranties? See Woodard v. Labrada, 2017 WL 3309765 (C.D. Cal. 2017);
Annot., 24 A.L.R.3d 465. Must the “as is” language be conspicuous? See §2-316(3)(a); Lumber Mut. Ins. Co. v. Clarklift of Detroit,
Inc., 224 Mich. App. 737, 569 N.W.2d 681 (1997); R.J. Robertson, Jr., A Modest Proposal Regarding the Enforceability of “As Is”
Disclaimers of Implied Warranties: What the Buyer Doesn’t Know Shouldn’t Hurt Him, 99 Com. L.J. 1 (1994). Would a
conspicuous statement of AS IS on a new recreational vehicle exclude the implied warranty of merchantability? See Chaudoin v.
Thor Motor Coach, Inc., 2017 WL 3485803 (E.D. Mich. 2017). Certainly it is clear that “As Is” will not protect p. 179a seller from
tort theories such as fraud. Sorchaga v. Ride Auto, LLC, 893 N.W.2d 360 (Wis. App. 2017).
 “AS IS” on the front windshield would (if the car were shown to the buyer) probably get rid of the implied warranties,
though not the express ones. Almost all (but not all) courts have agreed that the “AS IS” must be conspicuous to be
effective even though the Code does not say this; the cited Michigan case so holds.
 Professor Robertson’s article makes a persuasive case that an inconspicuous AS IS disclaimer should be effective if the buyer
actually saw it, since the buyer’s subjective knowledge appears to be the only relevant criterion in the actual language of
§2-316(3)(a).
 An “As Is” disclaimer will not get rid of express warranties, as the cited Luig holds; §2-316(3)(a) only applies to implied
warranties.
(c) The car salesman asks the buyer, “Would you like to examine the car?” and the buyer, who is in a hurry, says, “No.” Effective
disclaimer? See §2-316(3)(b); see also Official Comment 8.
 The inspection-type disclaimer arises under §2-316(3)(b) only where the seller has made a demand that buyer inspect; see
Official Comment 8, second paragraph.
 Here the seller has only asked a question, and that is not enough. I once bought a suit from a merchant who informed me
that it was store policy for the customer to try on the wares before the sale was made; that is the sort of statement that
would avoid liability for any defects a reasonable examination would have found.
 Also note the different inspection standards expected of a professional buyer as contrasted with a layman [an auto
mechanic buying a car should observe more than, say, a banker]. In any event, inspection vel non will not affect the express
warranties.

54
(d) Remember Ted Traveler (Problem 21), who walked into the men’s room of the bus depot and bought an expensive watch? We
decided there was no warranty of title in that transaction. However, a warranty of quality is a separate question. Are there
implied warranties in this sale? See §2-316(3)(c).
 There are no implied warranties in a sale in a washroom simply because common understanding (usage of trade?) tells us
that the buyer is not likely to ever see the seller again; the sale is necessarily “as is.”
 Casebook Problem
o If your client is the manufacturer of products and asks you to draft up contracts for the sale of the company’s products, should
you recommend disclaiming warranties? You could do this by making no express warranties except vague puffs — “This product
is outstanding!” — and carefully disclaim all implied warranties, mentioning “merchantability” conspicuously. Is this a good idea
either from a business or legal standpoint? Be prepared to discuss this in class.
 Because it is possible to disclaim warranties (and because courses like this one teach lawyers how to do it) there is an
obvious temptation to disclaim everything when selling a product. But both the seller and the seller’s attorney should stop
and consider what that really means. If there are no warranties then the risk that the product is not worth its price and will
completely fall apart lands on the buyer (who may fail to appreciate this burden in evaluating the deal). What does it say
about the product and its quality if there is no warranty at all? What does it say about the seller? “Well, okay,” the seller
may argue, “I’ll make an express warranty but I don’t want to make any implied warranties.” Hmm. Why not?
 The implied warranty of merchantability was created by the drafters to reflect basic fairness in the sale of a product. Look at
the list of its elements in §2-314 and ask if these are unreasonable burdens or merely obvious things a product should do.
 There are actually some sales contracts where sellers make an express warranty of merchantability, and good for them.
Those products are worth buying.
 As for the implied warranty of fitness for a particular purpose it also describes a situation (obvious reliance on the seller’s
expertise) which the seller can control by care in the negotiation process and/or the training of employees to be careful
when advising buyers with special needs for the item being sold beyond its normal use.
 The point here is that a seller who stands by its product can protect itself from difficulties the product causes buyers by care
in making the product and by procuring insurance against the damages it may cause.
 This might actually be a better business decision than trying to transfer the risks of product nonperformance to the hapless
buyer, and then bearing the expenses of defending lawsuits about attempted disclaimers of quality problems the seller
caused.
 Practice Pointer: Just because disclaimer is possible doesn’t mean it should always be used!
 Casebook Problem
o Joe College bought a new car from Flash Motors, relying on the seller’s extravagant claims about the car’s superior qualities. He
signed a purchase order on August 1, and the car was delivered two weeks later. In the glove compartment he found the
warranty booklet and on reading it was dismayed to learn that the actual written warranty was very limited in coverage. Is he
bound by the written warranty’s terms? What argument can he make? See Comment 7 to §2-313; §2-209; White & Summers
§13-5, at 582-583.
 The courts used to have no hesitancy in saying that a warranty limitation found in a later- delivered booklet is ineffective to
modify the terms expressly or impliedly agreed upon in the offer and acceptance process. A post-deal disclaimer, like the
one found later in the glove compartment, is not effective.
 The first case that follows in the text also reaches this result. However, the case after that reflects a disturbing trend to
change all of the above, under the “layered contracts” (sometimes called “rolling contracts”) theory favored by Judge
Easterbrook in ProCD and Hill v. Gateway (both cited in the case).
 Bowdoin v. Showell Growers, Inc.
o Bowdoin simply holds that a post-sale disclaimer is not effective because it did not form a part of the basis of the bargain
between the parties to the sale. Nor could it be made a part of contract by a prior course of dealing between the parties,
especially where they had only one prior transaction. This was the traditional common law result. What could be more obvious!
o … And then came the Seventh Circuit to muddy the waters.
 Rinaldi v. Iomega Corp.
o Rinaldi reflects a disturbing trend to change the traditional result, under the “layered contracts” (sometimes called “rolling
contracts”) theory favored by Judge Easterbrook in ProCD and Hill v. Gateway (both cited in the case). This theory is that the
contract between the manufacturer and the buyer at retail is not formed until the buyer has a chance to review any hidden
terms and assent to the same by continued use of the product (a second layer of contract on top of the one that has been
generated by the retail purchase).
o This is certainly a startling idea, and has all sorts of difficulties, legal and practical. First of all, the buyer deals with the retailer,
who sometimes will not take the product back (particularly software that has had its shrinkwrap removed), saying things like
“You bought it, there’s nothing wrong with it, there’s nothing we can do.” Secondly, the buyer is not in privity with the
manufacturer.
o As well, all the studies have shown that buyers rarely return products, even if unhappy with them. They have so much already
invested in the transaction that it is often easier to live with the difficulty than to package the product back up, ship it to the
manufacturer, and then hope that some remote flunky will promptly refund all the expenses involved.

55
 Distinguish Bowdoin from Rinaldi
o One major difference might be the emphasis on when the contract came into existence.
o If, as in Bowdoin, the offer and acceptance process was over before the disclaimer was made, the seller loses.
o But where the contract is not formed until the disclaimer is part of it, as arguably happened in Rinaldi, then the disclaimer is
effective since it is not first proffered post-sale.
o When do you think the contract is formed?
o Consider:
o Does not §2-207 come into play here?
o Who is the offeror and who is the offeree? Is not the shrink-wrap a written confirmation containing additional or different
terms?
o If sold to a consumer, do these additional or different terms become part of the agreement of the parties?
o Do consumers expressly agree to these terms by accepting and not rejecting the goods?
o What are reasonable consumer expectations?
o What if the consumer never reads the shrink-wrap T&C or if in the Internet of Things your printer orders supplies without your
involvement and the seller has posted online warranty disclaimers? Are those part of the contract?
 Wilson Trading Corp. v. David Ferguson, Ltd
o Limitations on the Warranty
o Wilson deals with two limits on seller’s ability to limit remedies.
o Under §2-719, the contract may limit remedies, such as providing that no damages are available, and the only remedy is repair
or replacement.
o But, if circumstances cause the remedy “to fail of its essential purpose, remedy may be had as provided in this Act.”
o In addition, consequential damages may be limited, unless the limitation is unconscionable (a much lighter restriction).
o In Wilson, the contract gave express warranties about yarn sold, but limited remedies to replacement and only if notice was
given before the yarn was processed. Latent defects, however, could not be detected until the yarn was processed. So, the
remedy failed of its essential purpose. That made the Article 2 remedies available. The court held the remedies made available
included consequential damages, despite the separate limitation on consequential damages.
 Canvas Quiz Questions
o Please assume that Zach purchased a new set of wooden lawn furniture from Boomtown, a large discount store. The furniture
was of very poor quality, with lots of splinters and even some visible rot. Please identify whether the following disclaimers of the
implied warranty of merchantability would be effective.
 The salesperson orally stated to Zach, "The warranty of - Yes, this would be an effective
merchantability is disclaimed." There was no written contract. disclaimer of the implied warranty
of merchantability.
 There was a written contract. It contained a clause stating, "There are - No, this would not be an effective
no warranties, express or implied, in connection with the sale of this disclaimer of the implied warranty
product." This language was not conspicuous. of merchantability.
 There was a written contract. It contained a clause stating, "There are - No, this would not be an effective
no warranties, express or implied, in connection with the sale of this disclaimer of the implied warranty
product." This language was conspicuous. of merchantability.
 A large red sign hanging on the furniture as displayed in the store - Yes, this would be an effective
stated "AS IS." Buyer pointed to the display and said to a salesperson, disclaimer of the implied warranty
"I want to buy this set of lawn furniture." of merchantability.
 A salesperson said to Zach, "It's store policy that you must inspect the - Yes, this would be an effective
furniture before purchasing it. Please take as long as you like." Zach disclaimer of the implied warranty
said, "No, I'd rather not." of merchantability.
 Zach inspected the furniture and said to the salesperson, "I think I can - Yes, this would be an effective
fix this up." disclaimer of the implied warranty
of merchantability.
 Zach had purchased many items from Boomtown over the years and - Yes, this would be an effective
was well aware of the store's policy that the store never made any disclaimer of the implied warranty
warranties, express or implied, about the quality of the goods it sold. of merchantability.
 There was a written contract. It contained a clause stating, "There are - Yes, this would be an effective
no warranties, express or implied, in connection with the sale of this disclaimer of the implied warranty
product. There is no warranty of merchantability." This language was of merchantability.
conspicuous.
o Barbara, who lives in Duluth, Minnesota, went to the paint store and told the salesperson, "I am going to paint the exterior of
my house here in Duluth and I need to purchase 15 gallons of paint. Could you help me?" The salesperson sold Barbara 15
gallons of an excellent brand of paint - except for the fact it was just not suitable for house exteriors in the cold, wet climate of
Duluth. Would the following language be effective to disclaim the implied warranty of fitness for a particular use?
56
 The salesperson said to her, "There are no warranties which - No, this language would not be an effective
extend beyond the description of this product as 'paint.'" disclaimer of the warranty of fitness for a
particular purpose.
 The written contract stated, "There are no warranties which -Yes, this language would be an effective
extend beyond the description on the face hereof." This disclaimer of the warranty of fitness for a
statement was conspicuous. particular purpose.
 The written contract stated, "There are no warranties which -No, this language would not be an effective
extend beyond the description on the face hereof." This disclaimer of the warranty of fitness for a
statement was not conspicuous. particular purpose.
1. Limitations on Recovery for Breach of Warranty
 Article 2 Authorizes Limitations on the Right to Recover for Breach of Warranty
o Section 2-316(4) provides that remedies for breach of warranty can be limited. It incorporates by reference two other code
sections: 2-718 and 2-719.
o Section 2-718 primarily deals with liquidated damages and the buyer’s right to restitution, which will be covered in a later
presentation.
o Section 2-719 primarily deals with modification of or limitation on the buyer’s right to sue for breach of warranty. This
presentation covers Section 2-719.
 Section 2-316(4): Limits to Recover for Breach of Warranty
Remedies for breach of warranty can be limited in accordance with the provisions of this Article on liquidation or limitation of
damages (UCC §2-718) and on contractual modification of remedy (UCC §2-719).
o E.g., the parties can agree on a “liquidated damages” provision to limit either party’s damages (Damages for breach by either
party may be liquidated in the agreement but only at an amount which is reasonable in the light of the anticipated or actual
harm caused by the breach, the difficulties of proof of loss, and the inconvenience or non-feasibility of otherwise obtaining an
adequate remedy. A term fixing unreasonably large, liquidated damages is void as a penalty. §2-718(1))
o E.g., the parties can contractually agree to limit, exclude or modify the type of or the amount of damages (the agreement may
provide for remedies in addition to or in substitution for those provided in this Article and may limit or alter the measure of
damages recoverable under this Article, as by limiting the buyer’s remedies to return of the goods and repayment of the price or
to repair and replacement of non-conforming goods or parts or limiting or excluding consequential damages. §2-719(1)(a), (3).)
 Notes on UCC §2-316(4)
o UCC §2-316(4) treats the limitation or avoidance of consequential damages as a matter of limiting remedies for breach, separate
from the matter of creation of liability under a warranty.
o If no warranty exists, there is of course no problem of limiting remedies for breach of warranty. Under subsection (4) the
question of limitation of remedy is governed by the sections referred to rather than by this section.
 Section 2-718(1): Liquidated Damages
Damages for breach by either party may be liquidated in the agreement but only at an amount which is reasonable in the light of the
anticipated or actual harm caused by the breach, the difficulties of proof of loss, and the inconvenience or non-feasibility of
otherwise obtaining an adequate remedy. A term fixing unreasonably large, liquidated damages is void as a penalty.
o Comment 1 to Section 2-718(1)
Under subsection (1) liquidated damage clauses are allowed where the amount involved is reasonable in the light of the
circumstances of the case. The subsection sets forth explicitly the elements to be considered in determining the reasonableness
of a liquidated damage clause. A term fixing unreasonably large, liquidated damages is expressly made void as a penalty. An
unreasonably small amount would be subject to similar criticism and might be stricken under the section on unconscionable
contracts or clauses.
 Permissible Limitations on Remedies
o Section 2-719(1) provides that an agreement for the sale of goods may limit remedies or alter the measure of damages, for
example in the following ways:
 By limiting the buyer’s remedies to return of the goods and repayment of the price; or
 By limiting the buyer’s remedies to repair and replacement of non-conforming goods and parts.
Subsection 1 also provides that these remedies are to be considered to be optional unless the contract expressly states that
they are exclusive.
 Section 2-719(1): Permissible Limitations on Remedies
Subject to the provisions of subsections (2) and (3) of this section and of the preceding section on liquidation and limitation of
damages,
(a) the agreement may provide for remedies in addition to or in substitution for those provided in this Article and may limit or
alter the measure of damages recoverable under this Article, as by limiting the buyer's remedies to return of the goods and
repayment of the price or to repair and replacement of non-conforming goods or parts; and
(b) resort to a remedy as provided is optional unless the remedy is expressly agreed to be exclusive, in which case it is the sole
remedy.
 Comment 1 to 2-719(1)(a)

57
Under this section parties are left free to shape their remedies to their particular requirements and reasonable agreements limiting
or modifying remedies are to be given effect.
However, it is of the very essence of a sales contract that at least minimum adequate remedies be available. If the parties intend to
conclude a contract for sale within this Article, they must accept the legal consequence that there be at least a fair quantum of
remedy for breach of the obligations or duties outlined in the contract.
Thus, any clause purporting to modify or limit the remedial provisions of this Article in an unconscionable manner is subject to
deletion and in that event the remedies made available by this Article are applicable as if the stricken clause had never existed.
Similarly, under subsection (2), where an apparently fair and reasonable clause because of circumstances fails in its purpose or
operates to deprive either party of the substantial value of the bargain, it must give way to the general remedy provisions of this
Article.
 Comment 2 to 2-719(1)(b)
Subsection (1)(b) creates a presumption that clauses prescribing remedies are cumulative rather than exclusive. If the parties
intend the term to describe the sole remedy under the contract, this must be clearly expressed.
 Two Restrictions on Limited Remedies: “Fail of Its Essential Purpose” and “Unconscionability”
o Subsection 2-719(1), which allows a seller to limit the buyer’s remedies, is subject to two restrictions that are contained in
subsections 2-719(2) and 2-719(3).
o Subsection 2-719(2) strikes down exclusive or limited remedies that “fail of their essential purpose.”
o Subsection 2-719(3) strikes down limitations on recovery for consequential damages that are “unconscionable.”
 An Exclusive or Limited Remedy That “Fails of Its Essential Purpose”
o Subsection 2-719(2) provides that if an exclusive or limited remedy such as a promise to repair or replace a defective product
“fails of its essential purpose” then the limitation is void and the buyer is entitled to pursue other remedies as provided in
Article 2. The Official Comment states that the limitation must provide “a fair quantum of remedy” or provide a party with “the
substantial value of the bargain.”
o The most common situation where this arises is where the contract limits the buyer’s remedy to repair or replacement of the
goods, but the seller is unable to repair a defective product within a reasonable time and refuses to replace it. In such a case the
remedy “fails of its essential purpose” and the buyer may pursue other remedies such as suing for damages.
 Section 2-719(2): An Exclusive or Limited Remedy That “Fails of Its Essential Purpose”
Where circumstances cause an exclusive or limited remedy to fail of its essential purpose, remedy may be had as provided in this
Act.
o Comment 2 to 2-719(2)
Under subsection (2), where an apparently fair and reasonable clause because of circumstances fails in its purpose or operates
to deprive either party of the substantial value of the bargain, it must give way to the general remedy provisions of this Article.
 Unconscionable Limitations on Consequential Damages
o Subsection 2-719(3) provides that a limitation or exclusion on recovery for consequential damages is invalid if it is
unconscionable.
o Limitations on recovery for personal injury resulting from consumer goods are “prima facie” unconscionable.
o Limitations on recovery for commercial loss are not “prima facie” unconscionable.
 Section 2-719(3): Unconscionable Limitations on Consequential Damages
Consequential damages may be limited or excluded unless the limitation or exclusion is unconscionable. Limitation of consequential
damages for injury to the person in the case of consumer goods is prima facie unconscionable but limitation of damages where the
loss is commercial is not.
o Comment 2 to 2-719(3)
Subsection (3) recognizes the validity of clauses limiting or excluding consequential damages but makes it clear that they may
not operate in an unconscionable manner. Actually, such terms are merely an allocation of unknown or undeterminable risks.
The seller in all cases is free to disclaim warranties in the manner provided in Section 2-316.
 The Link Between Subsections 2-719(2) and 2-719(3)
o Assume that a contract contains language limiting the buyer’s remedy to repair or replacement of the goods as well as language
prohibiting recovery for consequential damages.
o Assume further that the seller fails to repair or replace the goods within a reasonable time, so that the limited remedy “fails of
its essential purpose” within the meaning of Subsection 2-719(2).
o Does this mean that the limitation on consequential damages is automatically invalidated? Or is the seller’s failure to timely
repair or replace the goods merely “some evidence” that the limit on recovery for consequential damages is unconscionable?
 Wilson Trading Corp. v. David Ferguson, Ltd
o Wilson deals with two limits on seller’s ability to limit remedies. Under §2-719, the contract may limit remedies, such as
providing that no damages are available, and the only remedy is repair or replacement. If circumstances cause the remedy “to
fail of its essential purpose, remedy may be had as provided in this Act.” In addition, consequential damages may be limited,
unless the limitation is unconscionable (a much lighter restriction). In Wilson, the contract gave express warranties about yarn
sold, but limited remedies to replacement and only if notice was given before the yarn was processed. Latent defects, however,
could not be detected until the yarn was processed. So the remedy failed of its essential purpose. That made the Article 2
58
remedies available. The court held the remedies made available included consequential damages, despite the separate
limitation on consequential damages.
 Casebook Problem
o On November 1, Jack Frost of Portland, Maine, bought a snowmobile from King Cold Recreationland. Jack used the snowmobile
to get to work during the week in the winter and for fun on the weekends. The contract that he signed stated that the seller
warranted that the vehicle was merchantable, but that, in the event of breach, “the buyer’s remedy was limited solely to repair
or replacement of defective parts.” Moreover, the contract conspicuously stated that the seller was not responsible for “any
consequential damages.”
One week after he received the snowmobile, Jack noticed a strange rumble in the engine. He took the machine back to the King
Cold service department. The machine was returned to him in three days allegedly repaired. These events repeated themselves
three times over the next three weeks. Four weeks after he bought the snowmobile, Frost was seriously injured when it blew up
while he was riding it. The machine, which cost $1,200, was destroyed. Frost temporarily lost the use of his left arm, incurred
hospital expenses of $2,500, and lost pay of $1,600. Moreover, when he did return to work, he had to rent a snowmobile for
$40 a week until spring (16 weeks — spring is very late in Maine). In addition, a $350 camera he was carrying was also
destroyed. Frost brought suit against King Cold. King Cold defended on the ground that its liability was limited to the cost of
repair or replacement. Frost argued that the remedy limitation was “unconscionable” and failed of its “essential purpose.” All
the parties pointed to §§2-316(4), 2-302, 2-719, and 2-715. How should this suit come out? See Beal v. General Motors Corp.,
354 F. Supp. 423 (D. Del. 1973); Earl M. Jorgensen Co. v. Mark Constr., Inc., 56 Haw. 466, 540 P.2d 978 (1975).
 The elimination of liability for consequential damages for personal injury in the case of consumer goods (the arm, the
hospital expense, and the lost wages) is ineffective per §2-719(3), and Frost may recover the stated amounts under §2-
715(2)(b) (buyer can recover as consequential damages “injury to persons or property proximately resulting from any
breach of warranty” )...
 The leading case on this is the landmark decision of Henningsen v. Bloomfield Motors, Inc., 32 N.J. 358, 161 A.2d 69, 75
A.L.R.2d 1 (1960). The limitation on recovery of consequential property losses (here the camera) is valid if conscionable.
 Both the cited decisions at the end of the Problem raise the argument that a seller who has refused to repair or honor its
warranty waives his right to insist on the warranty limitations and is thus exposed to consequential damages. As to the “fail
of its essential purpose” argument, see the “minimum adequate remedy” language of Official Comment 1 to §2-719.
 Casebook Problem
o Since you are in charge of drafting the warranty provisions for your manufacturing client’s sales contracts, you have decided to
make some express warranties and disclaim all implied ones. For the express warranties you are thinking about limiting the
buyer’s remedies to just repair and replacement so that (as in the Georgia case just cited) your client at most will have to return
the p. 196purchase price but never pay consequential damages, which could be huge, particularly if there is personal injury
involved. Is this a good idea? Are there moral questions here? Who should bear these types of damages? See Hennings v.
Camping Time RV Centers, LLC, 2017 WL 4552896 (N.D. Ga. 2017).
 Many attorneys drafting sales contracts have done just what is suggested here: make some express warranties, disclaim all
the implied ones, and limit damages to repair and replacement only.
 Section 2-719 places some major restrictions on this: there has to be a “minimum adequate remedy” (the Official Comment
1 test), and it is unclear what that vague language commands. If the product malfunctions and the repair or replacement is
promptly made, well and good. But what if the malfunction causes major consequential damages to the buyer? These can
be as simple as minor property damage to realty or other chattels, or as serious as major hospital expenses or even death to
the hapless buyer.
 Those are likely to be dealt with under (3) of §2-719, where we find an unconscionable test, another vague standard. If the
buyer is a consumer the presumption in that subsection is that personal injury damages would have to be paid. What about
damages to property of the consumer?
 Pierce v. Catalina Yachts, Inc.
o Recent cases on §2-719 focus on the relationship between subsections (2) and (3), asking whether they are “dependent” or
“independent.” The majority authority in recent years, contrary to Wilson Trading above, has, as this court agrees, found the
clauses independent, so that a failure of essential purpose under (2) does not answer the (3) question of unconscionability when
it comes to the issue of disclaiming liability for consequential damages. See, e.g., Kearney & Trecker Corp. v. Master Engraving
Co., 527 A.2d 429 (N.J 1987)
o Here, a limitation on consequential damages (limiting remedy to repair of below- waterline blisters on a sailboat) was held
unconscionable, because of seller’s bad faith in failing to make promised repairs. So conduct long after contract formation may
be considered on the unconscionability question.
o The Razor case mentioned in the Note following the case also found the disclaimer of consequential damages unconscionable in
a car sale where the plaintiff did not even see it until after she had purchased the vehicle.
 Helena Chemical Co. v. Williamson
o Here the disclaimer of liability for consequential damages is also held unconscionable, and, surprisingly, this is a commercial
contract. However, the buyer was an inexperienced farmer buying defective seed that ruined his entire crop, and the courts
have always stretched to protect farmers, particularly those who are not agri-business corporate types.

59
o Compare, Helena Chemical with Commercial Real Estate Investment L.C. v. Comcast of Utah, Inc., 285 P.3d 1193 (Utah 2012)
(applying the Restatement of Contracts §339(1) test of unconscionability to uphold a liquidated damages clause).
 Belinda v. Smithers Tractors
o Belinda, who mows the lawns on several large private estates, purchased a used tractor from Smithers Tractors for $7,500. The
contract drawn up by Smithers stated conspicuously that "The sole and exclusive remedy for any defects in this product is that
seller will repair any defect that appears within three months of purchase or will replace the product." The contract also
provided that "Seller is not liable for any consequential damages beyond injury to the person." During the first two weeks
Belinda brought the tractor in to Smithers four times to fix a faulty transmission. After each repair the tractor worked for a day
or two and then the transmission failed again. Smithers refused to replace the tractor and insisted that Belinda should bring the
tractor back in for another attempted repair. Belinda refused, and instead had the tractor repaired by another dealer. Belinda
incurred $1,500 in expenses for renting another tractor during this period and $2,500 in expenses in having the transmission
repaired by another dealer. What recovery is Belinda entitled to?
o Belinda is entitled to recover $2,500 to repair the tractor, because Smithers breached its undertaking to repair or replace the
tractor in a timely manner. She is not entitled to recover the $1,500 in rental costs because it is barred by the contractual
limitation on recovery of consequential damages, and that clause is not unconscionable.
 Randall v. Cummins and Handyman Depot
o Randall purchased a power saw manufactured by Cummins from Handyman Depot, a large hardware store and lumber yard.
Unknown to both parties the design of the power saw was defective in that it was likely that the blade would come in contact
with the power cord. Both the warranty from the manufacturer and the warranty from the store restricted the buyer's remedy
for defects in the product to a full refund of the purchase price. The contracts also prohibited recovery for any consequential
damages.
o You know what happened next. Using the saw in a manner consistent with the manufacturer's instructions Randall severed the
power cord, giving him a terrible electric shock. The sparking from the severed electrical cord ignited the sawdust by the
workbench. Randall got his family to safety but he suffered serious burns to his hands causing him to miss two weeks of work.
And his house burned down.
Yes, Randall wants his money back for the defective saw ($80). He would also like to recover for his medical bills ($5,000), his
lost income ($2,000), his pain and suffering ($50,000), and the damage to his home ($180,000). The store and the manufacturer
have offered Randall $80, the purchase price of the saw. What is Randall entitled to under the U.C.C.?
Under the U.C.C. Randall is entitled to recovery of the $80 purchase price plus all of his consequential damages including injury
to person and injury to property. In this particular case any limit on the recovery of his consequential damages is
"unconscionable."
 Barton Batteries, Inc. v. Capitol Containers
o Barton Batteries, Inc. purchased a 100,000-gallon holding tank from Capitol Containers to store sulfuric acid at its factory in
West Philadelphia. The cost of the tank was $20,000. The contract between the parties expressly limited Barton's remedy for
any defect in the tank to return of the purchase price and prohibited any recovery for consequential damages. A faulty seal on
the tank broke and the factory was flooded with sulfuric acid, causing $2 million in property damage to Barton's factory and
causing $5 million in lost profits during the time that the factory was shut down. No-one was injured. Under the U.C.C., how
much can Barton recover from Capitol?
o Under the U.C.C. Barton can recover only $20,000, the purchase price of the tank. The remainder of the damages constitute
"commercial loss" and there is nothing unconscionable about an agreement excluding consequential damages for property
damage and lost profits as between two commercial enterprises.
 Hypothetical Problem
Plaintiff K&T was the manufacturer of the MM-180, a complex, computer-controlled machine tool that could perform automatic
machining on metal parts. Plaintiff sold an MM-180 to Master Company (Master) (defendant) for $167,000. The parties' agreement
contained a repair-and-replacement warranty and an exclusion of consequential damages. The MM-180 did not perform as
warranted. After the machine’s warranty expired, Master made a number of service calls to K&T and refused to pay for them. K&T
filed suit to recover the payments, and Master filed a counterclaim to recover lost-profit damages caused by the malfunctions of
the MM-180.
WARRANTY, DISCLAIMER, LIMITATION OF LIABILITY AND REMEDY: Seller warrants the products furnished hereunder to be free
from defects in material and workmanship for the shorter of (i) twelve (12) months from the date of delivery * * * or (ii) four
thousand (4,000) operating hours * * *. * * * * * * * * THE WARRANTY EXPRESSED HEREIN IS IN LIEU OF ANY OTHER WARRANTIES
EXPRESS OR IMPLIED INCLUDING, WITHOUT LIMITATION, ANY IMPLIED WARRANTY OF MERCHANTABILITY OR FITNESS FOR A
PARTICULAR PURPOSE AND IS IN LIEU OF ANY AND ALL OTHER OBLIGATIONS OR LIABILITY ON SELLER'S PART. UNDER NO
CIRCUMSTANCES WILL SELLER BE LIABLE FOR ANY INCIDENTAL OR CONSEQUENTIAL DAMAGES, OR FOR ANY OTHER LOSS, DAMAGE
OR EXPENSE OF ANY KIND, INCLUDING LOSS OF PROFITS ARISING IN CONNECTION WITH THIS CONTRACT OR WITH THE USE OF OR
INABILITY TO USE SELLER'S PRODUCTS FURNISHED UNDER THIS CONTRACT. SELLER'S MAXIMUM LIABILITY SHALL NOT EXCEED AND
BUYER'S REMEDY IS LIMITED TO EITHER (i) REPAIR OR REPLACEMENT OF THE DEFECTIVE PART OF PRODUCT, OR AT SELLER'S
OPTION, (ii) RETURN OF THE PRODUCT AND REFUND OF THE PURCHASE PRICE, AND SUCH REMEDY SHALL BE BUYER'S ENTIRE AND
EXCLUSIVE REMEDY.

60
o § 2-719. Contractual Modification or Limitation of Remedy.
(2) Where circumstances cause an exclusive or limited remedy to fail of its essential purpose, remedy may be had as provided
in this Act.
(3) Consequential damages may be limited or excluded unless the limitation or exclusion is unconscionable. Limitation of
consequential damages for injury to the person in the case of consumer goods is prima facie unconscionable but limitation
of damages where the loss is commercial is not.
 Clauses that try to prevent non-breaching party from recovering all remedies the law would normally provide. They
explicitly:
1. limit non-breaching parties to certain remedies specified in the contract, or
2. exclude certain available common law remedies upon breach.
 Most common kind of limitations clauses:
• Substituted remedies clause – i.e., one that seeks to substitute a certain remedy for those available at common law
– E.g., repair/replace clause in Kearney & Trecker – 2nd part of indented para
• Limitations on consequential damages – attempt to disclaim liability for consequential damages stemming from
breach of contract
– E.g., Kearney & Trecker – 1st part of indented para
o Substituted Remedies Clauses
 Under 2-719(1)(b) substituted remedies clauses are enforceable as the exclusive remedy for breach of contract if two
conditions are met:
1. The parties expressly agree that the substituted remedy is exclusive (2-719(1)(b))
2. The substituted remedies clause does not fail of its essential purpose (2-719(2))
• Why did the substituted remedies clause fail in Kearney & Trecker?
o Buyer’s Remedies When Substituted Remedies Clauses “Fail of Their Essential Purpose”
 Does the “failure of essential purpose” standard really protect non-breaching parties?
• What if both parties to a contract agree to a clause that states: “Buyer agrees to assume the sole risk of loss due to
failure of the machine except that Seller will try to repair in good faith.”
• Isn’t that clause fail proof? Is there any way plaintiff can get around such a problem?
 What remedies are available to P if the substituted remedies clause is found to be unenforceable?
o Limitations on Consequential Damages
 There was also a limitation on consequential damages in K&T. UCC 2-719(3) allows parties to limit consequential damages.
 Courts uphold clauses limiting consequential damages if they are not “unconscionable”
• Unconscionable = whether under circumstances existing at the time K formed in light of the general commercial
background and commercial needs of a particular case, the clause is so one-sided as to be oppressive.
 Unconscionability is a difficult standard to meet.
• Every case depends on the circumstances and evidence pertaining to the general commercial practices, needs of a
particular case and the operation of the clause against P. You really need to pay attention to the circumstances at the
time of contract in such cases.
o What happens when contract contains both clauses and substituted remedies clause fails? (actual issue in Kearney &
Trecker)
 There are two major approaches by the courts:
• Majority approach (maybe only just barely) – If the substituted remedies clause fails, the clause limiting consequential
damages still stands (unless it fails on its own as unconscionable).
– Why this approach?
• Minority approach – Both clauses (substituted remedies & clause limiting consequential damages) fail and are
unenforceable – B can go after all available remedies under the UCC.
– Why this approach?
 Canvas Quiz Questions
o Article 2 does not permit the seller to limit a buyer's remedies to repair and replacement of the goods; or to return of the
purchase price; or to restrict or eliminate recovery for consequential damages other than injury to the person.
 False - Section 2-719 permits contracts for the sale of goods to limit a buyer's remedies to repair and replacement of the
goods; or to return of the purchase price; or to restrict or eliminate recovery for consequential damages other than injury
to the person.
o Belinda, who mows the lawns on several large private estates, purchased a used tractor from Smithers Tractors for $7,500.
The contract drawn up by Smithers stated conspicuously that "The sole and exclusive remedy for any defects in this product
is that seller will repair any defect that appears within three months of purchase or will replace the product." The contract
also provided that "Seller is not liable for any consequential damages beyond injury to the person." During the first two
weeks Belinda brought the tractor in to Smithers four times to fix a faulty transmission. After each repair the tractor worked
for a day or two and then the transmission failed again. Smithers refused to replace the tractor and insisted that Belinda
should bring the tractor back in for another attempted repair. Belinda refused, and instead had the tractor repaired by

61
another dealer. Belinda incurred $1,500 in expenses for renting another tractor during this period and $2,500 in expenses in
having the transmission repaired by another dealer. What recovery is Belinda entitled to?
 Answer: Belinda is entitled to recover $2,500 to repair the tractor, because Smithers breached its undertaking to repair or
replace the tractor in a timely manner. She is not entitled to recover the $1,500 in rental costs because it is barred by the
contractual limitation on recovery of consequential damages, and that clause is not unconscionable. – Smithers’s failure to
repair or replace the tractor made the exclusive and limited remedy "fail of its essential purpose" and deprived Belinda of
"the substantial value of the bargain." Belinda is not entitled to recover the $1,500 in rental costs because it is barred by the
contractual limitation on recovery of consequential damages, and that clause is not unconscionable because this involves
commercial loss, not loss due to injury of the person. However, it could be argued that Smithers' failure to repair the tractor
made the limitation on consequential damages "unconscionable." The outcome would probably turn on whether it was
determined that Smithers had acted in "good faith."
 Belinda is not entitled to recover anything. The contract does not permit her to have the tractor repaired by another dealer,
and the rental costs are consequential damages that are barred by the contract.
 Belinda is entitled to recover the full $4,000. She is entitled to recover $2,500 to repair the tractor, because Smithers
breached its undertaking to repair or replace the tractor in a timely manner. She is also entitled to recover the $1,500 in
rental costs because Smithers' failure to repair the tractor made the limitation on consequential damages
"unconscionable."
o Randall purchased a power saw manufactured by Cummins from Handyman Depot, a large hardware store and lumber yard.
Unknown to both parties the design of the power saw was defective in that it was likely that the blade would come in contact
with the power cord. Both the warranty from the manufacturer and the warranty from the store restricted the buyer's
remedy for defects in the product to a full refund of the purchase price. The contracts also prohibited recovery for any
consequential damages.
You know what happened next. Using the saw in a manner consistent with the manufacturer's instructions Randall severed
the power cord, giving him a terrible electric shock. The sparking from the severed electrical cord ignited the sawdust by the
workbench. Randall got his family to safety, but he suffered serious burns to his hands causing him to miss two weeks of
work. And his house burned down.
Yes, Randall wants his money back for the defective saw ($80). He would also like to recover for his medical bills ($5,000), his
lost income ($2,000), his pain and suffering ($50,000), and the damage to his home ($180,000). The store and the
manufacturer have offered Randall $80, the purchase price of the saw. What is Randall entitled to under the U.C.C.?
 Under the U.C.C. Randall is only entitled to recover the amount that he paid for the saw -- $80.
 Under the U.C.C. Randall is entitled to recover only the $80 as the price of the saw plus $5,000 -- his medical bills resulting
from personal injury due to a defect in consumer goods. The remainder of his losses are "commercial" in nature, and the
limit on recovery for commercial loss is not prima facie unconscionable.
 Under the U.C.C. Randall is entitled to recover only the $80 as the price of the saw plus $5,000 (his medical bills) and $2,000
(his lost income) both of which resulted from personal injury due to a defect in consumer goods. The destruction of his
house is "commercial" in nature, and the limit on recovery for commercial loss is not prima facie unconscionable.
 Answer: Under the U.C.C. Randall is entitled to recovery of the $80 purchase price plus all of his consequential damages
including injury to person and injury to property. In this particular case any limit on the recovery of his consequential
damages is "unconscionable."- It's clear that the limitation on consequential damages for personal injury is unconscionable,
and that would include both Randall's medical expenses as well as his lost income. The question is, is the exclusion of
recovery for commercial loss also unconscionable? In my opinion the limit on consequential damages for injury to property is
"unconscionable" in this case in light of the obvious danger that this kind of defect presents to consumers and their families
in their homes. I don't think that a consumer should have to bear the risk of this type of loss. And in any event as a consumer
Randall could also sue the store and the manufacturer in tort for products liability, and it is likely that he would recover for
all injury to his person and his property.
o Barton Batteries, Inc. purchased a 100,000-gallon holding tank from Capitol Containers to store sulfuric acid at its factory in
West Philadelphia. The cost of the tank was $20,000. The contract between the parties expressly limited Barton's remedy for
any defect in the tank to return of the purchase price and prohibited any recovery for consequential damages. A faulty seal
on the tank broke and the factory was flooded with sulfuric acid, causing $2 million in property damage to Barton's factory
and causing $5 million in lost profits during the time that the factory was shut down. No-one was injured. Under the U.C.C.,
how much can Barton recover from Capitol?
 Answer: Under the U.C.C. Barton can recover only $20,000, the purchase price of the tank. The remainder of the damages
constitute "commercial loss" and there is nothing unconscionable about an agreement excluding consequential damages
for property damage and lost profits as between two commercial enterprises. – The courts would uphold the limitation on
recovery of consequential damages as applied to both property damage and lost profits - that both of these would be
considered "commercial loss" in a contract between two commercial enterprises. But an argument could be made that
damage to property is distinguishable from purely "economic loss."
 Under the U.C.C. Barton should recover $20,000 (the purchase price) plus $2 million (property damage resulting from the
breach). The clause prohibiting recovery for consequential damages is unconscionable as applied to property damage and is
valid only as it relates to "economic loss."
62
 Under the U.C.C. Barton should recover the $20,000 purchase price plus the entire $7 million for damage to property and
lost profits. The clause prohibiting recovery for consequential damages is unconscionable.
D. Defenses in Warranty Actions
1. Notice
 The Requirement of Notice for Breach of Warranty Claims
o Under Section 2-607(3), once the buyer has accepted goods and the buyer discovers that the goods do not conform to the
contract, the buyer must give the seller notice of the breach within a “reasonable time” after the buyer discovers or should have
discovered the defect.
o The courts have identified two reasons for requiring buyers to give sellers notice of a defect in the goods:
(1) To give the seller an opportunity to cure the defect; and
(2) To give the seller an opportunity to settle the matter out of court.
 Section 2-607(3): Notice for Breach of Warranty
Where a tender has been accepted
(a) the buyer must within a reasonable time after he discovers or should have discovered any breach notify the seller of breach or
be barred from any remedy; and
(b) if the claim is one for infringement or the like (subsection (3) of Section 2-312) and the buyer is sued as a result of such a breach
he must so notify the seller within a reasonable time after he receives notice of the litigation or be barred from any remedy over
for liability established by the litigation.
 Example: The Bad Micrometers
o Thoron Tools sold Academy Hardware store fourteen cartons of micrometers that turned out to be defective. Six months later,
without any prior notification to Thoron, Academy filed suit against Thoron for breach of warranty.
o The courts will dismiss Academy’s lawsuit on the ground that Academy failed to give Thoron notice of the breach in accordance
with Section 2-607(3).
 UCC §2-607(3) “Tender”
o What is a “tender”? We could search the UCC for a definition, but …
 §1-201 (general definitions) is silent.
 §2-103 (definitions and index of definitions) is likewise silent.
 From §1-102 to §1-309 (general provisions, definitions, applicability and general rules) – nothing.
 From §2-102 to §2-107 (Article 2 general construction) – silence.
 From §2-201 to §2-210 (Form, formation and readjustment of contract) – crickets.
 From §2-301 to §2-328 (general obligation and construction of contracts) – nada.
 From §2-401 to §2-403 (title, creditors and good faith purchasers) – zip.
o We need to look to Part 5 (§§2-501 – 2-513) – to §2-503 to learn that “tender of delivery requires that the seller put and hold
conforming goods at the buyer’s disposition and give the buyer any notification reasonably necessary to enable him to take
delivery. The manner, time and place for tender are determined by agreement and by this Article …”
o Sometimes called “due tender” it contemplates a party’s offer and a present ability to fulfill all of the conditions and must be
followed by actual performance if the other party shows it is ready to proceed.
o “Tender” connotes performance by the tendering party as puts the other party in default if he fails to proceed in some manner.
o The manner, time and place for tender and delivery are determined by agreement (e.g., F.O.B., F.A.S., C.I.F., C&F) and by the
Article’s gap filler provisions.
 UCC §2-507 Effect of Seller’s Tender and UCC §2-607(1) Effect of Buyer’s Acceptance
o §2-507 outlines the effect of seller’s tender and provides, “tender of delivery is a condition to the buyer’s duty to accept the
goods and, unless otherwise agreed, to his duty to pay for them. Tender entitles the seller to acceptance of the goods and to
payment according to the contract.”
o Thus, where seller tenders conforming goods, buyer has a duty to accept the goods* and pay for them. §2-607(1) provides, “the
buyer must pay at the contract price for any goods accepted.”
o Which begs the question, “what constitutes acceptance of the goods”?
 Subject to buyer’s right to reject non-conforming goods
 Section 2-606: What constitutes acceptance of the goods?
(1) Acceptance of the goods occurs when the buyer
(a) After a reasonable opportunity to inspect the goods signifies to the seller that the goods are conforming or that he will take
or retain them in spite of their non-conformity; or
(b) Fails to make an effective rejection [under §2-602(1)], but such acceptance does not occur until the buyer has had a
reasonable opportunity to inspect them; or
(c) Does any act inconsistent with the seller’s ownership; but if such act is wrongful as against the seller it is an acceptance only
if ratified by him.
o So, what happens when Seller tenders non-conforming goods? Does Buyer have rights?
 Section 2-601: Buyer’s Rights on Improper Delivery

63
Subject to [§§2-612, 2-718, 2-719], if the goods or the tender of delivery fail in any respect to conform to the contract, the buyer
may
(a) Reject the whole; or
(b) Accept the whole; or
(c) Accept any commercial unit or units and reject the rest
o §2-602 outlines the manner of a rightful rejection, providing “Rejections of goods must be within a reasonable time after their
delivery or tender. It is ineffective unless the buyer seasonably notifies the seller.”
o So, §2-606, §2-602 and §2-607(3) play together
 Compare UCC §§2-606, 2-602, 2-607(3)
o §2-606 Acceptance of the goods occurs when the buyer (a) After a reasonable opportunity to inspect the goods signifies to the
seller that the goods are conforming or that he will take or retain them in spite of their non-conformity; or (b) Fails to make an
effective rejection [under §2-602(1)]
o §2-602 outlines the manner of a rightful rejection, providing “Rejections of goods must be within a reasonable time after their
delivery or tender. It is ineffective unless the buyer seasonably notifies the seller.”
o §2-607(3) Where a tender has been accepted
(a) the buyer must within a reasonable time after he discovers or should have discovered any breach notify the seller of
breach or be barred from any remedy…
o So, what is a “reasonable time” and what constitutes adequate “notice”?
 Comment 4 to §2-607(3)(a): What is a Reasonable Time?
The time of notification is to be determined by applying commercial standards to a merchant buyer. “A reasonable time” for
notification from a retail consumer is to be judged by different standards so that in his case it will be extended, for the rule of
requiring notification is designed to defeat commercial bad faith, not to deprive a good faith consumer of his remedy.
o What are “commercial standards”? §1-303 provides for express terms, course of performance, course of dealing and usage of
trade.
o §1-205 provides, “whether a time for taking action required by the [UCC] is reasonable depends on the nature, purpose, and
circumstances of the action.”
o §1-302 provides, that “whenever the [UCC] requires an action be taken within a reasonable time, a time that is not manifestly
unreasonable may be fixed by agreement.”
o §1-304 “imposes an obligation of good faith in (the) performance and enforcement” of every contract and duty.
 UCC §2-607(3)(a) What is Adequate Notice?
o The content of the notification need merely be sufficient to let the seller know that the transaction is still troublesome and must
be watched.
 No need for buyer to use “magic language” or claim a reservation of rights.
 No need for buyer to specify all objections to the goods. Compare §2-605 (statement of defects upon rejection).
 No need for a formal claim for damages or threatened litigation.
 “Notice” under this section to preserve buyer’s rights must only inform the seller that the transaction is claimed to involve a
breach.
 UCC §2-607(3)(a) Summary
o So, if seller has tendered the goods and buyer has had an opportunity to inspect the goods and thereafter fails to make an
effective rejection under §2-602(1) (“Rejections of goods must be within a reasonable time after their delivery or tender. It is
ineffective unless the buyer seasonably notifies the seller”) …
o Under §2-606(1), Buyer has accepted the goods (Acceptance of the goods occurs when the buyer … Fails to make an effective
rejection under §2-602(1), but such acceptance does not occur until the buyer has had a reasonable opportunity to inspect
them) … and …
o Under §2-607(3)(a), Where a tender has been accepted, the buyer must within a reasonable time after he discovers or should
have discovered any breach notify the seller of breach or be barred from any remedy.
 UCC §2-607(3)(a) Note
o So, in a warranty action, a buyer loses all UCC rights if there is a failure to give the seller notice of the breach within a
reasonable time after the breach should have been discovered.
o Why such a drastic result?
o To preserve for the seller the right to inspect the goods (§2-515) and the right to cure (§2-508).
o And, to facilitate an early settlement of the dispute.
 Casebook Problem
o Pearl, a farmer, exhibited to Dave samples of her apples, but said that the bulk of the apples had less color and were one-fifth
smaller in size than the samples. Dave said, “Bring your apples to my warehouse; such size apples are worth $3 a bushel, and I
will pay you that for them.” Pearl agreed to do so.
The next day Pearl delivered 150 bushels of apples to Dave’s warehouse. These apples were not as good, on an average, as the
samples and were one-third smaller in size than the samples were. Dave, without inspecting the apples, delivered them ten days
later to his commission merchant, who the same day sold them on the market, bringing only $1.50 per bushel.
64
The commission merchant, immediately upon making the sale, called Dave and informed him of the price brought by the apples.
Dave was disgusted and decided to wait until Pearl billed him for the apples, at which time he would give her a piece of his
mind. Sixty days later Pearl billed Dave in the amount of $450 for the 150 bushels of apples. Dave refused to pay, telling Pearl
that the apples had not measured up to the contract. Pearl sued Dave. Dave contended that Pearl breached an express warranty
under the Uniform Commercial Code since a contract of sale by sample was involved. What result? Review §2-313.
o Did Pearl make an express warranty? By description? By sample?
o Did Pearl “tender” delivery?
o Were the apples conforming to the contract?
o Was the express warranty breached?
o Did Dave accept the delivery? How?
o Did Dave have an opportunity to inspect the apples? Did he inspect the apples?
o Did Dave notify Pearl of the breach? If so, when?
o Does Dave have a viable warranty action?
 Fitl v. Strek
o Take-away: Notice must be given within a “reasonable” time of when buyer discovered or should have discovered the breach of
warranty—which gives courts flexibility to consider the facts of the particular case.
o Fitl held notice given some two years after delivery of an altered collectible baseball card to be reasonable where the buyer
relied on the expertise of the dealer seller, had put the card in a safe deposit box during the interim, and gave notice promptly
after learning of the alteration.
o Based on Fitl, would your answer to Problem 39 change?
o Maybe Fitl should be trading assets instead on WAX Blockchain https://on.wax.io/wax-io/
 Casebook Problem
o Icarus Airlines ordered 40 new airplanes from the Daedalus Aircraft Company. Twenty were to be delivered on May 8 and the
rest on November 10. The first shipment actually came on September 11, but Icarus did not complain. The second came on
January 12 of the next year. On January 30 the president of Icarus wrote Daedalus that “[w]e are very disappointed by your late
shipment, which has caused us much expense and inconvenience.” Three months later Icarus sued, claiming some $24 million in
damages caused by the delayed deliveries. In its answer Daedalus responded by stating that it had received no notice of the
breach as to the first shipment and that the notice concerning the second shipment was defective because it did not announce
Icarus’s intention to claim a breach as a result of the late delivery. At trial, Icarus countered by stating that no notice is required
where, as here, the breach is obvious to the seller. As to the lack of formal notice of breach in the January letter, Icarus pointed
to Official Comment 4 to §2-607.
o As the trial court judge, how would you rule on the notice issues? See Aqualon Co. v. MAC Equipment, Inc.; Peavey Electronics
Corp. v. Baan U.S.A., Inc. (noting split among courts and holding that merchant did not give notice by merely giving notice of
problems, because notice requirement “exists to prevent commercial bad faith, promote mitigation and cure, enable the seller
to collect evidence while still fresh, and promote settlement”).
o What if on January 30 Icarus had filed suit against Daedalus rather than waiting three months? Would the notice requirement
have been satisfied? See Armco Steel Co. v. Isaacson Structure Steel Co., 611 P.2d 507 (Alaska 1980). In Armco, the court held
that filing suit was not sufficient notice. However, the dissenting judge said: “[F]iling suit without prior notice may be impolite
but it is not deceptive or dishonest and it certainly is no hindrance to ‘normal settlement through negotiation.’ Often, in fact,
serious settlement negotiations do not take place until a lawsuit is filed.” See also In re Bridgestone/Firestone, Inc. Tires
Products Liability Litigation. Some courts have held that consumers can give adequate notice by filing suit, particularly in
personal injury actions (see Buzadzhi v. Bexco Enter., Inc.), but others have not relaxed the law for consumers with complaints
alleging only financial losses (see In re Carrier IQ, Inc.,).
o Is notice required if the seller is already aware of the nonconformity on the delivery date? See Muir v. Nature’s Bounty, Inc.
 The Aqualon court quoted Learned Hand for this proposition: The plaintiff replies that the buyer is not required to give
notice of what the seller already knows, but this confuses two quite different things. The notice “of the breach” required is
not of the facts, which the seller presumably knows quite as well as, if not better than, the buyer, but of buyer’s claim that
they constitute a breach. The purpose of the notice is to advise the seller that he must meet a claim for damages, as to
which, rightly or wrongly, the law requires that he shall have early warning.
 Are the courts are right in requiring a notice of breach for late deliveries (Cf. the policy reasons—alerting the seller to the
seriousness of the breach, permitting planning and/or settlement, avoiding potential damages, etc.)?
 Official Comment 4 to §2-607 is ambiguous on the issue of whether the notice must threaten a lawsuit.
 Practice Pointer: In any event, the case highlights a practical step that lawyers would do well to remember: clients
complaining to a seller about defective goods are best advised to use the word “breach.”
 This does not mean that further performance must cease and litigation be the inevitable next step. In installment contracts
§2-609 (the right to adequate assurance of future performance) may be useful; see also §1-308 (performance under
reservation of rights).

65
 As for the filing of a lawsuit as sufficient notice, the courts have generally agreed that this is insufficient, though the cited
federal Indiana case did not require notice prior to the filing of a class action lawsuit where the defendants were well aware
of the problem before the complaint was served.
 Practice Pointer: But filing suit without any other notice is risky; no attorney who had thought through the issue would skip
over the giving of a more formal notice prior to suit.
 Casebook Problem
o Alonso Quijano invited his good friend Sancho to dinner and served him a pheasant for the meal. The wine specially uncorked
for the meal had been bottled by La Mancha Vineyards. It proved to be laced with a poisonous chemical, but only Sancho drank
enough to have a serious reaction: it put him in the hospital for eight months. When he was discharged, he hired an attorney
and filed suit against La Mancha Vineyards, which defended on the lack of notice required by §2-607(3)(a). How should this
come out? Read §2-318 and Official Comment 5 to §2-607. If the person who had been injured had been Alonso Quijano and if
he had given notice of breach to the retail seller, Carrasco Liquors, would that preserve his rights against the manufacturer, La
Mancha Vineyards? See Felice v. Invicta Watch Co. of America.
 As far as Sancho goes, he can potentially rely on §2-318 (Third Party Beneficiaries of Warranties Express or Implied), which
provides (in Alternative A, as enacted in Pennsylvania), “A seller’s warranty whether express or implied extends to any
natural person who is in the family or household of his buyer or who is a guest in his home if it is reasonable to expect that
such person may use, consume or be affected by the goods and who is injured in person by breach of the warranty. A seller
may not exclude or limit the operation of this section.”)
 Official Comment 5 to §2-607 says that notice must be given by a §2-318 beneficiary.
 However, in spite of this Official Comment, most courts have not required third party beneficiaries to give notice.
 As far as Alonso goes, if he had given notice of breach to the retail seller, would it preserve his rights against the
manufacturer?
 The best argument here is that when Alonso sues the manufacturer, he will do so as a third-party beneficiary of the
warranties made by the manufacturer to the retailer, as so is excused from the notice requirement by the same reasoning
as courts would apply to Sancho.
 If he is suing on an express warranty that the manufacturer made to him (through advertising, for example), the lack of
notice is harder to excuse, and may well bar his suit.
 Since he gave notice to the retailer, the latter should vouch in the manufacturer by giving a §2-607(5) notice; (Where the
buyer is sued for breach of a warranty or other obligation for which his seller is answerable over… he may give his seller
written notice of the litigation. If the notice states that the seller may come in and defend and that if the seller does not do
so he will be bound in any action against him by his buyer by any determination of fact common to the two litigations, then
unless the seller after seasonable receipt of the notice does come in and defend, he is so bound).
 The cited ACE American case held that a consumer’s notice to the retailer was sufficient notice to the manufacturer, noting
that the retailer is in the best position to identify the manufacturer and others who may be liable over, and also that the
UCC only requires the buyer to give notice to “his” seller. Not all courts agree, but that is the majority rule.
2. Burden of Proof
 The Burden of Proof in Breach of Warranty, Generally
o In a warranty action, Plaintiff has the burden of proving:
 Creation of the warranty
 Breach
 Actual causation
 Proximate causation
 Injury.
 The Burden of Proof in Breach Implied Warranty of Merchantability
o Under Section 2-607(4), once the buyer has accepted the goods, the buyer has the burden of proving breach of warranty. In
addition to satisfying requirements such as “notice” and “privity,” if the buyer claims that the seller breached the implied
warranty of merchantability, the buyer must prove:
(1) That a valid contract for the sale of goods was entered into;
(2) That the goods that were delivered and accepted were covered by an implied warranty of merchantability;
(3) That the goods were not “merchantable” within the meaning of Section 2-314;
(4) That the defect in the goods proximately caused damage to the plaintiff; and
(5) The measure of damages.
 Section 2-607(4): Burden of Proof in Breach of Warranty
The burden is on the buyer to establish any breach with respect to the goods accepted.
 Casebook Problem
o Richard Sharpe loved to eat the hamburgers at Wendy’s and routinely dined at the same franchise three times a week, never
going to a different Wendy’s in the two months before he was rushed to a hospital with trouble breathing. A small piece of a
Wendy’s fork was found in his lungs. He sued the local Wendy’s franchise owner for breach of the warranty of merchantability.
Is he going to have causation problems? See Manley v. Doe.

66
 Without showing some specific defect that caused the damage, Sharpe cannot meet the burden of proof.
 Note that some breaches of warranty are easier to prove. Where a product fails to meet promised standards of
performance, it would not be necessary to show why it did not live up to that warranty.
 In the cited case, based on the facts in the Problem, the court stated that it was not clear that the fork came from a
Wendy’s simply because it was a piece of a Wendy’s fork, and moreover the plaintiff had eaten at other Wendy’s in the past
and might have picked up one from there even some time ago.
 There are other causation issues: how do we know the fork piece was in the food—couldn’t he have bitten it off himself
when he was using the fork?
 Flippo v. Mode O’Day Frock Shops of Hollywood
o A brown recluse spider bit Gladys Flippo while she was trying on stretch pants in Rosie Goforth’s ladies clothing store, Mode
O’Day Frock Shops of Hollywood.
o Ms. Flippo proceeded to buy the pants, which triggered the implied warranty of merchantability.
o Her injury worsened.
o Her action for breach of warranty failed, because she could not meet the burden of showing that the spider was part of the
product or even show when the spider appeared.
o Cruelly, the court notes (in Fn. 2) that the spider could have been brought into the store by the plaintiff herself.
3. Privity
 Privity
o Warranty actions are actions on the contract, requiring privity (there is a “connection” between the parties – their contractual
relationship).
o But if Manufacturer sells to Wholesaler who sells to Retailer who sells to Buyer, there is lack of privity between Manufacturer
and Buyer. No vertical privity, but there is also horizontal privity.
o §2-318 (Third Party Beneficiaries of Warranties Express or Implied) provides (in Alternative A, as enacted in Pennsylvania), “A
seller’s warranty whether express or implied extends to any natural person who is in the family or household of his buyer or
who is a guest in his home if it is reasonable to expect that such person may use, consume or be affected by the goods and who
is injured in person by breach of the warranty. A seller may not exclude or limit the operation of this section.”)
 The Requirement of Privity in Breach of Warranty Claims
o Under the common law of contracts, a plaintiff could not sue any person for breach of contract unless the plaintiff had been in a
contractual relation (“privity”) with that defendant. For example, someone who purchased an item from a store could not sue
the manufacturer for breach of warranty (lack of vertical privity), and if someone other than the buyer was injured by the
product, they could not sue the store (lack of horizontal privity). The law began to change when the claim of “strict tort –
products liability” began to be recognized by the courts. Under the law of tort manufacturers and sellers were increasingly held
liable to any person who might foreseeably be injured by a product.
o As a result, the law of contract has begun to change as well. In certain circumstances – for example, in cases involving personal
injuries caused by defective products – Article 2 permits lawsuits for breach of warranty against sellers or manufacturers even
though the defendants did not sell the product directly to the injured person.
 Three Alternatives on Privity
o In light of all the ferment and disagreement over the law of privity, the drafters of the U.C.C. created three alternative versions
of the law of privity for states to choose from.
o Alternative A
 The most restrictive alternative on privity is Section 2-318(A). In addition to the buyer, it permits lawsuits by any natural
person who is injured in person by the product and who is in the family or household of the buyer or who is a guest of the
home if it would have been reasonable to expect that such a person would be affected by the product. More than half the
states have adopted Alternative A.
o Alternative B
 Section 2-318(B) would permit any natural person who is injured in person by the product to sue for breach of warranty so
long as it would have been reasonable to expect that such a person would be affected by the product.
 Alternative B differs from Alternative A in that the plaintiff need not have been a member of the buyer’s household or a
guest in the buyer’s home.
o Alternative C
 Under Section 2-318(C) would permit any natural person who is injured by the product to sue for breach of warranty so
long as it would have been reasonable to expect that such a person would be affected by the product.
 Section 2-318(C) differs from Alternative B in that it eliminates privity for all kinds of damages – not just for injuries to the
person.
 Section 2-318: Privity in Breach of Warranty
Alternative A
A seller's warranty whether express or implied extends to any natural person who is in the family or household of his buyer or who is
a guest in his home if it is reasonable to expect that such person may use, consume or be affected by the goods and who is injured in
person by breach of the warranty. A seller may not exclude or limit the operation of this section.

67
Alternative B
A seller's warranty whether express or implied extends to any natural person who may reasonably be expected to use, consume or
be affected by the goods and who is injured in person by breach of the warranty. A seller may not exclude or limit the operation of
this section.
Alternative C
A seller's warranty whether express or implied extends to any person who may reasonably be expected to use, consume or be
affected by the goods and who is injured by breach of the warranty. A seller may not exclude or limit the operation of this section
with respect to injury to the person of an individual to whom the warranty extends.
Are the Privity Rules of Section 2-318 Exclusive?
The doctrine of privity has been further diminished by the decisions of some courts interpreting Section 2-318 as non-exclusive; that
is, these courts have ruled that other parties, such as workers who are injured by products purchased by their employers, or patients
who are injured by products purchased a hospital, are entitled to sue the manufacturers of those products for breach of the implied
warranty of merchantability.
 Casebook Problem
o When Alan Turing needed a new computer, he went to Big Sale Department Store and bought the Enigma 1000, the best-selling
computer on the market, made by Code Manufacturing Company. The computer worked p. 218well for a few months, but then
had a spectacular electrical event that turned it into a useless piece of machinery and caused a fire, burning up much of Turing’s
office. Turing sued both Big Sale and Code Manufacturing, but the latter filed a motion for dismissal for lack of privity since it
had no direct dealings with Alan Turing. Code Manufacturing had made an express warranty that the computer would be defect
free, but this was only on a paper contained in the box that the computer came in. Code Manufacturing also argued that any
implied warranty of merchantability was only made to the retailer, Big Sale, with which it had direct dealings, and not to Turing,
with whom it did not. The trial court bought these arguments and dismissed Code Manufacturing from the lawsuit. Is this right?
Compare Hyundai Motor America, Inc. v. Goodin, (no privity needed to sue manufacturer) with Curl v. Volkswagen of America,
Inc., (manufacturer’s implied warranty to retailer did not create necessary privity with buyer). Suppose you are the attorney for
Big Sale. You are annoyed that your client really did nothing wrong except pass on a defective product from the manufacturer
but is nonetheless liable (due to a technicality — lack of privity — that this state stupidly adopts) to the consumer. Is there
anything you can do to make Code Manufacturing interested in defending this lawsuit even if privity would otherwise protect it?
See §2-607(5)(a).
 Courts’ ruling that privity protects a manufacturer from the buyer at retail, like the Ohio decision, are maddening. They
cause the bringing of two lawsuits when one would suffice, and make the retailer, who did not create the defective product,
liable to the buyer, and then have to bring a separate lawsuit, often in another state. Particularly where the manufacturer
gave an express warranty to the buyer should the implied warranty carry through to the retail buyer.
 We will return to this issue when we get to the Magnuson-Moss Act.
 In the meantime, the retailer should give the “vouching-in” notice of §2-607(5) so that the manufacturer is alerted to the
wisdom of joining the lawsuit and making all the arguments that might win it—failing to join means that the manufacturer
is bound by any common factual determinations decided in the first lawsuit and can’t relitigate them when pursued by the
retailer.
 Reed v. City of Chicago
o Horrific facts.
o Take-away: Reed shows that, in Illinois at least, a party may sue for breach of warranty who has not bought the product.
o The Illinois court held that privity is not a strict requirement. As the person who actually used the product, Reed was expected
to be affected by the product, indeed would be the only one who would suffer direct injury from breach of warranty.
o The parties to the sales contract would have assumed that the ultimate user would be the plaintiff (a detainee after arrest, who
killed himself using the “paper isolation gown” intended to be safe for possibly suicidal subjects).
4. Strict Products Liability
 The Differences Between the Implied Warranty of Merchantability and Strict Tort Products Liability
o Claims based on the implied warranty of merchantability and strict tort products liability are very similar in their essential
nature. Both types of claims seek redress for damages caused by defective products, and in many cases the plaintiff will sue the
defendant on both theories.
o But there are also many differences between these two legal rights. They originate in different areas of the law. Different types
of damages may be awarded as a remedy. There are several defenses that can be used to defeat a breach of warranty claim that
do not apply to claims for strict tort products liability. And different statutes of limitations apply to each type of claim.
 The Field of Law from Which Each Type of Claim Springs
o Claims for breach of the implied warranty of merchantability are based upon the law of contract. Today these claims are
governed by Article 2 of the Uniform Commercial Code.
o The law of strict tort products liability is based upon the law of tort. It is derived from Section 402A of the Restatement (Second)
of Torts, which says a defendant is subject to strict liability for injuries caused by an unreasonably dangerous defective product
only if the defendant is engaged in the business of selling the product.

68
 Section 402A, Restatement (Second) of Torts: SPECIAL LIABILITY OF SELLER OF PRODUCT FOR PHYSICAL HARM TO USER OR
CONSUMER
(1) One who sells any product in a defective condition unreasonably dangerous to the user or consumer or to his property is subject
to liability for physical harm thereby caused to the ultimate user or consumer, or to his property, if
(a) the seller is engaged in the business of selling such a product, and
(b) it is expected to and does reach the user or consumer without substantial change in the condition in which it is sold.
(2) The rule stated in Subsection (1) applies although
(a) the seller has exercised all possible care in the preparation and sale of his product, and
(b) the user or consumer has not bought the product from or entered into any contractual relation with the seller.
 Recoverable Damages
o As between a buyer and a seller, damages for breach of the implied warranty of merchantability are potentially quite broad:
unless limited by contract, the plaintiff may recover damages for all losses proximately caused by the defective product,
including expectancy damages for the diminution in value of the product itself, consequential damages for injury to person or to
other property, and incidental damages for other expenses incurred as a result.
o Damages for strict tort products liability are much more limited. Under this theory of law plaintiffs may recover damages only
for injury to a person and injury to property other than the product that caused the damage. There is no recovery for “economic
loss,” that is, for destruction of the product itself or for lost profits.
 Several Defenses to Breach of Warranty Are Not Applicable to Strict Tort Products Liability
o Actions for breach of the implied warranty of merchantability are subject to the normal defenses that apply to all lawsuits for
breach of contract:
 Lack of Timely Notice of Breach – U.C.C. Section 2-607(3)
 Lack of Privity – U.C.C. Section 2-318
 Contractual Disclaimer of the Warranty – U.C.C. Section 2-316
 Contractual Limitations on Recovery for Breach – U.C.C. Section 2-719
o None of these are valid defenses to a claim for strict tort products liability.
 Statute of Limitations
o The period of limitations for breach of warranty is measured differently than the period of limitations for strict tort products
liability. The period of limitations for breach of warranty under the U.C.C. is usually four years from the date that the goods
were delivered, while the period of limitations for strict tort products liability claims is usually one or two years from the date
that the person was injured by the defective product.
 Other Types of Product Liability Claims
o As noted above, when a person suffers a loss as a result of a defective product, they may sue for breach of the implied warranty
of merchantability and under the theory of strict tort products liability.
o They may also sue sellers or manufacturers for negligence, fraud, breach of an express warranty or the implied warranty of
fitness, or violation of consumer protection laws such as the Magnuson-Moss Warranty Act.
o Each type of claim has different elements, defenses, and statutes of limitation.
 Lincoln Company v. Detroit Diesel Corporation General Insurance
o The Supreme Court of Tennessee, like many courts, follows East River Steamship Corp. v. Transamerica Delaval, Inc., in applying
the “economic loss doctrine.”
o Under the majority version of the rule adopted in East River, as the Lincoln Company court states, there is no “recovery in tort
when a product damages itself without causing personal injury or damage to other property.” So, if the only damage is to the
product (and not to other property or personal injury) the plaintiff will likely need to use the UCC breach of warranty cause of
action as the cause of action. [or the Magnuson-Moss Act]
o The typical reason the plaintiff did not do so to begin with is that the statute of limitations has run on it, but not on the tort
theory (which begins at the date of discovery, not the date of sale).
 The “Economic Loss” Doctrine
o Whenever one person invades another person’s legal rights harm may occur. That harm may take a variety of forms. The victim
may suffer personal injury. The victim’s property may be damaged. The victim may not receive the benefit it reasonably
expected to gain from the other party’s performance under a contract. Or the victim may suffer “economic loss” as a result of
the conduct of the tortfeasor, seller of a defective and dangerous product, or party who has breached a contract.
o What is “economic loss”? Economic loss usually consists of lost profits, lost customers, or other lost business opportunities.
o In each of these cases the injured party will face certain barriers to recovering damages for “economic loss” from the negligent
tortfeasor, seller of a defective product, seller who breached a contract for the sale of goods, or party who has breached a
contract for providing services. The rules limiting recovery for economic loss are different under the law of torts, the law of strict
tort products liability, the law of contracts generally and the law of sales.
o The “Economic Loss” Doctrine in Tort
 Under the law of negligence, the rule is that if the victim has suffered purely economic loss then the victim cannot recover
damages. A victim of negligence may recover for economic loss only if the victim also suffered injury to person or injury to
property.
69
 The law of strict tort products liability has a rule that is stricter with respect to economic loss. Section 402A of the
Restatement (Second) of Torts provides when harm is caused by a defective product that is unreasonably dangerous the
injured person may recover for injury to person (including lost wages or salary) and for damage to property other than the
defective product itself. However, the majority rule is that recovery for “economic loss” is barred altogether.
o The “Economic Loss” Doctrine in Contract
 When asserting a claim for breach of warranty under the law of sales the buyer is subject to a host of contractual defenses
that do not apply to a claim for negligence or a claim for strict tort products liability.
(A) Did the seller make a warranty that became a term of the contract and if so, was the warranty effectively disclaimed?
(B) If the seller made a warranty and it was not disclaimed was the person who suffered the harm in privity with the
seller?
(C) Did the contract limit the remedy of the buyer to repair or replacement of defective parts or to a refund of the
purchase price?
(D) Did the contract limit or exclude the recovery of consequential damages?
(E) Did the contract establish liquidated damages as the sole remedy for any harm suffered by either party?
(F) Did the contract otherwise regulate the recovery of damages for example by requiring arbitration of disputes,
prohibiting class actions or class arbitrations, identifying the governing law, selecting a forum, or shortening the statute
of limitations? And finally,
(G) Did the seller’s breach of warranty proximately cause harm to the person asserting the breach and did the seller have
reason to know of the circumstances that resulted in economic loss to the buyer?
 Canvas Quiz Questions
o A buyer who has accepted goods and who discovers that the goods do not conform to an express or implied warranty must
give notice to the seller within a reasonable time after discovering the defect, or the buyer will forfeit the claim for breach of
warranty.
 True
o In addition to satisfying requirements such as "notice" and "privity" the plaintiff in a breach of warranty case has the burden
of proof as to all of the elements of the claim. Please place the following elements in their logical order:
1. A valid contract for the sale of goods was entered into.
2. The goods that were delivered and accepted were covered by an implied warranty of merchantability.
3. The goods were not "merchantable" within the meaning of Section 2-314.
4. A defect in the goods proximately caused damage to the plaintiff.
5. The measure of damages.
– As a matter of trial strategy an attorney might very well present this evidence in a different order. But from a legal
and logical standpoint I think that this order makes the most sense.
o Which of the following statements regarding the law of privity under Article 2 are correct?
 Section 2-318 of the "Uniform" Commercial Code provides three alternatives governing the law of privity for breach of
warranty.
 The State of Pennsylvania has adopted Alternative A, the most restrictive of the three alternatives under Section 2-318.
 Under Alternative A, a seller's warranty extends not only to the buyer but to any natural person in the family or the
household of the buyer or who is a guest in the home who is injured in the person by the product, if it would have been
reasonable to expect that such a person would be affected by the product.
 The requirement of privity has been abolished in breach of warranty cases under Article 2.
o Please identify whether each of the following legal principles applies to a claim for breach of the implied warranty of
merchantability, a claim for strict tort products liability, or both.
 The plaintiff must give reasonable notice - A claim for breach of the implied warranty of
of the breach to the defendant or the merchantability.
claim is barred.
 The claim arises out of the law of tort. - A claim based on strict tort products liability.
 Any person who is foreseeably injured by the product - A claim based on strict tort products liability.
may assert a claim; there is no requirement of privity.
 The claim may be barred or limited by proper - A claim for breach of the implied warranty of
contractual provisions that constitute a disclaimer, that merchantability.
provide a limited remedy such as return of the purchase
price, or that bar the recovery of consequential
damages.
 The claim provides a potential remedy to many persons - Both a claim for breach of the implied warranty of
who have been injured in person by a defective product. merchantability and a claim based on strict tort
products liability.
 This claim may be used to recover damages for the - A claim for breach of the implied warranty of
destruction of the product itself -- even if there is no merchantability.

70
other injury to person or property.
E. Magnuson-Moss Act
 The Scope of Magnuson-Moss
o The Magnuson-Moss Warranty Act applies to “written warranties” for “consumer products.”
o This federal consumer protection law supplements the Uniform Commercial Code, and where the two laws conflict, the
Magnuson-Moss Act preempts the U.C.C.
 Written Warranties
o “Written warranties” include any written affirmation of fact or promise about the material or workmanship of a consumer
product, as well as any undertaking to refund, repair, or replace the product. (Section 101(6)).
 Section 101(6): Written Warranties
The term “written warranty” means—
(A) any written affirmation of fact or written promise made in connection with the sale of a consumer product by a supplier to a
buyer which relates to the nature of the material or workmanship and affirms or promises that such material or workmanship is
defect free or will meet a specified level of performance over a specified period of time, or
(B) any undertaking in writing in connection with the sale by a supplier of a consumer product to refund, repair, replace, or take
other remedial action with respect to such product in the event that such product fails to meet the specifications set forth in the
undertaking, which written affirmation, promise, or undertaking becomes part of the basis of the bargain between a supplier
and a buyer for purposes other than resale of such product.
 Example: A Promise to Repair Is a “Written Warranty” Under Magnuson-Moss
o Fred purchases a used car that is covered by a disclaimer of all warranties, but which comes with a written promise to repair any
and all defects within the first 60 days.
o The promise to repair would not constitute an “express warranty” under the U.C.C. because it does not relate to the goods, but
it is a “written warranty” within the meaning of Magnuson-Moss.
 Example: An Oral Express Warranty Is Not a “Written Warranty” Under Magnuson-Moss
o Alan purchased a television set from a discount store. The store manager orally assured Alan that the television was brand new
and was in perfect working order. In fact, the television was used and defective.
o Under the U.C.C. the store manager’s statements constitute an express warranty, but Magnuson-Moss does not apply because
these statements were not in writing.
 Consumer Products
o Magnuson-Moss only applies to written warranties for “consumer products.”
o Under the U.C.C., a product is “consumer goods” if it was purchased or used for personal, family, or household purposes. Under
Magnuson-Moss, “consumer products” are goods that are normally used for personal, family, or household purposes.
 Section 101(1): Definition of “Consumer Product”
o The term “consumer product” means any tangible personal property which is distributed in commerce and which is normally
used for personal, family, or household purposes (including any such property intended to be attached to or installed in any real
property without regard to whether it is so attached or installed).
 Example of a Consumer Product
o Sharon purchases a laptop computer to use at the counter of the restaurant she owns. The laptop comes with a written
warranty. The computer is defective, and Sharon wishes to assert her rights under the Magnuson-Moss Warranty Act.
o Under the U.C.C., the laptop would not be considered to be “consumer goods” because it wasn’t purchased or used for
personal, family, or household purposes. However, the laptop would be considered to be a “consumer product” within the
meaning of the Magnuson-Moss Warranty Act because this is a product that is “normally” used for personal, family, or
household purposes. Accordingly, Sharon would have rights against the seller or the manufacturer under Magnuson-Moss.
 Required Form of Written Warranties
o Under Magnuson-Moss:
1. Written warranties on consumer products must be in simple and readily understood language (Section 102(a)); and
2. Written warranties on consumer products must be conspicuously designated as “full” or “limited” warranties (Section
103(a)).
 Required Content of Full Warranties
o Under Magnuson-Moss, full written warranties on consumer products:
1. May not disclaim any implied warranties such as the warranty of merchantability or warranty of fitness. (Section 108(a));
2. May not disclaim consequential damages for breach of any written or implied warranty except by conspicuous language on
the face of the warranty (Section 104);
3. Must at a minimum promise to remedy a defect within a reasonable time and without charge (Section 104); and
4. After a reasonable number of repair attempts, must permit the consumer to elect a refund or a replacement without charge
(Section 104).
 Limited Warranties

71
o Under Magnuson-Moss sellers and manufacturers are not required to give any warranties for the products that they sell.
Moreover, sellers and manufacturers may offer “limited warranties” instead of “full warranties.” Limited warranties are not
required to make the same set of promises as full warranties.
o The statutory policy assumed that purchasers would prefer “full warranties” and that products with “limited warranties” would
not be as commercially viable. That has not occurred.
o There are some limitations on the content of “limited warranties.” They may not disclaim implied warranties of merchantability
or fitness, but they may limit the duration of the implied warranties to the duration of the limited warranty, so long as it is at
least a “reasonable time.”
 Duration of Written Warranties
o Under Magnuson-Moss full and limited warranties may be limited to a certain period of time.
 For example, a warranty may be entitled “Full One-Year Warranty” or a “Limited One-Year Warranty.”
 The duration may be for any period of time, but no shorter than a “reasonable” period of time.
 What Magnuson-Moss Doesn’t Do
o Under Magnuson-Moss:
1. Sellers are not required to give buyers any warranties, express or implied;
2. If a seller does not give a written warranty, then the seller is free to disclaim implied warranties;
3. Magnuson-Moss does not apply to oral warranties; it neither permits nor prohibits them.
 Predicates to Litigation Under Magnuson-Moss
o Before suing under Magnuson-Moss, a customer must give the warrantor the opportunity to cure the defect. (Section 110(e)).
o In addition, before suing the customer must follow any approved informal dispute settlement procedure that may be
incorporated into the terms of the warranty. (Section 110(a)).
 Remedies Under Magnuson-Moss
o Under Magnuson-Moss if a customer prevails in litigation the customer is entitled to recover the costs and expenses of litigation
including attorney fees, unless the court finds that an award of attorney fees would be inappropriate. Section 110(d)(2). This
remedy makes it possible for consumers to sue even where their damages are relatively small.
Comparison of Warranty Provisions
UCC Magnuson-Moss
• Creation – oral (§2-313) or written; • Creation – written (Section 101(6)); Sellers are not required to give
Sellers are not required to give buyers buyers any warranties, express or implied.
express warranties, but warranties of • Application – consumer products – and promises to refund, repair or
title are automatic, and warranties of replace
fitness and merchantability are implied. • Required Form - Written warranties on consumer products must be in
• Application – all goods simple and readily understood language (Section 102(a)); on consumer
• Required Forms – N/A products must be conspicuously designated as “full” or “limited”
• Disclaimer of warranties - Seller can warranties (Section 103(a)).
modify or exclude the warranty of title • Disclaimer of warranties - If a seller does not give a written warranty,
under subject to §2-312(2) and disclaim then the seller is free to disclaim implied warranties.
express and implied warranties under §2- • Predicates to Litigation - A customer must give the warrantor the
316. opportunity to cure the defect (Section 110(e); In addition, the customer
• Predicates to Litigation - Buyers are must follow any approved informal dispute settlement procedure that
advised to provide notice of breach to may be incorporated into the terms of the warranty. (Section 110(a)).
sellers under §2-607(3); Filing suit may • Remedies - if a customer prevails in litigation the customer is entitled to
not be sufficient to constitute notice. recover the costs and expenses of litigation including attorney fees,
• Remedies - Sellers can limit remedies unless the court finds that an award of attorney fees would be
under §§2-718, 2-719 inappropriate. Section 110(d)(2)
o Fred purchases an automobile that is covered by a disclaimer of all warranties, but which comes with a written promise to
repair any and all defects within the first 60 days. Is this promise a warranty?
 UCC
• No. This undertaking would not constitute a “warranty” under the U.C.C.
• Why not?
 Magnuson-Moss
• Yes. It is a “written warranty” within the meaning of Magnuson-Moss.
• “Written warranties” include any written affirmation of fact or promise about the material or workmanship of a
consumer product, as well as any undertaking to refund, repair, or replace the product (Section 101(6)).
o Alan purchased a television set from a discount store. The store manager orally assured Alan that the television was brand
new and was in perfect working order. In fact, the television was used and defective.
 UCC
• Under the U.C.C. the store manager’s statements constitute an express warranty.
• Why? What section would you cite in support of this proposition?

72
 Magnuson-Moss
• Magnuson-Moss does not apply because these statements were not in writing.
• The Magnuson-Moss Warranty Act applies to “written warranties” for “consumer products.”
 Canvas Quiz Questions
o The Maguson-Moss Warranty Act
 Is a model act that has been adopted into law by the legislatures of each of the 50 states.
 Answer: Is a federal law that strengthens consumer protections with respect to warranties.
 Is a federal law that weakens consumer protections with respect to warranties.
 Is a Pennsylvania statute that supplants the provisions of the U.C.C. with respect to warranties.
o The Magnuson-Moss Warranty Act applies to the following situations:
 Sales of consumer goods that have a written warranty.
 Answer: Sales of consumer products that have a written warranty.
 Sales of goods that have a written warranty.
 Sales of goods that are covered by a warranty.
o Under Magnuson-Moss, "consumer products" are goods that are "normally" used for personal, family, or household
purposes. Under the U.C.C., "consumer goods" are goods that are actually used or bought for use for personal, family, or
household purposes.
 True
o Under Magnuson-Moss, written warranties on consumer products
 must be in simple and readily understood language.
 must be conspicuously designated as "full" or "limited" warranties.
 may disclaim the implied warranty of merchantability.
 are required.
o Under Magnuson-Moss, a "Full" warranty
 may not disclaim or limit the duration of any implied warranties such as the warranty of merchantability or the warranty of
fitness for a particular use.
 may not disclaim or limit consequential damages for breach of any written or implied warranty except by conspicuous
language on the face of the warranty.
 must at a minimum promise to remedy a defect within a reasonable time and without charge.
 after a reasonable number of repair attempts, must permit the customer to elect a refund or a replacement without charge.
o The Magnuson-Moss Warranty Act
 Does not require sellers to give buyers any warranties.
 Allows sellers to disclaim implied warranties if the seller does not give the buyer a written warranty.
 Does not apply to oral warranties; it neither permits nor prohibits them.
 Permits sellers who do give a written warranty to disclaim the implied warranty of merchantability.
o Before suing under Magnuson-Moss, a customer must give the warrantor an opportunity to cure the defect and must follow
any approved informal dispute settlement procedure that is incorporated into the terms of the warranty.
 True
F. Leases of Goods Under Article 2A
 Scope of Article 2A “Leases”
o $400 Billion worth of lease contracts in the USA in 2018.
o Modeled after Article 2, codified in 1987.
o §2A-102 (Scope) provides, “This Article applies to any transaction, regardless of form, that creates a lease.
o §2A-103(j) provides, “Lease” means a transfer of the right to possession and use of goods for a term in return for consideration,
but a sale, including a sale on approval or a sale or return, or retention or creation of a security interest is not a lease.”
 Sale of Goods or Lease of Goods?
o Sales of goods are governed by Article 2 of the U.C.C. Leases of goods are governed by Article 2A. It does not matter whether
the transaction is called a sale or a lease. What matters is whether the only reasonable outcome of the transaction is that the
buyer/lessee will retain the goods for the economic life of the goods; if so, the transaction is a sale of goods.
o It does not matter whether the transaction is called a sale or a lease. What matters is whether the only reasonable outcome of
the transaction is that the buyer/lessee will retain the goods for the economic life of the goods; if so, the transaction is a sale of
goods.
o The common scope issue is whether a transaction is a true lease (subject to Article 2A) or a sale on credit (subject to Articles 2
and 9), although characterized as a lease by the parties.
o The standards for determining whether a transaction is a sale, or a lease are set forth in Section 1-203.
 Section 1-203
(a) Whether a transaction in the form of a lease creates a lease or security interest is determined by the facts of each case.
(b) A transaction in the form of a lease creates a security interest if the consideration that the lessee is to pay the lessor for
the right to possession and use of the goods is an obligation for the term of the lease and is not subject to termination by the

73
lessee, and: (1) the original term of the lease is equal to or greater than the remaining economic life of the goods; (2) the lessee
is bound to renew the lease for the remaining economic life of the goods or is bound to become the owner of the goods; (3) the
lessee has an option to renew the lease for the remaining economic life of the goods for no additional consideration or for
nominal additional consideration upon compliance with the lease agreement; or (4) the lessee has an option to become the
owner of the goods for no additional consideration or for nominal additional consideration upon compliance with the lease
agreement.
(c) A transaction in the form of a lease does not create a security interest merely because: (1) the present value of the
consideration the lessee is obligated to pay the lessor for the right to possession and use of the goods is substantially equal to or
is greater than the fair market value of the goods at the time the lease is entered into; (2) the lessee assumes risk of loss of the
goods; (3) the lessee agrees to pay, with respect to the goods, taxes, insurance, filing, recording, or registration fees, or service
or maintenance costs; (4) the lessee has an option to renew the lease or to become the owner of the goods; (5) the lessee has
an option to renew the lease for a fixed rent that is equal to or greater than the reasonably predictable fair market rent for the
use of the goods for the term of the renewal at the time the option is to be performed; or (6) the lessee has an option to
become the owner of the goods for a fixed price that is equal to or greater than the reasonably predictable fair market value of
the goods at the time the option is to be performed.
(d) Additional consideration is nominal if it is less than the lessee's reasonably predictable cost of performing under the
lease agreement if the option is not exercised. Additional consideration is not nominal if: (1) when the option to renew the
lease is granted to the lessee, the rent is stated to be the fair market rent for the use of the goods for the term of the renewal
determined at the time the option is to be performed; or (2) when the option to become the owner of the goods is granted to
the lessee, the price is stated to be the fair market value of the goods determined at the time the option is to be performed.
(e) The "remaining economic life of the goods" and "reasonably predictable" fair market rent, fair market value, or cost of
performing under the lease agreement must be determined with reference to the facts and circumstances at the time the
transaction is entered into.
 Section 1-203(a): Practice Pointer
o Courts will pierce through the form of a transaction to determine its substance. You must carefully examine not only the words of
the purported “lease” of goods but look to the substance of the transaction.
o A transaction may be characterized as a “lease agreement” but it may actually be a contract for the sale of goods. You must know
the difference between a “true lease” and a “disguised sale” and the legal consequences of each type of contract.
o If a “lease agreement” is actually a sale of the goods, then the purported “lessee” actually acquires title to the goods and the
creditors of the lessee can seize the goods and resell them. If there is any doubt whether a transaction is a true lease or a disguised
sale the lessor should protect itself by filing a financing statement with the office of the Secretary of State.
 Summary of Section 1-203(b)
o A transaction is a sale if the required term of the lease is longer than economic life of the goods; if the lessee is bound to renew
the lease for the remaining economic life of the goods; or if the lease gives the lessee the option to become the owner or renew
the lease for the remaining economic life of the goods for no or nominal consideration.
 Example of a “Lease” That Is Really a “Sale”
o Frederick leases a large power washer from A-1 Tools for $80 per month for five years. When the “lease” was entered into the
expected life of the power washer was five years; it was expected that the washer would be worth nothing at the end of that
period. Under the lease Frederick does not have the power to cancel the lease; he is required to make the lease payments for
the full five-year period.
o This transaction is actually a “sale” governed by Article 2.
 Another Example of a “Lease” That Is Actually a “Sale”
o Sally leased a large trench digger from Rogers Equipment Company for four years, paying $320 per month. When the lease was
entered into it was expected that the trench digger would have an economic life of six years and that at the end of the four-year
lease term the trench digger would still be worth $5000. The lease provided that Sally could purchase the trench digger for $1 at
the end of the four-year lease term. Under the lease Sally does not have the power to cancel the lease during its four-year term.
o This transaction is actually a “sale” governed by Article 2.
 Example of a “Lease” That Really Is a Lease
o In either of the last two examples, assume that the lessee (Frederick or Sally) had the right to terminate the lease – that is, that
at any time Frederick or Sally could have returned the power washer or the trench digger to the lessor and canceled the lease.
o In that case the transaction would be a “true lease” governed by Article 2A.
 Other “True Leases”
o If at the end of the lease term the goods have a substantial residual value and the lessee either has no right to purchase the
goods, or the lessee does have the option to purchase the goods for an amount that is not nominal in relation to the anticipated
market value of the goods, then the transaction is a true lease subject to Article 2A.
 Casebook Problem
o BIG Machines, Inc., leased a computer to Helen’s Flower Shoppe for a five-year period. The machine was new and had cost BIG
Machines $10,000. Helen’s Flower Shoppe promised to pay $225 a month as rent. Is this a lease or a disguised sale? Is your
answer affected by the following considerations?

74
 Since Helen’s Flower Shoppe will have paid $13,500 as rent for the computer over the life of the lease, it has certainly more
than compensated the lessor for the value of the machine, though this does not answer the key question of whether the
computer will have any remaining economic life at the end of the lease term.
 Infact, §1-203(c)(1) expressly decries use of the fair market value of the goods as a deciding factor. Without more, it is
therefore impossible to say whether this is a true lease or a disguised sale.
(a) The lease provided that the lessee could terminate the lease at any time and return the computer to the lessor.
 If the lessee has a right to terminate at any time, a true lease has been affected. An escape clause is not an attribute of a
typical sale. Under the statutory language this result is reached by referring to the part of the definition excepting leases
“not subject to termination by the lessee.”
(b) Assume there was no such option as described in (a), but the goods had no value at the end of the five-year period.
 If the computer has no value at the end of the term, its economic life has been exhausted and, under the first (a) of the
definition, a disguised sale has occurred.
 One Consequence for the Seller: An Unperfected Security Interest
o If a transaction that is called a “lease” turns out to actually be a “sale,” it is essentially a sale on credit where the seller has the
right to repossess the goods from the buyer if payment is not made. The “lease” is actually a sale with retention of a “purchase
money security interest.”
o Secured transactions are governed by Article 9 of the U.C.C. In order to take priority over other creditors, a secured creditor
must usually “perfect” (that is, give public notice of) its security interest.
o Under Article 9, a purchase money security interest in consumer goods is automatically perfected. However, if the goods are not
consumer goods, the seller must file a financing statement with the appropriate government office to perfect its security
interest. If the seller fails to perfect its security interest in accordance with Article 9 of the U.C.C., the seller will probably lose
priority in the goods to another secured creditor of the buyer, a lien creditor, or a bankruptcy trustee.
o If the seller fails to perfect its security interest in accordance with Article 9 of the U.C.C., the seller will probably lose priority in
the goods to another secured creditor of the buyer, a lien creditor, or a bankruptcy trustee.
 Similarities Between Article 2 and Article 2A
o Article 2A was intentionally drafted to track Article 2, and in most respects the law governing leases of goods is the same as the
law governing the sale of goods.
o For example, under Section 2A-212 the lessor makes an implied warranty of merchantability, and under Section 2A-214 the
warranty may be disclaimed by express language mentioning merchantability or through the use of terms such as “as is” or
“with all faults.”
o A lessor makes the same warranties under Article 2A as a seller under Article 2; the lessor under Article 2A is analogous to a
seller under Article 2. However, this is not true in the case of a “finance lease.”
 Additional Provisions in Article 2A
o Article 2A contains a number of provisions for which there are no analogs in Article 2.
1. In the sections governing “consumer leases” Article 2A contains several protections for consumers. The Pennsylvania
Rental-Purchase Agreement Act also governs consumer “rent-to-own” contracts.
2. In the sections governing “finance leases” Article 2A holds suppliers liable for breach of warranty even though the lessee
was not in privity with the supplier, but releases lessors from responsibility for breach of warranty.
3. Article 2A contains specific provisions governing the lessor’s right to repossess the goods from the buyer. Thus, if the lessee
defaults the lessor may repossess and resell the goods.
4. In measuring monetary remedies, Article 2A grants lessors and lessees the “present value” of their expectancy damages.
Thus, Monetary remedies are reduced to a “present value” of the expected stream of payments.
 1. Consumer Leases
o Consumer leases are defined in Section 2A-103 as leases between a lessor who is regularly engaged in the business of leasing or
selling and a lessee who is an individual and who takes goods under the lease primarily for personal, family, or household
purposes.
o In Pennsylvania, consumer leases do not include leases where the total rent exceeds $25,000.
 13 Pa.C.S.A § 2A103
“Consumer Lease.” A lease that a lessor regularly engaged in the business of leasing or selling makes to a lessee who is an individual
and who takes under the lease primarily for a personal, family or household purpose, if the total payments to be made under the
lease contract, excluding payments for options to renew or buy, do not exceed $25,000.
 Protections Under Consumer Leases
o Article 2A contains a number of protections relating to consumer leases, including the following:
 Choice of law and choice of forum. Under section 2A-106, a consumer lease may not select the law of any jurisdiction other
than where the consumer resides, or any forum to which the consumer is not already subject.
 Unconscionability. Under Section 2A-108, unconscionability includes acts that induce a consumer to enter into a consumer
lease or in collecting a claim. Attorney fees may be awarded for unconscionable actions. Attorney fees may be awarded for
unconscionable actions. The Article 2A drafters have combined parts of §2-302 of the UCC with §5.108 of the Uniform
Consumer Credit Code to produce §2A-108. Unconscionable conduct (so-called “procedural unconscionability”) is now given

75
specific condemnation without a finding of substantive unconscionability, and an award of attorney’s fees is mandated in
the described situations. We still do not have a statutory definition of “unconscionability,” and perhaps that is as well.
 Risk of Loss. There is a strong presumption in §2A-219(1) that the lessor keeps the risk of loss. Absent clear agreement
otherwise (such as a signed lease agreement covering this issue), a court is not likely to transfer the risk to a consumer.
 Casebook Problem
o Dime-A-Minute Rent-A-Car rented a new sports car to Joseph Armstrong. Due to a snafu at the rental office, Armstrong did not
sign a rental agreement. As he was leaving the rental car lot, the car was struck by a city bus due to no fault of Armstrong (who
was unhurt). The sports car was totaled. Dime-A-Minute demanded that Armstrong look to his insurance to replace the car. Did
he have the risk of loss here? See §2A-219. If he had signed a rental agreement making him responsible for the car, would
that agreement be valid? See §§1-302, 2A-108.

 Armstrong had signed a lease giving him the risk of loss, query whether this is unconscionable or not? Actually, our
experience indicates that in car rentals, the consumer customer often agrees to accept the risk of loss!
 Practice Pointer: Make sure your auto insurance covers your use of rental cars before you rent!
 Finance Leases
o “Finance leases” are leases where “the lessor does not select, manufacture, or supply the goods.” These are situations where
the lessee selects a supplier from whom it wishes to obtain goods; the supplier sells the goods to another party (the “lessor”)
who is willing to finance the transaction; and the lessor leases the goods to the lessee.
o Typically, these kinds of leases are used because the party supplying the goods does not wish to lease the goods or extend credit
to the lessee; therefore, a third party is brought into the transaction to provide financing for the lease.
o With a finance lease the lessor is actually analogous to a lender, not a seller. A “finance lease” is analogous to a sale of goods
where the buyer borrows the money from a lender to purchase the goods from a seller. Under a finance lease the “finance
lessor” is not liable under the warranties; instead, the lessee must pay the finance lessor come “hell or high water” and the
lessee must seek any recovery for non-conformity from the supplier of the goods.
o Casebook Problem: Octopus National Bank and Chemicals of Tomorrow – the “hell or high water” clause
 Chemicals of Tomorrow Corp. decided that the safest way to work with hazardous materials was through the use of an
advanced robot. It persuaded Aurora Robotics, Inc., to design a robot that would meet its needs. To finance the purchase of
the robot, Chemicals of Tomorrow went to Octopus National Bank, which bought the robot from Aurora Robotics and then
leased it to Chemicals of Tomorrow. The robot was subject to both express and implied warranties, which became
important when the robot ran amok in one of Chemicals of Tomorrow’s laboratories and caused extensive damage.
• The lease between Octopus National bank and Chemicals of Tomorrow contains a clause stating that the lessee had to
pay the lessor even if the good did not work. See §2A-407
• The benefits to the lessee of the warranties made by the supplier to the lessor are in part offset by the enforcement of
the “Hell or High Water” clause, shielding the lessor from disputes between the supplier and the lessee. Thus, come
hell or high water, Chemicals of Tomorrow is going to have to pay the lease amounts to ONB no matter what the robot
does to its laboratories.
• Note that §2A-407’s Official Comment, first paragraph, states that the section is self-executing. Thus, even if the
contract had not contained an explicit “Hell or High Water” clause, the same legal result is reached.
• The section does not, however, automatically apply to a consumer finance lease. As to the efficacy of such a clause in a
consumer lease, the results would likely vary from state to state and case to case. In the absence of a special consumer
statute on point, the doctrine of unconscionability would be paramount; see §2A-108 for Article 2A’s unconscionability
provision.
o Casebook Problem
 Ambiance Hotel decided to acquire ten horse-carriages to be specially designed to carry its guests around the tourist areas
of the scenic city in which it is located. It had the plans for the carriages transmitted to Buggies, Inc., a carriage
manufacturer, which assured Ambiance that there would be no problem with the creation of the carriages. Ambiance
financed this transaction by having Octopus National Bank purchase the carriages from Buggies, Inc., and then lease them
to Ambiance for a ten-year period. Assume that this transaction qualifies as a finance lease; see §2A-103(1)(g).
(a) If the carriages are delivered to the hotel and Ambiance rejects them because they are the wrong color, must
Ambiance pay the lease amounts to Octopus National Bank? (You may assume that the finance lease contained a “hell
or high water” clause.) See §§2A-407(1), 2A-515.
• The “Hell or High Water” clause kicks into effect on the lessee’s acceptance of the goods; see §2A-407(1).
Assuming the Ambiance Hotel has not yet made an acceptance (as to which see §2A-515), it can avoid its
obligation to Bank.
(b) If the hotel accepts the carriages, but becomes upset when they constantly break down, can it revoke its acceptance
and refuse to pay the lessor? See §§2A-407, 2A-516 and its Official Comment, 2A. See GreatAmerica Fin. Serv. Corp. v.
Meisels.
• Revocation of acceptance in a finance lease can be had against the lessor (Bank here) only if the substantial defect
is in the finance lease itself and not in the goods.

76
• Frankly, I cannot imagine what the drafters mean by a defect in the lease agreement, but whatever that means it
did not happen in the facts of this Problem, so Ambiance is going to have to keep making its payments, while
seeking relief from the supplier.
 The Financer (“Lessor”) Is Not Liable for Breach of Warranty
o The principal legal consequence of a finance lease is that the lessor is not liable to the lessee for any express or implied
warranties. Instead, the supplier is liable to the lessee for breach of warranty, even though the supplier and the lessee are not in
privity with each other. This is governed by Section 2A-209, entitled “Lessee Under Finance Lease as Beneficiary of Supply
Contract.”
 3. Repossession and Re-Lease or Resale
o Section 2A-525 gives the lessor the right to repossess the goods if the lessee defaults.
o Section 2A-527 gives the lessor the right to re-lease or resell the goods in a commercially reasonable manner.
 4. Monetary Remedies for Lessee and Lessor
o The monetary remedies for breach of contract under Article 2A are the same as in Article 2. Like sellers and buyers, lessors and
lessees may claim restitution, expectancy, incidental and consequential damages.
o However, there is one key difference. Under Article 2A in computing expectancy damages a party is entitled to recover only the
“present value” of the expected stream of rental payments from the breaching party.
 Present Value as Measurement for Monetary Remedies
o Under Section 2A-518, if the lessor breaches by failing to deliver conforming goods the lessee may “cover” by leasing
comparable goods from another party. If the rental payments to cover are higher than the contract price, then the lessee is
entitled to the difference between the present value of the stream of rental payments under the cover lease and the present
value of the stream of payments under the original lease.
o Similarly, under Section 2A-527 if the lessee breaches by repudiating or failing to make rental payments then the lessor may re-
lease the goods to another party and recover the difference between the present value of the stream of payments under the
original lease and the present value of the stream of payments under the substituted lease.
 Section 2A-518(2)
Except as otherwise provided with respect to damages liquidated in the lease agreement (Section 2A-504) or otherwise determined
pursuant to agreement of the parties (Sections 1-302 and 2A-503), if a lessee's cover is by a lease agreement substantially similar to
the original lease agreement and the new lease agreement is made in good faith and in a commercially reasonable manner, the
lessee may recover from the lessor as damages (i) the present value, as of the date of the commencement of the term of the new
lease agreement, of the rent under the new lease agreement applicable to that period of the new lease term which is comparable to
the then remaining term of the original lease agreement minus the present value as of the same date of the total rent for the then
remaining lease term of the original lease agreement, and (ii) any incidental or consequential damages, less expenses saved in
consequence of the lessor's default.
 Section 2A-527(2)
Except as otherwise provided with respect to damages liquidated in the lease agreement (Section 2A-504) or otherwise determined
pursuant to agreement of the parties (Sections 1-302 and 2A-503), if the disposition is by lease agreement substantially similar to the
original lease agreement and the new lease agreement is made in good faith and in a commercially reasonable manner, the lessor
may recover from the lessee as damages (i) accrued and unpaid rent as of the date of the commencement of the term of the new
lease agreement, (ii) the present value, as of the same date, of the total rent for the then remaining lease term of the original lease
agreement minus the present value, as of the same date, of the rent under the new lease agreement applicable to that period of the
new lease term which is comparable to the then remaining term of the original lease agreement, and (iii) any incidental damages
allowed under Section 2A-530, less expenses saved in consequence of the lessee's default.
 Computing “Present Value”
o Assume that a lease contract called for payments of $1,000 per year for a period of 5 years for a total of $5,000. Assume further
that the applicable interest rate is 5% annually.
o At the beginning of the first year the present value of the entire stream of rental payments due under the lease contract would
be $4,329.48.
o Under Section 1-201, the parties may specify the interest rate in the lease contract, or, if the rate is not specified, a
“commercially reasonable rate” is used.
 Rent-to-Own Practices and Rental-Purchase Agreement Acts
o Persons who wish to use consumer goods for only a short time may choose to rent rather than to purchase those goods. For
example, a person visiting a city for only a short time may rent a washing machine for $25 per month rather than purchase one
for $400.
o However, many persons who lack money or credit may have to lease rather than purchase goods, and they may enter into
“rent-to-own” transactions. For example, rather than purchase a washing machine for $400 a person may enter into a contract
to rent it $25 per month, and the contract further provides that the person will become the owner of the washing machine after
36 payments, for a total of $900.
o Many states have also adopted Rental-Purchase Agreement Acts that protect consumers from a number of abusive practices
associated with “rent-to-own” contracts.

77
o In states that have enacted these laws, the transaction is subject to both the Rental-Purchase Agreement Act and Article 2A.
 Pennsylvania Rental-Purchase Agreement Act
o The Pennsylvania Rental-Purchase Agreement Act is contained at 42 Pa.C.S.A. §§ 6901-6911. It requires that a rental-purchase
agreement for consumer goods must make many disclosures, including the amount and timing of rental payments; prohibits
many abusive practices, including confession of judgment; provides that the lessee may reinstate the agreement after missing a
payment; requires that the rent be reduced after involuntary job loss, illness, pregnancy, or disability; and regulates
advertisements for rent-to-own transactions, including their accuracy and requiring disclosure of certain terms.
o Most significantly, Section 6905 provides: “The total amount charged by the lessor for the cost of lease services in a rental-
purchase transaction shall not exceed the cash price of the property.”
 Summary of Article 2A
o Article 2A applies to leases of goods. Many provisions of Article 2A are the same as provisions in Article 2, including those
governing the implied warranty of merchantability. Article 2A contains certain provisions that do not appear in Article 2. In
particular, Article 2A contains protections for consumers renting goods under consumer leases as well as protections for lessees
and lessors under finance leases. In addition, expectancy damages under Article 2A are measured by the present value of rental
payments.
Some states including Pennsylvania have enacted additional protections for consumers under Rental-Purchase Agreement Acts,
which supplement Article 2A.
 Detailed Summary of Key Points of Article 2A
1. A transaction may be characterized as a “lease agreement” but it may actually be a contract for the sale of goods. You must
know the difference between a “true lease” and a “disguised sale” and the legal consequences of each type of contract. If a
“lease agreement” is actually a sale of the goods, then the purported “lessee” actually acquires title to the goods and the
creditors of the lessee can seize the goods and resell them. If there is any doubt whether a transaction is a true lease or a
disguised sale the lessor should protect itself by filing a financing statement with the office of the Secretary of State.
2. Article 2A contains some protections for “consumer leases.” The Pennsylvania Rental-Purchase Agreement Act also governs
consumer “rent-to-own” contracts.
3. A lessor makes the same warranties under Article 2A as a seller under Article 2; the lessor under Article 2A is analogous to a
seller under Article 2. However, this is not true in the case of a “finance lease.” With a finance lease the lessor is actually
analogous to a lender, not a seller. A “finance lease” is analogous to a sale of goods where the buyer borrows the money from a
lender to purchase the goods from a seller. Under a finance lease the “finance lessor” is not liable under the warranties; instead,
the lessee must pay the finance lessor come “hell or high water” and the lessee must seek any recovery for non-conformity from
the supplier of the goods.
4. If the lessee defaults the lessor may repossess and resell the goods.
5. Monetary remedies are reduced to a “present value” of the expected stream of payments.
 Review/Assessment Questions
 How does a seller’s status as merchant affect warranties?
 When are express and implied warranties created?
 How can a seller disclaim implied warranties? Express warranties?
 What is the difference between basis of the bargain and reliance?
 What is the consequence of failing to give notice of breach of warranty?
 How does the Magnuson-Moss Act affect warranty law?
 Barnet burglarized Alban’s home and stole a painting, Blue Memphis. Finchley Gallery innocently bought the painting from Barnet
and sold it to Totteridge. Alban managed to track the painting down and demanded it back. Is Alban entitled to the painting?
o No, because Totteridge bought the painting in good faith with no notice that it was stolen. – A good faith buyer does not take
priority over the interest of the owner, unless something like the entrustment doctrine applies.
o No, because Alban and Totteridge had no contract with each other to rescind. – Alban is not trying to rescind a contract; Alban is
simply seeking return of Alban’s property.
o Answer: Yes. The entrustment doctrine does not protect Totteridge, because Alban did not entrust the painting to Finchley
Gallery. – The entrustment doctrine does not apply, because Alban did not entrust the painting to a merchant. Rather, Barnet
stole the painting from Alban. So, Barnet did not have title to the painting or the power to give title to Finchley Gallery. Likewise,
when Finchley Gallery sold the painting, Totteridge received only Finchley Gallery’s interest in the painting, which was nothing.
o Yes. “Buyer Beware” governs sales law.
 Is Totteridge entitled to recover from Finchley Gallery?
o No. Totteridge has not been harmed, because Totteridge is entitled to keep the painting.
o No. “Buyer Beware” governs sales law. - “Buyer Beware” no longer governs sales law. Rather, sellers make the warranties created
by Article 2, unless excluded. Finchley Gallery did not intend to deceive, so it did not defraud Totteridge. But warranty is breached
even if the seller does not know of the defect.
o Answer: Yes. Finchley Gallery breached the warranty of title. - Although Totteridge must return the painting to Alban, Totteridge
is entitled to recover from Finchley Gallery for breach of the warranty of title. Under §2-312, a seller warrants that the seller that

78
rightful title will be conveyed. Finchley Gallery breached the warranty, because it did not have title to the goods or power to
transfer title.
o Yes. Finchley Gallery defrauded Totteridge.
 Euston visited Pemulis Auto looking for a sports car. The salesperson showed Euston a 2001 Hal Roadster, telling Euston that the
Roadster had a top speed of 120 m.p.h. The manager, however, advised Euston that the car was not that speedy, and that the
salesperson was misguided. Euston purchased the car, signing a contract with a merger clause and clause conspicuously stating that
the car was sold “AS IS, with no warranties whatsoever.” The car disappointed Euston, who found it was capable of 120 m.p.h. only
when going steeply downhill. Was there a breach of warranty?
o No, due to the exclusion of warranties.
o Answer: No, because the top speed of the car was not an express warranty. - The only way to exclude an express warranty is not
to make it. An exclusion of warranty will not exclude express warranties, because any inconsistent language is inoperative to
exclude the express warranty. But here Pemulis Auto took steps to avoid any representation about the top speed of the car
becoming an express warranty. An express warranty is an affirmation of fact that is part of the basis of the bargain of the parties.
Pemulis Auto advised Euston that the car was not capable of 120 m.p.h. The contract also included a merger clause, indicating the
oral statements were not part of the agreement.
o No, because warranties must be in writing. – warranties do not need to be in writng.
o Yes, because express warranties are not excluded by language to the contrary.
 When Pemulis Auto decided to close one of its sales locations, it got a visit from the manager of a competitor, Stice Auto. The
manager purchased Pemulis Auto’s computer system. Within a week, the system burned up due to a faulty microchip. Did Pemulis
Auto breach any warranty?
o Pemulis Auto breached the implied warranty of merchantability, because Pemulis Auto is a merchant, and the computer system
was defective.
o Pemulis Auto breached an express warranty, because the system had a latent defect.
o Pemulis Auto breached the warranty of fitness for a particular purpose, because Pemulis Auto knew that Stice Auto desired a
functional computer system.
o Answer: Pemulis Auto did not breach any warranty. - Pemulis Auto did not make an implied warranty of merchantability, because
Pemulis Auto was not a merchant with respect to computer systems. Pemulis Auto did not make any express warranties, because
it made no representations about the computer system. Pemulis Auto did not make a warranty of fitness for a particular purpose,
because Stice Auto was not relying on Pemulis Auto’s skill or judgment to select suitable goods.
 Joelle’s stay at the Ennet Hotel is marred by bedbugs. When the hotel refused to refund her payment, she sued for breach of the
implied warranty of merchantability under §2-314. Will she succeed?
o Yes, because a hotel room with bed bugs is not merchantable.
o No. As long as someone could stay overnight in the room, it was merchantable.
o No, because there was no express warranty about bed bugs.
o Answer: No, because the Ennet Hotel did not make a warranty under §2-314. - The question reminds us that UCC Article 2 applies
only to sales of goods. This was a contract for services, lodging at a hotel. Article 2 does not apply, so §2-314 does not apply.
Chapter 5: Terms of the Contract
I. Gap Fillers
Formation in General
§2-204(3) Even though one or more terms are left open a contract for sale does not fail for indefiniteness if the parties have intended to
make a contract and there is a reasonably certain basis for giving an appropriate remedy.
But what if terms like Price, Quantity, Delivery, Time for Performance or Remedies are missing?
Commercial standards on the point of “indefiniteness” are intended to be applied, the UCC making provision elsewhere for missing
terms.

 The Function of the “Gap Filler” Provisions of Article 2


o Where the parties have entered into a contract but have failed to agree on certain necessary terms, those terms are supplied by
several provisions of Part 3 of Article 2.
 Good Faith and Unconscionability
o Good faith is a requirement of all transactions under the U.C.C. (Section 1-304) The obligations of good faith and reasonableness
may not be disclaimed. (Section 1-302)
o Furthermore, Article 2 provides that the courts may limit or refuse to enforce any contract or clause of a contract that is
unconscionable. (Section 2-302)
 The Duties of Seller and Buyer
o Section 2-301 establishes the central duties of the seller and a buyer of goods.
 Section 2-301: The Duties of Seller and Buyer
The obligation of the seller is to transfer and deliver and that of the buyer is to accept and pay in accordance with the contract.
 Other Gap-Filling Provisions of Article 2

79
o Sections 2-305 through 2-311 supply terms of a contract for the sale of goods in situations where the express agreement of the
parties does not include those terms. These provisions include:
 Section 2-305: Open Price Term
 Section 2-306: Outputs, Requirements, and Exclusive Dealings
 Section 2-307: Delivery in Single Lot or Several Lots
 Section 2-308: Absence of Specified Time for Delivery
 Section 2-309: Absence of Specific Time Provisions; Notice of Termination
 Section 2-310: Open Time for Payment or Running of Credit; Authority to Ship Under Reservation
 Section 2-311: Options and Cooperation Respecting Performance
 The Gap-Fillers Require the Parties to Act “Reasonably”
o In general, the gap-fillers supply terms to the contract that are “reasonable” – delivery within a reasonable time, payment of a
reasonable price, etc.
o Section 1-205 identifies three factors as bearing on the reasonableness of the time for taking an action:
o “Whether a time for taking an action required by [the Uniform Commercial Code] is reasonable depends on the nature,
purpose, and circumstances of the action.”
 Open Price Term
o Under Section 2-305 if the parties intended to conclude a contract but failed to agree upon a price for the goods then the
contract is for a “reasonable price” at the time of delivery.
o The contract may give the seller or the buyer the power to fix the price; in that case the seller or buyer must act in good faith.
o The contract may permit the price to be fixed by some agreed market or other standard; if the market or standard fails to
establish a price then the price is a reasonable price.
o If the parties did not intend for the contract to be binding unless the price was fixed or agreed upon then there is no contract,
and the parties must make restitution to each other of any goods that were delivered or price that was paid.
 Section 2-305: Open Price Term
(2) The parties if they so intend can conclude a contract for sale even though the price is not settled. In such a case the price is a
reasonable price at the time for delivery if
(a) nothing is said as to price; or
(b) the price is left to be agreed by the parties and they fail to agree; or
(c) the price is to be fixed in terms of some agreed market or other standard as set or recorded by a third person or agency and it is
not so set or recorded.
(3) A price to be fixed by the seller or by the buyer means a price for him to fix in good faith.
(4) When a price left to be fixed otherwise than by agreement of the parties fails to be fixed through fault of one party the other
may at his option treat the contract as cancelled or himself fix a reasonable price.
(5) Where, however, the parties intend not to be bound unless the price be fixed or agreed, and it is not fixed or agreed there is no
contract. In such a case the buyer must return any goods already received or if unable so to do must pay their reasonable value
at the time of delivery and the seller must return any portion of the price paid on account.
 §2-305(1) Open Price Term
o §2-305 applies when the price term is left open on the making of an agreement which is nevertheless intended by the parties to
be a binding agreement. The UCC rejects in these instances the formula that "an agreement to agree is unenforceable" if the
case falls within subsection (1) of this section and rejects also defeating such agreements on the ground of "indefiniteness".
Instead, the UCC recognizes the dominant intention of the parties to have the deal continue to be binding upon both.
o As to future performance, since the UCC recognizes remedies such as cover (Section 2-712), resale (Section 2-706) and specific
performance (Section 2-716) which go beyond any mere arithmetic as between contract price and market price, there is usually
a "reasonably certain basis for granting an appropriate remedy for breach" so that the contract need not fail for indefiniteness.
o Under some circumstances the postponement of agreement on price will mean that no deal has really been concluded, and this
is made express in the preamble of subsection (1) ("The parties if they so intend ") and in subsection (4). Whether or not this is
so is, in most cases, a question to be determined by the trier of fact.
 §2-305(2)-(4) Open Price Term
o Subsection (2), dealing with the situation where the price is to be fixed by one party rejects the uncommercial idea that an
agreement that the seller may fix the price means that he may fix any price he may wish by the express qualification that the
price so fixed must be fixed in good faith.
o Good faith includes observance of reasonable commercial standards of fair dealing in the trade if the party is a merchant.
(Section 2-103.) But in the normal case a "posted price" or a future seller's or buyer's "given price," "price in effect," "market
price," or the like satisfies the good faith requirement.
o Throughout the entire section, the purpose is to give effect to the agreement which has been made. That effect, however, is
always conditioned by the requirement of good faith action which is made an inherent part of all contracts within the UCC.
(Section 1-203.)
 Quantity Not Fixed

80
o Where the quantity of goods to be sold is not fixed there is no contract – unless the contract is an output, requirements, or
exclusive dealing contract.
o Contracts in which the quantity is measured by the output of the seller or the requirements of the buyer are valid but must be
exercised in good faith and may not be unreasonably disproportionate to any stated estimate or normal or comparable prior
outputs or requirements.
o Exclusive dealing contracts impose an obligation upon the seller to use best efforts to supply the goods and best efforts by the
buyer to promote their sale.
 Section 2-306: Open Quantity Term. Output, Requirements and Exclusive Dealings
(1) A term which measures the quantity by the output of the seller, or the requirements of the buyer means such actual output or
requirements as may occur in good faith, except that no quantity unreasonably disproportionate to any stated estimate or in
the absence of a stated estimate to any normal or otherwise comparable prior output or requirements may be tendered or
demanded.
(2) A lawful agreement by either the seller or the buyer for exclusive dealing in the kind of goods concerned imposes unless
otherwise agreed an obligation by the seller to use best efforts to supply the goods and by the buyer to use best efforts to
promote their sale.
 §2-306 Open Quantity Term. Output, Requirements and Exclusive Dealings
(1) The general approach of this Act which requires the reading of commercial background and intent into the language of any
agreement and demands good faith in the performance of that agreement.
(2) A contract for output or requirements is not too indefinite since it is held to mean the actual good faith output or requirements
of the particular party.
(3) A shut-down by a requirements buyer for lack of orders might be permissible when a shut-down merely to curtail losses would
not. The essential test is whether the party is acting in good faith.
(4) Similarly, a sudden expansion of the plant by which requirements are to be measured would not be included within the scope of
the contract as made but normal expansion undertaken in good faith would be within the scope of this section.
Subsection (2), on exclusive dealing, makes explicit the commercial rule embodied in this Act under which the parties to such
contracts are held to have impliedly, even when not expressly, bound themselves to use reasonable diligence as well as good
faith in their performance of the contract.
 Delivery in Single Lot or Several Lots
o Under Section 2-307 it is assumed that the goods are to be delivered in a single lot and that payment is due upon delivery –
however, if the circumstances give either party the right to delivery in lots the price due on delivery is to be apportioned, if
possible.
 Section 2-307: Delivery in Single Lot or Several Lots
Unless otherwise agreed all goods called for by a contract for sale must be tendered in a single delivery and payment is due only on
such tender but where the circumstances give either party the right to make or demand delivery in lots the price if it can be apportioned
may be demanded for each lot
 Absence of Specified Place for Delivery
o Under Section 2-308, the default place of delivery is the seller’s place of business; or, if the contract is for the sale of goods
identified to the contract which the parties know are in a different place, then that place is the place of delivery.
o Documents of title are delivered through customary banking channels.
 Section 2-308: Absence of Specified Place for Delivery
Unless otherwise agreed
(a) the place for delivery of goods is the seller's place of business or if he has none his residence; but
(b) in a contract for sale of identified goods which to the knowledge of the parties at the time of contracting are in some other
place, that place is the place for their delivery; and
(c) documents of title may be delivered through customary banking channels
 Absence of Specific Time Provisions; Notice of Termination
o Under Section 2-309 the time for shipment or delivery is within a reasonable time.
o A contract that calls for successive performances is valid for a reasonable time, but thereafter may be terminated at any time by
either party.
o Termination of a contract by one party requires reasonable notification.
 Section 2-309: Absence of Specific Time Provisions; Notice of Termination
(1) The time for shipment or delivery or any other action under a contract if not provided in this Article or agreed upon shall be a
reasonable time.
(2) Where the contract provides for successive performances but is indefinite in duration it is valid for a reasonable time but unless
otherwise agreed may be terminated at any time by either party.
(3) Termination of a contract by one party except on the happening of an agreed event requires that reasonable notification be
received by the other party and an agreement dispensing with notification is invalid if its operation would be unconscionable.
 §2-309 Open Delivery Term. Absence of Specific Time Provision; Notice of Termination

81
1. The reasonable time under this provision turns on the criteria as to "reasonable time" and on good faith and commercial
standards set forth in Sections 1-203, 1-204 and 2-103.
2. It thus depends upon what constitutes acceptable commercial conduct in view of the nature, purpose and circumstances of the
action to be taken.
3. Agreement as to a definite time, however, may be found in a term implied from the contractual circumstances, usage of trade
or course of dealing or performance as well as in an express term. Such cases fall outside of this subsection since in them the
time for action is "agreed" by usage.
4. The obligation of good faith under this Act requires reasonable notification before a contract may be treated as breached
because a reasonable time for delivery or demand has expired.
 Open Time for Payment or Running of Credit; Authority to Ship Under Reservation
o Under Section 2-310(a), in the absence of a contrary agreement payment is due at the time and place that the buyer is to
receive the goods.
o Under Section 2-505 a seller may procure a document of title that reserves in the seller a security interest in the goods. Under
Section 2-310(b) the buyer has the right to inspect the goods before payment is due.
 Section 2-310: Open Time for Payment or Running of Credit; Authority to Ship Under Reservation
Unless otherwise agreed
(a) payment is due at the time and place at which the buyer is to receive the goods even though the place of shipment is the place
of delivery; and
(b) if the seller is authorized to send the goods, he may ship them under reservation (see § 2-505), and may tender the documents
of title, but the buyer may inspect the goods after their arrival before payment is due unless such inspection is inconsistent with
the terms of the contract (Section 2-513); and
(c) if delivery is authorized and made by way of documents of title otherwise than by subsection (b) then payment is due regardless
of where the goods are to be received
(i) at the time and place at which the buyer is to receive delivery of the tangible documents or
(ii) at the time the buyer is to receive delivery of the electronic documents and at the seller's place of business or if none, the
seller's residence; and
(d) where the seller is required or authorized to ship the goods on credit the credit period runs from the time of shipment but post-
dating the invoice or delaying its dispatch will correspondingly delay the starting of the credit period.
 Leaving Particulars of Performance to Be Specified by One of the Parties
o Under Section 2-311, a contract for sale is not invalid simply because the agreement leaves one or more particulars of
performance to be specified by one of the parties.
o Unless otherwise agreed specifications relating to the assortment of goods are at the buyer’s option and specifications for
shipment are at the seller’s option.
o Where a party fails to make those specifications, the other party may be excused for any delay in performance; or the other
party may treat that failure as a breach.
 Section 2-311: Open Terms of Performance. Options and Cooperation Respecting Performance
(1) An agreement for sale which is otherwise sufficiently definite (subsection (3) of Section 2-204) to be a contract is not made
invalid by the fact that it leaves particulars of performance to be specified by one of the parties. Any such specification must be
made in good faith and within limits set by commercial reasonableness.
(2) Unless otherwise agreed specifications relating to assortment of the goods are at the buyer's option and except as otherwise
provided in subsections (1)(c) and (3) of Section 2-319 specifications or arrangements relating to shipment are at the seller's
option.
(3) Where such specification would materially affect the other party's performance but is not seasonably made or where one
party's cooperation is necessary to the agreed performance of the other but is not seasonably forthcoming, the other party in
addition to all other remedies:
(a) is excused for any resulting delay in his own performance; and
(b) may also either proceed to perform in any reasonable manner or after the time for a material part of his own performance
treat the failure to specify or to cooperate as a breach by failure to deliver or accept the goods.
 Casebook Problem
o Edwin Drake wrote to the Watsons Flat Motor Oil Company and said that he wanted to buy 100 cases of its motor oil, some
cases to be Type A (the expensive oil) and some to be Type B (a cheaper kind). He said he would let the company know later
how much he wanted of each type. The company told him that Type B was selling for $30 a case, but that since the price of Type
A was fluctuating, the sale price would have to be set by the company at the time of delivery. Drake agreed. The parties signed a
written contract for the delivery of 100 cases, types to be specified by Drake one week prior to the delivery date, which was set
for April 8. On April 1, the agent of the oil company called Drake to ask how much he would take of each type. Drake said, “April
Fool! I’m not taking any,” and hung up the phone. The company calls you, its attorney, for advice. In the past dealings that it has
had with Drake, he always has ordered 100 cases and has taken 50 to 65 percent in Type A and the rest in Type B. The usual
price for Type A has been $50 a case, but due to a Middle East oil situation, the price has now jumped to $125 a case. What
should the company do? See §§2-305, 2-311, 1-205; Citgo Petroleum Corporation v. US Lubes, LLC. If this were an international

82
sale of goods under the CISG, what result? See Article 65. What if Drake had simply said, “April Fool!” and hung up? Is this a
definite repudiation? See §§2-610, 2-611. What action can the oil company take to clear up Drake’s ambiguous statement?
Read §2-609 and its Official Comment; see Pittsburgh-Des Moines Steel Co. v. Brookhaven Manor Water Co. Would a seller have
the right to adequate assurance of performance, if shortly before making the goods seller learned that buyer had a poor
history of paying sellers? Starchem Laboratories, LLC v. Kabco Pharmaceuticals, Inc.
 The first three cited sections all permit Watsons Flat to designate for Drake the amount he would have taken; the only
limitation on its choice is the requirement that it act in good faith. This is a change from the common law rules, which either
denied liability on the ground of indefiniteness or held the seller’s recovery to the least expensive choice buyer could have
made. The key UCC section is §2-311 and its Official Comments.
 If the CISG applies, this result is even clearer. The seller is authorized by Article 65 to make the specification, tell the buyer
what has been done, and respond to a buyer correction (if any). This is a much neater resolution than the UCC version.
 If Drake makes an ambiguous repudiation (what does “April Fool!” mean in this context?), §2-609 should be used to
establish his breach vel non.
 The cited case held that an insecure seller could not use §2-609 to create new terms to the contract. In that case during
negotiations the buyer had refused to make partial payments during the construction of a water tank and the eventual
contract only provided buyer would make payment on completion. Seller became concerned about buyer’s financial
situation and refused to perform unless buyer gave assurance the payment would be made before construction began,
sending a §2-609 notice, but the court held there were no reasonable grounds of insecurity given that the contract didn’t
require this sort of proof prior to seller’s performance.
 Casserlie v. Shell Oil Company
o Casserlie Gas agreed to buy gasoline delivered by Shell Oil, at a price to be set by Shell Oil (“open price” term). Where the
contract calls for seller to set the price, seller must set the price in good faith; §2- 305(2).
o Shell Oil charged Casserlie Gas the same as its other local customers. Shell Oil, however, charged a lower price to jobbers, who
pick up the gasoline from Shell’s depot, and so did not require delivery by Shell. Jobbers were able to resell at a lower price, and
so Casserlie had difficulty competing with the stations that buy from jobbers.
o Casserlie argued that there would be bad faith unless Shell met both components of the good faith requirement: reasonable
commercial standards of fair dealing (objective standard) and honesty in fact (subject standard). As the Casserlie court noted,
some courts have taken this approach. But the court relied on the safe harbor in the comments, holding that the good faith
requirement would be met if the price was commercially reasonable and nondiscriminatory.
 Canvas Quiz Questions
o In general, Article 2 provides that if certain terms of the contract are intentionally left open or are not specified for some
other reason, then the parties must act "reasonably" -- act within a "reasonable time" or pay a "reasonable price," etc.
 True
o If the parties intended that there should be no contract unless the price was fixed or agreed upon, then there is no contract,
and the parties must make restitution to each other of any goods that were delivered or any price that was paid.
 True
o If the quantity of goods being sold under the contract is not fixed, then
 in all cases the amount of the goods that the seller is obligated to deliver, and that the buyer is obligated to pay for is "a
reasonable quantity."
 Answer: there is no contract unless the contract is an output, requirements, or exclusive dealing contract.
 the seller is deemed to have the power under the contract to set the quantity.
 the buyer is deemed to have the power under the contract to set the quantity.
o In the case of a contract that measures quantity by reference to the output of the seller or the requirements of the buyer:
 No quantity that is unreasonably disproportionate to any stated estimate or, if there is no stated estimate, any normal or
otherwise comparable prior output or requirements, may be tendered by the seller or demanded by the buyer.
 The parties must act in good faith in producing output or establishing requirements.
 The contract is void because the parties have failed to establish the precise quantity of goods that are subject to the
contract.
 The price to be paid is a "reasonable price" at the time of delivery.
o If the contract does not identify the place of delivery of the goods, then Article 2 provides that the place of delivery is the
buyer's place of business.
 False - If the place of delivery is left open, then Article 2 provides that the place of delivery is the seller's place of business; or
if the contract is for the sale of goods identified to the contract which the parties know are in a different place, then that
place is the place of delivery.
o If the contract leaves open the time of shipment or delivery, then the goods must be shipped or delivered within a
reasonable time.
 True
o If a contract for the sale of goods calls for successive performances but does not identify the time that the contract will
remain in force, then the contract is valid indefinitely and cannot be cancelled by either party.

83
 False - If a contract for the sale of goods calls for successive performances but does not identify the time that the contract
will remain in force, then the contract is valid for a reasonable time and thereafter it can be cancelled by either party.
o To terminate a contract, except upon the occurrence of an agreed-upon event, requires reasonable notice.
 True
o Absent a contrary agreement, the buyer must pay for the goods at the time and place that the buyer is to receive the goods.
 True
o If a contract leaves one or more particulars of performance to be specified by one of the parties,
 the contract is void.
 Article 2 requires the party who will specify those particulars to act in good faith and within limits set by commercial
reasonableness.
 If the party who is to specify the particulars fails to do so in a timely manner, then the other party is excused for any delay
in its performance; may choose to perform in any reasonable manner; or may treat the failure to specify as a breach.
II. Good Faith and Unconscionability
 Unconscionability and Good Faith
o Under Article 2 a contract must not be “unconscionable” when it is entered into and the parties must perform and enforce the
contract in “good faith.”
 Unconscionable Contracts
o Under Section 2-302 if a contract for the sale of goods is “unconscionable” then it is not enforceable.
o The UCC does not define the term “unconscionable,” but many cases have attempted to apply this ambiguous standard. A
contract may be “unconscionable” if it contains terms that are unfairly one-sided in favor of one party (substantive
unconscionability) and if the other party either had no opportunity or no power to bargain over those terms (procedural
unconscionability, or “contracts of adhesion”).
o Two prongs:
 Procedural unconscionability – unfair conduct in the formation of the contract
 Substantive unconscionability – unfairness in the terms of the resulting bargain
 Section 2-302: Unconscionable Contract or Clause
(1) If the court as a matter of law finds the contract or any clause of the contract to have been unconscionable at the time it was
made the court may refuse to enforce the contract, or it may enforce the remainder of the contract without the unconscionable
clause, or it may so limit the application of any unconscionable clause as to avoid any unconscionable result.
(2) When it is claimed or appears to the court that the contract or any clause thereof may be unconscionable the parties shall be
afforded a reasonable opportunity to present evidence as to its commercial setting, purpose and effect to aid the court in
making the determination.
 Official Comments to §2-302 (heavily redacted)
o This section is intended to allow the court to pass directly on the unconscionability of the contract or particular clause therein
and to make a conclusion of law as to its unconscionability. The basic test is whether, in the light of the general commercial
background and the commercial needs of the particular trade or case, the clauses involved are so one-sided as to be
unconscionable under the circumstances existing at the time of the making of the contract.
o Subsection (2) makes it clear that it is proper for the court to hear evidence upon these questions. The principle is one of the
prevention of oppression and unfair surprise (Cf. Campbell Soup Co. v. Wentz) and not of disturbance of allocation of risks
because of superior bargaining power.
o Under this section the court, in its discretion, may refuse to enforce the contract as a whole if it is permeated by the
unconscionability, or it may strike any single clause or group of clauses which are so tainted or which are contrary to the
essential purpose of the agreement, or it may simply limit unconscionable clauses so as to avoid unconscionable results
 Good Faith
o Under Section 1-304, all conduct that is subject to the Uniform Commercial Code must be performed in good faith.
 Section 1-304: Requirement of Good Faith
Every contract or duty within [the Uniform Commercial Code] imposes an obligation of good faith in its performance and
enforcement
 Definition of “Good Faith”
o Under Section 1-201(b)(20), the term “good faith” has a subjective component (“honesty in fact”) and an objective component
(“reasonable commercial standards of fair dealing”).
 Section 1-201(b)(20): Definition of “Good Faith”
“Good faith” … means honesty in fact and the observance of reasonable commercial standards of fair dealing.
 Good Faith Applies to Merchants and Non-Merchants and to All Sales of Goods
o The obligation of good faith applies to merchants and non-merchants alike. However, while the standard of fairness is the same
for all persons its meaning may vary depending upon the commercial setting.
 No Good Faith Disclaimer
o Under Section 1-302 the obligation of good faith may not be disclaimed.
 Section 1-302(b): No Good Faith Disclaimer
84
The obligations of good faith, diligence, reasonableness, and care prescribed by [the Uniform Commercial Code] may not be
disclaimed by agreement. The parties, by agreement, may determine the standards by which the performance of those obligations is
to be measured if those standards are not manifestly unreasonable. Whenever [the Uniform Commercial Code] requires an action to
be taken within a reasonable time, a time that is not manifestly unreasonable may be fixed by agreement.
 Unconscionability Questions
1. Is unconscionability defined in the Code? If asked, how would you define the meaning to a judge? Are the example cases in
the Official Comment helpful?
 The term is not defined; Leff says it is undefinable. It means some sort of basic unfairness, and, like obscenity, in theory we
all know it when we see it. The classic definition comes from Judge J Skelly Wright’s opinion in Williams v. Walker-Thomas
Furniture Co.:
• Unconscionability has generally been recognized to include an absence of meaningful choice on the part of one of the
parties together with contract terms which are unreasonably favorable to the other party. Whether a meaningful
choice is present in a particular case can only be determined by consideration of all the circumstances surrounding the
transaction. In many cases the meaningfulness of the choice is negated by a gross inequality of bargaining power. The
manner in which the contract was entered is also relevant to this consideration. Did each party to the contract,
considering his obvious education or lack of it, have a reasonable opportunity to understand the terms of the contract
or were the important terms hidden in a maze of fine print and minimized by deceptive sales practices? Ordinarily, one
who signs an agreement without full knowledge of its terms might be held to assume the risk that he has entered a
one-sided bargain. But when a party of little bargaining power, and hence little real choice, signs a commercially
unreasonable contract with little or no knowledge of its terms, it is hardly likely that his consent, or even an objective
manifestation of his consent, was ever given to all the terms. In such a case the usual rule that the terms of the
agreement are not to be questioned should be abandoned and the court should consider whether the terms of the
contract are so unfair that enforcement should be withheld.
2. Is the unconscionability issue one for the judge or the jury? What is the policy reason for the Code drafters’ decision on this
matter?
 The issue is to be decided as a “matter of law”; §2-302(1). The theory is that juries are too easily swayed by charges of
unconscionability, but that judges, being made of sterner stuff, are not.
3. Is the §2-302(2) hearing mandatory as a technical step prior to a finding of unconscionability? See Haugen v. Ford Motor Co.,
holding that the hearing is mandatory, and that summary judgment is inappropriate without such a hearing.
 Whether or not your jurisdiction has addressed this issue, enough courts have held the §2-302(2) hearing is mandatory that
it is a wise step to take.
 Sacramento E.D.M, Inc. v. Hynes Aviation Industries
o Here is an equipment lease in which the court uses the traditional procedural and substantive unconscionability analysis to
invalid the lease. There were obvious lies about the terms of the lease, and those terms were quite harsh, being in fact usurious.
Note that this is not a consumer case, but an application of the doctrine of unconscionability in a business transaction
(something of a rarity).
 Green Tree Servicing, LLC v. Duncan
o Here is an interesting Native American court using unconscionability under §2-302 of the UCC, which the Navajo nation had
adopted as part of their Navajo Nation Code, to invalidate a clause requiring the dispute to be subject to binding arbitration
over the repossession of a mobile home sold to a woman who was part of the Navajo Nation. The court notes a general policy
encouraging arbitration (and federal law has a similar provision across the United States outside of the tribal land). The court
decides that Navajo legal principles require “talking it out,” meaning that important contract principles must be understood by
all parties, and here it was not clear that the buyer understood she was giving up her legal right to go to court in exchange for an
arbitration procedure she didn’t comprehend at all.
o In the “Boilerplate” book discussed following the case Professor Radin makes a convincing argument that most consumers do
not understand arbitration and that it is often very unfair to them because the arbitrators are chosen by the seller and
consequently, they rule overwhelmingly in the seller’s favor. It’s difficult for consumers to find an attorney willing to take an
arbitration case. Combined with other factors such as lack of appeal, lack of a class action possibility, and lack of precedential
value in the ruling, arbitration is a bad deal for consumers. Sellers love it for all these reasons.
 Casebook Problem
o Professor Chalk went into the Swank Boating Company and inquired about the possibility of buying a sailboat. He told the
salesman that he knew nothing about sailboats but had always wanted to get into sailing. The salesman showed him a boat, a
handsome catamaran, costing $3,150. Chalk, delighted, signed a contract. On his way home from the store, he passed another
boat showplace and saw the same type of sailboat advertised at a price of $1,000. Subsequent investigation proved that the
highest price any other store was asking for the boat was $1,200. Chalk, who had done no comparison shopping, had not known
this. Does §2-302 permit Chalk to avoid the sale? See Morris v. Capitol Furniture & Appliance Co; Jones v. Star Credit Corp; For a
historical viewpoint, see Hamilton, The Ancient Maxim Caveat Emptor. See also Ellinghaus, In Defense of Unconscionability.
 The Professor should win even though there has been no comparison shopping:

85
• To hold otherwise is to say to the unscrupulous (but non-competitive) creditor: “Impose any terms you like—short of
fraud—and we will not condemn your unconscionable behavior as long as your debtor should have known enough to
have avoided you.” It is bad policy to give an unconscionability license to creditors on the condition that they may use it
only when they can catch the consumer behaving foolishly. Certainly, it was unwise for the professor to buy a boat
without looking around first, but how many of us have not had similar lapses? The penalty for this lack of wisdom may
have been bankruptcy, but that does not mean the creditor should keep his pound of flesh. Or, if you insist on
procedural unconscionability, approach the problem this way. The professor, when he bought the sailboat, believed, as
the merchant was bound to know, that the price was more or less competitive in the local community and the
merchant let him sign the contract under this delusion. There is procedural unconscionability in the merchant’s bad
faith appropriation of the professor’s money when the latter’s delusion is obvious. This is hardly the good faith dealing
the law requires.
 Canvas Quiz Questions
o What are some aspects of procedural and substantive unconscionability?
 A contract that was drafted and dictated by one party to the contract (an "adhesion contract" or "take it or leave it"
contract) may be procedurally unconscionable.
 A contractual term that is unduly harsh or unfair to one party may be substantively unconscionable.
 The doctrine of unconscionability is more likely to be applied to protect a consumer against a large commercial entity than
in a situation where the contract was negotiated by two commercial firms or two parties who are experienced
businesspersons.
 To be unconscionable, a contract must be void for illegality, fraud, or lack of capacity.
o Elite Contractors entered into an agreement with Philip and Sharon to replace all the windows in their home. The contract
was a form contract drafted by Elite with two pages of "fine print." The contract required the homeowners to pay the entire
purchase price in advance, and they did so. Once all the windows in the house had been removed, Elite notified the
homeowners that the price of windows and the price of labor would be double the original contract price. Elite then pointed
out that the fine print of the construction contract indeed states "Contractor has the right to raise the price of labor and
materials at any time and as it sees fit in its sole discretion." What legal principles should Philip and Sharon raise at this
point?
 The term of the contract permitting the contractor to raise the price at any time is substantively unconscionable because it
unduly harsh or unfair to one of the parties.
 The contract, or at least the term of the contract permitting the contractor to raise the price at any time, is procedurally
unconscionable because it was drafted by the contractor who had superior bargaining power in this instance and was
hidden in the fine print.
 Elite did not exercise good faith in performing the contract.
 Elite observed "reasonable commercial standards of fair dealing" in the home improvement industry. – This contract, or at
least the term of the contract in question, was unconscionable, and Elite did not act in good faith because it did not observe
reasonable commercial standards of fair dealing in this industry
III. Modification and Waiver
 Consideration Is Not Necessary for a Modification of a Contract …
o At common law a modification to a contract, just like the original contract, had to be supported by consideration. This created a
problem. Suppose that a seller incurs supply or production problems, and the contract is no longer profitable. The seller
contacts the buyer and requests a modest increase in the price to make performance feasible. Under the common law even if
the buyer agreed to the price increase this was not binding on the buyer because the modification was not supported by
consideration.
o Accordingly, Section 2-209(1) changes the common law by eliminating the requirement of consideration for a modification of
the contract.
 Section 2-209(1): Elimination of Consideration for Modification
An agreement modifying a contract within this Article needs no consideration to be binding.
 … But Good Faith Is Necessary
o The power to modify a contract without consideration might obviously be abused. For example, a seller might learn that its
buyer needs delivery of the goods by July 1, the agreed-upon delivery date, or it will lose its biggest customer. The seller
contacts the buyer and states, “I will deliver the goods on time on July 1 only if you agree to an increase in the price.” The buyer
agrees because it must, but after delivery the buyer tenders only the original contract price. Is the buyer bound by the
modification that it agreed to?
o No. Under Section 1-304 the U.C.C. imposes the obligation of good faith in every contract or duty, and in this case the seller did
not act in good faith. The attempted modification of the agreement is unenforceable.
 “No Oral Modification” Clauses
o The Parol Evidence Rule may bar proof of prior agreements or contemporaneous oral agreements, but it does not bar evidence
of subsequent oral or written agreements that modify the contract.

86
o Oral modification of a contract may be prohibited by Section 2-209(2). That section provides that the parties may, in a signed
agreement, prohibit modification of the contract except by a signed writing. If one of the parties is a merchant and the other is
not and if this clause appears on a form supplied by the merchant, then such a clause must be separately signed by the non-
merchant.
 Section 2-209(2): “No Oral Modification” Clauses
A signed agreement which excludes modification or rescission except by a signed writing cannot be otherwise modified or rescinded,
but except as between merchants such a requirement on a form supplied by the merchant must be separately signed by the other
party.
 Modifications and the Statute of Frauds
o Under Section 2-209(3) if the contract as modified would have to be evidenced by a writing under Section 2-201, the Statute of
Frauds, then the contract as modified must satisfy the Statute of Frauds.
 Section 2-209(3): Modifications and the Statute of Frauds
The requirements of the statute of frauds section of this Article (Section 2-201) must be satisfied if the contract as modified is within
its provisions.
 A Failed Modification May Be A Waiver
o Suppose that the parties orally agree to modify the contract, but the attempted modification is invalid because the contract
itself bars oral modification of the agreement or because the contract as modified is covered by the Statute of Frauds. Does the
modification have any legal effect whatsoever?
o Yes! Under Section 2-209(4) although the agreement fails as a proper modification of the agreement, it may operate as a
waiver.
 Section 2-209(4): A Failed Modification May Be A Waiver
Although an attempt at modification or rescission does not satisfy the requirements of subsection (2) or (3) it can operate as a
waiver.
 Example of a Waiver
o Suppose that a contract provided that the seller must ship the goods on December 1, and further provided that the contract
could not be modified by means of an oral agreement. Then the parties orally agree that the seller may ship the goods on
December 8 instead of December 1. The seller then ships the goods on December 8 and the buyer either rejects the goods for
lateness or the buyer sues the seller for breach of contract.
o Under Section 2-209(4), even though the contract was not properly “modified,” the law provides that the buyer “waived” its
right to delivery of the goods on December 1, and a court will find that seller is not liable to buyer for breach of contract.
o Section 2-209(4) greatly diminishes the effect of “no oral modification” clauses.
 Waivers Can Be Retracted
o A party who agrees to an oral modification under a contract with a “no oral modification” clause is given a second chance,
however.
o Under Section 2-209(5), a party may retract the waiver by notifying the other party within a reasonable time and before the
other party has materially changed its position in reliance on the waiver
o In the previous example, if the buyer had called the seller immediately after agreeing to the modification changing the delivery
date from December 1 to December 8 and stated, “No, you must deliver on December 1,” then the original delivery date would
remain in effect.
 Section 2-209(5): Waivers Can Be Retracted
A party who has made a waiver affecting an executory portion of the contract may retract the waiver by reasonable notification
received by the other party that strict performance will be required of any term waived, unless the retraction would be unjust in
view of a material change of position in reliance on the waiver.
 Canvas Quiz Questions
o Under Article 2 if a modification to a contract is not supported by separate consideration, then the attempted modification is
not effective.
 False - Under Section 2-209 consideration is not necessary to support a modification to a contract. This represents a change
from the common law, which did require separate consideration to modify a contract.
o The Parol Evidence Rule cannot be used to bar evidence of a modification of a contract.
 True - The parol evidence rule may be used to bar evidence of prior and contemporaneous oral agreements and previous
written agreements, but it has no application to contemporaneous written agreements or subsequent oral or written
agreements modifying the contract.
o Under Article 2 it is possible to bar oral modification of an agreement, and to require that any modification of an agreement
be in writing.
 True
o If the parties orally agree to modify a contract, but the contract itself bars oral modification, the attempted modification may
nevertheless operate as a "waiver" of the right to enforce the term of the contract that was modified.
 True

87
o The Fresh Laundry company produces detergent. One ingredient it uses is propylene glycol. Fresh Laundry's supplier for
propylene glycol is Baker Industries. Fresh and Baker entered into a contract for the sale of 12,000 gallons of propylene
glycol. The contract required Baker to deliver 1000 pounds of propylene glycol to Fresh's detergent factory in Chambersburg,
Pennsylvania, on the first of every month during 2015. The price was $50 per gallon.
The contract between Fresh and Baker also contained a clause stating: "This contract is the complete and exclusive statement
of the terms of the agreement between the parties. No other oral or written agreements are of any force or effect. This
agreement may not be modified orally or in writing, nor may any of its terms be waived by either party. Any attempted
modification, whether orally or in writing, shall be of no force or effect."
In February of 2015 there was an accident at Baker's plant and the necessary repairs would be costly -- so costly that the
contract with Fresh is no longer profitable and it would not make any sense to repair the equipment. Sanford, the owner
of Baker, contacted Fresh and requested that the contract price be raised to $60 per gallon so that Baker could at least break
even on the deal. Jones, the owner of Fresh, called Sanford up on the telephone and said, "Sure, I'll agree to the modification.
Make the repairs and ship the product as soon as you can, and we will pay you $60 per gallon."
Two days later Sanford emailed Jones and wrote, "I can't agree to raise the price to $60 per gallon. If you can't deliver the
product for $50 per gallon we will sue for breach of contract." In the last two days Baker had already incurred $7,000 in costs
for repairs to the equipment in its factory that it would not have incurred had Sanford not agreed to the modification.
Please determine which of the following statements is accurate about the application of Article 2 to this transaction.
 The oral modification of the agreement was barred by the clause barring oral modification of the contract.
 If the modification of the contract had been in a writing signed by Sanford, then Sanford would be bound by that
modification, even though the contract purported to prohibit a written modification of the contract.
 Even though the attempted oral modification of the contract was invalid under the terms of the contract, Sanford's
statement agreeing to the modification could be construed as a waiver.
 Sanford's retraction of his waiver was timely because he contacted Jones and withdrew his consent within a reasonable
time.
– The parol evidence rule does not bar modifications to a contract, whether oral or written; the original contract
properly barred oral modification of the contract, although a written modification that complied with the statute of
frauds would be effective regardless of any clause in the contract barring written modification; by orally agreeing
to the increase in price Sanford waived his right to enforce the contract price of $50 per gallon; and although
Sanford's retraction occurred quickly (within two days) Baker had already materially changed its position in reliance
on Sanford's waiver, and as a result it would be unjust for Sanford to insist on strict performance of the original
terms of the contract.
IV. Identification of Goods
 What Is “Identification” of Goods?
o Section 2-501 prescribes when goods are “identified” to a contract.
o It provides that existing goods can be identified to the contract by identifying them. (I know, not much help!)
o Future goods are identified to the contract when they are shipped, marked, or otherwise designated by the seller as the goods
that are subject to the contract.
o In general, growing crops can be identified to the contract when planted, and animal young can be identified when conceived, if
the crops are to be harvested or the young are to be born within 12 months.
 Section 2-501: Identification of Goods
(1) The buyer obtains a special property and an insurable interest in goods by identification of existing goods as goods to which the
contract refers even though the goods so identified are non-conforming and he has an option to return or reject them. Such
identification can be made at any time and in any manner explicitly agreed to by the parties. In the absence of explicit
agreement identification occurs:
(a) when the contract is made if it is for the sale of goods already existing and identified;
(b) if the contract is for the sale of future goods other than those described in paragraph (c), when goods are shipped, marked
or otherwise designated by the seller as goods to which the contract refers;
(c) when the crops are planted or otherwise become growing crops, or the young are conceived if the contract is for the sale of
unborn young to be born within twelve months after contracting or for the sale of crops to be harvested within twelve
months or the next normal harvest season after contracting whichever is longer.
(2) The seller retains an insurable interest in goods so long as title to or any security interest in the goods remains in him and where
the identification is by the seller alone, he may until default or insolvency or notification to the buyer that the identification is
final substitute other goods for those identified.
(3) Nothing in this section impairs any insurable interest recognized under any other statute or rule of law.
 Official Comment to §2-501
1. The present section deals with the manner of identifying goods to the contract so that an insurable interest in the buyer and the
rights set forth in the next section will accrue. Generally speaking, identification may be made in any manner "explicitly agreed
to" by the parties. The rules of paragraphs (1), (2) and (3) apply only in the absence of such "explicit agreement."

88
2. In the ordinary case identification of particular existing goods as goods to which the contract refers is unambiguous and may
occur in one of many ways. It is possible, however, for the identification to be tentative or contingent. In view of the limited
effect given to identification by this section, the general policy is to resolve all doubts in favor of identification.
 The Significance of Identification of Goods
o Whether or not goods are “identified to the contract” makes a difference in four circumstances:
1. In determining whether or not things are “goods” within the meaning of Section 2-105 and therefore subject to Article 2;
2. In determining whether or not title to goods can pass to a buyer under Section 2-401;
3. In determining whether performance of a contract is excused when the goods are damaged or destroyed under Section 2-
613;
4. In determining whether the buyer can require the seller to deliver the goods under Section 2-716 (3) (Replevin).
 Determining Whether Things Are Goods
o Under Section 2-105 in order to constitute “goods” things must be moveable “at the time of identification to the contract.”
o If the things are not “goods” when they are identified to the contract then Article 2 does not apply to the transaction.
 Section 2-105: Determining Whether Things Are Goods – Definition of “Goods”
“Goods” means all things (including specially manufactured goods) which are movable at the time of identification to the contract
for sale other than the money in which the price is to be paid, investment securities (Article 8) and things in action. “Goods” also
includes the unborn young of animals and growing crops and other identified things attached to realty as described in the section on
goods to be severed from realty (Section 2-107).
 Section 2-107: Determining Whether Things Are Goods – “Fixtures” or “Goods”
(1) A contract for the sale of minerals or the like (including oil and gas) or a structure or its materials to be removed from realty is a
contract for the sale of goods within this Article if they are to be severed by the seller but until severance a purported present
sale thereof which is not effective as a transfer of an interest in land is effective only as a contract to sell.
(2) A contract for the sale apart from the land of growing crops or other things attached to realty and capable of severance without
material harm thereto but not described in subsection (1) or of timber to be cut is a contract for the sale of goods within this
Article whether the subject matter is to be severed by the buyer or by the seller even though it forms part of the realty at the
time of contracting, and the parties can by identification effect a present sale before severance.
(3) The provisions of this section are subject to any third-party rights provided by the law relating to realty records, and the
contract for sale may be executed and recorded as a document transferring an interest in land and shall then constitute notice
to third parties of the buyer's rights under the contract for sale.
 Determining Whether Title to Goods Can Pass
o Under Section 2-401(1), title to goods cannot pass to the buyer until the goods have been identified to the contract.
 Section 2-401(1): Determining Whether Title to Goods Can Pass
Title to goods cannot pass under a contract for sale prior to their identification to the contract (Section 2-501), and unless otherwise
explicitly agreed the buyer acquires by their identification a special property as limited by this Act. …
 Excuse of Performance When Goods Are Damaged or Destroyed
o Under Section 2-613 the seller’s performance can be excused and the seller’s obligation to perform the contract can be avoided
if goods that were identified to the contract at the time of contracting are destroyed before risk of loss has passed to the buyer.
 Section 2-613: Excuse of Performance When Goods Are Damaged or Destroyed
Where the contract requires for its performance goods identified when the contract is made, and the goods suffer casualty without
fault of either party before the risk of loss passes to the buyer, or in a proper case under a “no arrival, no sale” term (Section 2-324)
then
(a) if the loss is total the contract is avoided; and
(b) if the loss is partial or the goods have so deteriorated as no longer to conform to the contract the buyer may nevertheless
demand inspection and at his option either treat the contract as avoided or accept the goods with due allowance from the
contract price for the deterioration or the deficiency in quantity but without further right against the seller.
 Determining Whether a Buyer Has a Right to Replevin
o A buyer has a right to “replevin” of goods under Section 2-716(3) only if those goods have been identified to the contract.
 Section 2-716(3): Determining Whether a Buyer Has a Right to Replevin
The buyer has a right of replevin for goods identified to the contract if after reasonable effort he is unable to effect cover for such
goods or the circumstances reasonably indicate that such effort will be unavailing or if the goods have been shipped under
reservation and satisfaction of the security interest in them has been made or tendered. In the case of goods bought for personal,
family, or household purposes, the buyer's right of replevin vests upon acquisition of a special property, even if the seller had not
then repudiated or failed to deliver.
 Casebook Problem
o Decide if identification per §2-501 has occurred in the following situations.
(a) Seller, a fisherman, contracts to sell his entire catch for the coming season. Does identification occur on the making of the
contract, on the catching of the fish, or on their packaging with a label indicating they belong to this particular buyer? See
Official Comment 6.

89
 The catching of the fish should do it; the parties “explicit agreement” is that all of the fish go to the buyer, so their catching
would, in §2-501(1)(b)’s words, make them “designated by the seller as goods to which the contract refers.”
(b) Three Ring Circus contracted to sell the unborn calf of the circus’s elephant Nancy as soon as the calf was born; the contract
was made when Nancy was two months pregnant. Does the identification occur on the date of contracting, on the calf’s birth,
or when the calf is marked for shipment?
 An elephant has a gestation period of 20 months so §2-501(1)(c) does not apply; the elephant’s birth would then be the
moment of identification. What if twins are born? Identification would then occur on the selection of the calf destined for
the buyer. For a real elephant case see Miller v. Kaye, 545 P.2d 199 (Utah 1975) (a Statute of Frauds issue, not
identification).
(c) Farmer Carl agreed to sell to Breakfast Cereals, Inc. one-half of the grain he had stored in Rural Silo, where Farmer Carl’s grain
was mixed with that of other farmers. Does identification occur on contracting or on segregation of the grain? Read Official
Comment 5. See §§2-105(3), 2-105(4).
 “Undivided share” is an unfortunate term, probably borrowed from Property law. Its opposite is a “divided share,” an
oxymoron. Apparently (we can find no discussion of this anywhere, by anybody), the drafters used an “undivided share” to
mean no more than that the share is mixed with others and is not yet segregated. If that is so, the farmer has sold an
undivided share to the cereal company, and identification would occur on contracting (since the fungible mass is in
existence and identified).
(d) Wonder Widgets contracted to sell 5,000 widgets to a buyer. Its warehouse contained 2 million widgets, all alike. Does
identification occur on contracting or when the goods are picked out and marked as pertaining to this contract?
 Official Comment 5 does not apply to this Problem since there is no share at being sold (seller owns all the widgets in the
warehouse), nor is the seller simply selling a share in the inventory. Seller is purporting to sell specific widgets to the buyer,
and their identification must await selection of the goods as clearly belonging to this particular buyer. Another relevant
factor in (d) is that here the seller can do something to identify the goods—more is possible and better under seller’s
control than the situation in (c).
(e) Laputa agreed to sell its wrecked helicopter to Sheeta and agreed to repair it extensively before delivery. After Sheeta paid
the contract price but before Laputa did any repairs, Laputa filed bankruptcy. When would the helicopter be identified to the
contract: at the time of contracting (meaning title would pass to Sheeta) or not until the helicopter was repaired (meaning
that it would stay in the bankruptcy estate)? See In re Phoenix Heliparts Inc.
 In the cited case the court held that the title passed (and identification occurred) when the contract was made in spite of
the fact that the helicopter still needed extensive repairs. The court quoted Official Comment 4: “In view of the limited
function of identification there is no requirement in this section that the goods be in deliverable state or that all of the
seller's duties with respect to the processing of the goods be completed in order that identification occur.”
 Canvas Quiz Questions
o Have the following items been identified to the contract?
 Charles ordered a set of wineglasses through Amazon. The order was - Yes, these goods have
forwarded to the dealer and the wineglasses were packed and labeled been identified to the
for shipment to the buyer but have not yet left the seller's warehouse. contract.
 The buyer and seller have agreed that the seller will manufacture four - No, these goods have
hundred ball caps to be designed according to the buyer's not yet been identified
specifications as set forth in the contract. to the contract.
 Ricardo sells all of his corn crop to First Factors. The crop has been - Yes, these goods have
planted but has not yet emerged from the soil. been identified to the
contract.
 Pamela is moving and agrees to sell her living room furniture to - Yes, these goods have
Raymond for $400. been identified to the
contract.
o Identification of goods to a contract has the following legal consequences:
 Under Section 2-105 if a thing would not be movable when it is identified to the contract of sale, then Article 2 does not
apply to the transaction.
 Under Section 2-401 title to goods cannot pass until the goods have been identified to the contract.
 Under Section 2-613 if the goods are destroyed before delivery the seller's duty to perform may be excused if the goods
were identified to the contract at the time that the contract was entered into.
 Under Section 2-716 a buyer has a right to replevin of goods from the seller only if the goods have been identified to the
contract and the buyer is unable to cover.
– Identification of the goods to the contract is a necessary aspect or element with respect to each of these legal
principles.
V. Risk of Loss: No Breach
A. General Rules
 General Rule

90
o Under the Uniform Sales Act, risk of loss stayed with the seller until title passed to the buyer. But the concept of “title” has been
unbundled – identification, shipment, delivery, receipt, inspection, rejection, cure, acceptance, revocation.
o Under the UCC, if the seller is a merchant, risk of loss passes when the goods are received by the buyer, and if the seller is not a
merchant, risk of loss passes when the seller tenders delivery.
 The Practical Problem
o If a seller and a buyer have entered into a contract for the sale of goods, and then the goods are lost, stolen, damaged, or
destroyed, who will take the loss?
o Specifically, does a buyer have to pay for the goods that were lost, stolen, damaged or destroyed?
o Which party has the risk of loss?
 Express Agreement, Course of Performance, Course of Dealing, and Trade Usage on Risk of Loss
o The seller and buyer may expressly agree how the risk of loss is to be assigned.
o Or the risk of loss may be determined by the parties’ course of performance, course of dealing, or trade usage.
o In the absence of express agreement, course of performance, course of dealing, or trade usage, Article 2 contains provisions
that assign the risk of loss to the seller or buyer.
 General Approaches to Assigning Risk of Loss
o At common law, the person who had “title” to goods bore the “risk of loss.” That is, the person who was the owner of the goods
would suffer monetary loss if the goods were damaged or destroyed. It followed that the owner of the goods was the person
who ought to purchase insurance on the goods.
o Article 2 of the Uniform Commercial Code takes a different approach to assigning the risk of loss. Under Article 2, instead of
making risk of loss turn upon an abstract concept such as “title,” the law assigns risk of loss on the basis of objective criteria
such as whether the goods have been identified to the contract, shipped, or delivered.
 General Rule Regarding Risk of Loss
o If a contract is not a shipment or a destination contract; and if the goods are not in the hands of a bailee; and if the goods are
conforming to the contract; and in the absence of any express agreement as to risk of loss or any course of performance, course
of dealing, or trade usage that determines the risk of loss – then the following general rules apply:
 If the seller is a merchant, then risk of loss passes to the buyer when the buyer receives the goods.
 If the seller is not a merchant, then risk of loss passes to the buyer when the seller tenders delivery of the goods
 Section 2-509(3): General Rule Regarding Risk of Loss
In any case not within subsection (1) or (2), the risk of loss passes to the buyer on his receipt of the goods if the seller is a merchant;
otherwise the risk passes to the buyer on tender of delivery.
Tender of Delivery
o Tender of delivery is governed by Section 2-503(1). Under that section of the Code tender of delivery is complete if the seller
notifies the buyer that the goods are available and holds the goods available to the buyer for a reasonable time. The buyer has
the duty to furnish facilities reasonably suited to receipt of the goods.
 Section 2-503(1): Tender of Delivery
Tender of delivery requires that the seller put and hold conforming goods at the buyer's disposition and give the buyer any
notification reasonably necessary to enable him to take delivery. The manner, time and place for tender are determined by the
agreement and this Article, and in particular
(a) tender must be at a reasonable hour, and if it is of goods they must be kept available for the period reasonably necessary to
enable the buyer to take possession; but
(b) unless otherwise agreed the buyer must furnish facilities reasonably suited to the receipt of the goods.
 Example: Merchant Seller
o David’s Hard as Nails Boutique agreed to purchase a set of expensive pedicure tools from Absolam Devices. The contract was
silent as to the time and place for delivery. Absolam packed the goods and labeled them for David and told David on Monday
that he could pick up the tools that day. David decided to put off picking up the tools for a week because he was busy. On Friday
Absolam’s store burned down. Absolam demands that David pay for the tools. Is David liable to Absolam? Had the risk of loss
passed to David?
 ANSWER: No, David did not bear the risk of loss. Because Absolam was a merchant, under Section 2-509(3) the risk of loss
was on Absolam until David received the goods, even though the goods had been identified to the contract. David does not
have to pay for the tools.
 Example: Non-Merchant Seller
o Fred, who is unemployed, is selling his personal stamp collection and he placed an ad in the local paper. On Monday Jenny, who
lives nearby, calls him up and agreed to purchase the stamp collection for $400. Fred says, “Great, you can pick it up right now.”
Jenny says that she is busy, and she will pick it up sometime soon.
On Friday Fred’s house burns down. Fred calls Jenny and tells her that she still owes him $400. Jenny contends that she isn’t
liable to pay for the stamp collection since it was destroyed. What conclusion would you reach? Is Jenny liable to pay for the
stamp collection? Had the risk of loss passed to Jenny before the goods were destroyed?
 ANSWER: The answer is not clear in this case. This is a difficult problem because Fred isn’t a merchant, so under Section 2-
509(3) risk of loss passed to Jenny as soon as Fred tendered delivery to her. Fred notified Jenny on Monday that she could
91
pick up the stamp collection, and Jenny had a further reasonable time to pick it up before the tender of delivery was
complete and the risk of loss passed to her. If she had waited a month to pick up the stamp collection the risk of loss would
clearly have passed to her because a “reasonable time” would have expired. A period of four days presents a close
question.
 Risk of Loss in Shipment and Destination Contracts
o Risk of loss under shipment and destination contracts is governed by Section 2-509(1).
o Section 2-509(1)(a) assigns the risk of loss for shipment contracts.
o Section 2-509(1)(b) assigns the risk of loss for destination contracts.
 Section 2-509(1): Risk of Loss in Shipment and Destination Contracts
Where the contract requires or authorizes the seller to ship the goods by carrier
(a) if it does not require him to deliver them at a particular destination, the risk of loss passes to the buyer when the goods are duly
delivered to the carrier even though the shipment is under reservation (Section 2-505); but
(b) if it does require him to deliver them at a particular destination and the goods are there duly tendered while in the possession
of the carrier, the risk of loss passes to the buyer when the goods are there duly so tendered as to enable the buyer to take
delivery.
 “Duly Delivered”
o In the case of a shipment contract, Section 2-509(1)(a) shifts the risk of loss from the seller to the buyer when the goods are
“duly delivered” to the carrier.
o Remember that under Section 2-504 with a shipment contract the seller must not only deliver the goods to the carrier, but the
seller must also make reasonable arrangements for shipment, obtain documents that will enable the buyer to obtain possession
of the goods, and promptly notify the buyer that the goods have been shipped. Failure to do any of these three things means
that the goods have not been “duly delivered” to the carrier, and as a result the seller will retain the risk of loss if the goods are
lost, stolen, damaged or destroyed en route.
 Example: Shipment Contract
o Asgarth Unlimited agreed to send Colson Research sixteen centrifuges. The contract price was $18,000 apiece. Asgarth delivered
the centrifuges to Safety Shipping, which specializes in carrying sensitive equipment, and made sure that the centrifuges were
carefully packaged. Asgarth obtained proper documents of title and delivered them to Colson and notified Colson when the
centrifuges were shipped. Unfortunately, the centrifuges were destroyed when the truck was struck by an avalanche. Who had
the risk of loss? Asgarth or Colson?
 ANSWER: Colson had the risk of loss. Asgarth “duly delivered” the centrifuges to the carrier within the meaning of Section
2-504, so under Section 2-509(1)(a) the risk of loss had passed to the buyer before the goods were destroyed. Colson must
pay Asgarth for the centrifuges.
 “Duly Tendered”
o Similarly, under destination contracts the risk of loss does not pass to the buyer until the goods have been “duly tendered.”
 Example: Destination Contract
o Rudy Rotors, of Flint, Michigan, entered into a contract to sell 500 drones to YouBuysIt.com, whose warehouse was located in
Biloxi, Mississippi. The contract called for the drones to be delivered “F.O.B. Biloxi,” which is a destination contract. The truck
carrying the drones was sideswiped on a highway in Tennessee, and the cargo was destroyed. Is the buyer liable to the seller
for the price of the goods? Did the buyer have the risk of loss when the goods were destroyed?
 ANSWER: Because this was a destination contract, the risk of loss remained on the seller until tender of delivery in Biloxi,
Mississippi. The buyer is not liable to pay for the goods.
 Goods in the Hands of a Bailee
o Risk of loss for goods in the hands of a bailee is governed by Section 2-509(2).
o Risk of loss passes to the buyer when the bailee receives possession of a document of title to the goods or when the bailee
acknowledges the buyer’s right to possession of the goods.
 What If the Goods Are Non-Conforming?
o If the goods do not conform to the contract, the risk of loss remains on the seller even after delivery.
o The topic of “Risk of Loss in the Event of Breach” is covered in a later presentation.
 Casebook Problem
o William College bought a car from Honest John, the friendly car dealer. He paid the price in full, and Honest John promised
delivery on the next Monday. On Monday the car was ready, and Honest John phoned College and said, “Take it away.” College
said he was busy and that he would pick it up the next day, to which Honest John agreed. That night the car was stolen from the
lot due to no fault of Honest John, who had taken reasonable precautions against such a thing. Who had the risk of loss? See
§2-509(3) and Official Comment 3. Ramos v. Wheel Sports Center. Might Honest John claim he was a bailee so that §2-509(2)
applies? See White & Summers §6-3; Galbraith v. American Motor Home Corp.
 Since Honest John is a merchant, risk of loss remains with John until College’s receipt of the goods, and, in a similar case,
the cited opinion so held. The policy is that the party who has possession has more opportunity to protect the goods and is
better able to insure them – so they should have more incentive to do so!

92
 White & Summers have an extensive discussion of the bailee argument, which appears to be a loser. To allow the seller to
claim bailee status (except where the buyer is paying separate bailment charges) violates the Code’s policy of keeping the
risk of loss on the party in possession who will then have more incentive to take good care of them (and who is likely to
have insurance).
 Casebook Problem
o Janice Junk decided to hold a garage sale to clean up her home and get some extra cash. In the course of the sale, which was a
huge success, her neighbor, Barbara Bargain, offered Junk $200 for her piano, and the two women shook hands. Junk said to
Bargain, “Take it away. It’s yours.” Bargain replied that she would come to get it the next day with four strong friends and a
truck. That night Junk’s home burned to the ground, and the piano was destroyed. Did the risk of loss pass from Junk to
Bargain? See §2-503. If Bargain never picked up the piano and if it was destroyed in a fire six months after the sale, what
result? See §2-709(1)(a).
 According to §2-308(a) the place for delivery here was the seller’s residence. Risk passes on seller’s tender of delivery since
Junk is not a merchant; §2-509(3).
 Has Junk made such a tender? Section 2-503(1)(a) says the tender lasts “for the period reasonably necessary to enable the
buyer to take possession,” probably keeping the risk on Junk until the next day (though this is a fact question).
 Six months’ delay presents more problems. At some point Junk passes the risk to Bargain, but per the “commercially
reasonable time” requirement of §2-709(1)(a), at some point the risk also passed back to Junk. A court would likely say this
occurred when it became obvious that the buyer was in breach and not going to come pick up the piano.
 Canvas Quiz Questions
o At common law, risk of loss depended upon which party had "title" to the goods. Under Article 2 the risk of loss depends
upon objective factors such as whether the goods have been identified to the contract, shipped, delivered, or received, as
well as whether the goods are conforming to the contract.
 True
o In the absence of any express terms in the contract assigning risk of loss, and in the absence of any course of performance,
course of dealing or trade usage on the subject, and if the contract is neither a shipment contract, a destination contract, nor
a contract where the goods are in the hands of a bailee, risk of loss is allocated as follows when the seller is a merchant or the
seller is a non-merchant:
 Answer: If the seller is a merchant, risk of loss passes to the buyer when the buyer receives the goods. If the seller is not a
merchant risk of loss passes to the buyer when the seller tenders delivery of the goods to the buyer.
 If the seller is a merchant, risk of loss passes to the buyer when the seller tenders delivery of the goods to the buyer. If the
seller is not a merchant risk of loss passes to the buyer when the buyer receives the goods.
 Whether the seller is a merchant or a non-merchant, risk of loss passes to the buyer when the goods are received by the
buyer.
 Whether the seller is a merchant or a non-merchant, risk of loss passes to the buyer when the seller tenders delivery of the
goods to the buyer.
o In shipment and destination contracts the risk of loss is allocated as follows:
 With shipment contracts, risk of loss passes to the buyer when the goods have been "duly delivered" to the carrier; with
destination contracts, risk of loss passes to the buyer when the goods have been "duly tendered" to the buyer at the
destination.
 With both shipment and destination contracts, risk of loss passes to the buyer when the goods have been "duly delivered"
to the carrier.
 With both shipment and destination contracts, risk of loss passes to the buyer when the goods have been "duly tendered"
to the buyer at the destination.
 With both shipment and destination contracts, the parties are free to allocate risk of loss by agreement.
o If the goods are not conforming to the contract, risk of loss may stay with the seller even after delivery.
 True
B. Delivery Terms
 Overview
o Shipment contract – seller gets the goods to the carrier.
o Destination contract – seller gets the goods to the buyer.
o Section 2-509(1) creates a presumption for each type of contract as to the shifting of the risk of loss.
o Parties have developed a long list of delivery terms that determine the responsibilities of the seller and the buyer – delivery,
insurance, and who is to pay the cost of freight.
 Delivery Terms Are Usually Carefully Spelled Out and Gap Fillers Should Not Be Necessary
o In the usual commercial contract, the delivery terms are carefully set forth in detail, including where the goods are to be
delivered, who will pay for shipment, who is responsible for insuring the goods and who will bear the risk of loss if the goods are
damaged or destroyed prior to delivery. It is not normally necessary to rely upon “gap fillers.”
o However, when these terms are omitted Article 2 will provide them.
 Gap Fillers as to Place and Time of Delivery

93
o If the contract is silent as to where delivery is to occur, and if the delivery term is not supplied by any course of performance,
course of dealing, or trade usage, then under Section 2-308 the place for delivery is the seller’s place of business.
o Under Section 2-309, the default time for shipment or delivery is a “reasonable time.”
 Section 2-308: Gap Fillers as to Place of Delivery
Unless otherwise agreed
(a) the place for delivery of goods is the seller's place of business or if he has none his residence; but
(b) in a contract for sale of identified goods which to the knowledge of the parties at the time of contracting are in some other
place, that place is the place for their delivery; and
(c) documents of title may be delivered through customary banking channels.
 Section 2-309: Gap Fillers as to Time of Delivery
(1) The time for shipment or delivery or any other action under a contract if not provided in this Article or agreed upon shall be a
reasonable time.
(2) Where the contract provides for successive performances but is indefinite in duration it is valid for a reasonable time but unless
otherwise agreed may be terminated at any time by either party.
Termination of a contract by one party except on the happening of an agreed event requires that reasonable notification be received
by the other party and an agreement dispensing with notification is invalid if its operation would be unconscionable.
 Shipment Contracts
o If the contract requires or authorizes the seller to send the goods to the buyer, the U.C.C. presumes that the seller is to ship the
goods. This is called a “shipment contract.”
o Under Section 2-504, under a shipment contract the seller has three duties: (a) deliver the goods to the carrier and make a
reasonable contract for shipment; (b) obtain and deliver to the buyer a document that will enable the buyer to obtain
possession of the goods; and (c) notify the buyer of the shipment.
 Section 2-504: Shipment Contracts
Where the seller is required or authorized to send the goods to the buyer and the contract does not require him to deliver them at a
particular destination, then unless otherwise agreed he must
(a) put the goods in the possession of such a carrier and make such a contract for their transportation as may be reasonable having
regard to the nature of the goods and other circumstances of the case; and
(b) obtain and promptly deliver or tender in due form any document necessary to enable the buyer to obtain possession of the
goods or otherwise required by the agreement or by usage of trade; and
(c) promptly notify the buyer of the shipment.
Failure to notify the buyer under paragraph (c) or to make a proper contract under paragraph (a) is a ground for rejection only if
material delay or loss ensues.
 Destination Contracts
o Contracts that expressly require the seller to deliver the goods to a specific destination are “destination contracts.”
Risk of Loss in Shipment and Destination Contracts
o Section 2-509(1)(a) assigns the risk of loss for shipment contracts.
o Section 2-509(1)(b) assigns the risk of loss for destination contracts.
o Risk of loss is the subject of other presentations.
 Section 2-509(1): Risk of Loss in Shipment Contracts
Where the contract requires or authorizes the seller to ship the goods by carrier
(a) if it does not require him to deliver them at a particular destination, the risk of loss passes to the buyer when the goods are duly
delivered to the carrier even though the shipment is under reservation (Section 2-505); but
(b) if it does require him to deliver them at a particular destination and the goods are there duly tendered while in the possession
of the carrier, the risk of loss passes to the buyer when the goods are there duly so tendered as to enable the buyer to take
delivery.
 Abbreviations for Delivery Terms
o Delivery terms are often reduced to terms such as F.O.B. (free on board), F.A.S. (free alongside), C.I.F., C.&F., and “Ex-Ship.”
These terms indicate whether the contract is a shipment contract or a destination contract; which party is required to pay for
shipment; whether the seller is required to load the goods; whether the seller is required to obtain insurance covering the
goods; and which party bears the risk of loss during shipment.
 Example: F.O.B. (Free on Board)
o For example, under Section 2-319(1):
o F.O.B. [named place of shipment] is a shipment contract;
o F.O.B. [named place of destination] is a destination contract; and
o F.O.B. [vessel, car, or other vehicle] means that the seller is also responsible for loading the goods.
 Section 2-319(1): Risk of Loss in Destination Contracts
Unless otherwise agreed the term F.O.B. (which means “free on board”) at a named place, even though used only in connection with
the stated price, is a delivery term under which

94
(a) when the term is F.O.B. the place of shipment, the seller must at that place ship the goods in the manner provided in this Article
(Section 2-504) and bear the expense and risk of putting them into the possession of the carrier; or
(b) when the term is F.O.B. the place of destination, the seller must at his own expense and risk transport the goods to that place
and their tender delivery of them in the manner provided in this Article (Section 2-503);
(c) when under either (a) or (b) the term is also F.O.B. vessel, car or other vehicle, the seller must in addition at his own expense
and risk load the goods on board. If the term is F.O.B. vessel the buyer must name, the vessel and in an appropriate case the
seller must comply with the provisions of this Article on the form of bill of lading (Section 2-323).
 Example: Figs from Pittsburgh to Wheeling
o Organic Specialties LLC grows food crops hydroponically in a downtown Pittsburgh office building. Dinnertime Corporation
prepares frozen dinners at its plant in Wheeling, West Virginia. Dinnertime has ordered three thousand pounds of fresh figs
from Organic. The contract calls for the figs to be shipped “F.O.B. Wheeling.” Is this a shipment contract governed by Section 2-
319(1)(a) or a destination contract governed by Section 2-319(1)(b)?
 ANSWER: This is a destination contract and is therefore governed by Section 2-319(1)(b).
 Incoterms
o International contracts recognize a set of shipping terms (“incoterms”) that are different from the abbreviations are recognized
by the U.C.C. Incoterms are three-letter trade terms that determine whether the contract is a shipment or a destination
contract, who will bear the expense of shipment, who will bear the risk of loss, and several other factors.
o Be careful! Some of the incoterms are identical to the delivery abbreviations used by the U.C.C. but they have different
meanings under international law. For example, in international usage the term “F.O.B.” is used only for non-container shipment
by water.
 Casebook Problem
o Seller in New York City contracted to sell 80 boxes of clothing to buyer in Savannah, Georgia. The delivery term was “$1,800
F.A.S. S.S. Seaworthy, N.Y.C.” Seller delivered the 80 boxes to the dock alongside the S.S. Seaworthy and received a bill of lading
from the ship as a receipt. Before the boxes could be loaded, the dock collapsed, and everything thereon disappeared into the
water. Under §2-319(2) must buyer pay the $1,800 anyway? What if the delivery term had been “Ex-ship S.S. Seaworthy,
Savannah,” and the boxes had been properly unloaded just before the dock collapsed? Would §2-322 make the buyer pay?
 Seller, which is located in New York, is to deliver 80 boxes of clothing, “$1,800 F.A.S. S.S. Seaworthy, N.Y.C.”
 Seller put the clothing on the dock by the ship. The dock collapsed, destroying the shipment.
 Under §2-319(2), seller has done all it need do to pass the risk of loss to the buyer: get the goods to the ship and get a
receipt (a bill of lading, being the usual item) for them...
 Were the term “ex ship,” §2-322 passes the risk of loss to the buyer on proper unloading, so seller wins again.
 Casebook Problem
o Seller in Detroit, Michigan, contracted to sell and ship 50 automobiles to buyer in Birmingham, Alabama. Assume lightning
strikes, destroying the vehicles after the carrier has received them but before they are loaded on board the railroad car that was
to take them to Birmingham. Who had the risk of loss if (a) the contract said, “F.O.B. Detroit”? (see §2-319(1)(a)); (b) the
contract said, “F.O.B. railroad cars Detroit”? (see §2-319(1)(c) and Pagano v. Occidental Chem. Corp.); or (c) the contract said,
“C.I.F. Birmingham”? (see §2-320). If the buyer had inspected the automobiles and accepted them at the factory, would the
risk shift to buyer or remain on seller until the automobiles reached the railroad cars? See Southern Recycling, LLC v. Gibbs
International, Inc.
 Seller in Detroit contracted to sell and ship 50 autos to Birmingham, Alabama. The cars are destroyed after they are loaded
on the carrier but before being delivered to the railroad which was to take them to Birmingham.
• FOB Detroit – Buyer has risk of loss. Seller had to make the goods available to the carrier, no duty to load them. 2-
319(1)(a). The risk of loss had passed to the buyer.
• If the terms had been “F.O.B. vessel, car, or other vehicle,” the seller would have the obligation to load and supervise
the loading of the goods onto the truck.
• FOB railroad cars Detroit – Seller has risk of loss until goods are loaded on that vehicle. 2-319(1)(c)
• CIF Birmingham – According to 2-320(2)(b), CIF destination requires loading. Therefore, seller had the risk of loss. But
Comment 1 to 2-320 says it is a shipment contract, and risk of loss passes to the buyer upon delivery to the carrier.
 Official comment to Restatement Section of Contracts, Section 224: Illustration 3
o A contract to sell and B to buy goods to be shipped “C.I.F.,” payment to be “on arrival of goods.” Risk of loss of the goods passes
from A to B when A, having otherwise complied with the C.I.F. term of the contract, puts the goods in the possession of the
carrier (§ 2-320(2)). If the goods are lost in transit, B is under a duty to pay the price when the goods should have arrived (§§ 2-
709(1)(a), 2-321(3)). The arrival of the goods is not a condition of B's duty to pay for the goods.
 Casebook Problem
o The dispatcher for Perfect Pineapples, Inc., had just finished loading five boxcars of the company’s product on board the cars of
an independent railroad carrier when he received a notice from PPI’s sales department that the company had agreed to sell one
boxcar load to Grocery King Food Stores “F.O.B. seller’s processing plant.” The dispatcher agreed to divert one of the boxcars to
Grocery King, but before he could do so, a hurricane destroyed all five boxcars and their contents. Who bears the risk of loss?
See Official Comment 2 to §2-509 and §2-501.

95
 Unsold goods were loaded on five boxcars. Then dispatcher received word that one boxcar would be sold to Grocery King.
Before the dispatcher selected one of the boxcars for diversion to Grocery King, they were all destroyed by hurricane.
 Goods destroyed before they are identified to the contract. Seller has the risk of loss. Buyer does not acquire a special
property in the goods until identification to the contract. 2-501
 Since the goods are not identified, seller retained the risk of loss. This is not the sale of an “undivided share” since the
goods are segregated into different boxcars at the time of contracting. You might argue that since the goods from which
identification was to be made were all destroyed, thus surely the buyer’s portion went with them, but courts would not
likely so hold. Until a particular car was selected no identification occurred sufficient to permit buyer to insure the goods
and making buyer responsible for their destruction prior to that moment is unfair.
• Policy – buyer could not inspect the goods until they are identified to the contract.
 Cook Specialty Co. v. Schrlock
o Seller sold a $28,000 press to buyer, FOB seller’s warehouse. Seller only purchased $5,000 worth of insurance. Press was
delivered to the carrier, loaded, took off, and fell off the truck in transit. The District Court (the trial court) was considering
whether to grant summary judgment to the seller.
o Text: Under 2-509, risk of loss passed to the buyer when the goods were delivered to the carrier and the seller has made a
“reasonable contract” for their shipment.
o A “reasonable contract” for shipment is governed by 2-504.
o Buyer claimed that seller did not make a reasonable contract for shipment of the press because it did not purchase enough
insurance.
o Precedent: There is one case – La Casse v. Blaustein – where the court mentioned an inadequate amount of insurance as one
aspect of a contract for shipment that was unreasonable. However, there were a lot of other factors in that case as well – the
package for the item was misaddressed, which is a “theft-tempting” notation; the seller was told to spend up to $50 on
shipment but spent $10; and the seller insured a $1,600 shipment for only $200.
o In another case, Miller v. Harvey, the seller failed to declare the value of the goods, thus forfeiting the buyer’s claim against the
carrier.
o The court distinguished those cases – La Casse was “reckless” and the other “improper.”
o Plaintiff raises Comment 3, which states that a seller does not make an adequate contract if it makes an agreement with the
carrier that undervalues the shipment and thus cuts off the buyer’s remedy in the event of loss. The court says that this is very
different than not procuring adequate insurance.
o The court concludes that the reason for the allocation of the risk of loss in 2-509 is to notify that party to purchase insurance –
in this case, the buyer!
o “For this reason, the UCC has specifically established mercantile symbols which delineate the risk of loss in a transaction so that
the appropriate party might obtain insurance on the shipment. The contract in this case was FOB Seller’s Warehouse. Plaintiff
clearly bears the risk of loss in transit.”
 Rheinberg-Kellerei GmbH v. Vineyard Wine Co.
o Seller = Rheinberg-Kellerei, a West German wine producer; Buyer = Vinyard Wine Co, a North Carolina wine distributor
o The 620 cases of wine that Vinyard Wine Co. purchased from Rheinberg-Kellerei cost $8,600. Seller was to ship the wine, and
buyer was to pay upon delivery.
o The written confirmations were exchanged in August, but the wine wasn’t shipped until November 29. In the meantime, the
buyer’s owner, Cremilde Blank, kept phoning Sutton’s office to find out when the shipment would come, but received no
answer. The seller notified its agent Sutton that the wine was being shipped on November 27, but Sutton did not notify the
buyer.
o The seller’s bank notified the buyer’s bank (Wachovia Bank) that the wine was being shipped and sent it the paperwork on
December 27. The bank mailed this to the buyer on December 29. On January 24 the buyer learned that the ship, the
MS München, had been lost at sea sometime between December 12 and December 24.
o Which party has the risk of loss?
o The trial court found that the seller did not comply with Section 2-504 in that it did not notify the buyer that the goods had been
shipped, and that this prevented the buyer from being able to purchase insurance to cover the goods. While the seller gave
notice to its agent Frank Sutton, he did not pass on the notice to the buyer, and the buyer did not receive any of this information
until weeks after the cargo was lost.
o Obviously, the buyer refused to pay for the wine.
o This was a shipment contract. Normally the risk of loss passes to the buyer when the goods are “duly delivered” to the buyer. 2-
509(1)(a). The trial court entered judgment on behalf of the buyer.
o The seller does not challenge the facts as found by the trial court, only the legal conclusions to be drawn from those facts.
o Text: The Court quotes Sections 2-504 and 2-509(1)(a).
o Policy: “The requirement of prompt notification by the seller, as used in 2-504(c), must be construed as taking into
consideration the need of a buyer to be informed of the shipment in sufficient time for him to take action to protect himself
from the risk of damage to or loss of the goods while in transit.”

96
o Given the myriad factual situations which arise in business dealings and keeping in mind the commercial realities, whether
notification has been “prompt” within the meaning of UCC will have to be determined on a case-by-case basis under all the
circumstances.” Citing Hawkland.
o The court holds that the risk of loss stayed on the seller because the seller had failed to “promptly notify the buyer of the
shipment” under 2-504(c), and that the buyer was entitled to “reject the shipment” pursuant to 2-504(c).
o But was buyer really “rejecting the shipment”? Maybe so …
o Post-script: The evidence from a lifeboat torn from the ship indicates that it was hit by a “rogue wave” during a huge storm – a
wall of water 80 to 100 feet high. She drifted for 33 hours with a list of 50 degrees without electricity or propulsion.
o The ship was considered “unsinkable”, so it sailed into the storm. Many distress calls were heard, but no-one was able to find
the ship before it disappeared. Rescuers abandoned the search 2 days before Christmas. All hands were lost at sea.
 Casebook Problem
o The University of Beijing in China ordered video equipment to be shipped from Applied Technology, Inc., in San Jose, California.
If nothing is said about the subject, as a matter of international law, will this create a shipment or a destination contract? See
CISG Articles 67 and 69. If the parties had been negotiating for the purchase of this equipment but had not gotten around to
signing the contract until the goods were already on board an airplane crossing the Pacific Ocean, does the buyer have the
risk of loss only from the moment of the signing of the contract or from the delivery of the equipment to the air carrier? See
Article 68.
 Article 67 permits the parties to agree to anything they want in terms of risk of loss, but seems to presume a shipment
contract, with the risk passing to the buyer whenever conforming goods are “handed over to the first carrier,” though it can
be argued that the parties have not agreed, so Article 69 creates a destination contract by default. If the contract is signed
while the goods are en route, Article 68 at least dates the risk of loss from the moment of contracting, and even earlier,
back to the moment of shipment “if the circumstances so indicate,” whatever that means.
 Casebook Problem
o Dime-A-Minute Rent-A-Car rented a new sports car to Joseph Armstrong. Due to a snafu at the rental office, Armstrong did not
sign a rental agreement. As he was leaving the rental car lot, the car was struck by a city bus due to no fault of Armstrong (who
was unhurt). The sports car was totaled. Dime-A-Minute demanded that Armstrong look to his insurance to replace the car. Did
he have the risk of loss here? See §2A-219. If he had signed a rental agreement making him responsible for the car, would
that agreement be valid? See §§1-302, 2A-108.
 There is a strong presumption in §2A-219(1) that the lessor keeps the risk of loss.
 Absent clear agreement otherwise (such as a signed lease agreement covering this issue), a court is not likely to transfer the
risk to Armstrong.
 If he had signed a lease giving him the risk of loss, query whether this is unconscionable or not?
 Actually, our experience indicates that in car rentals, the consumer customer often agrees to accept the risk of loss – be
sure your auto insurance covers your leased vehicles!
 Canvas Quiz Questions
o Sandoval Appliances orally agreed to sell Ipswitch Cleaners four industrial washer-dryers for $2,100 apiece. The appliances
are in the storage area in the back of Sandoval's store. The signed purchase order and invoice reflected the terms as to
quantity and price but neglected to state the time and place of delivery. In the absence of any course of performance, course
of dealing, or trade usage, when and where are the washer-dryers to be delivered?
 Answer: Under this contract Ipswitch must pick up the washer-dryers at Sandoval's store within a reasonable time. - In the
absence of an agreement or any course of performance, course of dealing, or trade usage as to time and place of delivery,
Article 2 provides that the goods are to be delivered at seller's place of business at a reasonable time.
 Under this contract Sandoval must ship the washer-dryers to Ipswitch within a reasonable time.
 Under this contract Sandoval must deliver the washer-dryers to Ipswitch's store within a reasonable time.
 The contract fails for lack of an essential term.
o If the contract requires the seller to "send" the goods to the buyer, then under Article 2 this is considered a "destination
contract" and the seller has the duty to deliver the goods to the buyer's place of business.
 False - If the contract requires the seller to "send" the goods to the buyer, then under Article 2 this is considered a "shipment
contract" and the seller has the duty to deliver the goods to a carrier and make arrangements for shipment to the buyer's
place of business.
o Which of the following are duties that a seller has under a "shipment contract"? The seller must:
 Put the goods in the possession of a carrier
 Make a contract for the transportation of the goods that is reasonable under the circumstances
 Procure and deliver to the buyer any documents necessary to enable the buyer to obtain possession of the goods
 Promptly notify the buyer of the shipment
– Under a shipment contract the seller has all four of these duties.
o Which of the following statements about trade abbreviations are accurate?
 In North America, "F.O.B. [seller's place of business]" is a shipment contract.
 In North America, "F.O.B. [buyer's place of business]" is a delivery contract.

97
 A careful attorney should expressly set forth the delivery terms in a contract and not rely solely on abbreviations.
 Delivery terms are consistent all over the world. Terms like "F.O.B." mean the same thing in domestic and international
trade.
 Review/Assessment Questions
 When does a party have a right to adequate assurance of performance?
o Under §2-609, “When reasonable grounds for insecurity arise with respect to the performance of either party the other may in
writing demand adequate assurance of due performance and until he receives such assurance may if commercially reasonable
suspend any performance for which he has not already received the agreed return.”
o This is an important tool for a contract party who is not sure whether the other party will perform.
 Are unfair contract terms necessarily unconscionable?
o Unconscionability requires more than unfair contract terms, rather requires extreme unfairness. As the Official Comment puts it,
there must be a showing that “in the light of the general commercial background and the commercial needs of the particular
trade or case, the clauses involved are so one-sided as to be unconscionable under the circumstances existing at the time of the
making of the contract."
o Courts will usually require both procedural and substantive unfairness, as the book discusses.
 What is the effect of this provision in Amazon’s Terms of Use: “All purchases of physical items from Amazon are made pursuant to
a shipment contract. This means that the risk of loss and title for such items pass to you upon our delivery to the carrier”?
o If an Amazon package is stolen from buyer’s doorstep or porch, the risk of loss is on buyer (i.e. buyer still has to pay).
o Note that Amazon may not enforce that clause and may agree to reship the item.
 What are the meanings of F.O.B., C.I.F., C. & F., F.A.S., and ex-ship?
o The book spells out their meaning at pages 271-272. Please review these terms.
o The key is that the parties can concisely agree as to where the risk of loss shifts from seller to buyer (and that contracting parties
should be aware of the implications of those terms, and negotiate or insure accordingly)
 Does risk of loss depend on ownership of the goods or whether they have been accepted by buyer?
o Risk of loss does not depend on ownership of the goods!
 Curious Liquids signed a contract to sell Pemulis Auto its requirements of motor oil for the year, in monthly shipments at a price of
$5 per quart. Pemulis Auto usually needed about 3,000 quarts per month. After fracking-related earthquakes shut down production
of many wells, motor oil prices spiked. Pemulis Auto signed another contract to sell Josephson Jobber 100,000 quarts at $10 per
quart. Pemulis Auto then informed Curious Liquids that the requirement for the next month would be 103,000 quarts. Is Curious
Liquids obliged to deliver 103,000 quarts next month, which would be ruinous for it?
o Yes. Curious Liquids made a bad deal but is bound by the contract.
o No. The parties have no sales contract, because they did not agree about the most basic term, the quantity of goods. - Section 2-
306 permits quantity to be measured by the seller’s output or buyer’s requirements
Answer: No. Pemulis Auto is entitled only to its good faith, actual requirement. Its demand is not good faith or its actual
requirement. - Article 2 does allow the parties flexibility in forming a contract, without specifying all terms. Section 2-306(1)
means the “actual output or requirements as may occur in good faith.”. Here, Pemulis Auto asked for more than its actual
requirements, as contemplated at the time of the agreement. Nor did Pemulis Auto act in good faith, where it misrepresented its
actual requirements to take advantage of changing market conditions.
o Yes. Pemulis Auto has an actual requirement for 103,000 gallons, because it is obliged to deliver 100,000 quarts to Josephson
Jobber and needs 3,000 quarts for its own operations.
 On June 1, Pemulis Auto signed a contract to sell Redford a “2016 Model Excalibur Hardtop” automobile. A week later, Pemulis Auto
received three 2016 Model Excalibur Hardtops from the manufacturer. Two days after that, Pemulis Auto placed a sticker on one of
the three cars, designating it for delivery to Redford. On July 1, Redford tendered payment, Pemulis Auto signed the certificate of
title and handed over the keys, and Redford drove away. When were goods identified to the contract?
o On June 1, when the parties signed a contract specifically identifying the car to be sold. - Identification of goods to the contract
is not the same as specifying what the seller is obliged to deliver.
o When Pemulis Auto received three cars, meaning it could now deliver one to Redford.
Answer: When Pemulis Auto put a sticker on one of the cars. – Section 2-501 states how the seller, in effect, marks specific
goods as the ones to be delivered to the buyer. If the contract is for goods already existing and identified, identification occurs
when the contract was made. §2-501(1)(a). When the contract was made, Pemulis Auto was obliged to deliver a 2016 Model
Excalibur Hardtop, but no particular car had been designated to deliver to Redford. Even when three Hardtops were delivered,
no particular one was designated for Redford. Rather, when Pemulis Auto put a sticker on one, that identified that particular car
to the contract.
o When the car was paid for and driven away. - Identification of goods to the contract does not require the goods to be paid for
and delivered.
 Word got around that Pemulis Auto had some 2016 Model Excalibur Hardtops in stock. Pompeia soon made a contract with Pemulis
Auto to buy one as her retirement present to herself. Pompeia wired the full payment, and Pemulis Auto parked one of the cars to
deliver to her the next p. 283day. It was stolen overnight. Is Pompeia entitled to either get her payment back or a different car
delivered?
98
o No. She had bought and paid for the car, so it was her car that was stolen. - The risk of loss does not depend on whether
payment has been made.
o No. She would be reasonably required to obtain insurance for the car. - The parties can agree that one should provide insurance
but did not do so.
o Answer: Yes. The risk of loss would not pass to her until delivery, which did not occur. - The general rule, for nonmerchant
buyers, is that risk of loss passes when the goods are delivered. §2-509(3). There are other rules where the goods are being
transported, §2-509(1), or held by a bailee, §2-509(2), but neither is the case here. Likewise, the parties can agree about when
the risk of loss will pass, but did not do so here, so the general rule will apply.
o Yes. A merchant is required to insure the goods. - The parties can agree that one should provide insurance but did not do so.
 Blackhorse Goods signed a contract to sell 100 washing machines to Otto Automats. The washing machines were stored in a
warehouse. Blackhorse Goods negotiated the warehouse receipt to Otto Automats, who promised to pay the contract price in 30
days. Before that time, the warehouse was destroyed by a fire, through no fault of the warehouse operator. Is Otto Automats
obliged to pay for washing machines that are now scrap?
o No. Blackhorse Goods has not delivered the goods.
o No. The machines still belonged to Blackhorse Goods.
Answer: Yes. The risk of loss of the goods passed to Otto Automats with the warehouse receipt. - When goods are held by a
bailee, the risk of loss passes when a negotiable document covering the goods is delivered to the buyer. §2-509(2)(a). The
provision also addresses the issue when there is no document covering the goods or when the document is not negotiable.
o Yes. The risk of loss passed to Otto Automats upon signing the contract.
 Obergefell Furniture, in Boston, MA, entered into a contract with Unterwasser Healthcare, in Charleston, S.C., for the sale of medical
devices, to be delivered by a trucking company. In which case would the risk of loss pass from the seller to the buyer when the
goods were tendered to the carrier?
o If the parties did not specifically address the issue in the contract.
o If they used the shipping term “C.I.F. Charleston.”
o If the used the shipping term “F.O.B. Boston.”
o Answer: All of the above. - This question illustrates the role of shipping terms. In a transportation contract, if the parties do not
address the issue, the contract is a “shipment” contract, meaning risk of loss shifts upon shipment. The parties may decide when
the risk of loss shifts. The term C.I.F. designates a shipment contract, so the risk of loss would also shift upon shipment in B.
F.O.B. means the risk of loss shifts at the place designated. F.O.B. Boston means risk of loss shifts in Boston, where the seller is,
so this is also a shipment contract.
Chapter 6: Performance of the Contract
I. Installment Sales
 Definition of an “Installment Contract”
o Section 2-612(1) defines an installment contract as one that requires or authorizes goods to be delivered in separate lots to be
separately accepted.
 Section 2-612(1): Definition of an “Installment Contract”
An “installment contract” is one which requires or authorizes the delivery of goods in separate lots to be separately accepted, even
though the contract contains a clause “each delivery is a separate contract” or its equivalent.
 The Perfect Tender Rule Does Not Apply to Installment Contracts
o The “perfect tender rule” does not apply to installment contracts. A non-conformity in goods delivered in an installment does
not constitute a breach unless it “substantially impairs the value” of the installment.
o Furthermore, just because the goods delivered in one installment substantially impair the value of that installment does not
mean that the entire contract has been breached. To constitute a breach of the entire contract, the non-conformity must
“substantially impair the value” of the entire contract.
 Right to Reject an Installment for Substantial Breach
o The buyer may reject an installment if the non-conformity substantially impairs the value of that installment and cannot be
cured.
o In other words, with installment contracts the buyer has a duty to work with the seller – to allow the seller to cure, or to accept
payment in lieu of rejection for minor non-conformities.
 Section 2-612(2): Right to Reject an Installment for Substantial Breach
The buyer may reject any installment which is non-conforming if the non-conformity substantially impairs the value of that
installment and cannot be cured or if the non-conformity is a defect in the required documents; but if the non-conformity does not
fall within subsection (3) and the seller gives adequate assurance of its cure the buyer must accept that installment.
 Right to Cancel the Contract for Substantial Breach
o If a breach “substantially impairs the value of the whole contract” then the buyer may cancel the contract and sue for damages.
o However, if the buyer does not notify the seller of the breach within a reasonable time or if the buyer brings an action only with
respect to past installments then the whole contract is reinstated.
 Section 2-612(3): Right to Cancel the Contract for Substantial Breach

99
Whenever non-conformity or default with respect to one or more installments substantially impairs the value of the whole contract
there is a breach of the whole. But the aggrieved party reinstates the contract if he accepts a non-conforming installment without
seasonably notifying of cancellation or if he brings an action with respect only to past installments or demands performance as to
future installments.
 Example: The Faulty Gyroscopes: Breach of an Installment or Breach of the Whole Contract?
o Milan Rockets Inc. manufactures rockets used to launch satellites into orbit around the earth. The company does not maintain a
large inventory of parts but prefers to purchase parts on an ongoing basis as it constructs the rockets. Gyroscopes are a key
component of each rocket; they keep the rockets on course and prevent potentially disastrous accidents. Milan entered into a
contract with Sage Company to purchase gyroscopes for the rockets. Sage agreed to deliver 14 gyroscopes per month to Milan
on the 15th of every month of the year 2016, starting on January 15th. The first two shipments were fine, but in the third
shipment on March 15th four of the gyroscopes failed after a half hour of testing. This delayed the launch of four rockets. Does
Milan have the right to reject the March shipment and sue for damages? Does Milan have the right to cancel the whole
contract?
 ANSWER: If the defect in the four gyroscopes substantially impaired the value of the March installment and cannot be
cured, then Milan may sue for breach. The delay in the launch of four rockets would constitute a substantial impairment of
the value of the installment. If the defect in the four gyroscopes substantially impairs the value of the whole contract, then
Milan may cancel the whole contract. What argument would you make on behalf of Milan that this nonconformity
substantially impairs the value of the whole contract, and that this impairment cannot be cured?
 Casebook Problem
o Travis Galleries developed a market in copies of famous statues. It ordered monthly shipments of the statues from Ersatz
Imports, agreeing to take 12 shipments of 20 statues each over the coming year. The first month all of the statues arrived upside
down in their cartons. The manager of Travis Galleries was amazed that most had survived the trip in this condition. Only one
was broken, and a phone call to Ersatz Imports resulted in a promise to ship a replacement at once. The next month the statues
were again packaged p. 287upside down, and half of them were broken. Does §2-612 permit rejection for this reason? Assume
that Ersatz replaced the broken statues within a week, but that the next month the shipment contained no statues at all.
Instead, Ersatz had mistakenly shipped Travis poor copies of 20 French Impressionist paintings. Travis Galleries called you, its
attorney. May it reject this shipment? Under what theory? May it now cancel the remainder of the Ersatz contract? In any
rejection situation, is it wise for attorneys to ask their clients what really underlies their own actions?
 Section 2-612 does all it can to keep the contract going in spite of deficiencies in the tender of some shipments.
 On the other hand, at some point enough is enough and the disappointed buyer must be allowed to escape from its
contractual bonds.
 Judging this point is the trick, and it is impossible to lay down hard and fast rules about the matter.
 As to any one installment the legal test is substantial performance. Even if the goods do not substantially comply, the buyer
must accept the goods if the defect can be cured and the seller gives adequate assurance of that cure.
 For the second shipment of statutes, half of them are broken and this is likely to be deemed a substantial impairment of
the value of this installment, but since Ersatz replaced the broken statues within one week, the cure has been made.
 The next month the seller ships the wrong goods, and at this point buyer’s confidence in seller may be so destroyed that §2-
612(3) permits the buyer to declare that a “breach of the whole” contract has occurred, and buyer will take no further
deliveries.
 Whether a breach of the whole has in fact occurred is a factual question (and a wide-awake attorney might make use of §2-
609’s right to adequate assurance as to future performance in order to force the issue).
 Cherwell-Ralli, Inc. v. Rytman Grain Co.
o First line of the case: “This case involves a dispute about which of the parties to an oral installment contract was the first to be in
breach.”
o Plaintiff Cherwell-Ralli delivered meal to the defendant Rytman Grain in weekly installments. The buyer was behind in
payments. Buyer finally wrote a check in the amount of $9,800, but when a truck driver that was not employed by the seller told
the buyer it was his last run because the seller’s plant was closing, the buyer stopped payment on the check. The buyer owed
the seller $21,000 at this point. The buyer then said it would not pay unless it received assurances of future deliveries.
o The trial court entered judgment for the seller-plaintiff and dismissed the buyer’s counterclaim. Stopping payment on the check
certainly had a substantial effect on the value of the buyer’s performance and justified the seller in canceling the contract and
suing for damages.
o The buyer argued that the seller in an installment contract is not permitted to cancel the contract without first demanding
assurances under Section 2-609. The court responds, “This is not the law.”
o And the seller did not reinstate the contract by suing for past performance only – as the court said, “as the buyer well knew” the
seller was cancelling the remainder of the contract and was suing for the 19 shipments that the buyer hadn’t paid for. The seller
was not suing for past performance only and trying to keep the contract open.
o Finally, the buyer argued that the seller had to answer the buyer’s demand for assurances. The court held that the buyer had no
right to “demand assurances” of future conduct since it was in breach of the whole contract. “The buyer could not rely on its
own nonpayments as a basis for its own insecurity.”

100
o The seller’s plant closed because of a surplus, not a shortage of grain.
o Take-away:
 The buyer Rytman Grain materially breached first. Therefore, the buyer does not have the right to demand assurance of
due performance from the seller Cherwell Ralli under §2-609.
 Also, the breach by the buyer was a total breach (paying with a bad check). The seller was entitled to cancel the contract
and sue for the $21,000 that the buyer owed for prior shipments received by the buyer.
o The case shows that there is a breach of the whole contract if breach as to one installment substantially impairs the value of
the contract. This applies to both buyers and sellers, so where buyer stopped payment on the check for an installment (relying
only on a rumor from a truck driver) and failed to pay for outstanding shipments, seller could terminate the contract.
o The court also rejected buyer’s argument that seller had failed to provide adequate assurance of performance under §2-609 - a
useful innovation in Article 2 that lawyers should have in their arsenal—allows a party to require assurances where it has
reasonable grounds for insecurity.
o So, where seller’s plant burns down, buyer may require adequate assurance of performance (e.g., seller can show another
source for the goods). If seller cannot provide assurance, buyer can terminate the contract and buy elsewhere.
o Likewise, if buyer’s store closes, seller may require assurance about buyer’s ability to accept and pay for next month’s shipment
of inventory.
o But in Cherwell-Ralli, buyer’s own failure to pay could not give rise to buyer being entitled to adequate assurance about seller’s
performance
 Summary: Installment Contracts
o An installment contract is one that requires goods to be delivered in separate lots to be separately accepted.
o A non-conformity in goods delivered in an installment does not constitute a breach unless it “substantially impairs the value” of
the installment and cannot be cured. To constitute a breach of the whole contract, the non-conformity must “substantially
impair the value” of the whole contract. If a breach “substantially impairs the value of the whole contract” then the buyer may
cancel the whole contract.
 Summary: Substantial Performance
o In installment sales, “substantial performance” is still the law, as it was under the common law. (See, e.g., Jacob & Youngs v.
Kent, by Cardozo). Ellen Peters’ famous article covers this:
o The basic reason for the difference between the common law “substantial performance / material breach” rule and Article 2’s
“perfect tender rule” is because imperfect goods can be returned to the seller, which in a services or construction contract, the
buyer cannot return the service provider’s performance to the service provider. Cardozo actually mentions this point in Jacob &
Youngs v. Kent.
 Canvas Quiz Questions
o Fitzsimmon's Aviary has contracted to sell exotic birds to Sam's Flying Casino. The contract calls for a different species of exotic
bird to be delivered to the casino on the 1st of each month for a period of two years. The birds would be auctioned off to
customers at the end of each month.
In the fourth month of the contract Fitzsimmon's delivered Myna birds to the casino. One of the birds would land on the
shoulder of patrons and squawk, "Sucker!" (An animal caretaker at the Aviary had trained the bird to say that.) Sam Andrews,
the owner of the casino, says to you, his attorney, "I want to cancel the whole contract. Am I entitled to do that?" Which of the
following advice would you give him?
 "Sam, you are entitled to sue the aviary for breach of this installment because of the wiseacre Myna bird, but you aren't
entitled to cancel the whole contract."
 "Sam, you are entitled to cancel the whole contract."
 "Sam, you are entitled to cancel the whole contract, and we can do that later, but for now let's just sue the aviary for the
damages you suffered from the delivery of the Myna bird."
 Answer: "Sam, keep that bird. It's worth a million dollars at the auction!"
II. Firm Offers
 Article 2 Changes the Common Law with Respect to “Firm Offers”
o At common law a party could retract an offer at any time prior to acceptance, even if the party had promised to leave the offer
open.
o The courts at common law reasoned that since there was no consideration in return for the promise to keep the offer open, an
offer could be retracted.
o This common law rule is changed by the “firm offer rule” of Section 2-205.
 Elements of a “Firm Offer”
o Section 2-205 applies only to firm offers made by “merchants.” (Merchants are defined in Section 2-104(1).)
o A firm offer must be in the form of a “signed writing.” (“Signed” and “writing” are defined in Section 1-201.)
o A firm offer remains open for the period stated, or if no period is stated for a “reasonable time”; but in no event longer than
three months.
 Section 2-205: “Firm Offer”

101
An offer by a merchant to buy or sell goods in a signed writing which by its terms gives assurance that it will be held open is not
revocable, for lack of consideration, during the time stated or if no time is stated for a reasonable time, but in no event may such
period of irrevocability exceed three months; but any such term of assurance on a form supplied by the offeree must be separately
signed by the offeror.
o “Signed” §1-201(b)(37)
o “Writing” §1-201(b)(43)
 Official Comments to §2-205
o This section is intended to modify the former rule which required that "firm offers" be sustained by consideration in order to
bind, and to require instead that they must merely be characterized as such and expressed in signed writings.
o The primary purpose of this section is to give effect to the deliberate intention of a merchant to make a current firm offer
binding. The deliberation is shown in the case of an individualized document by the merchant's signature to the offer, and in the
case of an offer included on a form supplied by the other party to the transaction by the separate signing of the particular clause
which contains the offer.
o "Signed" includes authentication but the reasonableness of the authentication must be determined in the light of the purpose of
the section.
o A handwritten memorandum on the writer's letterhead purporting in its terms to "confirm" a firm offer already made would be
enough to satisfy this section.
o Similarly, an authorized telegram will suffice, even though the original draft contained only a typewritten signature.
o Despite settled courses of dealing or usages of the trade whereby firm offers are made by oral communication and relied upon
without more evidence, such offers remain revocable since authentication by a writing is the essence of this section.
o The three-month period during which firm offers remain irrevocable under this section need not be stated by days or by date.
o If the offer states that it is "guaranteed" or "firm" until the happening of a contingency which will occur within the three-month
period, it will remain irrevocable until that event.
o A promise made for a longer period will operate under this section to bind the offeror only for the first three months of the
period but may of course be renewed. If supported by consideration it may continue for as long as the parties specify. This
section deals only with the offer which is not supported by consideration.
o Protection is afforded against the inadvertent signing of a firm offer when contained in a form prepared by the offeree by
requiring that such a clause be separately authenticated.
If the offer clause is called to the offeror's attention and he separately authenticates it, he will be bound; Section 2-302 may operate,
however, to prevent an unconscionable result which otherwise would flow from other terms appearing in the form.
 Canvas Quiz Questions
o Teresa made an offer to sell a diamond tie clip for $1,000 and promised that this offer would remain open. Under what
circumstances would this offer not be binding on Teresa?
 If Teresa is not a merchant.
 If the offer is not in the form of a signed writing.
 After a reasonable time, no longer than three months.
 At any time because under the law if a promise to keep an offer open is not supported by consideration the offeror may
retract the promise at any time, even if the promise was by a merchant in the form of a signed writing.
III. Perfect Tender Rule
 The U.C.C. Adopts the “Perfect Tender Rule”
o Under the general common law of contracts, if a party tenders performance that is not perfect but that does not constitute a
material breach, then the other party could not reject the tender and cancel the contract. This was the “substantial
performance” standard rejected by Judge Learned Hand.
o But there is an exception to this rule for the sale of goods; traditionally if the goods failed to conform to the contract in any
respect then under the common law the buyer was entitled to reject the goods. The exception that was applicable to the sale of
goods is called the “perfect tender rule.”
o The exception that was applicable to the sale of goods is called the “perfect tender rule.” Section 2-601 retains the “perfect
tender rule” for the sale of goods.
o Section 2-601 retains the “perfect tender rule” for the sale of goods.
 Section 2-601: Buyer’s Rights on Improper Delivery – the “Perfect Tender Rule”
Subject to the provisions of this Article on breach in installment contracts (Section 2-612) and unless otherwise agreed under the
sections on contractual limitations of remedy (Sections 2-718 and 2-719), if the goods or the tender of delivery fail in any respect to
conform to the contract, the buyer may
(a) reject the whole; or
(b) accept the whole; or
(c) accept any commercial unit or units and reject the rest.
 But Should the Code Be Taken Literally?
o Section 2-601 states that if the seller’s performance fails to conform to the contract “in any respect” then the buyer may cancel
the whole contract.
102
o What if the goods were delivered by U.P.S. instead of Fedex, as called for in the contract, but they arrived on time?
o What if the goods had a trivial defect that was easily repaired by a minor adjustment?
o What if the goods that were delivered were actually of higher quality than the goods specified in the contract?
o Would a court actually rule in favor of a buyer who cancelled the contract in those circumstances?
 Answer: Good Faith
o Under Section 1-304 good faith is an obligation in the performance of every contract. Section 1-201(b)(20) provides that “good
faith” means “honesty in fact and the observance of reasonable commercial standards of fair dealing.”
o A buyer who rejects goods for an immaterial or insubstantial defect runs the risk of being found not to have acted in good faith
and would incur liability for breach of contract.
o Partial acceptance (§2-601(c)) in good faith is recognized and the buyer's remedies on the contract for breach of warranty and
the like, where the buyer has returned the goods after transfer of title, are no longer barred.
 Mitigation of Damages
o Under the common law, a party had a duty to mitigate its damages. Although this concept is not expressly set forth in Article 2
of the U.C.C., it may constitute one aspect of the buyer’s duty of “good faith.”
o A buyer who rejects goods for insubstantial reasons might be prevented from recovering damages on the ground that the buyer
has to mitigate its damages. If the buyer would have suffered only nominal damages if the buyer had accepted the goods, the
buyer may be estopped from suing. If the seller offers compensation for any minor non-conformity, to mitigate damages the
buyer ought to accept the compensation instead of rejecting the goods and suing for breach.
o If the buyer would have suffered only nominal damages if the buyer had accepted the goods, the buyer may be estopped from
suing. If the seller offers compensation for any minor non-conformity, to mitigate damages the buyer ought to accept the
compensation instead of rejecting the goods and suing for breach.
 Cure
o Moreover, even if the buyer rightfully rejects goods that are not conforming the seller may have the right to “cure” – that is, to
deliver goods that conform to the contract – pursuant to Section 2-508.
o If a buyer rejects goods because they are not “perfect,” the buyer still must allow the seller to cure the defect.
o The seller’s right to cure is covered in §2-508.
What can a Buyer do on receipt of nonconforming goods?
(a) reject the whole; or
(b) accept the whole; or
(c) accept any commercial unit or units and reject the rest.
(a) Note: rejection of the whole must be done in “good faith” with Buyer subject to duty to mitigate. And, Seller has the
right to cure (§2-508).
(b) Note: a buyer accepting a non-conforming tender is not penalized by the loss of any remedy otherwise open to him
(but cf. §2-718, §2-7190.
(c) Note: Partial acceptance in good faith is recognized and the buyer's remedies on the contract for breach of warranty
and the like, where the buyer has returned the goods after transfer of title, are no longer barred. The only limitation on
partial acceptance is that good faith and commercial reasonableness must be used to avoid undue impairment of the
value of the remaining portion of the goods. This is the reason for the insistence on the "commercial unit" in paragraph
(c).
 Casebook Problem
o Stella Speculator, a wealthy investor, signed a contract with Swank Motors to buy five new cars. All five were to be delivered on
October 1. When the cars arrived, she test drove each of them and then returned two of them, saying she would keep the other
three. She rejected the two cars because the audio system did not work in one of them (she was a great music lover) and the
carpeting in the trunk of the other was ripping. Swank Motors offered to repair both defects. When Speculator refused to
permit repair, Swank sued. Answer these questions:
(a) Does the common law doctrine de minimis non curat lex (“the law does not notice small defects”) survive §2-601? If so, in
spite of the tiny defects, the cars would be conforming. See Official Comment 2 to §2-106; see D.P. Technology Corp. v.
Sherwood Tool, Inc., (where goods specially manufactured, buyer not in good faith in rejecting because of insubstantial delay in
delivery).
 “De minimus non curat lex” is probably still with us; as to whether it applies here depends on how minor the defects are.
 If Speculator’s experience with audio systems is that they are hard to repair if they do not work right at first, a court might
well permit her to reject.
 As to the carpet ripping in the trunk, that seems more than de minimus. With a new car the buyer should not have to live
with a repaired trunk interior (or a money allowance); cf. §2-508 (re: cure).
(b) A seller has a right to cure in some circumstances; see §2-508. Is this section of use to Swank Motors? See Wilson v. Scampoli,
 In Wilson, (reprinted at p. 291 after the next Problem), the court held that a buyer too hastily rejected a television set that
was easily repaired. Cf. §2-508.

103
(c) Suppose Swank can demonstrate that it is common for car sellers to correct small defects. Will Swank succeed if it argues that
such correction is a usage of trade (§1-303(c)) and thus either that the goods are conforming or that because of this usage of
trade, the parties have impliedly agreed that a §2-601 perfect tender is not required? See §§1-302, 1-201(b)(3).
 Usage of trade (or the parties’ past course of dealing) can be used to establish the standards by which the perfectness of
the tender is measured. Swank might very well succeed with this cure argument, which is the argument that the seller in
effect used in Wilson.
• Query: What are the limits on the buyer’s right to reject for minor defects?
 Canvas Quiz Questions
o Under the common law if the seller delivered goods that were non-conforming in a trivial or non-substantial respect, then
the buyer had the right to reject the goods. The U.C.C. changed the law and eliminated the "perfect tender rule."
 False - It is actually the reverse. At common law a buyer did not have the right to reject a tender of delivery for trivial or
insubstantial reasons, but rather had the right to reject only on account of a "material breach." The U.C.C. actually replaced
the "material breach" rule with a "perfect tender rule" in Section 2-601.
o Should the perfect tender rule be taken literally? Will the courts actually permit a buyer to reject goods and cancel the
contract on account of a trivial or insubstantial defect? What other principles might come to bear upon such a situation?
 The buyer must act in good faith. A buyer won't be permitted to cancel a contract on account of some trivial defect when
the real reason is that the buyer simply does not wish to perform its obligations under the contract.
 The duty to mitigate damages. A buyer has the right to cancel a contract for any conceivable non-conformity. But if the non-
conformity is easily corrected that might be the more economically efficient result.
 The seller's right to cure. Section 2-508 gives the seller the right to cure non-conformities in certain situations.
– The perfect tender rule is the law under Section 2-601, but the requirements of good faith, mitigation, and cure
militate against a literal application of the perfect tender rule.
o Tim ordered an espresso machine from Coffee Nuts. When it arrived, Tim noticed right away that the company had sent the
wrong size - they sent a smaller machine than Tim had ordered, although they charged him less as well. Instead of repacking the
machine and sending it back to Coffee Nuts, Tim decided to use the machine at a party he hosted and for several friends who
visited on different evenings. After two weeks of use Tim is ready to return the machine. Does he have the right to reject the
coffee maker?
 Answer: Not now! Tim's "exercise of ownership" over the espresso machine negated his right to reject it. If Coffee Nuts
wishes it may take the position that Tim has accepted it.
 Yes, Tim can reject the machine and return it for a full refund. It is non-conforming to the contract.
o Chaz ordered a three-piece suit online from NYC Fashions. When the suit arrived, he found that the pants were not pleated
correctly. He likes the vest and the coat, though. He notifies the seller that he is rejecting the pants but has decided to keep
the vest and the jacket, and that he will pay for those two items only.
 Answer: If NYC objects Chaz will not be permitted to reject only the pants. The entire three-piece suit is a "commercial
unit," and the buyer must accept or reject a commercial unit as a whole, not in parts.
 Chaz may accept or reject the pants, vest, and jacket individually. Each of these is a separate "commercial unit."
 Chaz may accept or reject the pants, vest, and jacket individually because Chaz is not a merchant. - It makes no difference
that Chaz is not a merchant.
o Chaz also ordered two three-piece suits from SF Wardrobe. When they arrived, one fit him but the other did not. He wants to
reject the one that doesn't fit and accept the other. He may do this because each suit is a commercial unit.
 True - Each suit is a separate commercial unit.
o If the defect is something that the seller could have cured, the buyer has the duty to specify the defect; otherwise, the
rejection is ineffective.
 True
o Once the buyer has accepted the goods
 The buyer is liable to pay the contract price.
 The buyer cannot reject the goods.
 The buyer may not under any circumstances revoke its acceptance.
o Yummy in the Tummy is a store specializing in selling desserts. It ordered 1000 frozen banana cream pies from The Value of Pie
for $5000, and Yummy paid the entire amount in advance. Unfortunately, instead of banana cream the manufacturer shipped
1000 cherry custard pies, which do not sell as well, and sell for less than banana cream. Unfortunately, the frozen pies have a
short shelf life; if Yummy ships the pies back to Value they won't be worth much at all. Yummy asks Value for instructions but
Value does not respond. What may Yummy do?
 Put the pies on the sidewalk in the hot sun and sue for the return of the entire purchase price.
 Answer: Yummy is a merchant. Under Section 2-603 Yummy has a special responsibility with respect to perishable goods
that it has rightfully rejected. Yummy should keep the pies frozen and resell them in a commercially reasonable manner,
then sue for damages to recover any shortfall in the purchase price, any loss for the "benefit of the bargain," and any extra
expenses incurred in storing and selling these non-conforming pies.

104
 Yummy rightfully rejected the goods and Value has not communicated any instructions. Yummy has the right under Section
2-604 to store, reship, or resell the goods, at its discretion.
– Under these circumstances, involving perishable goods, the merchant buyer had a special responsibility to make
reasonable efforts to sell the goods for the seller's account.
IV. Cure
 The Seller’s Right to Cure
o In most circumstances the seller must be given an opportunity to “cure” a breach of contract or breach of warranty.
o Problem: Seller has not made a “perfect tender” and buyer has rejected the goods.
o Solution: The seller has the right in some circumstances to cure the defective performance.
o In the real world: In most circumstances the seller must be given an opportunity to “cure” a breach of contract or breach of
warranty.
o Rule: Cure is governed by Section 2-508. Section 2-508(1) applies if the time for performance has not yet expired. Section 2-
508(2) applies if the time for performance has expired.
 Section 2-508: Cure by Seller of Improper Tender or Delivery; Replacement.
(1) Where any tender or delivery by the seller is rejected because non-conforming and the time for performance has not yet
expired, the seller may seasonably notify the buyer of his intention to cure and may then within the contract time make a
conforming delivery.
(2) Where the buyer rejects a non-conforming tender which the seller had reasonable grounds to believe would be acceptable with
or without money allowance the seller may if he seasonably notifies the buyer have a further reasonable time to substitute a
conforming tender.
 If Time for Performance Has Not Expired
o If the seller delivers non-conforming goods to the buyer and the buyer rejects those goods, under Section 2-508(1) if the time
for delivery of the goods has not yet expired the seller has the right to “cure” by delivering goods that are conforming to the
contract.
If Time for Performance Has Expired
o If the delivery date has already passed and the buyer rejects the goods as non-conforming, then Section 2-508(2) applies.
o Under Section 2-508(2), the seller may have a “further reasonable time” to deliver conforming goods.
 The Seller Must Act in Good Faith
o In order to have the right to “cure” under Section 2-508(2) the seller must reasonably and in good faith have believed that the
goods would be acceptable to the buyer, with or without a money allowance.
o If a seller deliberately delivers unacceptable goods to gain time, that is not an action that is in good faith, and the seller would
not have the right to cure.
o Seller’s duty of good faith mirrors buyer’s duty of good faith.
 “Enough is Enough”
o A seller is not entitled to an unlimited amount of time or an unlimited number of attempts to cure a defect to nonconforming
goods.
o If a seller is unable to cure a defect within a reasonable time or after a reasonable number of attempts, the buyer is entitled to
reject the goods and to be reimbursed for any portion of the purchase price that has been paid.
 Query
o Under what circumstances does a seller have the right to cure a non-conformity in the goods?
o How much time does a seller have and how many attempts is a seller entitled in exercising the right to cure?
o Does a seller have a right to cure if the buyer’s “faith” in the product has been “shaken”?
 Zabriskie Chevrolet, Inc. v. Smith
o Automobile dealer sued buyer of new automobile for balance of purchase price and incidental charges, and buyer
counterclaimed for return of his deposit and incidental charges.
o The NJ Superior Court held that where buyer's wife in driving new automobile home from dealer's showroom found it practically
inoperable because of transmission so defective as to require replacement and where buyer almost concomitantly with
discovery of failure called dealer and cancelled sale, there was no ‘acceptance’ of the automobile by the buyer.
o Plaintiff urges that under §2-508 it had a right to cure the nonconforming delivery.
o After examining §2-508, the Court framed the issue: “The inquiry is as to what is intended by ‘cure,’ as used in the Code. This
statute makes no attempt to define or specify what a ‘cure’ shall consist of. It would appear, then, that each case must be
controlled by its own facts.”
o “The ‘cure’ intended under the cited section of the Code does not, in the court's opinion, contemplate the tender of a new
vehicle with a substituted transmission, not from the factory and of unknown lineage from another vehicle in plaintiff's
possession. It was not the intention of the Legislature that the right to ‘cure’ is a limitless one to be controlled only by the will of
the seller.”
 Casebook Problem
o On August 8, Francis and Sophie Ferdinand ordered a new car from Princip Motors for $22,000. The car was scheduled for
delivery “no later than September 1” (it had special accessories that had to be installed at the factory). On August 15, Princip
105
Motors told the Ferdinands that the car was ready, so they picked it up. Halfway home (three miles from the car dealer), the
engine blew up without warning. The Ferdinands were not hurt, but the engine was destroyed. On being informed that the
Ferdinands wanted their money back, Princip made the following responses:
(a) Princip offered to take an engine out of a car of the same model and install it in the original automobile (which was otherwise
undamaged). Does §2-508 require the Ferdinands to accept either of these cure offers?
Zabriskie rejected this argument.
(b) Princip refused to refund the money; instead, it claimed a right to give the Ferdinands a new car to be delivered fresh from the
factory on August 20. Does §2-508 require the Ferdinands to accept either of these cure offers?
The argument here is that per §2-508(1) the time for performance (“within the next few weeks”) has not yet passed.
But would it fail here because of the “Shaken Faith” Doctrine? Would their faith be “shaken” if they were given a new car? Per
the Zabriskie court, ““For a majority of people the purchase of a new car is a major investment, rationalized by the peace of
mind that flows from its dependability and safety. Once their faith is shaken, the vehicle loses not only its real value in their eyes
but becomes an instrument whose integrity is substantially impaired and whose operation is fraught with apprehension.”
 Wilson v. Scampoli
o The seller wants to take the new color television back to the shop for adjustment. Does the seller have that right, or is the buyer
entitled to a new television?
o Wilson gives an example of how buyer’s right to perfect tender is affected by seller’s right to cure.
o Buyer refused to allow seller to inspect and correct the newly delivered color television set, rather insisting on receiving another
brand new set.
o But §2-508 gives seller a right to cure in such circumstances. There is a right to cure if seller reasonably believed buyer would
accept the goods. This could be because seller thinks that buyer would take the goods despite a known defect, or because seller
reasonably does not know about the defect, such as here, where a retailer delivers goods from a wholesaler.
o Questions following Wilson
1. Is the court following the perfect tender rule §2-601?
• The court does not address the question clearly, but apparently it is saying that because of the ease of adjustment this
was a perfect tender rule, with usage of trade permitting the seller to tinker with this complex machine.
2. What portion of §2-508 is relevant to this case?
• Subsection (2) is the surprise rejection rule, cited by the court. Query whether this subsection is to be read as saying:
• (a) seller may cure whenever seller knew of the problem but thought buyer wouldn’t care, or
• (b) seller didn’t know of the problem (and shouldn’t have), but now that it has come up, seller should have a right to fix
it.
• Practice pointer: you should advise your buyer clients that rejection/revocation without allowing the seller a chance to
fix the problem is usually a losing legal tactic.
4. Can buyer reject the goods and refuse to return them?

• Section 2-711(3) permits a rightfully rejecting/revoking buyer to refuse to return the goods, claim a security interest in
them for the amount of payments made or expenses incurred, and (if, as is likely, the seller does not promptly repay
the claimed amount) resell the goods under §2-706.
• Practice pointer: Since this gives the buyer much leverage in any settlement discussion, your buyer-clients should not
be too quick to re-deliver the offending goods to the seller.
 Summary of Cure
o In most cases a seller has the right to cure any nonconformity in goods that have been delivered and rejected by the buyer. If
the time for the seller’s performance has not yet expired when the goods are rejected, the seller has an unqualified right to
deliver conforming goods. If the time for the seller’s performance has expired, a seller who is acting in good faith has a further
reasonable time to cure any nonconformity.
o A seller does not have an unlimited amount of time nor is the seller allowed an unlimited number of attempts to cure a defect. A
seller must cure any nonconformity within a reasonable time.
 Canvas Quiz Questions
o If the seller delivers non-conforming goods to the buyer and the buyer discovers the defect and notifies the seller and the
time for performance has not yet expired, then the seller may notify the buyer of its intention to cure and then within the
contract time make a conforming delivery.
 True
o Yo Yo Industries agreed to deliver 1000 yoyos to Games Unlimited on December 1. Yo Yo was unable to produce the goods on
time so it deliberately shipped 1000 defective yo yos to Games Unlimited, which were delivered on December 1. Two days later
Yo Yo notified Games Unlimited that it had mistakenly shipped the wrong product and it sent 1000 conforming yo yos, which
arrived on December 4. What are the legal consequences that would flow from this series of events?
 Games Unlimited must accept the conforming yo yos because the seller has a right to cure.
 Yo Yo did not have a right to cure in this situation because the time for performance had expired and Yo Yo did not act in
good faith.

106
 Yo Yo breached the contract and did so in bad faith. Games Unlimited therefore has the right to cancel the contract, even
though conforming goods were delivered only three days late.
 Yo Yo breached the contract by delivering conforming goods three days late, but this is an insubstantial breach and Games
Unlimited does not have the right to cancel the contract because of it.
– While the breach was fairly minor, Games Unlimited does not have to continue to deal with a supplier who acts in
bad faith. I think that Games Unlimited has the legal right to cancel the entire contract.
B. Performance and Breach Under Article 2 of the U.C.C.
 The Sale of Goods
o Contracts for the sale of goods are ubiquitous. Every car that is sold, every online purchase from Amazon or eBay, every piece of
food that is ordered at a restaurant, served over fast-food counter, or picked up at the grocery store involves a contract for the
sale of goods.
o Article 2 of the Uniform Commercial Code establishes a three-stage process governing performance and breach by buyers and
sellers of goods. A breach may occur at any of these three stages.
 Three-Stages at Which a Breach May Occur Under Article 2
1. Before the Goods Are Delivered
 The first-time frame in which a breach may occur is before performance. If either of the parties repudiates the contract
before their own performance is due, then the contract is canceled, and the aggrieved party is entitled to damages for
breach of contract.
 Example: Anticipatory Repudiation by Seller
• If Cooper Technologies entered into a contract with Carolina Circuits to sell Carolina 2,500 high-performance graphics
cards for a total cost of $30,000, and before the delivery date Cooper notified Carolina that it was not going to honor
the contract, then Cooper would be liable to Carolina for breach of contract.
2. When the Goods Are Delivered
 The second time frame within which a breach may occur under Article 2 is when the goods are delivered. When the goods
are delivered the buyer must decide whether to accept or reject the goods. If the buyer rejects the goods, then in most
circumstances the seller must be given an opportunity to cure any non-conformities in the goods.
 Seller’s Duty to Deliver: If the seller fails to deliver the goods on time, then the seller has breached.
 The “Perfect Tender Rule”: The seller is obligated to deliver goods that conform precisely to the contract. (The “perfect
tender rule.”)
 Inspection, Acceptance, Rejection, Cure: If the seller delivers the goods on time, then the buyer must choose whether to
accept or reject them. The buyer has a reasonable time to inspect the goods before deciding whether to accept or reject
them. If the buyer decides to reject the goods, then the buyer must notify the seller within a reasonable time and identify
the particular defect that is the basis for rejection. The seller then has a reasonable time within which to cure the defect. If
the seller cannot cure the defect, then the buyer may return the goods and sue for breach. If the buyer wrongfully rejects
the goods, then the seller may sue for breach
 Legal Consequences of Buyer’s Acceptance of the Goods: Once the buyer accepts the goods then the buyer may not reject
the goods; the buyer is obligated to pay for the goods; and if the buyer wishes the revoke acceptance, the burden of proof
shifts to the buyer to prove that the goods are substantially nonconforming.
 Example: Wrongful Rejection by Buyer
• Cooper Technologies entered into a contract with Carolina Circuits to sell Carolina 2,500 high-performance graphics
cards for a total cost of $30,000. When the cards were delivered Carolina inspected them, found that they were non-
conforming, returned them to Cooper, and canceled their order. Carolina did not give Cooper an opportunity to cure
the defect in the cards.
• Assuming that Cooper could have cured the defect if given the opportunity, then Carolina has breached the contract
and Cooper is entitled to recovery for any damages it sustained.
3. After the Buyer Has Accepted and Paid for the Goods
 The third time frame when a breach of contract for the sale of goods may occur is after acceptance. If it turns out that the
goods are non-conforming then the buyer may be entitled to sue for breach of warranty, which is measured by the
difference between the actual value of the goods as delivered and the value that the goods would have had if they had
been as warranted.
 In the alternative, under certain circumstances a buyer is entitled to revoke its acceptance of nonconforming goods and
return the goods to the seller. The attempted exercise of this right to revocation of acceptance may result in a breach by
the seller or by the buyer.
 Latent Defects and the Buyer’s Right to Revoke Acceptance
• After the buyer has accepted the goods the buyer has an additional reasonable time to discover any latent defects in
the goods. Once a latent defect is discovered the buyer must notify the seller of the defect and the seller has a
reasonable time within which to cure the defect. If the seller is unable to cure, and if the non-conformity substantially
impairs the value of the goods to the buyer, then the buyer may revoke its acceptance and return the goods to the
seller. The buyer would then be entitled to the return of its purchase price as well as any expectancy, incidental and
consequential damages.
107
• In the alternative, a buyer may elect to retain the non-conforming goods. The buyer would then be entitled to
expectancy damages for breach of warranty, which is measured by the difference in value between the goods as
delivered and the value that the goods would have had if they had been conforming to the contract.
• If a buyer wrongfully revokes acceptance, then the buyer has breached and if the seller has thereby suffered a loss the
seller would be entitled to recovery for breach of contract.
 Example: Rightful Revocation of Acceptance by Buyer
• Cooper Technologies entered into a contract with Carolina Circuits to sell Carolina 2,500 high-performance
graphics cards for a total cost of $30,000. When the cards were delivered Carolina inspected them but Carolina
failed to discover a latent defect in the cards. Carolina paid Cooper for the cards.
• Six months later the latent defect surfaced. Carolina promptly notified Cooper of the defect and gave Cooper an
opportunity to cure it. Cooper was unable to cure the defect.
• Carolina has a choice. If the defect is substantial Carolina may return the defective cards to Cooper and assert its
rights for breach of contract, which will include a return of the purchase price. Or Carolina may retain the cards
and assert its rights to breach of warranty, which is measured by the difference between the value that the cards
would have had if they had been conforming to the contract and the actual value that the cards had when they
were delivered to Carolina.
Performance and Breach Flowchart
Contract Formed
Seller – Duty to Deliver Goods Buyer – Duty to Accept Goods and Pay
Repudiation
If Seller repudiates, Buyer entitled to damages If Buyer repudiates, Seller gets damages
Anticipatory Repudiation
Seller: “cannot and will not Buyer: “Will not accept or pay for goods”
deliver goods” Seller entitled to damages
Buyer entitled to damages
Failure of Delivery
Seller fails to deliver goods on time - Breach! Buyer entitled to damages (e.g., cover)
Delivery of Nonconforming Goods
Goods are delivered but they do not conform to contract -> Perfect Tender Rule
Buyer 1. Accept the whole
has a. Acceptance triggers duty to pay
Options b. Acceptance subject to revocation (below)
2. Rightfully reject the whole
a. Good faith duty implied in perfect tender rule
b. Seller has right to cure
(i) If Seller cures, Buyer has duty to pay
(ii) If Seller Fails to cure
(a) Buyer returns goods and entitled to damages
(b) Buyer retains goods and entitled to damages
c. If Buyer wrongfully rejects, Seller entitled to damages
3. Accept part and reject part
After Acceptance
Buyer has accepted the goods but discovers a nonconformity
Buyer 1. Sue for breach of warranty (measured by the difference between the actual value of the goods as delivered and
has the value that the goods would have had if they had been as warranted)
Options 2. Notify Seller of Nonconformity if nonconformity is substantial
a. Seller has reasonable time to cure
(i) If Seller cures, Buyer has duty to pay
(ii) If Seller Fails to cure
(a) Revoke acceptance (but if Buyer wrongfully revokes acceptance, Seller is entitled to damages) and
return the goods to Seller. This is a material breach by Seller giving Buyer the right to damages.
(b) Retain the nonconforming goods and sue for breach of warranty.
C. Acceptance and Rejection
 After Seller Delivers
1. After delivery, buyer must either accept or reject the goods.
2. Rejection of goods is governed by §2-602, §2-603, and §2-605.
3. Acceptance of goods is governed by §2-606 and §2-607.
4. Buyer’s Options are listed in §2-601.
 Buyer’s Rights on Improper Delivery UCC §2-601

108
o Subject to the provisions of this Article on breach in installment contracts (Section 2-612) and unless otherwise agreed under
the sections on contractual limitations of remedy (Sections 2-718 and 2-719), if the goods or the tender of delivery fail in any
respect to conform to the contract (i.e., the Perfect Tender Rule), the buyer may
(a) reject the whole; or
(b) accept the whole; or
(c) accept any commercial unit or units and reject the rest.
 Conduct Constituting Acceptance of Goods
o Under Section 2-606(1), a buyer is deemed to have accepted goods if the buyer has not rejected them after a reasonable time
for inspection has passed.
o A buyer is also deemed to have accepted goods if the buyer exercises ownership over the goods.
 Section 2-606(1): Acceptance of Goods
Acceptance of goods occurs when the buyer
(a) after a reasonable opportunity to inspect the goods signifies to the seller that the goods are conforming or that he will take or
retain them in spite of their non-conformity; or
(b) fails to make an effective rejection (subsection (1) of Section 2-602), but such acceptance does not occur until the buyer has
had a reasonable opportunity to inspect them; or
(c) does any act inconsistent with the seller's ownership; but if such act is wrongful as against the seller it is an acceptance only if
ratified by him.
 Acceptance of a “Commercial Unit”
o A buyer must accept or reject goods by “commercial units.” A buyer may not accept part of a commercial unit and reject the
rest.
o A “commercial unit” is an item or items that is a single whole for purposes of sale, the division of which would impair its
character or value on the market.
 Section 2-606(2): Acceptance of a “Commercial Unit”
Acceptance of a part of any commercial unit is acceptance of that entire unit.
o Section 2-105 defines the term “commercial unit”: “Commercial unit” means such a unit of goods as by commercial usage is a
single whole for purposes of sale and division of which materially impairs its character or value on the market or in use. A
commercial unit may be a single article (as a machine) or a set of articles (as a suite of furniture or an assortment of sizes) or a
quantity (as a bale, gross, or carload) or any other unit treated in use or in the relevant market as a single whole.
 Example: Eggs for Breakfast
o Shiloh is hungry this morning, so Shiloh went to the grocery store, opened a package of a dozen eggs, picked out two eggs and
went to the cashier to pay for them. The cashier said, “You can’t just buy two eggs. Buy a dozen or nothing!” Shiloh replied,
“Your store offered eggs for sale and I want to purchase these two. An egg is an egg; they are all the same. Each egg is a
commercial unit, and I want to purchase these two.” Who is right, Shiloh or the cashier?
 ANSWER: The cashier is correct. While it is true that all eggs are basically the same, they are packaged by the dozen, and it
would be difficult for the store to try to sell a container with ten eggs.
 The commercial unit in this case is a package of a dozen eggs.
 Legal Effect of Acceptance
o Under Section 2-607(1) once the buyer has accepted the goods the buyer must pay the contract price.
o Under Section 2-607(2) once the buyer has accepted the goods the buyer may not reject the goods. (in certain circumstances
revoke acceptance or may sue for any defect in the goods.)
o Under Section 2-607(2) if made with knowledge of a non-conformity cannot be revoked because of it unless the acceptance
was on the reasonable assumption that the non-conformity would be seasonably cured.
o But acceptance does not of itself impair any other remedy provided by this Article for non-conformity
 Section 2-607(1) and (2): Legal Effect of Acceptance
(1) The buyer must pay at the contract rate for any goods accepted.
(2) Acceptance of goods by the buyer precludes rejection of the goods accepted and if made with knowledge of a non-conformity
cannot be revoked because of it unless the acceptance was on the reasonable assumption that the non-conformity would be
seasonably cured but acceptance does not of itself impair any other remedy provided by this Article for non-conformity.
 Rejection of Goods
o As noted in the presentation on the Perfect Tender Rule, under Section 2-601 if the tender of delivery fails “in any respect” to
conform to the contract, the buyer may reject the goods.
o Section 2-601 states that the buyer may “reject the whole,” “accept the whole,” or “accept any commercial unit or units and
reject the rest.”
 Section 2-601: Rejection of Goods
Subject to the provisions of this Article on breach in installment contracts (Section 2-612) and unless otherwise agreed under the
sections on contractual limitations of remedy (Sections 2-718 and 2-719), if the goods or the tender of delivery fail in any respect to
conform to the contract, the buyer may
(a) reject the whole; or
109
(b) accept the whole; or
(c) accept any commercial unit or units and reject the rest.
 Time for Rejection and Requirement of Notice
o A buyer has a reasonable time to inspect the goods after delivery. Under Section 2-602(1), the buyer must reject the goods
within a reasonable time after delivery.
o A rejection is not effective unless the buyer seasonably notifies the seller of the rejection within a reasonable time.
 Section 2-602(1): Manner of Rightful Rejection
Rejection of goods must be within a reasonable time after their delivery or tender. It is ineffective unless the buyer seasonably
notifies the seller.
 Example: The Sweater
o Randy purchased a size Large wool sweater online from Good Sheepment, LLC. The sweater arrived on July 2, but Randy did not
try it on until September 20, which was a cool and windy day. Randy then discovered that the company’s sizes run small, and the
Large sweater was not large enough for Randy. Has Randy waited too long to reject the sweater? Or has Randy accepted the
sweater?
 ANSWER: This is a close call. Section 2-602 gave Randy a “reasonable time” after delivery to inspect and reject the sweater,
and normally two and half months would seem to be longer than a reasonable time.
 However, Official Comment 4 to Section 2-602 explains that a “reasonable time” may be longer for buyer who is a consumer
than for a buyer who is a merchant.
 Official Comment 1 to Section 2-602 explains that Contract provisions limiting the time for rejection fall within the rule of the
section on “Time” and are effective if the time set gives the buyer a reasonable time for discovery of defects. What
constitutes a due “notifying” of rejection by the buyer to the seller is defined in Section 1-202(d).
 Official Comment 4 to Section 2-607
o Different Standards for Commercial and Consumer Buyers
The time of notification [of rejection] is to be determined by applying commercial standards to a merchant buyer. “A reasonable
time” for notification from a retail consumer is to be judged by different standards so that in his case it will be extended, for the
rule of requiring notification is designed to defeat commercial bad faith, not to deprive a good faith consumer of his remedy.
o Content of Notification
The content of the notification need merely be sufficient to let the seller know that the transaction is still troublesome and must
be watched. There is no reason to require that the notification which saves the buyer’s rights under this section must include a
clear statement of all the objections that will be relied on by the buyer, as under the section covering statements of defects
upon rejection (Section 2-605). Nor is there reason for requiring the notification to be a claim for damages or of any threatened
litigation or other resort to a remedy. The notification which saves the buyer’s rights under this Article need only be such as
informs the seller that the transaction is claimed to involve a breach, and thus opens the way for normal settlement through
negotiation.
 Buyer Must Hold the Goods for Seller Unless the Buyer Is Owed Money on Account of the Goods
o Section 2-602(2) provides that after rejection the buyer may not exercise ownership over the goods.
o Section 2-602(2) also provides that the buyer has a duty hold the goods for a reasonable time to allow the seller to remove
them, unless the buyer has a “security interest” in the goods under Section 2-711(3). Under that section a buyer has a security
interest in goods that it is rightfully rejecting to the extent that the buyer has already paid for the goods or has incurred
expenses relating to the goods. Ultimately the buyer has the right to resell those goods to recover its payment or expenses.
 Section 2-602(2): Buyer Must Hold the Goods for Seller Unless the Buyer Is Owed Money on Account of the Goods
Subject to the provisions of the two following sections on rejected goods (Sections 2-603 [Merchant Buyer’s Duties as to Rightfully
Rejected Goods] and 2-604 [Buyer’s Options as to Salvage of Rightfully Rejected Goods]),
(a) after rejection any exercise of ownership by the buyer with respect to any commercial unit is wrongful as against the seller; and
(b) if the buyer has before rejection taken physical possession of goods in which he does not have a security interest under the
provisions of this Article (subsection (3) of Section 2-711), he is under a duty after rejection to hold them with reasonable care
at the seller's disposition for a time sufficient to permit the seller to remove them; but
(c) the buyer has no further obligations with regard to goods rightfully rejected.
 Merchant Buyer Must in Good Faith Follow Seller’s Instructions or Otherwise Resell the Go
o Under Section 2-603, if the seller does not have an agent or place of business at the place of rejection then a merchant buyer
who has rejected goods must follow the seller’s reasonable instructions respecting the goods. If the goods are perishable or
threaten to decline in value and the buyer has not received any instructions from the seller then the buyer has the duty to resell
the goods. The buyer is entitled to reimbursement for any reasonable expenses incurred in caring for or reselling the goods.
o Any such actions of the buyer that are in good faith do not constitute acceptance or conversion of the goods.
 Section 2-603: Merchant Buyer Must in Good Faith Follow Seller’s Instructions or Otherwise Resell the Go
(1) Subject to any security interest in the buyer (subsection (3) of Section 2-711), when the seller has no agent or place of business
at the market of rejection a merchant buyer is under a duty after rejection of goods in his possession or control to follow any
reasonable instructions received from the seller with respect to the goods and in the absence of such instructions to make

110
reasonable efforts to sell them for the seller's account if they are perishable or threaten to decline in value speedily. Instructions
are not reasonable if on demand indemnity for expenses is not forthcoming.
(2) When the buyer sells goods under subsection (1), he is entitled to reimbursement from the seller or out of the proceeds for
reasonable expenses of caring for and selling them, and if the expenses include no selling commission then to such commission
as is usual in the trade or if there is none to a reasonable sum not exceeding ten per cent on the gross proceeds.
(3) In complying with this section, the buyer is held only to good faith and good faith conduct hereunder is neither acceptance nor
conversion nor the basis of an action for damages.
 Buyer’s Rights If the Seller Gives No Instructions
o Under Section 2-604, if the seller does not give the buyer instructions with respect to rightfully rejected goods within a
reasonable time the buyer may store, reship, or resell the goods.
 Section 2-604: Buyer’s Options as to Salvage of Rightfully Rejected Goods
Subject to the provisions of the immediately preceding section on perishables if the seller gives no instructions within a reasonable
time after notification of rejection the buyer may store the rejected goods for the seller's account or reship them to him or resell
them for the seller's account with reimbursement as provided in the preceding section. Such action is not acceptance or conversion.
 Buyer’s Duty to Specify the Defect in the Goods Justifying Rejection
o Under Section 2-605, if the buyer in rejecting the goods does not identify a particular defect then that defect cannot be used to
justify rejection if the seller could have cured the defect.
o Also, as between merchants, if the seller in writing requests a full and final written statement of all defects, the buyer must
identify a particular defect justifying rejection or that defect cannot be used to justify rejection.
o Practice Pointer: when representing Seller, make demand in writing for a “full and final written statement of all defects on
which the buyer proposes to rely.”
 Section 2-605: Waiver of Buyer’s Objections by Failure to Particularize
(1) The buyer's failure to state in connection with rejection a particular defect which is ascertainable by reasonable inspection
precludes him from relying on the unstated defect to justify rejection or to establish breach
(a) where the seller could have cured it if stated seasonably; or
(b) between merchants when the seller has after rejection made a request in writing for a full and final written statement of all
defects on which the buyer proposes to rely.
(2) Payment against documents made without reservation of rights precludes recovery of the payment for defects apparent in the
documents.
 Seller’s Rights with Respect to Wrongfully Rejected Goods
o If a buyer wrongfully rejects goods, Article 2 contains several provisions governing seller’s remedies, starting with Section 2-703
o Among Seller’s remedies are §2-702 – §2-710
 Koviack Irrigation and Farm Services, Inc. v. Maple Row Farms, LLC
o Did Maple Row Farms accept or reject the irrigation equipment that was delivered by Koviack Irrigation?
o More specifically, did Maple Row Farms notify Koviack Irrigation that it was rejecting the irrigation equipment within a
reasonable time after delivery?
o Rejection of an irrigation pump that never worked properly was the issue here. The court looked at the reasonableness of the
defendant’s rejection and found that satisfactory.
o The pump had to be tested and and that couldn’t happen until the harvest season rolled around.
o Defendant adequately conveyed its dissatisfaction with the pump the court finds, so plaintiff’s cause of action for the price fails.
 Casebook Problem
o Midwestern Seafoods, headquartered in Iowa, ordered 50 live lobsters from Maine Exports, “F.O.B. Portland.” On September 1,
Maine Exports loaded p. 298the lobsters on board an airplane in Portland, from where they were flown to Boston and then to
Des Moines. Maine Exports failed to notify Midwestern Seafoods of the date of the flight until two days later, when
Midwestern’s purchasing agent called to inquire. He then made a few calls and located the lobsters in Des Moines, where they
had been sitting for a day. Midwestern signed a receipt and picked the lobsters up. Twenty of them were clearly dying (15 due
to bad handling by Maine Exports before they were handed over to the airline and 5 due to damage in transit); the other 30
were fine. Midwestern decided, for reasons that are unclear, that it wanted none of the lobsters.
(a) Is the seller’s failure to notify Midwestern of the shipment a ground for rejection? See §2-504.
 Yes, but only if “material loss or delay” occurs as a result of the delay. §2-504, a factual inquiry.
(b) May Midwestern reject because of the 20 defective lobsters? See §§2-601, 2-503, 2-509(1), 2-510(1).
 Yes, under 2-601. Normally, risk of loss would have shifted to the buyer when the goods were shipped in Portland, but in
this case the failure to notify the buyer meant that risk of loss stayed on the seller.
 Since some of the lobsters were damaged by the seller’s conduct prior to shipment, seller retained the risk of loss on the
whole shipment per §2-510(1). Thus all 50 lobsters are seller’s problem; §2-601 permits buyer to reject the whole if any
units are defective.
(c) How quickly must Midwestern act if it wishes to reject? What technical steps is it required to take? See §§2-602, 1-205.
 Within a reasonable time to inspect, under §2-602.

111
 Midwestern must give notice within a reasonable time, take reasonable care of the goods, and make no further use of
them.
(d) Must Midwestern reship the goods to Maine Exports if the latter offers to pay the freight? See §2-602(2) with its Official
Comment 2; §§2-603, 2-604.
 Since Midwestern is a merchant and the seller has no agent at the market of rejection, the buyer must take reasonable
steps to deal with the goods; so yes, Midwestern must reship the lobsters.
 The buyer is entitled to compensation, including a commission for resale of the goods. §2-604.
(e) If Midwestern decides to keep 30 of the lobsters for resale, is this allowed? See §§2-602(2)(a), 2-606; cf. §§2-601, 2-105(6);
Annot., 67 A.L.R.3d 363.
 Under §2-601, Midwestern may accept and reject by commercial units.
 If Midwestern clearly rejects the rest, §2-601 permits it to accept or reject by commercial units. The question then turns on
what is the relevant commercial unit, which would appear to be each individual lobster (absent a usage of lobster trade we
don’t appreciate). Thus, Midwestern can accept 30 lobsters and reject the rest.
(f) If Midwestern rejects the goods, must it give its reasons in the notice of rejection? What penalty is there for not doing so?
See §2-605 and its Official Comment 2.
 Yes. Under §2-607(3), it must give notice of breach, and under §2-605, if it does not give a reason, it waives curable defects.
 Practice Pointer: Midwestern should give a list of reasons (but is well advised to make it clear that the list is not
exhaustive). Per §2-605 it waives non-mentioned curable defects.
(g) If Midwestern gives a valid notice of rejection within a reasonable period of time after the lobsters are delivered, what
should it then do with the lobsters? See §2-602(2).
 The buyer must take reasonable care of the lobsters, under §2-602, must follow any reasonable instructions of the seller
under §2-603, and if no instructions are received has the duty to resell any perishable goods for the seller’s account.
 The risk of their loss remains with seller.
 Ramirez v. Autosp
o The Ramirez’s purchased an RV and traded in their old van. On the delivery date, August 3, the paint was scratched, the electric
and sewer hookups were missing, and the hubcaps were not installed. On August 14, workers were still touching up the paint,
but the cushions were soaking wet. Ramirez offered to take the RV with a $2,000 discount, but the dealer offered only $250.
Ramirez kept calling the dealer to find out when the van would be ready, but it never was. On September 1, Ramirez’s were
invited to come in, but they waited for one and a half hours while the dealer ignored them. And seller sold their trade-in for
$5,000.
o The court discusses the common law and says that the perfect tender rule was the rule at common law. A 1947 case disputed
that, but the Uniform Sales Act had a version of the perfect tender rule.
o This is contrasted later in the opinion with the statement that general contract law permits rescission only for material
breaches. Page 289
o The harshness of the perfect tender rule is mitigated by the seller’s right to cure.
o The buyer has a right to perfect tender under §2-601, but the seller has a right to cure under §2-508. The seller has to cure
within a reasonable time.
o In determining what a “reasonable time” for the seller to cure is, the courts take into account:
i. Change of position by the buyer;
ii. Amount of inconvenience to the buyer;
iii. Length of time needed by the seller to cure; and
iv. Ability of the seller to salvage the goods by resale to others.
o This is a case involving rejection, not revocation, so the perfect tender rule applies.
o The court held that rejection may be made for many minor defects (hence the “perfect tender” rule), subject to seller’s right to
cure—and seller had not cured the defects.
o The statement of the issue is: “A further problem, however, is identifying the remedy available to a buyer who rejects goods
with insubstantial defects that the seller fails to cure within a reasonable time.”
o Text and Case Law and Official Comments: The court engages in a long discussion about the equitable remedy of “rescission.”
Apparently, the buyer’s attorney filed a claim for “rescission” and “restitution.”
o The court finds that the equitable remedy of rescission has been replaced by the UCC rights of “rejection” and “revocation”
under Sections 2-602 and 2-608, and the concomitant right of the buyer to cancel the contract under Section 2-711.
o The right of “restitution” is the buyer’s right to return of the purchase price under Section 2-711.
o Intra-textual argument? The buyer’s right to reject for minor defects versus the buyer’s right to revoke acceptance only for
substantial defects is consistent with other legal changes that occur with acceptance of the goods. The buyer is liable for the
price; the buyer may not reject the goods; and where the goods are rejected for nonconformity, the burden is on the seller to
prove that the defects have been corrected.
o Policy argument. “Underlying the Code provisions is the recognition of the revolutionary change in business practices in this
country.” Faceless manufacturers and consumers. Mass produced “with all kinds of defects.” There is a “reasonable expectation
that the buyer will return the goods and that the seller will repair or replace them.”

112
o Policy: Today, “recognizing this commercial reality,” the reasonable expectation is that there may be some minor
nonconformities, but that the seller will cure any defects. However, “the buyer should have the option of cancelling if the seller
does not provide conforming goods.”
o “Although the Ramirezes gave Autosport ample time to correct the defects, Autosport did not demonstrate that the van
conformed to the contract on September 1. In fact, on that date, when Mr. and Mrs. Ramirez returned at Autosport’s invitation,
all they received was discourtesy.”
o Ramirezes were entitled to the return of the value of their trade-in, which the trial court fixed at $4,700.
o Questions Following Ramirez
1. Did the defendant’s grounds for cure fit within §2-508?
• Did the seller really have grounds to believe that this van was acceptable in its condition?
• Section 2-508 permits cure in only two circumstances, neither of which appears applicable here: rejection before the
time has passed for tender and rejection for reasons that the seller has grounds for believing the buyer would accept
the goods. Here the time for tender had passed and the difficulties encountered by the Ramirezes were such that the
seller was always aware of their objections. Thus this cure does not fit within the statutory language.
2. Is the court creating a right to cure outside of §2-508?
• Whaley says that the courts take a very expansive view of the seller’s right to cure – basically for any curable defects.
• That being so, the court is of necessity creating a new ground for cure, one that is apparently available in any situation
in which cure is reasonably possible. In the answers to the questions following Wilson we noted the predilection the
courts have for a broad reading of the right to cure, and here is a particularly striking example.
 Casebook Problem
o Ulysses Sinon ran a dude ranch in Troy, Colorado. He decided to erect a statue of a giant horse near the entrance to the ranch as
a tourist attraction. The horse was specially manufactured by Epeius of Paris and arrived in six boxes to be assembled by Sinon.
When the horse was put together, Sinon was displeased with the appearance of the tail. The horse had been designed by
Epeius, and the scale model Sinon had seen when he decided to buy the horse had had a different tail. Sinon removed the tail
and substituted one of his own design. He returned the original to Epeius along with a letter of rejection. In the meantime, Sinon
painted the rest of the horse black (in the delivered state it was white) and used it extensively in advertising for the ranch. The
horse failed to attract new business to the ranch. After three months of display, Sinon took it down and shipped it back to
Epeius with a letter of rejection that stated that the problem with the tail made the horse unattractive and unusable. Epeius
sues.
o Did Sinon make a rejection or an acceptance? See Technology and Supply Management, LLC v. Johnson Controls Building
Automation Systems, LLC.
 He accepted the horse. His actions were inconsistent with the seller’s ownership under §2-602.
 His tail substitution and repainting are acts inconsistent with the seller’s ownership; he is treating the horse as his own and
not the seller’s. Further, one can only reject and accept by commercial units. The horse would seem to be one unit, so he
could not, as he first attempted to do, reject just the tail.
o If the tail did not conform to the model, is that a ground for rejection? See §2-601.
 Yes – but he must accept or reject by commercial unit. He should have rejected the horse. §2-601.
 The model he saw created an express warranty per §2-313 and the non-conforming tail would be a breach of seller’s
perfect tender requirement, so rejection would have been possible had he not made such use of the horse.
o If Sinon had made a technical acceptance, does that fact preclude a suit for breach of warranty? See §2-607(2).
 Yes. The model is a form of express warranty, and the statue was nonconforming. §2-313.
 An acceptance does not preclude a warranty suit (it only precludes rejection)
o What steps should Sinon take to preserve his legal rights? See §2-607(3)(a).
 He must give notice of the defect with a reasonable time under §2-607(3)(a). And he must particularize the defect in the
statue. §2-605.
o What reasons lie behind the notice requirement? See §§2-508, 2-515.
 To preserve the seller’s right to cure under §2-508 and to inspect the goods in order to resolve the dispute under §2-515.
Plateq Corp. of North Haven v. Machlett Laboratories, Inc.
o Seller is Plateq Corporation. Buyer is Machlett laboratories.
o “the principal issue is whether the buyer accepted the goods before it attempted to cancel the contract of sale.”
o This was a trial to the court, and the judge entered findings and ruled in favor of the seller and awarded the seller damages in
the amount of $14,000.
o The trial court found the following facts: The buyer Machlett ordered two specially constructed lead-covered steel tanks from
Plateq originally due on August 9, 1976. There were some difficulties in construction, and it was delayed. The seller finally
declared the tanks ready for pick-up. Buyer’s engineer Albert Yannello came on October 11 and noted some remaining
deficiencies; the seller agreed to fix them. Yannello stated that buyer would send trucks to pick them up in the next day or two.
Instead, the buyer sent a letter cancelling the contract.

113
o Text of the law: Under Section 2-606, if buyer does NOTHING after a reasonable opportunity to inspect, then there is an
acceptance. Here, the buyer went further and “signifies to the seller … that he will take … them in spite of their non-
conformity.”
o This was acceptance. Under §2-607(2), once goods have been accepted it is too late to reject them.
o In this case, there were no grounds to revoke acceptance of the goods under §2-608. Therefore, the seller was entitled to the
price of the goods under §2-703 and §2-709, because they could not be resold.
o The buyer asserts that the seller’s conduct was not sufficient to constitute a tender of delivery, and that the buyer’s conduct was
not sufficient to constitute acceptance. “The difficulty with this argument is that it is inconsistent with the underlying facts
found by the trial court.”
o Note: it is very difficult to convince an appellate court to reverse the factual findings of the trial court.
o Procedurally, the buyer had to show the appellate court that the trial court’s findings were “clearly erroneous.” They cannot
base their appeal on the conflicting evidence in the record – there was “evidence of record” to support the findings of the trial
court.
o “The defendant cannot sustain its burden of establishing that a trial court’s findings of fact are clearly erroneous by the mere
recitation in its brief of conflicting testimony entirely unsupported by reference to pages of the transcript.”
o Practice pointer: In other words, cite to pages in the record to establish that testimony or exhibits were admitted below and are
what you say they are.
o The legal analysis continues. Under §2-606, acceptance occurs when the buyer after a reasonable opportunity to inspect the
goods signifies to the seller that he will take in spite of their nonconformity or fails to make an effective rejection.
o The appellate court cites the evidence supporting the trial court’s finding that the buyer “signified” that it was taking the goods,
and the evidence that the buyer had a reasonable opportunity to inspect the goods.
o There was also an independent ground for finding that the buyer accepted the goods; the buyer failed to make an effective
rejection of them! In rejecting goods a buyer must identify the particular defect that justifies the rejection. 2-605(1). The buyer
failed to do this.
o The buyer then argues that the seller did not tender the goods. “That argument founders, however, on the trial court’s finding
that the seller was ready to make a tender on the day following the last inspection and would have done so but for its receipt of
the defendant’s telegram of cancellation.”
o Once the conclusion is reached that the defendant accepted the tanks, a number of legal rules come into play. The buyer may
no longer reject the goods. If the buyer wants to revoke the buyer must prove that the goods were substantially non-
conforming. The buyer is liable for the price.
o “Since the defendant has provided no basis for any argument that the trial court was clearly erroneous in finding that the
defendant had not met its burden of proof to show that the goods were substantially nonconforming, we can find no error in
the conclusion that the defendant’s cancellation constituted an unauthorized and hence wrongful revocation of acceptance.”
o Finally, the buyer in its brief although not in the assignments of error challenges the remedial portions of the trial court’s order.
However, it is not challenged that the tanks were specially manufactured, and that any effort to resell them would be
unavailing. Accordingly, the seller was entitled to recover the entire purchase price under Sections 2-703 and 2-709.
o Takeaways:
 If buyer accepts the goods, several consequences follow. Buyer is obliged to pay the price (subject to counterclaims for
breach by seller). Buyer can no longer reject the goods using the perfect tender rule, rather can only revoke acceptance
(which has a much higher standard meet). Buyer must, within a reasonable time, notify seller of any breach of warranty or
be barred from remedy.
 Plateq illustrates that acceptance does not require buyer to say, “I accept the goods” or even to physically accept delivery
of the goods.
 Rather, under §2-606, acceptance occurs if buyer, after reasonable opportunity to inspect, indicates buyer will take them,
even if nonconformities exist (or if buyer fails to make an effective rejection, or if buyer treats them as buyer’s property).
 Summary: Acceptance and Rejection of Goods
o Once a buyer has accepted goods the buyer is obligated to pay for the goods. Also, the buyer may not reject the goods, but in a
proper case the buyer may revoke its acceptance.
o A buyer is deemed to have accepted goods if the buyer has not rejected them within a reasonable time for inspection. A buyer is
also deemed to have accepted goods if the buyer exercises ownership over the goods. When nonconforming goods are
delivered, the buyer may accept the whole, reject the whole, or accept any commercial units and reject the rest.
o A rejection is not effective unless the buyer notifies the seller of the rejection within a reasonable time. A buyer must notify the
seller of the particular non-conformity within a reasonable time and must give the seller a reasonable opportunity to cure the
defect.
o A merchant buyer who has rightfully rejected goods must follow the seller’s instructions with respect to the goods, and in the
absence of instructions may store, reship, or resell the goods. If the goods are perishable or speedily declining in value, then a
merchant buyer also has the duty to resell the goods for the merchant’s account. The buyer also has the right to retain and
resell non-conforming goods as security for the return of any purchase price that has been paid or expenses that the buyer has
incurred with respect to the goods.
114
 Canvas Quiz Questions
o A buyer is deemed to have accepted goods if the buyer has not rejected them after a reasonable time for inspection has
passed.
 True - A buyer is deemed to have accepted goods if the buyer has not rejected them after a reasonable time for inspection
has passed, according to §2-606(1)(b). Thus, buyer's must be careful to inspect the delivery and, if there is a nonconformity,
reject the goods within a reasonable time for inspection or they will be DEEMED to have accepted them!
o A buyer may accept all or part of a commercial unit and reject the rest.
 False - A buyer must accept or reject goods by “commercial units.” A buyer may not accept part of a commercial unit and
reject the rest. Under Section 2-606(2), “Acceptance of a part of any commercial unit is acceptance of that entire unit.”
o Acceptance of goods occurs when the buyer:
 after a reasonable opportunity to inspect the goods signifies to the seller that the goods are conforming or that he will take
or retain them in spite of their non-conformity
 fails to make an effective rejection after the buyer has had a reasonable opportunity to inspect the goods
 does any act inconsistent with the seller's ownership
 takes delivery of the goods from the seller
o A “commercial unit” is an item or items that is a single whole for purposes of sale, the division of which would impair its
character or value on the market.
 True - Section 2-105 defines the term “Commercial unit” to mean such a unit of goods as by commercial usage is a single
whole for purposes of sale and division of which materially impairs its character or value on the market or in use. A
commercial unit may be a single article (as a machine) or a set of articles (as a suite of furniture or an assortment of sizes)
or a quantity (as a bale, gross, or carload) or any other unit treated in use or in the relevant market as a single whole.
o Acceptance has certain legal effects, including (select all that apply, if any):
 The buyer must pay at the contract rate for any goods accepted
 The buyer cannot reject the goods, no matter what - The "no matter what" language is what makes the answer on
revocation incorrect.
 The buyer cannot revoke acceptance if made with knowledge of a non-conformity, no matter what
 Acceptance precludes the buyer from any remedy under Article 2 - Acceptance does not of itself impair any other
remedy provided by this Article for non-conformity
o If the goods or the tender of delivery fail in any respect to conform to the contract, the buyer may:
 accept the whole delivery
 reject the whole delivery
 accept any commercial unit or units and reject the rest
 reject the whole and, if this is an installment contract, cancel the contract
D. Revocation of Acceptance.
 What Circumstances Allow a Buyer to Accept Goods and Then Revoke Its Acceptance?
o Under Section 2-608(1) a buyer may revoke its acceptance of goods if
 the buyer discovered the defect and the seller promised that the non-conformity would be cured, and the seller failed to
cure it; or
 if the buyer failed to discover the defect before acceptance either because of the difficulty of discovery or because of the
seller’s assurances.
 Revocation of Acceptance for a Commercial Unit
o A buyer may revoke acceptance of an entire lot that is substantially nonconforming or of a commercial unit that is substantially
nonconforming.
o Obviously, a buyer may not revoke acceptance of a portion of a commercial unit.
 Substantial Impairment of Value
o When the goods are initially delivered to the buyer, the “perfect tender rule” applies; if the goods are not conforming to the
contract in any respect the buyer can reject them.
o However, under Section 2-608(1) once the buyer accepts the goods the buyer may revoke its acceptance only if the non-
conformity substantially impairs the value of the goods to the buyer.
 Substantial Impairment “To Him”
o Section 2-608 adopts a subjective standard for determining whether the value of the nonconforming goods is substantially
impaired. It provides that a buyer may revoke acceptance if the nonconformity substantially impairs the value of the goods “to
him” – that is, to the buyer.
 Section 2-608(1): Revocation of Acceptance
The buyer may revoke his acceptance of a lot or commercial unit whose non-conformity substantially impairs its value to him if he
has accepted it
(a) on the reasonable assumption that its non-conformity would be cured, and it has not been seasonably cured; or
(b) without discovery of such non-conformity if his acceptance was reasonably induced either by the difficulty of discovery before
acceptance or by the seller's assurances.

115
 Time for Revocation of Acceptance
o Under Section 2-608(2) a buyer must revoke its acceptance within a reasonable time after the buyer discovered or should have
discovered that the buyer had a right to revoke and before any substantial change in the condition of the goods that does not
result from the defect.
 Section 2-608(2): Time for Revocation of Acceptance
Revocation of acceptance must occur within a reasonable time after the buyer discovers or should have discovered the ground for it
and before any substantial change in condition of the goods which is not caused by their own defects. It is not effective until the
buyer notifies the seller of it.
 Buyer’s Remedies After Revocation of Acceptance
o Under Section 2-608(3) a buyer who rightfully revokes their acceptance of the goods has the same rights as a buyer who has
rightfully rejected the goods.
 Section 2-608(3): Buyer’s Remedies After Revocation of Acceptance
A buyer who so revokes has the same rights and duties with regard to the goods involved as if he had rejected them
 Example: The Crane
o Big Sky Construction LLC is a Montana company that specializes in building high-rise office buildings. Big Sky ordered a 20-ton
crane from Canary Manufacturing for a price of $900,000 to be delivered to Butte on February 20. The crane was delivered on
time, but there were several mechanical problems with the slewing unit which Canary promised to fix. Big Sky put the crane into
service, but the slewing unit never worked properly despite Canary’s best efforts. After two months, during which the crane had
been out of service for a total of six weeks, the slewing unit broke again, and Big Sky informed Canary that it was revoking its
acceptance, returning the crane to Canary, and demanding the return of its purchase price. The crane was still in excellent
condition with the exception of the slewing unit. Under these circumstances is Big Sky entitled to revoke its acceptance and
return of the purchase price, or is Big Sky required to retain the crane and be limited to a suit for breach of warranty as to
accepted goods?
 ANSWER: The defect in the slewing unit was apparent when Big Sky accepted the crane, and in the absence of other facts
two months would be too long a period for Big Sky to wait until revoking its acceptance. However, in this case Big Sky
accepted the crane based on Canary’s assurances that the defect in the slewing unit would be repaired. Canary was
unsuccessful in its attempts to cure the defect. The condition of the crane had not otherwise deteriorated. Under these
circumstances, Big Sky revoked its acceptance within a reasonable time.
 Because of the defective slewing unit, the crane was out of service for six weeks over a two-month period. This defect
substantially impaired the value of the crane to Big Sky.
 Under these circumstances, Big Sky was entitled to revoke its acceptance, return the crane, and the return of its purchase
price.
 Waddell v. L.V.R.V., Inc.
o Seller = LVRV; Buyer = Waddells
o After a bench trial the trial court entered judgment for the Waddells allowing them to revoke their acceptance of the RV and
granted them attorney fees and costs. The court also denied LVRV indemnification against the manufacturer Coachmen. LVRV
appeals
o Even after acceptance, buyer can avoid liability for the price if buyer can revoke acceptance under §2-608. Revocation requires:
 The non-conformity substantially impairs the value of the goods to buyer (i.e. not just failure to make perfect tender)
 Buyer accepted reasonably expecting cure by seller, or reasonably not knowing of defect, because it was hard to discover or
because of seller’s assurances
 Buyer acts within a reasonable time of when buyer should know of defect, and before substantial change in condition of
goods not caused by the defect.
o The Nevada Supreme Court adopts the reasoning of the Oregon Supreme Court in finding a two-part test to determine whether
a nonconformity substantially impairs the value of the goods to the buyer:
 Subjective element – the value of the goods to the buyer
• Subjective Impairment: Subjectively the Waddells purchased the motor home to enjoy that lifestyle. The seller’s
advertising had promised “freedom.”
 Objective element – whether the non-conformity in fact substantially impairs the value of the goods to the buyer.
• Objective impairment: In the 18 months after acceptance, the motor home had problems with air conditioning,
heating, batteries, and the engine overheating. It was in the shop for 7 of those 18 months. “This testimony is sufficient
to demonstrate an objective, substantial impairment of value.”
o Waddell held that buyers of a motor home could revoke acceptance.
o The motor home, in the eighteen months after purchase, had problems with air conditioning, heating, batteries, and with the
engine overheating.
o The court did not decide whether substantial impairment of value to the buyer included a subjective standard.
o Rather, because the many problems with the motor home caused the buyers to have “shaken faith” in the motor home, there
was substantial impairment under either standard.

116
o Revocation after eighteen months was within a reasonable time under the facts of this case, where the seller’s service
department had the motor home for seven of those months, buyers regularly complained of the problems, and seller continued
unsuccessful attempts to repair the motor home.
o Objectively, they were unable to enjoy the item as they intended.
o The court quotes precedent from several cases, including Ohio where it says that there is a substantial impairment of value if
the defects “shakes the buyer’s faith or undermines his confidence in the reliability and integrity of the purchased item.”
o And the Supreme Judicial Court of Massachusetts cosmetic or minor defects that go unrepaired or other defects that
circumscribe the use of the goods can substantially impair the value of the goods to the buyer.
o And the federal district court in Nevada has ruled that the seller’s inability to correct defects in motor vehicles creates a major
hardship and an unacceptable economic burden on the consumer.”
o “Accordingly, we conclude that substantial evidence exists to support revocation of acceptance under §2-608.
o Revocation within a Reasonable Time
• Footnote 10 – reasonableness of time is a question of fact
 The seller also claimed that the time for revocation of acceptance had passed.
 “That’s all.” But the court quoted a Mississippi case that the seller may not “postpone revocation in perpetuity by fixing
everything that goes wrong.” Rather, “there comes a time when ... the buyer is entitled to say, ‘That’s all,’ and revoke.”
 A Nevada court stated that the “time for revocation of acceptance will be tolled while the seller attempts repairs.”
o “The Waddells gave Wheeler’s several opportunities to repair the defects before revoking their acceptance. Because Wheeler’s
was unable to repair the defects after a total of seven months, the Waddell’s were entitled to say, ‘That’s all,” and revoke their
acceptance, despite Wheeler’s good-faith attempts to repair the RV.” The reasonable time was tolled during the time of the
repairs.
o The court found that the district court’s decision was supported by substantial evidence and was not clearly erroneous.
o End of case: “The district court did not err in allowing the Waddells to revoke their acceptance of the RV within a reasonable
time because chronic engine overheating problems substantially impaired the RV’s value to the Waddells.”
o Questions Following Waddell
1. If buyer has paid for the car, then buyer may keep the car as security for return of the purchase price. §2-711(3).
• Buyer need not let seller have the car if the price has been paid. To that extent the buyer has a security interest in the
car and may realize on it per §2-706 if the seller will not refund the money.
• If the buyer does not have a security interest in the car, buyer must behave as if buyer had rejected it originally, and §2-
602 requires the buyer to take reasonable care of the car. It is seller’s responsibility to come pick it up.
2. Most courts say that the buyer is liable for the value of the use of the car.
• The Lemon Laws often provide for this, including the Pennsylvania Lemon Law. However, Connecticut in Barco (cited in
the Question) held that the buyer is not liable, reasoning that there is nothing in §2-608 to that effect, and also that
sellers are not liable to buyers for the value of the interest they earned on the purchase price.
• Is the Barco court wrong? There is a lot of authority allowing an offset for use, usually measured by rental value (not to
mention §1-305). The Moore court made buyer take an offset for the use of the goods.
3. May the buyer continue to use the goods?
• There is a risk that this will be considered to be acceptance under §2-602 and §2-606. If so, the buyer will have only a
suit for breach of warranty of accepted goods under §2-714.
• Practice Pointer: Post-rejection use is legally dangerous, but courts are becoming more liberal in forgiving it. If you are
worried that post-rejection use will result in acceptance, you might live with the goods and simply sue for breach of
warranty damages.
4. Is the seller entitled to more than one opportunity to cure? See Car Transportation Brokerage Company, Inc. v. Blue Bird
Body, (holding that because the electrical system for a motor coach is a complex piece of equipment, buyer must give seller
more than one chance to cure before revoking acceptance).
• As the Blue Bird case says, yes, it may be reasonable to give the seller more than one attempt to cure. Wilson v.
Scampoli, too!
5. Could a buyer revoke acceptance after buyer has already resold nonconforming goods at a loss? See §§2-706(1), 2-711(3);
Summit Auto Sales, Inc. v. Draco, Inc.
• Normally on revoking acceptance the buyer must then surrender the goods to the seller, but per §2-711(3) if the buyer
has paid for the goods or had expenses in connection with them the buyer may claim a security interest in the goods,
refuse to return them, and then resell them using the same procedures sellers use in §2-706.
– The court in the cited case applied this idea to the revocation of acceptance of automobiles bought in the United
States and then sold overseas that proved to have reduced value because of their prior usage as taxis.
 Practice Pointer: when you are the rejecting/revoking buyer don’t be so quick to surrender the offending goods
 Note on Lemon Laws
o If there is a substantial defect in the auto that makes it stay more than 30 days in the shop or 3 unsuccessful attempts to cure
it, the dealer must refund the purchase price (less reasonable cost for use) or replace the vehicle.
 Casebook Problem

117
o The day after Alice Bluegown bought her new car, the right rear fender fell off. May she use §2-608 or must she give the car
dealer a right to cure? Pretend she is sitting in your office expecting an immediate answer; glance at §2-608 and decide. The
next day she took the car back to have the fender repaired; this made her late for work. The dealer fixed it, and the fender gave
her no more trouble. However, the first time it rained all the paint washed off the car. May she revoke now? She took the car
back to the dealer when the rain stopped and rode the bus to work (late again). The car dealer did a nice job repainting the car.
Two weeks later the engine quit on her when she was in the middle lane of a superhighway at rush hour. The car had to be
towed to the car dealer, and Alice missed an important sales meeting. The car dealer fixed the engine. Now Alice is back in your
office. The car’s trunk will not open. Must she permit them to fix it, or can she revoke? See Foss, The Seller’s Right to Cure
When the Buyer Revokes Acceptance: Erase the Line in the Sand. She has missed enough work to worry about hurting her
career. She’s also concerned that the car is going to keep breaking down right through and past the warranty period.
 Whether Alice can revoke because the fender fell off will depend in large part on the “substantial impairment” to her
personally.
 The “Shaken Faith” doctrine may help her here.
 On the other hand, §2-608(2)(a) mentions “cure” and some commentators have suggested that seller should usually be
permitted one cure attempt.
 When the paint comes off the substantial impairment becomes more obvious and the courts are more likely to let her out.
 As subsequent things go wrong, the idea of cumulative defects as a “substantial impairment” becomes more viable and
revocation should be permitted.
o Should she demand assurances under §2-609?
 I suppose she could. You could use §2-609 in this sort of situation, demanding that the car dealer furnish adequate
assurance that the car is now in good shape (a tough request to fulfill).
o If she decides to revoke acceptance and if the court agrees that this is allowed, would it also permit her to recover for the
cost of a rental car used as substitute transportation while she was attempting to purchase a new car? See McGinnis v.
Wentworth Chevrolet Co.
 The Oregon Supreme Court was not willing to give rental car expenses as an incidental damage, saying that rental costs
were not readily translatable into a comparable value figure for computation of loss of bargain. That seems wrong:
incidental damages are attempts at mitigating loss and the rental car may help keep down consequential employment
losses.
o If she goes out and buys a new car, can she make the first car dealer pay for it? See §2-712.
 If she buys a new car, the courts should not view this as a damage seller must pay for. Presumably the car she now buys is
worth what she pays for it, so where are her damages?
 Casebook Problem
o Suppose in the last Problem the contract between the dealer and Bluegown explicitly limits the remedy for breach to repair
or replacement of defective parts. The dealer argues that all defects have been promptly and successfully repaired and that
the remedy of revocation of acceptance is therefore unavailable to Bluegown. See §2-719(2); Durfee v. Rod Baxter Imports,
Inc.; see also Andover Air Ltd. Partnership v. Piper Aircraft Corp.
 Argue that this limited remedy has “failed of its essential purpose” under §2-719(2) … but the Durfee case is quite explicit in
stating that if the repairs are made successfully each time then the limitation is valid.
 Even so, at some point the court may feel that enough is enough and that repair and replacement do not afford her a
“minimum adequate remedy,” the test from Official Comment 1 to §2-719.
 The “Shaken Faith Doctrine” may help her here. The Andover case held in favor of the buyer’s revocation of acceptance of a
defective airplane, stating that limiting the remedy (as the contract did) to replacement of a defective landing gear (costing
tens of dollars) even though the gear was the cause of the plane's crash landing (resulting in damages in excess of
$100,000), was unconscionable.
 Casebook Problem
o Arthur Author ordered an expensive computer (the ION #740) from ION Business Machines. ION sent him model #745, a newer
and better version of the machine he had ordered, at the same price. When he saw the computer, he liked it and wrote them a
letter of acceptance, enclosing a check in payment. However, when he began to use it, he was horrified to learn that the
computer was turned on by a hidden switch under the front panel. Arthur Author’s father had lost a finger when he reached
under a machine to activate it. Arthur had witnessed the accident as a child. Arthur sent a notice of revocation of acceptance to
ION, stating that the #740 had a visible switch and explaining that the hidden switch on the #745 brought back childhood
memories that kept him from wanting the computer. Does §2-608 permit him to revoke for this reason? See Official Comment
2. Is §2-508(2) relevant? How would you advise ION to respond to Arthur Author’s letter? See Annot., 98 A.L.R.3d 1183.
 Arthur has a decent argument that the nonconformity substantially impairs the value of the machine “to him”.
 However, it might have been something that he should reasonably have discovered upon inspection. If a court decides that
Arthur should have noticed the hidden switch prior to his acceptance, this fact may also keep him from being able to
revoke.

118
 This is a hard case. Official Comment 2 is clearly in Arthur’s favor, since it adopts a subjective test. Obviously, Arthur could
have rejected for this reason (there is a breach of a “model” express warranty §2-313), but revocation is harder to justify
where seller had no reason to know of the highly unusual problem.
 Certainly, the seller has a good argument that this is just the sort of “surprise” revocation that ought to give rise to an
opportunity to cure under §2-508(2).
 Casebook Problem
o After his car had broken down with the same defect six times, Zack Taylor decided to revoke acceptance and return the car to
Fillmore Motors, the p. 321dealership that had sold him the vehicle but that had been unable to repair it. To Zack’s dismay, he
discovered that Fillmore Motors had gone bankrupt and was out of business. Zack is now in your office with this issue: may he
revoke acceptance against the manufacturer of the car (which had covered its product with a limited warranty)? Compare
Newmar Corp. v. McCrary (Nevada), with Gasque v. Mooers Motor Car Co. (Virginia); Gregory J. Krabacher, Revocation of
Tripartite Rolling Contracts: Finding a Remedy in the Twenty-First Century Usage of Trade.
 Some states have ruled that revocation may only occur against the party with whom one is in privity.
 Magnuson-Moss Warranty Act should help (written limited warranty; consumer Buyer), but the courts are evenly split on
this question.
 Whaley thinks that the courts that deny this remedy are wrong.
 The two cited cases reflect the extremes.
• Virginia would not allow revocation against the manufacturer on the theory that only the retailer is the “seller”
mentioned in §2-608; Nevada did allow the direct revocation where the manufacturer was the creator of the defective
product and involved itself in the sales transaction.
• Particularly where the “seller” is bankrupt, the Nevada position seems fairest. If the retail seller has made no
warranties, so that the only warranties are the express written ones given by the manufacturer, a result holding that
revocation is not available is very hard on the buyer. Since this is a common fact situation, I think the revocation against
the manufacturer should be permitted.
 For consumer goods it may be possible to get relief against the manufacturer pursuant to §110(d) of the Magnuson-Moss
Act. It allows a lawsuit against the warrantor for legal and equitable relief (and revocation of acceptance, a statutory form
of rescission, is an equitable remedy), though some courts hold that in their jurisdictions privity is a requirement in
Magnuson-Moss actions (though Congress could not have intended this result).
 Summary – Revocation of Acceptance
o Revocation of acceptance is governed by Section 2-608.
o Even though a buyer has accepted nonconforming goods, in some circumstances a buyer may revoke its acceptance.
o A buyer may revoke its acceptance if the nonconformity was something that the buyer could not have discovered by means of a
reasonable inspection, or where the buyer discovered the nonconformity and nevertheless accepted the goods based upon the
seller’s assurances that the nonconformity would be cured.
o A buyer must notify a seller that it is revoking its acceptance of the goods within a reasonable time and before the value of the
goods has deteriorated due to factors other than the nonconformity.
o A buyer may revoke its acceptance only if the nonconformity substantially impairs the value of the goods to the buyer.
 Canvas Quiz Questions
o In order for the buyer to revoke acceptance, is it sufficient that the goods are non-conforming to any extent - would a trivial
defect justify revocation of acceptance? Or is it necessary that the defect substantially impairs the value of the goods?
 Answer: In order for the buyer to revoke its acceptance of the goods, there must be a defect that substantially impairs the
value of the goods.
 Any non-conformity of the goods, no matter how trivial, may justify revocation of acceptance of the goods.
o A buyer may revoke its acceptance of the goods if:
 the buyer discovered the defect, the seller promised to cure it, and the seller failed to cure it.
 the buyer failed to discover the defect before acceptance because of the difficulty of discovery.
 the buyer failed to discover the defect before acceptance because of the seller's assurances.
 the buyer chose not to inspect the goods before acceptance.
o A buyer must revoke its acceptance of the goods within a reasonable time after the buyer discovers or should have
discovered the defect and before any change occurs in the condition of the goods which is not caused by its own defects.
 True
o A buyer who has rightfully revoked its acceptance of the goods may pursue the same remedies as if it had rightfully rejected
the goods.
 True
o Janet is the Athletic Director at Eastern Pennsylvania Polytechnic. During the summer she ordered a time clock from Wyandot's
Chronos for Eastern's varsity basketball court. The clock operated perfectly except for one thing: the time clock is a bit fast -- it
counts down 35 seconds in only 33 seconds, a fact that Janet did not discover until the second game of the season when a fan
for the opposing team timed the time clock with a stopwatch. Janet wants to return the time clock to Wyandot's and obtain a
refund. Which of the following questions are potentially relevant to determining whether she may revoke her acceptance?

119
 Could Janet have discovered the defect before acceptance by a reasonable inspection of the goods?
 Did this defect substantially impair the value of the time clock?
 Was Janet dissuaded from inspecting the time clock earlier because of the seller's assurances?
 Did Janet notify the seller of her intent to revoke her acceptance of the time clock within a reasonable time after
discovering the defect?
A. Risk of Loss in Case of Breach
 Risk of Loss under the Uniform Sales Act and the Uniform Commercial Code
o Under the Uniform Sales Act, risk of loss stayed with the seller until title passed to the buyer. But the concept of “title” has been
unbundled – identification, shipment, delivery, receipt, inspection, rejection, cure, acceptance, revocation.
o Under the UCC, if the seller is a merchant, risk of loss passes when the goods are received by the buyer, and if the seller is not a
merchant, risk of loss passes when the seller tenders delivery.
o UCC §2-509 dealt with situations where a seller and a buyer have entered into a contract for the sale of goods, and then the
goods are lost, stolen, damaged, or destroyed. Specifically, whether a buyer has to pay for the goods that were lost, stolen,
damaged or destroyed.
o UCC §2-510 deals with situations where one party is in breach.
 Rules for Risk of Loss in the Absence of Breach
o The rules governing risk of loss in the absence of breach were covered in a previous presentation. If the goods are conforming to
the contract, under Section 2-509 the risk of loss shifts to the buyer when the following events occur:
 If the seller is a merchant and it is not a shipment or delivery contract – when the buyer receives the goods;
 If the seller is not a merchant and it is not a shipment or delivery contract – when the seller tenders delivery of the goods;
 Shipment contract – when the seller duly delivers the goods to the carrier;
 Destination contract – when the carrier duly tenders delivery of the goods to the buyer at the place of delivery.
 Effect of Breach on Risk of Loss Section 2-510
(1) Where a tender or delivery of goods so fails to conform to the contract as to give a right of rejection the risk of their loss
remains on the seller until cure or acceptance.
(2) Where the buyer rightfully revokes acceptance, he may to the extent of any deficiency in his effective insurance coverage treat
the risk of loss as having rested on the seller from the beginning.
(3) Where the buyer as to conforming goods already identified to the contract for sale repudiates or is otherwise in breach before
risk of their loss has passed to him, the seller may to the extent of any deficiency in his effective insurance coverage treat the
risk of loss as resting on the buyer for a commercially reasonable time.
 Who Has the Risk of Loss If the Goods That Were Destroyed Were Non-Conforming?
o Under Section 2-510(1), if the goods that are tendered for delivery do not conform to the contract, the risk of loss remains on
the seller until the seller cures the defect or until the buyer accepts the goods.
o If the buyer rejects nonconforming goods, the risk of loss remains on the seller. However, once the buyer accepts the goods
(even if those goods are non-conforming) the buyer has the risk of loss.
 Section 2-510(1): Non-conforming Goods
Where a tender or delivery of goods so fails to conform to the contract as to give a right of rejection the risk of their loss remains on
the seller until cure or acceptance.
 Example: The Cantaloupes
o Super Duper Grocery, Inc., of Austin, Texas, ordered four tons of size 15 cantaloupes from Apex Produce Distributors of Phoenix,
Arizona. The contract provided that the goods were to be shipped F.O.B. Phoenix on August 10. In accordance with the contract
the cantaloupes were duly delivered to the carrier on August 10. However, the cantaloupes spoiled in transit before they
reached Austin. The buyer rejected the melons. In addition, an inspection revealed that the melons were size 12, much smaller
than what was called for in the contract. Who had the risk of loss when the melons were destroyed? Is Super Duper liable to
pay Apex for the cantaloupes?
 ANSWER: Under Section 2-509 with a shipment contract the risk of loss passes to the buyer when the seller duly delivers
the goods to the carrier.
 However, in this case the goods were nonconforming because they were too small, so Section 2-510 applies. Under Section
2-510, the risk of loss as to nonconforming goods stays on the seller until the nonconformity is cured or until the buyer
accepts the goods. In this case the goods were destroyed before cure or acceptance, so the risk of loss remained on the
seller. Even though this was a shipment contract, Super Duper is not liable to pay for the cantaloupes.
 What If the Buyer Rightfully Revokes Acceptance?
o If the goods are not conforming to the contract, the buyer accepted the goods, and then the buyer rightfully revokes its
acceptance, under Section 2-510(2) the risk of loss shifts back to the seller to the extent that the buyer’s insurance does not
fully cover the loss.
o Section 2-510(2): Rightfully Revocation Acceptance, Risk of Loss
Where the buyer rightfully revokes acceptance, he may to the extent of any deficiency in his effective insurance coverage treat the
risk of loss as having rested on the seller from the beginning.
 Example: The Bad Transmission

120
o Micki purchased a new 2013 Nissan Pathfinder from Patterson Nissan. Within a few days it became apparent that the car had a
bad CVT transmission. In the first month that Micki owned the car the dealership tried six times to repair the transmission. The
transmission began lugging again, so Micki called the dealership and notified it that Micki was revoking acceptance of the car
and wished to return it. The dealership refused and told Micki that there was really nothing wrong with the transmission and
that Micki would have to live with it. That evening the car was stolen from Micki’s driveway. Micki’s insurance policy for theft
had lapsed for non-payment. Who bore the risk of loss?
 ANSWER: Once Micki accepted the car, Micki had the risk of loss. However, once Micki rightfully revoked acceptance of the
car, the risk of loss shifted back to the dealership as to any deficiency in Micki’s insurance. Since Micki didn’t have any
insurance whatsoever on the car against theft, the dealership will bear the entire risk of loss in this case.
 However, once Micki rightfully revoked acceptance of the car, the risk of loss shifted back to the dealership as to any
deficiency in Micki’s insurance. Since Micki didn’t have any insurance whatsoever on the car against theft, the dealership
will bear the entire risk of loss in this case.
 Query whether Micki’s revocation was rightful or wrongful. At least the dealer would argue that point!
 What If the Buyer Breaches by a Wrongful Anticipatory Repudiation?
o If the buyer breaches by wrongfully repudiating the contract before the seller has performed, then to some extent the risk of
loss shifts to the buyer. Under Section 2-510(3), if the goods have been identified to the contract before the buyer breaches,
the breaching buyer has the risk of loss to the extent that the seller’s insurance does not fully cover the loss.
o Under Section 2-510(3), if the goods have been identified to the contract before the buyer breaches, the breaching buyer has
the risk of loss to the extent that the seller’s insurance does not fully cover the loss.
 Section 2-510(3): What If the Buyer Breaches by a Wrongful Anticipatory Repudiation?
Where the buyer as to conforming goods already identified to the contract for sale repudiates or is otherwise in breach before risk
of their loss has passed to him, the seller may to the extent of any deficiency in his effective insurance coverage treat the risk of loss
as resting on the buyer for a commercially reasonable time.
 Jakowski v. Carole Chevrolet, Inc.
o This is a case where the goods were non-conforming. Accordingly, the risk of loss stayed on the seller until “cure or acceptance.”
o Seller = Carole Chevrolet; Buyer = Jakowski
o Seller sold buyer a 1980 Camaro. Buyer picked it up, but seller called to say that the coating hadn’t been applied and to bring it
back it.
o That night the car was stolen. Who had the risk of loss?
o Rule: Section 2-510(1) – where goods are nonconforming, risk of loss stays on the seller until cure or acceptance.
o Under the perfect tender rule, Section 2-601 the court ruled that the car was nonconforming (lacking the undercoating and
finish hadn’t been applied).
o Furthermore, the buyer had not yet accepted the car. Merely taking possession of a vehicle is not “acceptance.” The law gives
the buyer a reasonable time for acceptance of the goods. The “reasonable time for inspection” had not yet elapsed (following
Zabriskie Chevrolet), and the seller had not cured the defect.
o While commentators have imagined a host of difficulties with Section 2-510, the courts have not had difficulty applying it.
o “For present purposes it is adequate to hold simply that where a seller obtains possession of the goods in an effort to cure
defects in them so as to comply with his end of the bargain, he is under a contractual duty to redeliver them to the buyer. In
failing to do so, he has breached the contract.”
o Under Section 2-711, the buyer was entitled to return of the purchase price, plus interest.
o Note that the same result would apply if buyer revoked acceptance (and the goods were uninsured). §2-510(2).
 Casebook Problem
o The Lamia Museum’s director, Mandrake Griffin, ordered three new pieces for the museum: an Egyptian sphinx, an Old-World
gargoyle, and an Etruscan statue of a centaur. These objects d’art were purchased under separate contracts from Empusa
Exports of London, England. All were to be shipped “F.A.S. S.S. Titanic” on or about April 9, on their way to the museum, which
was located in New Jersey. The parties agreed that New Jersey law would apply. Prior to April 9, Empusa Exports received a call
from Griffin cancelling the purchase of the centaur statue. Empusa protested the cancellation but agreed to ship the other two
pieces. Empusa’s manager discovered that the sphinx was a phony, but kept her mouth shut and shipped it anyway. She also
discovered that the gargoyle’s condition was such that it could not survive the exposure to sea air, so she decided to send it by
air in spite of the contract’s F.A.S. Titanicterm. This decision proved wise since the Titanic encountered an obstacle on its sea
voyage and foundered, taking the sphinx with it. The gargoyle arrived in good condition, and Griffin wrote a letter to Empusa
accepting the gargoyle and enclosing the museum’s check. A week later Griffin learned that the gargoyle was not from the “Old
World,” but instead had been cast in Hoboken many years p. 326ago, had somehow found its way to Europe, and now was
home again. He sent Empusa a letter demanding that the museum’s money be returned and stating that he canceled the sale.
Before Empusa could respond, two things happened: the museum burned to the ground, and the centaur statue was stolen
from Empusa’s warehouse (through no fault of Empusa, which was not negligent in guarding it). Both the museum and Empusa
were fully insured. Answer these questions:

121
(a) By shipping the other two objects after the museum refused to take the centaur statue, did Empusa waive its right to sue
for the repudiation? See §§2-209(1), 2-106(3), 2-106(4). Would §1-308 have helped Empusa? What should it have done to
use this section?
 Probably not. Empusa’s protest should be sufficient to protect its legal position, though a use of §1-308’s more formal
phrases is wiser.
 Practice Pointer: Section 1-308 allows a party to announce to the other side that it is performing with a “full
reservation of rights,” including the right to dispute all that has gone on before.
 This avoids any argument about waiver, and alerts the other side to the seriousness of what will be argued as a breach.
(b) Which party took the risk of loss on
(1) the centaur?
• Because of the buyer’s anticipatory breach, §2-510(3) puts the risk of loss on the buyer to the extent of any
deficiency in the seller’s insurance coverage. But seller was fully insured, so buyer is not liable.
(2) the sphinx?
• The seller did not make a perfect tender, so the risk of loss remained on the seller until cure or acceptance. Seller is
liable for the loss.
(3) the gargoyle?
• The buyer rightfully revoked its acceptance, so the risk of loss shifted back to the seller to the extent of any
deficiency in the buyer’s insurance; however, the gargoyle was fully insured, so the seller is not liable.
(c) When Empusa shipped the gargoyle by air instead of by sea, could Lamia have treated this as an imperfect tender and
rejected the gargoyle for that reason? See §§2-503(1), 2-503(2), 2-504, 2-614.
 No, this was not a breach. Under §2-614(1), the seller is required to make the change and the buyer is required to
accept it.
(d) The Lamia Museum’s insurance policy with the Pegasus Insurance Company contains two clauses relevant to §2-510. One
provided that on payment of a claim the insurance company was subrogated to any claim its insured had against any other
person. The other stated that the policy should not be deemed to provide protection for any claim where the risk of loss
rested with another person. What is the effect of these provisions? See Official Comment 3 to §2-510; White & Summers
§6-7.
 Lamia Museum’s insurance policy gave the insurance company the right to deny payment in any situation where the
risk of loss would have rested on another person but for the policy.
 This clause should be ineffective as against public policy – an attempt to evade the clear allocation of §2-510(3).
 These provisions are ineffective; see Official Comment 3 to §2-510. The cited authorities are well worth reading on this
complicated issue. White & Summers, at §6-7, argue (on both the subrogation and risk of loss shifting argument, which
they deem “fancy footwork” in the insurance policy) that such attempts to deny a party the benefit of the other party’s
insurance should be ineffective, as contrary to the letter and purpose of §2-510 (and, they argue, public policy).
 Summary: Risk of Loss – Breach
o If goods that are tendered for delivery are not conforming to the contract, then the risk of loss remains on the seller until cure
or acceptance.
o If the buyer rightfully rejects nonconforming goods, the risk of loss remains on the seller.
o If the buyer accepted nonconforming goods but then rightfully revokes acceptance, then the risk of loss shifts back to the seller
to the extent that the buyer’s insurance does not fully cover the loss.
o If the buyer breaches by wrongfully repudiating the contract after the goods have been identified to the contract but before the
seller has performed, then to some extent the risk of loss shifts to the buyer. In that case the buyer has the risk of loss to the
extent that the seller’s insurance does not fully cover the loss.
 Canvas Quiz Questions
o Basically, if a seller ships non-conforming goods the risk of loss tends to stay on the seller after it normally would have, and if
a buyer wrongfully repudiates a contract then the risk of loss tends to shift to the buyer earlier than it would have.
 True
o If a seller ships goods that are non-conforming the risk of loss will stay on the seller until the defect is cured or until the buyer
accepts the goods.
 True
o If a buyer rightfully revokes acceptance of non-conforming goods, the entire risk of lost shifts back to the seller.
 False - If the buyer rightfully revokes acceptances of goods that are non-conforming, the risk of loss shifts back to the seller
to the extent that the buyer's insurance does not fully cover the loss.
o If the buyer breaches the contract by wrongfully repudiating a contract before the seller has performed, then the risk of loss
shifts to the buyer to the extent that the seller's insurance does not fully cover the loss.
 True
o Aston and Gerald sell high-end dolls. They obtained what they thought was a rare 19th century eight-legged walking doll and
advertised it for sale. Jonathan, who lives in Flint, Michigan, purchased it for $7,000. Under the contract, Aston and Gerald were
required to send the doll by Courier Express to Jonathan's apartment in Flint. The doll arrived, Jonathan loved it, and after

122
spending two days carefully inspecting it he sent Aston and Gerald an email thanking them for the item and promising to
purchase more items from them. A month later the doll was ruined when the bathtub in the upstairs apartment overflowed.
The flood did one good thing, though. The immersion (which ruined the doll) revealed that the doll was a fake. Jonathan did not
have renter's insurance or any insurance coverage that extended to the doll. When did the risk of loss shift in this case? What
legal rights does Jonathan have?
 Answer: The risk of loss shifted to Jonathan when he accepted the doll. It is too late to revoke acceptance because the doll
has been destroyed, and its destruction was unrelated to the fact that it was a fake. Jonathan had the risk of loss when the
doll was destroyed, and he must sue Aston and Gerald for breach of warranty.
 The risk of loss shifted to Jonathan when he accepted the doll, but he should now revoke his acceptance and the entire risk
of loss will shift back to Aston and Gerald.
 The risk of loss never shifted to Jonathan because the defect was not discoverable by means of a reasonable inspection.
Aston and Gerald had the risk of loss.
 The risk of loss shifted to Jonathan when he accepted the doll, but he should now revoke his acceptance and the risk of loss
will shift back to Aston and Gerald to the extent of his insurance deficiency, which is the entire amount.
B. Excuse of Performance.
 Excuse versus Breach
o Normally if a party fails to perform its duty under a contract it constitutes a breach of contract.
o However, if a party’s performance is excused then its failure to perform its duty is not a breach of contract. Instead, the legal
consequence of excuse of performance is as if the contract had been rescinded. Each party must make restitution to the other
of any partial performance, but neither party is liable to the other for any expectancy or consequential damages.
 Two Types of Excuse: Casualty to Identified Goods and Failure of a Presupposed Condition
o Article 2 of the U.C.C. excuses a party’s performance in two circumstances. First, under Section 2-613, the seller’s performance
may be excused if there has been “casualty to identified goods.” Second, under Section 2-615, either party’s performance may
be excused if an event preventing performance has occurred that the parties assumed would not occur, such as a natural
disaster or adoption of a government regulation.
o Section 2-614 permits substituted methods of delivery or payment.
 “Casualty to Identified Goods”
o Under Section 2-613, if the contract requires for its performance goods identified when the contract is made, and if those goods
are destroyed before the risk of loss has shifted to the buyer, then under Section 2-613 the seller’s performance is excused.
o If the loss is partial the buyer may demand inspection and at its option either treat the contract as avoided or accept the goods
with due allowance.
 Section 2-613: “Casualty to Identified Goods”
Where the contract requires for its performance goods identified when the contract is made, and the goods suffer casualty without
fault of either party before the risk of loss passes to the buyer, or in a proper case under a “no arrival, no sale” term (Section 2-324)
then
(a) if the loss is total the contract is avoided; and
(b) if the loss is partial or the goods have so deteriorated as no longer to conform to the contract the buyer may nevertheless
demand inspection and at his option either treat the contract as avoided or accept the goods with due allowance from the
contract price for the deterioration or the deficiency in quantity but without further right against the seller.
 Example: Casualty to Identified Goods: Lot Number 1285
o Naomi purchased 3 tons of nitrogen fertilizer for her farm from Grozit, a local feed and fertilizer distributor in Carlisle,
Pennsylvania. Ever careful, before Naomi signed the purchase agreement, she visited the warehouse, inspected the fertilizer,
and had 3 tons of it set aside for her and designated “Lot Number 1285.” The purchase agreement dated March 1 specified that
Grozit would deliver “Lot Number 1285” to Naomi on May 1. Unfortunately, a small fire destroyed Lot Number 1285 on April 15.
Grozit has other fertilizer of the same type that is available, but the market price has risen and Grozit insists that the prior
agreement is now void and that Naomi must sign a new purchase agreement at a higher price if she wants the fertilizer. Is
Grozit within its rights?
 ANSWER: Yes, Grozit is within its rights to insist upon a new contract. Grozit’s duty to perform the original contract was
excused when the goods were destroyed by fire because the goods were required to be identified at the time when the
contract was entered into. Naomi contracted for those particular goods, and when those goods were destroyed the seller’s
obligation to perform under the contract was destroyed as well. The goods were identified when the contract is made; the
goods suffered casualty without fault of either party; and the casualty occurred before the risk of loss passes to the buyer.
 Example: Casualty to Goods That Were Not Identified at the Time of Contract
o Change the facts to the previous problem. Assume that Naomi simply contracted to purchase three tons of nitrogen fertilizer
from Grozit. After the contract was entered into Grozit packed and labeled three tons of fertilizer that it intended to ship to
Naomi. Then a small fire at the warehouse destroyed the three tons of fertilizer that had been set aside for Naomi. In that
situation would Grozit’s obligation to perform under the contract be excused on account of the casualty to the goods?
 ANSWER: No! In this situation although the goods were destroyed after they were identified to the contract, the goods
were not required to be identified to the contract when the contract was entered into. Grozit’s duty to perform is not

123
excused, and it must either deliver three tons of fertilizer to Naomi at the original contract price or be considered to be in
breach of contract.
 Substituted Performance: Failure of Carrier or Failure of Means of Payment
o Under Section 2-614, if the agreed manner of delivery fails or becomes commercially impracticable but a commercially
reasonable method of delivery is available, the seller must tender and the buyer must accept the substitute method of delivery.
* Remember the gargoyle (shipped by air and not by sea - F.A.S. S.S. Titanic)?
o Similarly, if the agreed manner of payment fails because of government regulation, the seller may withhold delivery until the
buyer provides a substantially equivalent method of payment.
 Section 2-614: Substituted Performance: Failure of Carrier or Failure of Means of Payment
(1) Where without fault of either party the agreed berthing, loading, or unloading facilities fail or an agreed type of carrier becomes
unavailable or the agreed manner of delivery otherwise becomes commercially impracticable, but a commercially reasonable
substitute is available, such substitute performance must be tendered and accepted.
(2) If the agreed means or manner of payment fails because of domestic or foreign governmental regulation, the seller may
withhold or stop delivery unless the buyer provides a means or manner of payment which is commercially a substantial
equivalent. If delivery has already been taken, payment by the means or in the manner provided by the regulation discharges
the buyer's obligation unless the regulation is discriminatory, oppressive or predatory.
 “Excuse by Failure of Presupposed Conditions”
o This is the heading of Section 2-615. This is the “gap filler” provision of Article 2 on excuse of performance. If the parties have
not adopted a force majeure clause, then under Section 2-615 a party’s performance is excused if performance is rendered
“impracticable” by “the occurrence of a contingency the non-occurrence of which was a basic assumption of the contract” or by
“compliance in good faith with any applicable foreign or domestic governmental regulation or order.”
 Section 2-615(a): “Excuse by Failure of Presupposed Conditions”
Except so far as a seller may have assumed a greater obligation and subject to the preceding section on substituted performance:
(a) Delay in delivery or non-delivery in whole or in part by a seller who complies with paragraphs (b) and (c) is not a breach of his
duty under a contract for sale if performance as agreed has been made impracticable by the occurrence of a contingency the
non-occurrence of which was a basic assumption on which the contract was made or by compliance in good faith with any
applicable foreign or domestic governmental regulation or order whether or not it later proves to be invalid.
 Increased Cost Alone Does Not Excuse Performance
o Every contract is entered into in order to protect against changes in the market price. The buyer normally assumes the risk that
the price will drop and that it could have obtained the goods at a lower price if it had not entered into the contract. The seller
assumes the risk that the market price will rise and that it could have obtained a better price for the goods had it not entered
into the contract. The whole point of entering into a contract is to minimize the risk of loss against fluctuations in the market.
Accordingly, changes in market price alone do not justify excusing a party’s performance and cancelling the contract.
o What matters is the cause of the change in the ability of the party to perform. Was there some unforeseen event that impaired
the ability of the party to perform – an event that the parties had assumed would not occur?
 Comment 4 to Section 2-615: Increased Cost Alone Does Not Excuse Performance
Increased cost alone does not excuse performance unless the rise in cost is due to some unforeseen contingency which alters the
essential nature of the performance. Neither is a rise or a collapse in the market in itself a justification, for that is exactly the type of
business risk which business contracts made at fixed prices are intended to cover. But a severe shortage of raw materials or of
supplies due to a contingency such as war, embargo, local crop failure, unforeseen shutdown of major sources of supply or the like,
which either causes a marked increase in cost or altogether prevents the seller from securing supplies necessary to his performance,
is within the contemplation of this section. (See Ford & Sons, Ltd., v. Henry Leetham & Sons,)
 Example: Excessive Rain
Bob is a potato broker. Bob entered into a contract with McDonald’s to provide 80 tons of potatoes to be delivered by November 1.
There was excessive rain in the Midwest that ruined the potato crop, thus raising the price of potatoes nationwide by 25%, making
the contract with McDonald’s relatively unprofitable for Bob. Bob claims that this unforeseen event should excuse his performance
under the contract. McDonald’s demands that he either deliver the potatoes or be liable for breach of contract. Who wins?
 Add a few more facts. All of Bob’s contracts for obtaining potatoes were with farmers in the Midwest, none of whom have
any potatoes for sale. McDonald’s was aware of Bob’s regional affiliations with farmers and contracted with potato brokers
in many different regions around the country specifically to protect against localized and regional crop failures. Does this
change the outcome?
(b) McDonald's should win. Bob's "production capacity" is not destroyed. He can obtain potatoes from elsewhere to perform the
contract. Bob took the risk that the price of potatoes would rise. McDonald's entered into this contract to protect itself against
an increase in prices. Bob's performance is not excused.
(c) Bob's performance is excused. This time a basic assumption of the contract was that there would be potatoes produced in the
Midwest. The contract is avoided.
 Force Majeure Clauses
o A written contract may expressly define the events that will excuse a party’s performance.
o Such a clause is called a “Force Majeure” clause.

124
o Force Majeure clauses may be written in two ways.
 Clause 1: Neither SELLER nor BUYER shall be considered in breach of this contract to the extent that its performance of any
duty under this contract is prevented by natural disasters (such as extreme weather events, earthquakes, drought, tidal
waves, and floods); fires or explosions; war, terrorism, or other armed hostilities; public riot, disorder or commotion; labor
strikes, lockouts, or other labor action, unless solely restricted to employees of the party claiming that its performance is
excused.
 Clause 2: Neither SELLER nor BUYER shall be considered in breach of this contract to the extent that its performance of any
duty under this contract is prevented by an event beyond the control of that party, including but not limited to natural
disasters (such as extreme weather events, earthquakes, drought, tidal waves, and floods); fires or explosions; war,
terrorism, or other armed hostilities; public riot, disorder or commotion; labor strikes, lockouts, or other labor action,
unless solely restricted to employees of the party claiming that its performance is excused.
 Example: Train Wreck and Nuclear Contamination
o Suppose that the seller is unable to complete the manufacture of the goods because a train crashed near its factory spilling
thousands of gallons of radioactive slurry, necessitating evacuation, and that it will take months if not years to decontaminate
the factory facility.
 Under Clause 2 (the open-ended force majeure clause) the seller’s performance would be excused, but under the Clause 1
the seller’s failure to perform would likely not be excused. The clause would probably be interpreted to mean that the seller
assumed the risk of events not listed in the clause and would be in breach of the contract for failure to deliver the goods.
The seller would actually be worse off with Clause 1 than if there had been no force majeure clause because Section 2-615
would likely have excused the seller’s performance.
 Partial Excuse and Allocation
o If an event only partially excuses a seller’s ability to perform its duty under the contract, the seller may allocate production
and deliveries among its customers in any manner that is fair and reasonable.
 Section 2-615(b): Partial Excuse and Allocation
(b) Where the causes mentioned in paragraph (a) affect only a part of the seller's capacity to perform, he must allocate production
and deliveries among his customers but may at his option include regular customers not then under contract as well as his own
requirements for further manufacture. He may so allocate in any manner which is fair and reasonable.
Seller’s Duty to Notify Buyer of Excuse
o If an event occurs that excuses the seller’s duty to perform the contract, the seller must promptly notify the buyer of this fact.
o If the seller’s performance is partially excused, the seller must promptly notify the buyer of the estimated quota made
available to the buyer.
 Section 2-615(c): Seller’s Duty to Notify Buyer of Excuse
(c) The seller must notify the buyer seasonably that there will be delay or non-delivery and, when allocation is required under
paragraph (b), of the estimated quota thus made available for the buyer.
 Casebook Problem
o Virgil Escapement had always wanted a sundial for his garden, and he ordered one for $250 from Horology Timepieces, Inc. The
latter had 12 sundials of the type Escapement ordered in its storage room when an earthquake shook the building. All 12 fell
over, and all but 3 were smashed. The remaining three were slightly damaged. Escapement, on being informed of the problem,
insisted on the right to look over the three remaining sundials and to select one for his purchase, possibly at a reduced price due
to the damage. Horology comes to you. Is §2-613 or §2-615 relevant? Must it let Escapement pick out a sundial, and must it
offer to let him purchase at a reduced price, or can it simply cancel without fear of legal liability? For the test for impossibility
of performance in international sales, see CISG Article 79; cf. Articles 71 and 72.
 §2-613 does not apply because the contract did not require the goods to be identified when the contract was entered into.
As a result, the buyer does not have the right to inspect the goods and pick one out and buy it at a reduced price.
 However, Section §2-615 does apply. Under §2-615, when a portion of the seller’s capacity has been affected, the seller
may proceed “in any manner which is fair and reasonable.” In this case, with only 3 sundials left, the seller may prefer to
allocate them to its regular customers.
 In cases of partial loss, the difference between §2-613 and §2-615 is that §2-613 gives the buyer the right to inspect and
accept goods with due allowance and §2-615 gives the seller the right to allocate production and deliveries among current
customers, regular customers not under contract, and its own requirements.
 Casebook Problem
o Suppose the following, using the basic facts of the last Problem. When Horology received Virgil’s order, one of their
salespersons immediately put a red tag on one of the sundials. It said, “Hold for Virgil Escapement.” Then the earthquake
occurred, and miraculously only Virgil’s sundial was destroyed. The other 11 sundials, all exactly like Virgil’s, were undamaged.
When Virgil demanded his sundial, Horology pleaded §2-613. Will that section excuse them? See Valley Forge Flag Co. v. New
York Dowel & Molding Import Co.
 What if the salesperson had tagged Virgil’s order as soon as the contract was made, and it was destroyed in the
earthquake? Does Section 2-613 apply in this case, excusing the seller’s performance?

125
 No, again, the contract did not require that the goods be identified at the time that the contract was entered into. Virgil is
entitled under the contract to one of the other sundials.
 The cited case reaches this result as to fungible goods and adds that there must be a meeting of the minds as to a specific
item before it is identified.
 Arabian Score v. Lasma Arabian Ltd.
o Lasma sold Arabian a colt, Score, for $1 million. However, under paragraph 5 of the agreement, as part of the deal Lasma
promised to spend $250,000 of the purchase price promoting Score as a stallion. Under paragraph 4 of the agreement, if Lasma
determined (in its sole discretion) that Score was not a Star Stallion, and it did not spend the money to promote Score, then any
remaining funds would be returned to Arabian.
o Score passed away after siring two foals. Lasma had only spent $50,000 promoting the horse. Arabian sues Lasma for the other
$200,000, on the theory that further promotion of a dead horse is useless, and that Lasma should just pay it the money in
accordance with paragraph 4.
o Arabian sought cancellation of the promotion portion of the contract.
o The court declined. The colt’s death was unfortunate but also foreseeable (as evidenced by the insurance policy purchased for
him, unfortunately from an insurer that failed). Promoting dead horses had been done in the past, to burnish the “owning
entity's reputation and to increase the value of the stallion’s progeny.” So, the purpose of the contract was not frustrated.
o Note that the court applied the common law doctrine of commercial frustration. UCC’s §2-615 Excuse by Failure of Presupposed
Conditions, by its terms, applies only where seller’s performance becomes impracticable. Official Comment 9, however, states
that buyers may be entitled to the protection of the section in an appropriate case.
 Casebook Problem
o In the mid-1960s, in an effort to boost sales of its nuclear reactors, Westinghouse Corporation agreed to sell 27 utility
companies 80 million pounds of uranium over the next 20 years. The average sale price per pound was $10. When
Westinghouse made the sale, it actually owned only 15 million pounds of uranium. By the mid-1970s, the price of uranium had
risen to $40 a pound. In late 1975, Westinghouse announced that it would not honor its contract. The utilities sued.
Westinghouse argued that the best evidence in the late 1960s and early 1970s indicated uranium prices would be stable over
the long term. The Corporation claimed that the price rise was unforeseeable and that the contracts were excused under §2-615
as “commercially impracticable.” In particular, Westinghouse blamed the 1973 oil embargo and worldwide price fixing for the
“unpredictable” price rises. See Official Comments 4 and 5 to §2-615. How should the dispute be resolved? See Publicker Indus.
v. Union Carbide Corp.; Ecology Services, Inc. v. Granturk Equipment, Inc.; Eagan, The Westinghouse Uranium Contracts:
Commercial Impracticability and Related Matters, 18 Am. Bus. L.J. 281 (1980); Rochester Gas and Electric Corporation v. Delta
Star, (seller of goods at a fixed price was not relieved of obligation where price of component increased due to Hurricane
Katrina). If you could advise Westinghouse on how to avoid this problem in the future, what would you suggest? See §2-305.
 The trial judge told Westinghouse that it was going to lose, so Westinghouse settled the case.
 But why?
 In the Publicker case cited in the problem, the court was unwilling to let a price rise excuse compliance with the fixed price
contract, saying that fixing a price also fixes the risk of the seller that a market fluctuation will cause the contract to become
unprofitable.
 In Ecology Services the problem was the failure of a key supply source, but the court held that such a problem is foreseeable
and a risk that is on the seller unless the contract expressly provides otherwise.
 The courts have been very reluctant to let price rises from the seller’s supplier be an excuse for avoiding fixed price
contracts with the buyer.
 Another set of famous cases rejecting the doctrine of frustration was the Suez Canal Cases, a trilogy of cases decided by the
British courts. The court said in those cases that the unforeseen cost must be more than merely onerous or expensive but
must be “positively unjust.”
 These cases have been followed by American courts, most notably in the Transatlantic case by Judge J. Skelly Wright. The
Suez Canal cases, and the Transatlantic case are discussed later.
 So, what would you advise to avoid the problem in the future?
 The solution would be for the seller not to enter into fixed price contracts, but instead tie costs to some sliding scale.
Otherwise, the courts feel that the fixed price contract has already allocated the risk of market change, and it would be
unfair to disturb that allocation.
 To avoid this problem in the future the seller might insist on an open price term – Section 2-305.
 Anecdote:
• The famous (and controversial) case of Aluminum Co. of America v. Essex Group, 499 F. Supp. 53 (W.D. Pa. 1980), is one
in which the court completely rewrote the contract to give each party the profits originally expected. The Alcoa fact
pattern, however, is unique.
• In that case the parties had employed eminent economist Alan Greenspan (later chair of the Federal Reserve System)
to devise a pricing index for the sale of aluminum. When the index chosen proved to be dramatically out of line with
the expectations of the parties (because of the unexpected actions of OPEC in the early 1970s), the pricing formula

126
awarded one party huge gains while costing the other a loss of $75 million - the court reformed the contract to bring it
back into line with the original understanding.
• The case was settled rather than appealed, leaving on the books a citation permitting a court enormous freedom to
rewrite contracts where the need to do so is strong.
 Louisiana Power & Light Co. Allegheny Ludlum Industries, Inc.
o Seller = Allegheny Ludlum; Buyer = Louisiana Power & Light
o The seller Allegheny Ludlum purported to cancel the contract because of rising costs of supplying aluminum tubing. The buyer
Louisiana Power & Light sued for its cover costs in purchasing aluminum tubing.
o The buyer – that is, the plaintiff – moved for summary judgment. The trial court in this case granted summary judgment to the
buyer on the question of the seller’s liability but held that the reasonableness of the cost of cover is a material issue of fact for
trial.
o This is a trial court decision applying Section 2-615. This case gives many examples of situations where sellers were held to
contracts despite increases in price.
o FACTS: Allegheny Ludlum had agreed to provide condenser tubing to Louisiana Power & Light for a price of $1.1 million. It then
wrote to LP&L that its costs were rising, and the price would have to be higher. LP&L then wrote to demand assurances from
Allegheny. Allegheny wrote back that it needed more compensation, but it did not give assurances. LP&L then wrote back to say
that Allegheny had repudiated the contract, and that it would sue for damages.
o Recall our Discussion of §2-609 – the right to demand assurances.
o Recall that repudiation = breach, giving rise to buyer’s right to sue for damages §2-713 (Buyer’s Damages for Non-Delivery or
Repudiation)
o Allegheny claimed excuse of performance under 2-615. Court states that there must be 3 elements under 2-615:
1. A “contingency” must occur;
2. Performance must be made “impracticable”; and
3. The non-occurrence of the contingency must have been a basic assumption on which the contract was made.
 The effect of the contingency must be to make the contract “vitally different” from the one that was entered into.
o The burden of proof is on the person claiming excuse. Citing the Eastern Airlines case of the 5th Circuit.
o “The undisputed facts in this case show that Allegheny has not and cannot meet its burden of proof as to this defense because it
is unable to show that performance under the contract was commercially impracticable.”
o The party claiming excuse must show that its performance would be “especially severe and unreasonable,” citing the Gulf Oil
case.
o Allegheny’s costs increased only 38%.
o There is a long quote from the Eastern Airlines case discussing the “Suez Cases” – the lost must be “positively unjust.” “Those
cases offered little encouragement to those who would wield the sword of commercial impracticability.”
o Quote from Judge Skelly Wright in Transatlantic, refusing to find impracticability due to “added expense alone,” particularly
where the promisor “can legitimately be presumed to have accepted some degree of abnormal risk.”
o In American Trading and Production Co. v. Shell Oil, a Suez Canal case, the court stated, “Mere increase in cost alone is not a
sufficient excuse for non-performance.”
o Iowa Electric, costs increased 54%, did not constitute commercial impracticability.
o Footnote on the Alcoa case - where one party stood to lose and the other stood to gain $60,000,000, that was impracticable –
but in this case it was $428,000 on a $1.2 million contract.
o The key holding in the case:
 “As the jurisprudence indicates, Allegheny’s performance under the contract was not commercially impracticable. Even if
Allegheny were to show that its increased costs constituted a contingency, the non-occurrence of which was a basic
assumption on which the contract was made, it still could not show that performance had been rendered commercially
impracticable as a result. Allegheny bears the burden of proof as to each of the requisite three elements on its claim of
commercial impracticability. Being unable to prove all three elements, its defense must fail.”
o The contract contained a clause permitting LP&L to cancel the contract at any time and pay Allegheny its disbursements and
expenses.
 Note – this is actually a termination clause.
 Note – there is a difference between termination, rescission, and cancellation.
o Section 2-106 of Article 2 of the Uniform Commercial Code defines, among other concepts, the ending of the contract, as
follows:
 "Termination" occurs when either party pursuant to a power created by agreement or law puts an end to the contract
otherwise than for its breach. On "termination" all obligations which are still executory on both sides are discharged but any
right based on prior breach or performance survives.
 "Cancellation" occurs when either party puts an end to the contract for breach by the other and its effect is the same as
that of "termination" except that the cancelling party also retains any remedy for breach of the whole contract or any
unperformed balance.

127
 A” rescission” restores the parties as though the agreement was never written. A rescission also releases the parties from
any and all present or future liability.
o Allegheny claimed that the cancellation clause was unconscionable.
o The court quotes Section 2-302 and Official Comment 1. Unconscionability is a question of law for the court. However, Section
2-302(2) provides that there must be “a reasonable opportunity to present evidence as to commercial setting, purpose, and
effect to aid the court in making its determination.” The Court held that this could not be determined on summary judgment.
o LP&L did not act in bad faith when it refused to renegotiate the contract. The Court Cites Sections 1-203 and 2-103(1)(b). There
is “no obligation” to even discuss renegotiation of a contract.
o LP&L is allowed to cover, but the reasonableness of its cover, timeliness, mitigation, and costs, have to be determined as a
question of fact – not on summary judgment.
 Summary: Excuse of Performance
o If a party’s performance is excused, then its failure to perform its duty is not a breach of contract. If the contract requires the
goods to be identified at the time that the contract is entered into, and if those identified goods are destroyed before the risk of
loss has shifted to the buyer, then the seller’s performance is excused under Section 2-613. Under Section 2-615 a party’s
performance is also excused if performance is rendered “impracticable” by “the occurrence of a contingency the non-
occurrence of which was a basic assumption of the contract” or by “compliance in good faith with any applicable foreign or
domestic governmental regulation or order.” If an event only partially excuses a seller’s ability to perform its duty under the
contract, the seller may allocate production and deliveries among its customers in any manner that is fair and reasonable. The
seller must promptly notify the buyer if an event occurs that excuses its failure to perform the contract.
o A “force majeure” clause may be used to expressly identify the events or types of events that would excuse performance. But
use them with care!
 Canvas Quiz Questions
o Normally if a party fails to perform its duty under a contract it constitutes a breach of contract. However, if a party's
performance is excused then failure to perform its duty is not a breach.
 True
o Which of the following is the best statement of the rule regarding "casualty to identified goods" under Section 2-613?
 Answer: If the contract requires for its performance goods identified when the contract is made and if those goods are
destroyed before risk of loss has passed to the buyer, then the seller's performance is excused.
 If goods are destroyed after they have been identified to the contract and before the risk of loss has passed to the buyer,
then the seller's performance is excused.
 If goods are destroyed before they have been identified to the contract and before risk of loss has passed to the buyer, then
the seller's performance is excused.
 If the contract requires for its performance goods identified when the contract is made and if those goods are destroyed
after risk of loss has passed to the buyer, then the seller's performance is excused.
o If the agreed manner of delivery fails, the seller must tender, and the buyer must accept a commercially reasonable
substitute; and if the agreed manner of payment fails the buyer must provide a substantially equivalent method of payment.
 True
o An increase in costs excuses the performance of the seller.
 False
o Bob is a potato broker. Bob entered into a contract with McDonald's to provide 80 tons of potatoes to be delivered by
November 1. There was excessive rain in the Midwest that ruined the potato crop, thus raising the price of potatoes nationwide
by 25%, making the contract with McDonald's relatively unprofitable for Bob. Bob claims that this unforeseen event should
excuse his performance under the contract. McDonald's demands that he either deliver the goods or be liable for breach of
contract. Who wins?
 Answer: McDonald's should win. Bob's "production capacity" is not destroyed. He can obtain potatoes from elsewhere to
perform the contract. Bob took the risk that the price of potatoes would rise. McDonald's entered into this contract to
protect itself against an increase in prices. Bob's performance is not excused.
 Bob should win. A basic assumption of the contract was that there would not be such excessive rain that it would ruin the
potato crop for the entire Midwest. Bob's duty is excused, and the contract is avoided.
o Bob is a potato broker. Bob entered into a contract with McDonald's to provide 80 tons of potatoes to be delivered by
November 1. There was excessive rain in the Midwest that ruined the potato crop, thus raising the price of potatoes nationwide
by 25%, making the contract with McDonald's relatively unprofitable for Bob. All of Bob's contracts for obtaining potatoes were
with farmers in the Midwest, none of whom have any potatoes for sale. McDonald's was aware of Bob's regional affinations
with farmers and contracted with potato brokers in many different regions around the country specifically to protect against
localized and regional crop failures. Bob claims that this unforeseen event should excuse his performance under the contract.
McDonald's demands that he either deliver the goods or be liable for breach of contract. Who wins?
 Answer: Bob's performance is excused. This time a basic assumption of the contract was that there would be potatoes
produced in the Midwest. The contract is avoided.

128
 Bob's performance is not excused. Bob's "production capacity" is not affected in that he could contract to purchase
potatoes elsewhere in the country to satisfy his duty to McDonald's under the contract. Bob must either deliver the
potatoes or be in breach of contract.
o Which of the following statements regarding "force majeure" clauses are accurate?
 The express terms of a force majeure clause imposing greater obligation on the seller will control over the code provisions
relating to excuse of performance by failure of presupposed conditions.
 There is a risk that a force majeure clause could be read exclusively; that is, that the clause would be interpreted to mean
that events not mentioned in the clause do not justify excuse of performance.
 Force majeure clauses in contracts are in general ineffective because Code provisions regarding excuse preempt force
majeure clauses.
o Snap Industries manufactures rubber bands and also wraps small containers with rubber bands for other companies. Its rubber
band factory is in Peoria. The factory's output was already stretched to the breaking point when a tornado destroyed 60% of the
factory's production capacity. Snap normally uses about 10% of the factory's output for its own container wrap business. In
addition to customers under contract, it has a few regular customers whom it wishes to be able to serve while the factory is
being repaired. Which of the following is the most accurate statement regarding Snap's duty to allocate its limited production
capacity among its customers?
 Answer: Snap may allocate production among customers under contract, its own requirements, and regular customers not
currently under contract in any manner which is fair and reasonable.
 Snap may allocate production among customers under contract in any manner which is fair and reasonable, but it may not
make allocation for its own requirements or for any customers not under contract.
 Snap may allocate production among customers under contract and its own requirements in any manner which is fair and
reasonable, but it may not make allocation to any customers not under contract.
 All existing contracts are avoided, and Snap may allocate production in any manner it chooses.
o A seller must give seasonable notice to the buyer that there will be a delay or non-delivery due to excuse of performance.
 True
 Review/Assessment Questions
 Is a buyer obliged to accept the goods if the seller has substantially performed?
o No. The perfect tender rule requires buyer to accept only if there has been a perfect tender, not mere substantial performance.
Section 2-601.
 What are the exceptions to the perfect tender rule?
o Installment contracts, seller’s right to cure, de minimis (if the relevant court follows that approach), and any provision in the
contract.
 How can a buyer avoid accepting the goods?
o By effectively rejecting them.
 What is an ineffective rejection?
o See §2-602: failing to reject within a reasonable time after delivery, failing to notify seller – or doing anything “inconsistent with
seller’s ownership” §2-607(c).
 Can a buyer make an effective rejection that breaches the contract?
o Yes, buyer may effectively reject the goods, meaning buyer is not liable for the price. But buyer will still be liable for damages for
wrongful rejection, such as the difference between the price and the market price, or seller’s lost profits.
 Is a buyer bound by an acceptance of the goods if seller has breached?
o Yes (unless buyer can revoke acceptance, under the heightened standard of §2-608). Buyer, having accepted the goods, is liable
for the price, but has a counterclaim for damages from the breach of warranty.
 What is the difference between rejection and revocation of acceptance?
o Buyer can always reject the goods tendered (although it may be a wrongful rejection, making buyer liable for damages). If Buyer
accepts, Buyer can revoke only if Buyer meets the standards set in §2-608. In particular, if Seller has not breached, Buyer will
definitely not be able to revoke. There can be wrongful rejection, but not a wrongful revocation.
 What is the effect of breach, by seller or buyer, on risk of loss?
o If one party breaches, the risk of loss will be on them longer than otherwise. See §2-610.
 When will a party be excused from performance by failure of presupposed conditions?
o Courts have required a strong showing under §2-615. As the Louisiana Power & Light case states, a party will not be excused
simply because the contract has become burdensome or financially costly. Rather, impracticability may be found where “an
event occurs which renders performance so ‘vitally different’ from that which is anticipated, the contract cannot be reasonably
considered to govern and performance under that contract is excused.
 Wambach Retail ordered 10,000 folding chairs from Lloyd Furniture, to be delivered in 5 shipments of 2,000 chairs, on the first
days of July, August, and September. The first shipment arrived on July 1, missing a chair. On July 2, may Wambach Retail
rightfully reject the shipment?
o Yes. In sales law, buyer may reject a seller’s tender unless it is a perfect tender.
o Yes. The missing chair can never be delivered on time, because July 1 is in the past, so the defect cannot be cured.
129
o Answer: No. The non-conformity is not substantial enough to reject.
o No. Buyers must always accept a good faith attempt by sellers to meet the contract requirements.
– In sales contract generally, a buyer may reject a tender “if the goods or the tender of delivery fail in any respect to
conform to the contract.” §2-601. This requirement of perfect tender protects buyers from being required to accept
and pay unless the seller delivers exactly as promised. But a lower standard exists for installment contracts, where
the parties have repeated dealings and so the buyer is at less of a bargaining disadvantage. In installment
contracts, a buyer may reject only “an installment which is non-conforming if the non-conformity substantially
impairs the value of that installment and cannot be cured or if non-conformity is a defect in the required
documents.” One missing chair of 2,000 is not substantial (unless there were special circumstances, not present
here). Even if it were, it could be quickly cured. We are not told of any problems with the documents. Grounds for
rejecting an installment delivery are lacking.
 A question with a slight but important difference: can Wambach Retail reject the shipment?
o No. The non-conformity is not substantial enough to reject.
o Answer: Yes. A buyer can reject a shipment, even if the rejection is wrongful, although buyer would then be in breach. -
Wambach Retail does not have grounds to reject the shipment. But Wambach Retail can still reject the shipment. That would be
an effective, but wrongful rejection. Wambach Retail would not be liable for the price, because it did not accept the goods, but
would be liable for any damages, such as Lloyd Furniture reselling the chairs at a loss.
o No. If seller makes a tender that may not be rejected, then buyer cannot reject the tender.
o Yes. Buyer always has complete discretion about whether to accept a tender.
 Lloyd Furniture carefully counted the chairs for a different contract, a single delivery of 1,000 chairs to Rapinoe Raptures, made on
the agreed delivery date. Finding the shipment 1 chair short, Lloyd made the delivery as scheduled, arranging for another chair to be
delivered the following day. But Rapinoe Raptures rejected the tender of 999 chairs and refused to allow Lloyd Furniture to cure the
defect, because the delivery date was now past. Does Lloyd Furniture have a right to cure the defect?
o No. The delivery date is past and so the defect cannot be cured.
o No. The shaken faith doctrine would apply to prevent cure.
o Answer: Yes. Lloyd Furniture may cure the defect, where it reasonably believed the tender was conforming. - A seller does not
always have a right to cure. Under §2-508, a seller has a right to cure if the time for performance has not expired (not applicable
here) or the seller had reasonable grounds to believe the tender would be accepted. Here, the seller could reasonably believe
that one missing chair of one thousand would not result in rejection of the delivery.
o Yes. A seller always has a right to cure.
 Pemulis Auto signed yet another contract, to sell a 2016 Model Excalibur Hardtop to Hamlet. As agreed, Hamlet showed up Monday
morning at Pemulis Auto to pick up the car. But after Hamlet paid and the papers were signed, Hamlet abruptly decided to wait a
couple of days before driving away. That delay breached the contract, which required Hamlet to take delivery upon payment. The
next day, the car was struck by lightning and destroyed. Pemulis Auto did not have insurance covering the car. Is Hamlet entitled to
return of the price paid?
o No. Once the car was paid for and the papers signed, the car belonged to Hamlet.
o Answer: No. Hamlet was in breach, so the risk of loss was on Hamlet. - Normally, when the buyer is not a merchant, the risk of
loss does not pass until the goods have been delivered. §2-509(3). But when one party breaches, a different set of rules may
apply under §2-510. Where the buyer is in breach, the seller may treat the risk of loss as resting on the buyer for a commercially
reasonable time. §2-510(3).
o Yes. Hamlet had not taken delivery, so the risk of loss was still on the seller.
o Yes. Pemulis Auto was at fault for failure to obtain insurance. - The parties may agree on different rules for risk of loss or for one
party to obtain insurance, but they did not do so here. So, Pemulis Auto was not required to obtain insurance.
 Pemulis Auto sold a used Hekla sedan to Rapinoe, assuring her that the engine had been thoroughly tested for reliability and safety.
Rapinoe agreed to take the car, signed the papers to get title transferred, and set off for home in the car. The engine burst into
flames after a mile. Pemulis Auto offers to set off the price of a new engine but refuses to take the car back. Is Rapinoe obliged to
pay for the car, minus the cost of a new engine?
o Yes. Rapinoe accepted the car, so is obliged to pay for it, minus damages for the breach.
o Yes. Rapinoe should have tested the car herself.
o No. Rapinoe may revoke her acceptance.
o Answer: No. A buyer is never obliged to pay for defective goods.
o Yes. Rapinoe cannot revoke acceptance, because the car is damaged and cannot be returned intact.
– Rapinoe accepted the car. She agreed to take the car, and had ownership transferred, either of which would be
acceptance. §2-606. Normally a buyer that accepts the goods must pay the price, with a counterclaim for any
breach by seller. §2-607. But sometimes a buyer may revoke acceptance. §2-608. Rapinoe meets the requirements
for revocation of acceptance. She accepted the goods with assurances from the seller that the engine was not
defective. §2-608(1)(b). The non-conformity substantially impairs the value of the car to Rapinoe. §2-608. There has
been a substantial change in the condition of the goods, but that does not prevent revocation where that change
was caused by the defect in the goods. §2-608(2).
130
Chapter 7: Remedies
I. Special Remedies
 The Purpose of Remedies Under the U.C.C.:
o Benefit of the Bargain – Not Penalties
o The U.C.C. is not a moral document – its purpose is not to punish people who break their promises.
o According to Section 1-305 its purpose is to ensure that when a contract is breached the law will award the non-breaching party
the “benefit of the bargain” – to put that party in as good a position as if the other party had fully performed.
o Punitive damages may be awarded under the common law where a party is guilty of fraud or bad faith, but Article 2 does not
award punitive damages for breach of contract, nor does it enforce remedies that are the equivalent to a penalty.
 Section 1-305(a): Special Remedies
(a) The remedies provided by [the Uniform Commercial Code] must be liberally administered to the end that the aggrieved
party may be put in as good a position as if the other party had fully performed but neither consequential or special damages
nor penal damages may be had except as specifically provided in [the Uniform Commercial Code] or by other rule of law.
 Special Remedies Under Article 2 – Remedies Other Than Monetary Damages for Breach
o Article 2 of the U.C.C. contains a number of provisions that give sellers and buyers the right to restitution, as well as expectancy,
consequential, and incidental damages for breach of contract. Those remedies for sellers and buyers are covered in the next
two presentations in this series. This presentation reviews a number of special remedies provided by Article 2. These include:
1. Liquidated damages for either party under Section 2-718(1);
2. Breaching Buyer’s right to restitution under Section 2-718(2);
3. Buyer’s right to specific performance under Section 2-716(1);
4. Buyer’s right to replevin under Section 2-716(3);
5. Buyer’s right to recover the goods under Section 2-502; and
6. Seller’s right of reclamation of the goods under Section 2-702.
 Special Remedies for the Seller Under Other Laws
o Article 9 of the UCC: If the seller and the buyer enter into a security agreement giving the seller a security interest in the goods
to secure payment of the purchase price, then the seller has a “purchase money security interest,” which gives the seller a
powerful legal right to repossess the goods for non-payment of the purchase price.
o Bankruptcy Code: Even without an Article 9 security interest, a seller has some legal rights in bankruptcy. Section 546(c) of the
Bankruptcy Code gives the seller the right to reclaim goods from the bankruptcy trustee if those goods were delivered to the
buyer within 45 days before bankruptcy commenced at a time when the buyer was insolvent.
A. Remedies on Insolvency
 Remedies on Insolvency
o The UCC adopts the “equitable” definition of insolvency – inability to pay bills as they become due, AND/OR the bankruptcy
definition, liabilities greater than assets (which is more difficult to prove). §1-201(b)(23).
o Seller’s right to Reclamation on Buyer’s insolvency. §2-702.
o Buyer’s right to recover the price on Seller’s insolvency. §2-502.
 Casebook Problem
o Ratchett Tools delivered a truckload of inventory to Madoff Hardware. The next day, Ratchett Tools read that Madoff Hardware
had shut its doors, failed to pay its many creditors, and has few assets to go after. Can Ratchett get the inventory back? See §2-
702(2). Would it help sellers to include a clause in their sales contract forms making buyers promise they had the
wherewithal to pay for goods ordered? See §2-702(2). What if Ratchett learned of the financial distress before delivery? §2-
702(1). What if Madoff Hardware files for bankruptcy? See 11 U.S.C. §546(c).
 Reclamation by the seller from an insolvent buyer under § 2-702.
 Ratchett Tools delivered goods to Madoff Hardware, who then promptly shut its doors and was not paying creditors.
 §2-702(2) allows Ratchett Tools to reclaim the inventory it delivered, provided it acted within 10 days.
 If the buyer, within three months of delivery, represented that it was solvent, then 10-day limit does not apply.
 If the buyer declares bankruptcy then the seller can recover goods delivered within 45 days of filing. BC §546(c).
 It might help sellers to include representations of solvency in sales contracts. Where there has been misrepresentation in
writing of solvency, the period to reclaim is extended to three months—although showing that there was a
misrepresentation of solvency might be difficult. If Madoff Hardware is in bankruptcy, § 546(c) of the Bankruptcy Code
allows the seller to exercise its reclamation rights.
 Practice Pointer: Most unpaid sellers will not be able to use reclamation because of the short time limits. But for those few
that are well-advised of their rights and act quickly, reclamation can save the day. Without reclamation, an unpaid seller
with no collateral merely has an unsecured claim in the bankruptcy proceeding—which in the typical bankruptcy yields zero
dollars.
B. Liquidated Damages
 1. Liquidated Damages
o Under Section 2-718(1) the parties may agree to fix the amount of damages for breach of the contract. However, the amount of
the liquidated damages must be “reasonable” in light of:

131
 The anticipated or actual harm;
 The difficulty of proving loss; and
 The inconvenience or nonfeasability of obtaining an adequate remedy.
• A term fixing unreasonably large liquidated damages is void.
 Liquidated Damages, §2-718(1)
o This is a sliding scale. The greater the difficulty of proving loss, the more leeway that the parties have in setting a fixed sum as
liquidated damages.
o Obviously, if the amount of liquidated damages is tied to the specific breach that occurred or to market values at the time and
place of the breach the more likely that the clause will be upheld.
o A term fixing unreasonably large liquidated damages is void.
o With Leases under Article 2A, the UCC provides a formula for damages. §2A-504. A penalty clause would surely be void.
o No one knows for sure the answer to the question posed about the ability of the parties in a lease transaction to contract for a
penalty amount, but surely the courts would not allow that; see §1-305.
o Our law has a tremendous bias against penalty clauses in contracts.
 Section 2-718(1): Liquidated Damages
(1) Damages for breach by either party may be liquidated in the agreement but only at an amount which is reasonable in the light
of the anticipated or actual harm caused by the breach, the difficulties of proof of loss, and the inconvenience or nonfeasibility
of otherwise obtaining an adequate remedy. A term fixing unreasonably large liquidated damages is void as a penalty.
 Example: Liquidated Damages and Aluminum
o On February 1 Klinginsmith Mining agreed to sell and Von Damme Construction agreed to buy 400 tons of aluminum at a price
of $2,000 per ton (total of $800,000) to be delivered on June 1. The price of aluminum is very volatile – the price may double or
be cut in half over the course of a year – so the parties agreed that if either party breached the agreement the measure of
damages would be $200,000. On June 1 Klinginsmith failed to deliver the aluminum. Von Damme was able to “cover” (purchase
the aluminum from another seller) at a lower price, so it actually saved money as a result of the breach. Is Von Damme entitled
to recover $200,000 in liquidated damages from Klinginsmith?
 ANSWER: Yes! A liquidated damages clause is valid so long as the amount agreed to is reasonable in light of actual or
anticipated damages. In this case the amount is reasonable in light of what the buyer’s anticipated damages might have
been had the market price increased. At the time that they entered into the agreement both parties wanted to protect
themselves from the risk of market fluctuation. The amount of the liquidated damages in this case is not so
disproportionate to the risk of market fluctuation as to constitute a penalty. Under these circumstances the courts will
uphold a provision for liquidated damages, even though the buyer suffered no loss as a result of the breach.
C. Breaching Buyer’s Right to Restitution
 2. Breaching Buyer’s Right to Restitution
o If the buyer breaches the contract after making partial payment, Section 2-718(2) provides that the buyer is nevertheless
entitled to restitution. The buyer’s right to restitution of the payment is subject to offset for:
 Any liquidated damages owed to the seller;
 If there is no provision for liquidated damages, twenty percent of the purchase price or $500, whichever is smaller;
 Damages owed to the seller under any other provision of Article 2; or
 The value of any benefits received by the buyer as a result of the contract.
 Section 2-718(2) and (3): Breaching Buyer’s Right to Restitution
(2) Where the seller justifiably withholds delivery of goods because of the buyer's breach, the buyer is entitled to restitution of any
amount by which the sum of his payments exceeds
(a) the amount to which the seller is entitled by virtue of terms liquidating the seller's damages in accordance with subsection
(1), or
(b) in the absence of such terms, twenty per cent of the value of the total performance for which the buyer is obligated under
the contract or $500, whichever is smaller.
(3) The buyer's right to restitution under subsection (2) is subject to offset to the extent that the seller establishes
(a) a right to recover damages under the provisions of this Article other than subsection (1), and
(b) the amount or value of any benefits received by the buyer directly or indirectly by reason of the contract.
 Casebook Problem
o The zoo officials for the Minerun (West Virginia) Zoo contracted to buy an elephant from the zoo in White Cliffs, Delaware. The
terms of the deal were that the West Virginia zoo would deliver a black bear worth $300 as a down payment and pay $100 a
month for 20 months, at the end of which time the Delaware zoo would deliver the elephant. The bear was tendered and
accepted. The West Virginia zoo duly made its $100 payments for 15 months before it ran out of money and could pay no more.
The West Virginia zoo comes to you. Can it recover the $1,500 it has paid? The bear? Look at §§2-718(2), 2-718(3), and 2-718(4).
Assuming the bear was and is still worth $300, calculate the amount that the West Virginia zoo is likely to recover in a restitution
action.

132
 West Virginia zoo wanted to purchase an elephant from a Delaware zoo for $2,300. It sent the Delaware zoo a bear worth
$300 and made 15 installment payments (out of 20) of $100 each, then it ran out of money. How much may the breaching
buyer recover from the Delaware zoo?
 Section 2-718(2) applies. Buyer paid $1,800. There was no provision for liquidated damages, and buyer did not receive any
benefits under the contract, so buyer is liable for an offset in the amount of 20% of purchase price ($2,300) or $500,
whichever is less. 20% of $2,300 is $460, so buyer may recover $1,800 less $460, or $1,340.
 Note: Maximum penalty of $500
D. Buyer’s Right to Specific Performance
 3. “Buyer’s Right to Specific Performance”
o The title of Section 2-716 is “Buyer’s Right of Specific Performance or Replevin.” Under Section 1-107, “Section captions are part
of the Uniform Commercial Code.” Accordingly, only buyers have the right of specific performance.
o Under Section 2-716(1) a seller may be required to perform the contract (that is, deliver the goods) when the goods are
“unique” or in “other proper circumstances.” This is called “specific performance.”
 Section 2-716(1) and (2):
(1) Specific performance may be decreed where the goods are unique or in other proper circumstances.
(2) The decree for specific performance may include such terms and conditions as to payment of the price, damages, or other relief
as the court may deem just.
 “Unique” Goods and “Other Proper Circumstances”
o Comment 2 to Section 2-716 observes that traditionally specific performance was granted in cases involving “unique” goods
such as heirlooms or priceless works of art.
o Section 2-716(1) expands the common law and extends the buyer’s right of specific performance to particularly reliable sources
of supply such as output or requirements contracts or in other situations where the buyer is unable to “cover,” that is, where
the buyer cannot obtain the goods from another source.
 Comment 2 to Section 2-716
In view of this Article's emphasis on the commercial feasibility of replacement, a new concept of what are “unique” goods is
introduced under this section. Specific performance is no longer limited to goods which are already specific or ascertained at the
time of contracting. The test of uniqueness under this section must be made in terms of the total situation which characterizes the
contract. Output and requirements contracts involving a particular or peculiarly available source or market present today the typical
commercial specific performance situation, as contrasted with contracts for the sale of heirlooms or priceless works of art which
were usually involved in the older cases. However, uniqueness is not the sole basis of the remedy under this section for the relief
may also be granted “in other proper circumstances” and inability to cover is strong evidence of “other proper circumstances”.
 Example: Specific Performance for Granite
o Escobedo’s Construction is building a new office tower for Atlas Holding Company, and the contract calls for granite facing on
the exterior of the building. Escobedo entered into a contract with Jim’s Quarry to supply the granite that would be cut into
panels and attached to the exterior walls of the building. When the building was half finished Jim contacted Escobedo and
stated, “I’ve found a truly desperate buyer who will pay me double what you are paying for the granite – way more than market
value. I will pay you all the money damages you are entitled to plus a premium, but I am cancelling our contract as of today. “If
Escobedo tries to use granite from another source the color and the pattern won’t match what has already been installed on
the building. What is your advice to Escobedo?
 ANSWER: “Escabedo, you are entitled under the law to a court order requiring Jim’s Quarry to deliver the granite at the
contract price.”
 This is a relatively clear example of goods that are “unique.” Under Section 2-716(1) Escobedo is entitled to specific
performance of the contract. Jim’s Quarry will be required to honor the contract and deliver the granite to Escobedo.
E. Buyer’s Right to Replevin
 4. Buyer’s Right of Replevin of Identified Goods
o Under Section 2-716(3) if the seller breaches and the buyer is unable to “cover” the buyer has the right to recover identified
goods from the seller even though the goods are not “unique.” This is called the right of “replevin.”
 Section 2-716(3): Buyer’s Right of Replevin of Identified Goods
(3) The buyer has a right of replevin for goods identified to the contract if after reasonable effort he is unable to effect cover for
such goods or the circumstances reasonably indicate that such effort will be unavailing or if the goods have been shipped under
reservation and satisfaction of the security interest in them has been made or tendered. In the case of goods bought for
personal, family, or household purposes, the buyer's right of replevin vests upon acquisition of a special property, even if the
seller had not then repudiated or failed to deliver.
 Example: Buyer’s Right of Replevin – Ink Cartridges
o Saul, the office manager for the law firm of Saul, Saul, and Saul, ordered 160 ink cartridges (Model HP 1601) from Office Frenzy
on July 1, to be delivered August 1. On August 1 Office Frenzy had 400 of the cartridges in stock, but decided to keep them for
resale to one of its other regular customers, and it notified Saul that it would not be honoring the contract.Saul contacted office
supply stores and even the manufacturer and it cannot find any HP 1601 cartridges that will be available for at least 6 months. Is
Saul entitled to replevin of the cartridges from Office Frenzy under Section 2-716(3)?

133
 ANSWER: No. These goods were not “identified to the contract,” which is a requirement for the right of replevin under
Section 2-716(3). However, Saul might be entitled to “specific performance” under Section 2-716(1), if Saul can convince a
court that this situation qualifies as “other special circumstances” where specific performance is warranted.
F. Buyer’s Right to Recover the Goods
 5. Buyer’s Right to Recover Identified Goods If Payment Has Already Been Made
o Under Section 2-501 the buyer obtains a “special property” in goods when those goods are identified to the contract.
o Under Section 2-502 if the buyer has paid all of the purchase price, then in certain situations the buyer has the right to recover
identified goods from the seller, and if the buyer has paid part of the purchase price the buyer may recover the identified goods
by tendering the rest of the purchase price to the seller.
o In what situations does Section 2-502 apply? It applies if the goods are purchased for personal, family, or household purposes
and the seller fails or refuses to deliver, and in all cases if the seller becomes insolvent within ten days after receipt of the first
installment of the price.
 Section 2-501(1):
(1) The buyer obtains a special property and an insurable interest in goods by identification of existing goods as goods to which
the contract refers even though the goods so identified are non-conforming and he has an option to return or reject them. Such
identification can be made at any time and in any manner explicitly agreed to by the parties. In the absence of explicit
agreement identification occurs
(a) when the contract is made if it is for the sale of goods already existing and identified;
(b) if the contract is for the sale of future goods other than those described in paragraph (c), when goods are shipped, marked
or otherwise designated by the seller as goods to which the contract refers;
(c) when the crops are planted or otherwise become growing crops, or the young are conceived if the contract is for the sale of
unborn young to be born within twelve months after contracting or for the sale of crops to be harvested within twelve
months or the next normal harvest season after contracting whichever is longer.
 Section 2-502:
(1) Subject to subsections (2) and (3) and even though the goods have not been shipped a buyer who has paid a part or all of the
price of goods in which he has a special property under the provisions of the immediately preceding section may on making
and keeping good a tender of any unpaid portion of their price recover them from the seller if:
(a) in the case of goods bought for personal, family, or household purposes, the seller repudiates or fails to deliver as
required by the contract; or
(b) in all cases, the seller becomes insolvent within ten days after receipt of the first installment on their price.
(2) The buyer's right to recover the goods under subsection (1)(a) vests upon acquisition of a special property, even if the seller had
not then repudiated or failed to deliver.
(3) If the identification creating his special property has been made by the buyer he acquires the right to recover the goods only if
they conform to the contract for sale.
 Example: Buyer’s Right to Recover the Goods – Golf Clubs
o Freya ordered a set of golf clubs from Dixie Links, a profitable golf supply distributor. The clubs cost $3,200, a price that Freya
considered to be a bargain. The same clubs would cost twice that amount from other suppliers. Freya is a professional golfer
and she intended to use the clubs in tournaments. She paid the entire purchase price in advance and awaited the clubs. Dixie
Links then sends her an email stating that although the clubs were packed and labeled for shipment to her it will not be able to
ship the clubs. Dixie offered to either fully refund her money or give her a “store credit. “Is Freya entitled to recover the golf
clubs from Dixie under Section 2-502?
 ANSWER: No, Freya is not entitled to recover the clubs from Dixie under Section 2-502(1)(a) or (b). The golf clubs were
identified to the contract, but they were not bought for personal, family, or household use, nor did Dixie Links become
insolvent after Freya prepaid the contract price.
 Furthermore, under Section 2-716 Freya is not entitled to specific performance unless the goods are “unique” and she is not
entitled to replevin unless she is unable to “cover.” In this case the golf clubs are not unique, and Freya could purchase the
goods from another seller. Accordingly, Freya will have to settle for money damages – restitution (the return of her
purchase price), and the difference between the cover price and the contract price, if she purchases the clubs from another
seller.
G. Seller’s Right of Reclamation
 6. Seller’s Right of Reclamation
o Under Section 2-702 a seller has the right to reclaim goods that were delivered to the buyer on credit if the seller discovers after
delivery that the buyer was insolvent when it received the goods.
The seller must assert its right of reclamation within ten days after the buyer’s receipt of the goods, unless the buyer misrepresented
its solvency within three months before the delivery.
The seller’s right to reclaim the goods under Section 2-702 is subject to the rights of a buyer in the ordinary course or other good
faith purchaser.
 Section 2-702: Seller’s Right of Reclamation

134
(1) Where the seller discovers the buyer to be insolvent, he may refuse delivery except for cash including payment for all goods
theretofore delivered under the contract and stop delivery under this Article (Section 2-705).
(2) Where the seller discovers that the buyer has received goods on credit while insolvent, he may reclaim the goods upon
demand made within ten days after the receipt, but if misrepresentation of solvency has been made to the particular seller in
writing within three months before delivery the ten-day limitation does not apply. Except as provided in this subsection the
seller may not base a right to reclaim goods on the buyer's fraudulent or innocent misrepresentation of solvency or of intent to
pay.
(3) The seller's right to reclaim under subsection (2) is subject to the rights of a buyer in ordinary course or other good faith
purchaser under this Article (Section 2-403). Successful reclamation of goods excludes all other remedies with respect to them.
 Example: Seller’s Right of Reclamation – The Miracle Drug
o Anthrogen, a drug distributor, sent a shipment of the incredibly expensive drug Naglazyme to the New York City Metropolitan
Area Health Network, Inc., on credit. The contract price for this drug was $495,747. The amount was enough to take care of one
patient for one year for the treatment of Maroteaux-Lamy Syndrome. Unfortunately, the health network became insolvent as
soon as it received delivery of the drug. The network did not have enough money to pay for the drug, and the patient, Karl
Krumpf, who is uninsured, could not pay for it either.
o Is Anthrogen entitled to reclaim the drug from the network?
 ANSWER: Yes, under these facts Anthrogen is entitled to recover the Naglazyme from the health network. The only way
that Anthrogen could be blocked would be if Karl were a buyer in the ordinary course or a good faith purchaser.
o If Anthrogen demands the return of the medicine within ten days, is Anthrogen entitled to reclaim the medicine under
Section 2-702?
A. Anthrogen is not entitled to reclaim the medicine under Section 2-702 because it should have demanded cash on delivery
from the health network.
B. Anthrogen satisfied two factors of Section 2-702 in that the health network was insolvent when it received the goods on
credit, and Anthrogen demanded return of the goods within ten days of the health network's receipt of the goods.
C. Anthrogen is not entitled to reclaim the medicine under Section 2-702 because this is a life-saving drug and reclamation
would be against public policy.
D. If Karl Krumpf is a buyer in the ordinary course under Section 1-201(9) or a good faith purchaser under Section 2-403, then
Anthrogen is not entitled to reclaim the goods from the health network.
 Discussion
• Under 1-201(9) whether Karl is a buyer in the ordinary course does not depend on whether he already paid for it,
because a BIOC is someone who may have purchased the item on unsecured credit!
• Rather, a BIOC is someone who either has possession or has a right to possession of the item as against its seller. For
that, turn to Section 2-502 on a buyer’s right to recover the goods, and Section 2-716(3) on replevin.
• Under §2-502 the buyer must have paid all or a part of the price – under the facts, Karl paid nothing, but Medicaid may
have paid something.
• Section 2-716(3) seems a more likely scenario, this is a consumer good, and if Karl is unable to effect cover this section
may apply.
• I wonder if Karl has a right to specific performance under Section 2-716(1) because the goods are unique or in other
proper circumstances.
• This may also be a sale to a good faith purchaser under §2-403(2).
• The drug company “entrusted” the goods to a merchant that deals in goods of that kind (the hospital, which regularly
sells drugs).
• But did Karl give value within the meaning of Section 1-204?
• Did Karl give consideration?
• Maybe, if the hospital was going to be reimbursed by Medicaid …?
• Krumpf’s basic problem – was there a “sale” of the goods to him? Was there an exchange of the drugs for a price? If he
has Medicaid, then it was a sale. If not then it was just a gift, and he is not a BIOC nor is he a good faith purchaser.
 ANSWER: B and D are correct. Anthrogen has satisfied two of the elements of a seller's claim for reclamation under Section
2-702, in that the health network was insolvent when it received the goods on credit and Anthrogen demanded return of
the goods within ten days of the buyer's receipt of the goods.
 If the patient Karl Krumpf does not qualify as a buyer in the ordinary course or a good faith purchaser, then Anthrogen is
entitled to return of the goods.
 Canvas Quiz Questions
o The purpose of the remedies that are available under the U.C.C. is to punish people who fail to keep the promises they have
made in contracts.
 False - The purpose of the remedial provisions of the U.C.C. is not punishment but rather to award the non-breaching party
the "benefit of the bargain" -- to put that party in as good a position as if the other party had fully performed.
o The amount of liquidated damages that is fixed in a contract must be reasonable in light of:
 the anticipated or actual harm.

135
 the difficulty of proving loss.
 the inconvenience or nonfeasibility of obtaining an adequate remedy.
 the gap filler provision of Article 2 establishing liquidated damages.
o Wilbur, in an impulsive moment, purchased a large silk painting of a kitty with sad eyes from Touching Moments Arts. The price
was $1,000. Wilbur paid half of the money ($500) down and was due to pay the balance when the painting was to be delivered
the following week. However, after receiving vigorous advice from his wife, (whom he had failed to consult before ordering the
painting) Wilbur decided to cancel the order and try to get his down payment back. What additional information do you need
in order to calculate the amount of restitution Wilber is entitled to?
 Is there a liquidated damages clause, and how much is it?
 Is Wilbur liable to Touching Moments for any damages under Article 2 -- for example, was Touching Moments able to resell
the painting, and if so, for how much?
 Did Wilbur receive any benefits, directly or indirectly, by reason of the contract?
 Is Wilbur entitled to specific performance of this contract in light of the fact that the painting is unique?
o Freya ordered a set of golf clubs from Dixie Links, a profitable golf supply distributor. The clubs cost $3,200, a price that Freya
considered to be a bargain. The same clubs would cost twice that amount from other suppliers. Freya is a professional golfer
and she intended to use the clubs in tournaments. She paid the entire purchase price in advance and awaited the clubs. Dixie
Links then sends her an email stating that although the clubs were packed and labeled for shipment to her it will not be able to
ship the clubs. Dixie offered to either fully refund her money or give her a “store credit.” Which of the following statements
about the facts and the law are correct?
 The golf clubs are "consumer goods" within the meaning of the Uniform Commercial Code.
 The golf clubs were identified to the contract.
 The golf clubs are unique.
 Freya is entitled to the return of her purchase price.
 Under Section 2-716(1) Freya is entitled to specific performance -- that is, Dixie must send her the clubs.
 Freya is entitled to replevin of the golf clubs under Section 2-716(3).
 Freya is entitled to recover the clubs under Section 2-502.
 If Freya purchases the clubs at a higher price from another supplier, Freya will be entitled to recover the difference between
the price she paid the other supplier and the original contract price with Dixie.
o Anthrogen, a drug distributor, sent a shipment of the incredibly expensive drug Naglazyme to the New York City
Metropolitan Area Health Network, Inc., on credit. The contract price for this drug was $495,747. The amount was enough to
take care of one patient for one year for the treatment of Maroteaux-Lamy Syndrome. Unfortunately, the health network
became insolvent as soon as it received delivery of the drug. The network did not have enough money to pay for the drug,
and the patient, Karl Krumpf, who is uninsured, could not pay for it either. Anthrogen is entitled to reclaim the drug from the
network.
 True – Probably
II. Seller’s Remedies
 Seller’s Remedies
o The 700 Club. §§703-710
o Distinguish situations of accepted goods and unaccepted goods
o A seller may recover incidental damages in all cases where the buyer breached the contract.
o Incidental damages include all commercially reasonable expenses incurred with stopping shipment, storage, and resale of the
goods. A party is not entitled to recover attorney fees incurred in negotiations or litigation with the opposing party.
 Seller’s Remedies on Buyer’s Breach
o There are four measures of expectancy damages the seller may be entitled to as a result of the buyer’s breach:
o the entire contract price,
o the contract price minus the resale price,
o the contract price minus market value, or
o the contract price minus the cost of the goods (lost profit on the sale).
 Seller’s Rights When a Buyer Breaches the Contract
o Under Section 2-703 if the buyer breaches the contract, the seller may:
 Withhold delivery;
 Stop delivery;
 Identify goods to the contract;
 Resell and recover damages for the difference between the contract price and the resale price;
 Recover damages for non-acceptance or in a proper case for the price of the goods; or
 Cancel the contract
 Section 2-703: Seller’s Remedies on Buyer’s Breach

136
Where the buyer wrongfully rejects or revokes acceptance of goods or fails to make a payment due on or before delivery or
repudiates with respect to a part or the whole, then with respect to any goods directly affected and, if the breach is of the whole
contract (Section 2-612), then also with respect to the whole undelivered balance, the aggrieved seller may
(a) withhold delivery of such goods;
(b) stop delivery by any bailee as hereafter provided (Section 2-705);
(c) proceed under the next section respecting goods still unidentified to the contract;
(d) resell and recover damages as hereafter provided (Section 2-706);
(e) recover damages for non-acceptance (Section 2-708) or in a proper case the price (Section 2-709);
(f) cancel.
 List of Seller’s Remedies for Damages
A seller may recover the following types of money damages after the buyer has breached the contract:
1. Incidental damages – 2-710
 Commercially reasonable shipping, storage, insurance and resale costs … but not attorney fees
2. Action for the price – 2-709
 Buyer accepted the goods
 Goods destroyed after risk of loss passed to buyer
 Impossible to resell the goods
3. Contract price minus resale price – 2-706
 Seller resold goods
4. Contract price minus market value – 2-708(1)
 Seller did not resell goods*
5. Contract price minus cost of goods – “lost profits” on resale – 2-708(2)
 “Lost volume” situations
A. Accepted Goods
 1. Incidental Damages
o Under Section 2-710, when the buyer breaches the contract, the seller may in all cases recover expenses incurred as a result of
the breach, to the extent those expenses are commercially reasonable. Typically, these are expenses related to stopping
shipment, storage, and resale of the goods.
o However, in the United States normally each party pays its own attorney fees in lawsuits dealing with torts, contracts, or the law
of property. Attorney fees are awarded only under certain civil rights statutes or consumer protection statutes. This same rule
applies to the sale of goods. Each party is responsible for its own attorney fees that are incurred in resolving disputes with the
other party.
 Section 2-710: Incidental Damages
Incidental damages to an aggrieved seller include any commercially reasonable charges, expenses or commissions incurred in
stopping delivery, in the transportation, care and custody of goods after the buyer's breach, in connection with return or resale of
the goods or otherwise resulting from the breach.
 Example: Incidental Damages – The Press
o Whitney Incorporated of Kansas sold a four-ton press for $35,000 to Xenia Machine Shop, which was located in Oklahoma.
While the press was en route to Xenia, Xenia cancelled the order. Whitney called the trucking company and had them turn
around and return the press to their warehouse in Kansas, at an extra cost of $970. Whitney’s workers had to work extra hours
to unload the press and prep it for resale, costing Whitney an estimated $620. Whitney’s account manager spent an extra $830
in time and expenses to resell the press. And their attorney spent time writing the new contract for resale ($200) and in haggling
with Xenia over how much Xenia owed Whitney for breach of contract ($400). Assuming that all of these amounts were
commercially reasonable, how much of these expenses are recoverable as damages incidental to the breach?
 ANSWER: All of Whitney’s damages are incidental to the breach except the last one – the $400 in attorney fees spent in
negotiations with Xenia. Whitney is entitled to recover all of the other costs it incurred in stopping shipment, storage, and
resale (including the $200 in attorney fees it incurred in preparing the contract for resale).
 2. Action for the Price
o Under Section 2-709, the seller is entitled to recover the entire purchase price from the buyer in three circumstances:
 When the buyer has accepted the goods;
 When the goods were destroyed after risk of loss had passed to the buyer; or
 When the seller is unable to resell the goods despite reasonable effort.
 Section 2-709(1): Action for the Price
(1) When the buyer fails to pay the price as it becomes due the seller may recover, together with any incidental damages under the
next section, the price
(a) of goods accepted or of conforming goods lost or damaged within a commercially reasonable time after risk of their loss has
passed to the buyer; and
(b) of goods identified to the contract if the seller is unable after reasonable effort to resell them at a reasonable price or the
circumstances reasonably indicate that such effort will be unavailing.

137
 If the Seller Recovers the Entire Purchase Price, the Buyer Is Entitled to the Goods
Under Section 2-709(2), if the seller recovers the purchase price from the buyer and if the goods are still in existence then the buyer
is entitled to the goods. If the seller resells the goods, the buyer must be credited with the proceeds.
 Section 2-709(2): If the Seller Recovers the Entire Purchase Price, the Buyer Is Entitled to the Goods
(2) Where the seller sues for the price, he must hold for the buyer any goods which have been identified to the contract and are still
in his control except that if resale becomes possible, he may resell them at any time prior to the collection of the judgment. The
net proceeds of any such resale must be credited to the buyer and payment of the judgment entitles him to any goods not
resold.
 Casebook Problem
o Backslappers Auto Sales sold a new blue sports car to Dwane Diletante on credit. He accepted the car and drove it for a month.
He then sent Backslappers a notice of revocation of acceptance and gave as his reason the recent repainting of his garage in a
color that clashed with the blue car. The notice stated that Diletante had parked the car down the block from his home (away
from the clashing garage) and that Backslappers should come and get it. Dwane also refused to make any more car payments.
Three days after Backslappers received the notice, the car disappeared and has never been found. May the seller recover the
price under §2-709? Who had the risk of loss? See §§2-608(1) and (3), 2-606(1), 2-510; note Official Comment 5 to §2-709.
There is a good discussion of this problem in White & Summers §8-3. Would it make a difference if Diletante had rejected the
goods for the same reason? Would a seller be entitled to recover the price if the market price for the goods had declined
considerably after the delivery date? See Brewer v. J-Six Farms.
 The “normal” remedy is an action for the price, 2-709.
 Dilettante, the buyer, revoked acceptance of his car after a month because the color clashed with his garage, left it parked
on the street where it was stolen. This was a wrongful revocation. The risk of loss had passed to the buyer upon receipt of
the car. (This was risk of loss/no breach, governed by 2-509, not 2-510.) Dilettante is liable for the price under 2-709 –
accepted goods, risk of loss on the buyer.
 If Diletante had wrongfully rejected the car and had returned the car to the seller then the seller’s remedy would not be for
the contract price but for something less – the loss due to substitute performance, for example, or lost profits.
 Diletante has revoked for an impermissible reason—there is no warranty as to this so there is no non-conformity at all. He
has also breached his §2-602 duty to take reasonable care of the goods.
 Section 2-510(2) is not relevant since the buyer did not “rightfully” revoke. Instead §2-509 is the key section; Diletante got
the risk of loss on his receipt of the car; §2-509(3).
 Seller may use §2-709(1)(a) to keep the risk of loss on the buyer since he has made an “acceptance” of the goods.
 If wrongful “rejection” occurs there has been no “acceptance” but the key to the passage of the risk under §2-509(3) is
“receipt.” After receipt, Diletante gets the risk of loss, which in spite of his “rejection” remains on him for a “commercially
reasonable time” per §2-709(1)(a) (a question of fact).
B. Unaccepted Goods
 Unaccepted Goods
o If the seller has the goods because they were not delivered or they were returned by the buyer, then the seller may resell and
recover under §2-706 or simply hold the goods and recover under §2-708(1), market value remedy. Or the seller may recover
for its “lost profits” under §2-708(2).
 3. Contract Price Minus Resale Price
o Under Section 2-706, when the buyer breaches and the seller resell the goods, the seller is entitled to recover from the buyer
the difference between the contract price and the resale price.
 Section 2-706(1): Contract Price Minus Resale Price
(1) Under the conditions stated in Section 2-703 on seller's remedies, the seller may resell the goods concerned or the undelivered
balance thereof. Where the resale is made in good faith and in a commercially reasonable manner the seller may recover the
difference between the resale price and the contract price together with any incidental damages allowed under the provisions
of this Article (Section 2-710), but less expenses saved in consequence of the buyer's breach.
 The Resale Must Be “Commercially Reasonable”
o Section 2-706(2)-(6) prescribes rules for the seller’s resale of goods after a buyer has breached. It provides that “every aspect of
the sale including the method, manner, time, place and terms must be commercially reasonable.”
o If the seller uses a private sale the seller must give the buyer reasonable notice of his intention to resell. If the seller uses a
public sale the seller may buy the goods.
o The seller is not accountable to the buyer for any profit made on any resale.
 Section 2-706(2)-(6): Pules for the Seller’s Resale of Goods
(2) Except as otherwise provided in subsection (3) or unless otherwise agreed resale may be at public or private sale including sale
by way of one or more contracts to sell or of identification to an existing contract of the seller. Sale may be as a unit or in parcels
and at any time and place and on any terms but every aspect of the sale including the method, manner, time, place and terms
must be commercially reasonable. The resale must be reasonably identified as referring to the broken contract, but it is not
necessary that the goods be in existence or that any or all of them have been identified to the contract before the breach.
(3) Where the resale is at private sale the seller must give the buyer reasonable notification of his intention to resell.

138
(4) Where the resale is at public sale
(a) only identified goods can be sold except where there is a recognized market for a public sale of futures in goods of the kind;
and
(b) it must be made at a usual place or market for public sale if one is reasonably available and except in the case of goods
which are perishable or threaten to decline in value speedily the seller must give the buyer reasonable notice of the time
and place of the resale; and
(c) if the goods are not to be within the view of those attending the sale the notification of sale must state the place where the
goods are located and provide for their reasonable inspection by prospective bidders; and
(d) the seller may buy.
(5) A purchaser who buys in good faith at a resale takes the goods free of any rights of the original buyer even though the seller fails
to comply with one or more of the requirements of this section.
(6) The seller is not accountable to the buyer for any profit made on any resale. A person in the position of a seller (Section 2-707)
or a buyer who has rightfully rejected or justifiably revoked acceptance must account for any excess over the amount of his
security interest, as hereinafter defined (subsection (3) of Section 2-711).
 4. Contract Price Minus Market Value
o Under Section 2-708(1), if the seller chooses not to resell the goods, the seller is entitled to recover from the buyer the
difference between the contract price and the market value of the goods at the time and place of tender.
 Section 2-708(1): Contract Price Minus Market Value
(1) Subject to subsection (2) and to the provisions of this Article with respect to proof of market price (Section 2-723), the measure
of damages for non-acceptance or repudiation by the buyer is the difference between the market price at the time and place for
tender and the unpaid contract price together with any incidental damages provided in this Article (Section 2-710), but less
expenses saved in consequence of the buyer's breach.
 May a Seller Sue for the Market Price Remedy If It Has Resold the Goods?
o Some courts have held that if a buyer breaches and the seller resells the goods, the seller has a choice; the seller may sue for
either the difference between the contract price and the resale price or may sue for the difference between the contract price
and the market price.
 5. Contract Price Minus Cost of Goods: “Lost Profits”
o Under Section 2-708(2), if the seller is a “lost volume” seller, then the seller may recover the profit that it expected to make on
the sale; that is, the difference between the contract price and the cost of the goods to the seller.
o A “lost volume” seller is a party who can prove that it had more supply of its goods than there was demand for them – that the
buyer’s breach did not “free up” goods for resale but was simply a “lost sale.”
 Section 2-708(2): “Lost Profits”
(2) If the measure of damages provided in subsection (1) is inadequate to put the seller in as good a position as performance would
have done then the measure of damages is the profit (including reasonable overhead) which the seller would have made from
full performance by the buyer, together with any incidental damages provided in this Article (Section 2-710), due allowance for
costs reasonably incurred and due credit for payments or proceeds of resale.
 Example: Studebaker Wagonaire
o One of the worst selling cars of all time was the Studebaker Wagonaire. In 1966 only 618 of these station wagons were sold.
Please assume that Janine signed a contract to purchase a Wagonaire in 1966 from Stud’s Studes (a Studebaker dealership) for
$2,000, but the next day she thought better of it and backed out of the contract. Please assume further that Stud paid the
manufacturer $1,750 for the car. Finally, please assume that Stud had three Wagonaires on the lot when Janine backed out of
the contract and that after she breached Stud was able to sell only one of them – for $2000. Janine contends that Stud is
entitled to recover the contract price ($2,000) minus the resale price, which she says is $2,000 – and therefore Stud’s damages
are zero. Which of the following remedies is Stud entitled to?
A. Under Section §2-709 Stud is entitled to recover the entire price from Janine -- $2,000.
B. Under Section §2-706(1), Stud is entitled to recover the contract price ($2,000) minus the resale price ($2,000) of the car. In
other words, Stud recovers nothing.
C. Under Section §2-708(1), Stud is entitled to recover the contract price ($2,000) minus the market value ($1,800) of the
car, equal to $200.
D. Under Section §2-708(2), Stud is entitled to recover his "lost profits" on the sale of car: the contract price ($2,000) minus
the cost of the car ($1,750), a total of $250.
 The action for the price (§2-709) is appropriate if it’s true that the car really cannot be resold … if there is no market for the
car, then it makes no sense to speak about the “market price” or “cover price” or even “lost profits” – the dealer lost $2,000
when the car was not sold! Of course, the dealer would have to make the car available to Janine if she is required to pay for
it.
 Another way of looking at it is if there is a market for the car, then Stud’s remedy should be $200 – the difference between
the market price and the contract price.
 If the car would eventually sell for $2,000 but Stud could have obtained as many cars as he wanted for $1,750, then Stud
should recover $250 – his lost profit on the sale to Janine that did not occur.

139
 Stud didn’t really “resell” the car that Janine contracted for; he would have made that sale anyway. “Resale” is the only
remedy that I think really does not apply.
 Answer: Under Section §2-708(2), Stud is entitled to recover his "lost profits" on the sale of car: the contract price ($2,000)
minus the cost of the car ($1,750), a total of $250.
 Casebook Problem
o Lannie Light was the sole proprietor of Light’s Bulbs, a lighting fixtures business in Austin, Texas. She contracted to sell 80 neon
light fixtures to Signs, Inc., a firm in San Antonio. The price was $1,500 “F.O.B. Austin,” and the shipment date was to be March
15. On March 5, Signs, Inc., phoned Light and told her that the deal was off, but Lannie refused to agree to a cancellation. She
went to her warehouse and picked 80 of the fixtures from her large stock. Then she posted a notice on the bulletin board near
the cash register in her store, stating that 80 of the fixtures would be sold to the person making the best offer. Carl Customer
(who was always buying these types of items) saw the sign and offered Light $1,000 for the fixtures. Light sold Customer the
goods and took payment. Now Light comes to you. She tells you that on March 5 the fixtures were selling on the open market at
$800 for 80 and that on March 15 the price for 80 such fixtures were $900 in Austin and $800 in San Antonio. Answer these
questions:
(a) Does the UCC permit Light to select goods from the warehouse after the buyer repudiates? See §2-704.
 Yes, 2-704.
(b) Was the resale proper? See §2-706 and Official Comment 2.
 No. She gave no §2-706 notice to the buyer, did no advertising, and did not conduct an auction. The penalty may be that
Lannie loses all right to recovery!
(c) Suppose Signs, Inc. had no cash for the purchase price, but offered to take the light fixtures on credit, even offering to provide
a motorcycle as collateral. Could Light sell the light fixtures to someone else at a lower price and recover damages? Star
Funding, Inc. v. Vault Minerals, LLC.
 In the cited case the court held that an offer to give collateral is irrelevant to the issue of whether the resale was proper.
The proper question is whether the resale was done in a proper manner, which has nothing to do with the offer of
collateral.
(d) If Light’s damages are measured under §2-708(1), what amount may she collect? What amount under §2-706?
 Under §2-708(1) (Repudiation), she recovers for the difference between contract price and market price at time and place
of tender, which was March 15, FOB Austin. Thus, the damages are $600.
 Under §2-706 (Resale), she would recover $500, the difference between the contract price and the resale price.
 HOWEVER – If Lannie is a “lost volume seller” she could assert a claim for the profit that she lost on the sale under §2-
708(2). Contract price less cost of goods sold (including reasonable overhead).
• Note: a “lost volume seller” is a seller who sells to a buyer after a previous buyer has breached a contract for sale but
who would have been able to make a sale to the second buyer even if the first buyer had not breached.
(e) Does Light have the choice between the §2-706 (Resale) computation and the §2-708 (Repudiation) computation? See Official
Comment 1 to §2-703; White & Summers §8-7.
 The seller violating §2-706 is limited to a resale remedy, here apparently $500 plus the costs of resale (and buyer may
argue that a more commercially reasonable resale would have brought more, so that seller loses all right to a deficiency.
 Article 9 foreclosure sales §9-626 presumes that a proper sale would have brought the full amount due unless the seller
proves otherwise.
 Seller may counter that this is really a “lost volume” problem and §2-708(2) is the only relevant damages section.
 Peace River Seed Co-Operative, Ltd. v. Proseeds Marketing, Inc.
o Here, the Supreme Court of Oregon comes out firmly in favor of allowing a seller who resold goods the buyer refused to take to
use the market price-contract price differential measure of damages in §2-708 even if it will give the seller more than the resale
amount (§2-706).
o As the court discusses, the commentators have disagreed on this issue, but a plain reading of the statutory language allows
sellers an election of remedies (see §2-703) and gives seller the alternate possibilities.
o The court notes that both parties understood that the market price differential was the standard, so there is nothing unfair in
applying it. Moreover, where there is a fixed price contract, as here, each party assumes the risk, up or down, of fluctuations,
and limiting the seller to the resale price relieves the buyer of this risk when it goes in buyer’s favor.
 Casebook Problem
o Fun in the Sun, Inc., sells swimming pools. Its president comes to your law office with this problem. A customer named Esther
Swimmer ordered one of the standard above-ground pools, retailing for $2,000. The pool’s components are purchased by Fun in
the Sun at a wholesale price of $800 and are assembled into the finished product. The assembly process costs the seller $400.
Swimmer has now repudiated the contract, and Fun in the Sun wants to sue. The current market price is $2,000 for such a pool.
Fun in the Sun is sure it can find another buyer at that price if it resells the pool. Does it have damages? How are they
measured? See §2-708(2), along with Official Comment 2. See also White & Summers §§8-8 to 8-13.
 Buyer repudiated in the purchase of an above ground pool. Seller can simply resell the pool to someone else at the same
price.
 This is a “Lost volume seller.”

140
 Contract price, $2,000. Cost of pool, $800. Cost of installation, $400.
 Recovery should be $800.
 Fun in the Sun has an apparent profit loss of $800, though this figure may go up or down depending on expenses saved or
incurred by the variation in volume.
 Teradyne, Inc. v. Teledyne Industries, Inc.
o Teradyne gives an example of a lost volume seller.
o Teledyne breached its promise to buy a transistor test system. Teradyne sold the system to another buyer at a similar price.
o Teradyne had no damages in the sense of difference between market and resale price. But Teradyne would have sold a system
to the second buyer anyway. Its damages were that it had one less sale and lost the profit and payment for overhead from that
possible sale.
o The case also illustrates a common issue: deciding which costs were variable costs (which seller does not incur and so do not
count as damages) and which are fixed costs (overhead attributable to that sale, for which seller gets damages, because
overhead would be apportioned over the sales made by seller).
o The court below identified the following as variable costs to be deducted: direct labor costs of production, material charges,
sales commission, and expenses. The appeals court added to that the labor costs of testers, shippers, and installers.
o Questions following Teradyne:
1. Exactly who qualifies as a “lost volume seller”? If a college student advertises his guitar for sale in the campus
newspaper, he contracts to sell it to a caller, the caller then backs out, and the student sells the guitar to someone else,
can the student qualify as a lost volume seller?
 The student in the question cannot qualify as a lost volume seller because he does not have an unlimited inventory. A
lost volume seller must be able to satisfy all customers from stock. Obviously, a casual sale of a discrete item will not
require lost volume treatment.
2. Why does the court draw a distinction between overhead costs and variable expenses (which the court calls “direct
costs”)? Which was which here?
 Variable expenses (like the wages of the testers in the case) really are saved by the production of one less item, so they
are not overhead lost.
3. Section 2-708(2) specifically requires a subtraction of the “proceeds of resale.” Doesn’t this language always defeat the
lost volume seller, who will typically have resold the unaccepted goods? See R.E. Davis Chem. Corp. v. Diasonics, Inc.
 The courts considering the issue, including the cited case, all agree that “proceeds of resale” is a drafting error in §2-
708(2), and cannot be meant to apply to a lost volume seller. In Teradyne, the court discusses this issue in Part 1 of the
opinion.
4. For a case requiring the seller to measure damages at no more than the lost profit amount where the market
price/contract price differential under §2-708(1) would overcompensate the seller? see Purina Mills, L.L.C. v. Less.
 The Purina Mills case turns 2-708(2) into a MAXIMUM recovery.
5. The lost volume concept also applies to lessors of goods. See §2A-529’s Official Comment 2 and C.I.C. Corp. v. Ragtime, Inc.
 The lost volume concept also applies to leases. 2A-529.
 Casebook Problem
o Milo Veep, sales agent for the Complex Computer Corporation (CCC), negotiated a contract whereby his company was to design
and manufacture a special computer that would regulate the timing of subway trains for the City of Plantation, Mississippi. The
price was $20,000 F.O.B. CCC’s plant in Atlanta, Georgia. When the computer was half completed, the City of Plantation
underwent a change of administration, and the new city leaders decided to dump the subway renovations. They phoned CCC
and canceled the computer order. Now Veep phones your law office for advice. To help in your decision, Veep states that as
scrap the computer and its components p. 368are now worth $5,000. Veep has heard that three other cities have subway
systems similar to Plantation’s, and if the computer is finished, they might be enticed to buy it at a price between $15,000 and
$20,000. On the other hand, it will cost CCC $9,000 to complete the computer.
(a) Should CCC stop the manufacture of the computer and sell it for scrap or complete manufacture and then try to resell it? See
§2-704(2) and its Official Comment 2.
 Under §2-704, the seller may do either if “commercially reasonable.”
(b) If CCC completes manufacture and then, after a good faith effort, is unable to find a new buyer for the computer, can it make
Plantation pay for the finished product? See §2-709(1)(b); Official Comment 1 to §2-704.
 Yes, under §2-709(1)(b).
 This is an almost purely factual question; CCC must exercise “reasonable commercial judgment” and if it does so may
recover the §2-706 damages, including the incidental damages cost of conducting the resale.
 If no substitute buyer can be found, §2-709(1)(b) certainly permits CCC to recover the price from the city.
 Practice Pointer: The savvy lawyer will ask the client: “If there were no chance of a lawsuit, what would you have done?”
Since this is essentially a business question, not a legal one, the client is in the best position to decide what a reasonable
commercial judgment is.
 Casebook Problem

141
o Stern Oil entered into a ten-year Motor Fuel Supply Agreement with James Brown, which required Brown each year to purchase
a certain amount of ExxonMobil branded fuel from Stern Oil for Brown’s two convenience stores. Brown breached the
agreement and stopped purchasing fuel from Stern. As a result, Stern Oil did not receive profits from a 1.5 cent markup per
gallon or a 1.25 percent discount from ExxonMobil. Brown conceded the 1.5 cent markup per gallon was duly awarded as lost
profits but argued that the lost discount was not recoverable as damages. Should it be? See §§1-305(a), 2-715(2), 1-103, 2-
708(2), Stern Oil Co. v. Brown.
 In the cited case the court noted that sellers cannot get consequential damages under the UCC but may recover “direct
damages.”
 Here, the buyer knew that seller was reselling goods it bought in volume from another seller and that the price to the seller
would be greater if the buyer did not buy the volume contracted for.
 Since §1-305 commands the courts to put the injured party in the position that performance would have, it seemed fairest
to the court to call this a direct damage and not merely consequential, and thus recoverable.
 The case mentioned in the text immediately following the Problem (Nobs Chemical) directly contradicts this result. Which is
right? White and Summers in §8-16 of their treatise criticize Nobs Chemical and conclude that it was a mistake for the
drafters not to allow sellers to recover consequential damages, forcing the courts that see justice in doing so to use covert
means to allow such a recovery.
 Casebook Problem
o Lawyer Portia Moot decided to rent a computer from Machines Unlimited and use it in her office. The computer arrived, and
Portia found it most satisfactory, but her struggling practice made it difficult for her to make the lease payments on time. After
she had missed two payments in a row, Machines Unlimited sent a goon to her office to repossess the computer. Portia was not
there at the time, but her loyal administrative assistant protested mightily when the goon grabbed the machine, at one point
blocking the door with her body, but she was shoved aside, and the computer was taken. The lease still had a year to run, with
payments of $100 due each month. Machines Unlimited sued Portia for $1,200.
(a) Was Machines Unlimited’s repossession valid? See §2A-525. What remedy does Portia have if it was not?
 Self-help repossession under Article 2A, like self-help repossession under Article 9 [§9-609], is allowed only if it can be
accomplished without a breach of the peace, generally defined to include any debtor objection to the repossession (like the
administrative assistant blocking the door).
 The policy here is to avoid pitched battles over goods. The only remedy mentioned in the Official Comment is an injunction,
but the drafters surely would have also given damages. The most popular theory will probably be conversion (as it is in
Article 9), but a court might also treat the wrongful repossession the same as if the seller had never delivered, and measure
damages under the rules for non-delivery. Replevin per §2A-521 should also work.
(b) Assuming there was no problem with the repossession, is the lessor required to try to mitigate damages by re-leasing the
machine? See §2A-529 and its Official Comment.
 As originally written, §2A-529 was the most criticized provision in Article 2A. The original wording apparently permitted the
lessor to repossess, relet the goods, and still sue the lessor for the unpaid rent, past and present. The 1990 rewrite of this
section made it clear that that is not allowed.
 The Official Comment has an extended discussion of what is now permitted, but note the evil language at the beginning:
“Absent a lease contract provision to the contrary”(!!!). This appears to permit a clause reaching the original result, and, if
this is true, will most surely lead to standard boilerplate making the lessee pay the full contracted-for rent even where
repossession occurs. Query whether the courts will declare this unconscionable?
(c) Could the lessor avoid any possible duty to mitigate by so stipulating in the lease agreement? See §§1-302, 2A-503; Homer
Kripke, Some Dissonant Notes About Article 2A.
 In the cited article, Professor Kripke is suspicious that the broad grant of authority to modify remedies granted by §2A-503(1)
would permit the disclaiming of any duty to mitigate. A court might so hold, though resort to §1-302 could persuade a court
that this is the disclaimer of a requirement of reasonableness, which of course is not allowed.
 Canvas Quiz Questions
o Attorney fees incurred by a party in negotiation or litigation with an opposing party are not normally awarded for breach of
contract. However, if reasonable attorney fees are in good faith incurred in connection with the resale of goods following the
other party's breach, then the aggrieved party may recover those fees as damages incidental to the breach.
 True - The costs associated with negotiation with or litigation against the breaching party are not recoverable by the non-
breaching party, but costs that are incurred in good faith and that are reasonable in amount in connection with the resale of
goods are recoverable by the non-breaching party as incidental to the breach.
o One of the worst selling cars of all time was the Studebaker Wagonaire. In 1966 only 618 of these station wagons were sold.
Please assume that Janine signed a contract to purchase a Wagonaire in 1966 from Stud’s Studes (a Studebaker dealership) for
$2,000, but the next day she thought better of it and backed out of the contract. Please assume further that Stud paid the
manufacturer $1,750 for the car. Finally, please assume that Stud had three Wagonaires on the lot when Janine backed out of
the contract and that after she breached Stud was able to sell only one of them – for $2000. Which of the following remedies is
Stud entitled to?
 Under Section 2-709 Stud is entitled to recover the entire price from Janine -- $2,000.

142
 Under Section 2-706(1), Stud is entitled to recover the contract price ($2,000) minus the resale price ($2,000) of the car. In
other words, Stud recovers nothing.
 Under Section 2-708(1), Stud is entitled to recover the contract price ($2,000) minus the market value (?) of the car.
 Under Section 2-708(2), Stud is entitled to recover his "lost profits" on the sale of car: the contract price ($2,000) minus the
cost of the car ($1,750), a total of $250.

Seller’s Expectancy Damages

How Buyer Breached How Seller Reacted Seller’s Expectancy Remedy

Goods were destroyed after risk of loss had shifted Full Performance. Seller is entitled to the entire
to the buyer. contract price. 2-709

Buyer accepted the goods, and the goods are


conforming to the contract. Buyer did not pay for
the goods.

Buyer accepted the goods and failed to give seller Seller was unable to resell the goods
proper notice of any breach. Buyer did not pay for despite a good faith effort.
the goods.
Substitute Performance. Seller is entitled to the
Seller resold the goods in a commercially
Seller may elect contract price minus the resale price. 2-706
reasonable manner.
the remedy
Buyer repudiated the contract and seller retained Market Value. Seller is entitled to the contract price
Seller made no attempt to resell the goods
the goods, or buyer wrongfully rejected the goods, minus the market price at the time and place of
or resold the goods in a commercially
and the goods were returned to the seller. delivery. 2-708(1)
unreasonable manner.

Seller resold the goods to another Lost Profits. Seller is entitled to the contract price
customer but would have made the minus the variable costs of acquiring or producing the
second sale anyway (lost volume seller). goods (lost profits). 2-708(2)

 Answer: Arguments can be made for more than one of these outcomes.
III. Buyer’s Remedies
 Buyer’s Special Remedies
o The 700 Club. §§711 - 717
o In general, if the seller breaches the contract, the buyer may be entitled to restitution, expectancy damages, incidental
damages, and consequential damages.
o If the goods are never delivered to the buyer or if the goods are returned to the seller, the buyer is entitled to the return of any
portion of the purchase price that has been paid. A buyer is also entitled to retain the goods as security for the return of the
purchase price and incidental expenses.
o There are three types of measures for the recovery of expectancy damages: cover price minus contract price, market price
minus contract price, and value of the goods as warranted minus actual value of the goods.
o A previous presentation reviews a number of special remedies that are available to buyers under Article 2:
1. Liquidated damages for either party under Section 2-718(1);
2. Breaching Buyer’s right to restitution under Section 2-718(2);
3. Buyer’s right to specific performance under Section 2-716(1);
4. Buyer’s right to replevin under Section 2-716(3); and
5. Buyer’s right to recover the goods under Section 2-502.
 This presentation is concerned with the buyer’s right to recovery of monetary damages on account of the seller’s breach of
contract.
 Buyer’s Monetary Remedies for Breach Under the Common Law of Contract
Under the common law of contract there are four basic types of money damages that buyers can recover when the seller breaches
the contract: restitution (return of the purchase price); direct expectancy damages (benefit of the bargain); incidental damages
(expenses for receipt, storage, reshipment, and resale); and consequential damages (injury to person or property or to the business of
the buyer).
All four of these types of monetary damages for breach of contract are available to buyers under Article 2.
 The Buyer May Be Entitled to All Four Types of Monetary Damages
In the usual case where the seller breaches and the goods were either never delivered or were nonconforming and were rightfully
returned to the seller the buyer is entitled to restitution, expectancy, and incidental damages. If the seller’s breach proximately caused
consequential injury to person or property or other economic loss, then the buyer is entitled to compensation for those damages as
well.
 Example: The Wrong Textbook
o Using his debit card Fred purchased a new textbook online for $200 (including shipping and handling) from Discount Texts.
When the book arrived, Fred discovered that it was not new as advertised but rather an out-of-date edition of the textbook,
worth nothing to him and worth $20 on the market. He returned the book to the seller at a shipping cost of $15 and demanded
the return of his purchase price. He has received no answer from Discount Texts. It cost Fred $300 to purchase the book from

143
another source. How much is Fred entitled to under each category of monetary damages: restitution, expectancy, incidental,
and consequential?
 ANSWER: Fred is entitled to the following damages:
• Restitution (return of the purchase price): $200
• Expectancy Damages (cover price minus contract price): $100
• Incidental Damages (reshipping): $15
• Consequential Damages: None
 The following slide contains a more detailed list of a buyer’s remedies for damages.
 List of Buyer’s Remedies for Money Damages Under Article 2
o Restitution
1. Recovery of payment from breaching seller when the seller has the goods – Section 2-711(1)
2. Recovery of payment from breaching seller when the buyer has the goods – Section 2-711(3)
o Expectancy Damages
3. Buyer never receives the goods or returns the nonconforming goods and “covers” – Cover price minus contract price –
Section 2-712
4. Buyer never receives the goods or returns the nonconforming goods and does not cover – Market price minus contract
price (buyer’s market damages) – Section 2-713
5. Buyer accepts nonconforming goods and does not revoke acceptance (breach of warranty) – Value had the goods been as
promised minus actual value – Section 2-714
o Incidental Damages
6. In all situations the buyer is entitled to reimbursement for expenses reasonably incurred as a result of the breach for
shipping and receipt, inspection, storage, reshipping, and resale of the goods – Section 2-715(1)
o Consequential Damages
7. Recovery for any loss resulting from requirements that the seller had reason to know and that could not reasonably be
prevented by cover or otherwise – Section 2-715(2)(a)
8. Recovery for injury to person or property proximately resulting from any breach of warranty – Section 2-715 (2)(b)
 1. Buyer’s Right to Restitution of the Purchase Price If Goods Never Delivered or Returned
o If the buyer never received the goods, or if the buyer rightfully rejected the goods and returned the goods to the seller, or if the
buyer rightfully revoked its acceptance and returned the goods to the seller, then under Section 2-711(1) the buyer is entitled to
the return of any portion of the purchase price that it has paid.
 Section 2-711: Buyer’s Right to Restitution of the Purchase Price If Goods Never Delivered or Returned
(1) Where the seller fails to make delivery or repudiates or the buyer rightfully rejects or justifiably revokes acceptance then with
respect to any goods involved, and with respect to the whole if the breach goes to the whole contract (Section 2-612), the buyer
may cancel and whether or not he has done so may in addition to recovering so much of the price as has been paid
(a) “cover” and have damages under the next section as to all the goods affected whether or not they have been identified to
the contract; or
(b) recover damages for non-delivery as provided in this Article (Section 2-713).
 2. Buyer’s Right to Hold and Resell the Goods as Security for Return of the Purchase Price
o Normally under the law governing acceptance and rejection if the buyer exercises ownership over the goods the buyer is
deemed to have accepted the goods and is liable for the purchase price – and if the buyer has paid for the goods the buyer is
not entitled to return of the purchase price.
o However, there is an exception to this rule that is created to protect the right of the buyer to restitution. Under Section 2-
711(3), if the buyer has rightfully rejected goods or rightfully revoked acceptance, but the buyer still has possession of the
goods, the buyer may hold the goods as security for return of the purchase price or any expenses reasonably incurred by the
buyer as a result of the seller’s breach and may resell the goods to compensate for those losses.
 Section 2-711(3): Buyer’s Right to Hold and Resell the Goods as Security for Return of the Purchase Price
(3) On rightful rejection or justifiable revocation of acceptance a buyer has a security interest in goods in his possession or control
for any payments made on their price and any expenses reasonably incurred in their inspection, receipt, transportation, care
and custody and may hold such goods and resell them in like manner as an aggrieved seller (Section 2-706).
 3. Expectancy Damages for Breach If Buyer Covers: Cover Price Minus Contract Price
o Under Section 2-712, if the seller failed to deliver the goods or if the goods have been rightfully rejected or the buyer has
rightfully revoked acceptance the buyer may “cover” – purchase substitute goods – and if the buyer covers it is entitled to
recover the difference between the cost of cover and the contract price. In covering the buyer must act in good faith and
without unreasonable delay, and the cover purchase must be reasonable.
o This measure of damages gives a buyer who has covered the “benefit of the bargain” – the protection against an increase in the
market price that the buyer bargained for in the original contract.
 Section 2-712: Cover Price Minus Contract Price
(1) After a breach within the preceding section the buyer may “cover” by making in good faith and without unreasonable delay any
reasonable purchase of or contract to purchase goods in substitution for those due from the seller.

144
(2) The buyer may recover from the seller as damages the difference between the cost of cover and the contract price together
with any incidental or consequential damages as hereinafter defined (Section 2-715), but less expenses saved in consequence of
the seller's breach.
(3) Failure of the buyer to effect cover within this section does not bar him from any other remedy.
 4. Expectancy Damages for Non-Delivery: Market Price Minus Contract Price
o Under Section 2-713, if the seller fails to deliver the goods or if the buyer rightfully rejected the goods or rightfully revoked
acceptance of the goods, then as expectancy damages the buyer is entitled to recover the difference between the market price
of the goods at the time that the buyer learned of the breach, minus the contract price.
o This measure of damages gives a buyer who has not covered the “benefit of the bargain” – the protection against an increase in
the market price that the buyer bargained for in the original contract.
 Section 2-713: Market Price Minus Contract Price
(1) Subject to the provisions of this Article with respect to proof of market price (Section 2-723), the measure of damages for non-
delivery or repudiation by the seller is the difference between the market price at the time when the buyer learned of the breach
and the contract price together with any incidental and consequential damages provided in this Article (Section 2-715), but less
expenses saved in consequence of the seller's breach.
(2) Market price is to be determined as of the place for tender or, in cases of rejection after arrival or revocation of acceptance, as of
the place of arrival.
 Cover Is an Exclusive Remedy
o The Official Comment to Section 2-713 indicates that “cover” is an exclusive remedy. If the buyer enters the market to purchase
the goods from another source, the buyer is limited to its remedy of the difference between the contract price and the price
that the buyer paid to cover.
 Official Comment 5 to Section 2-713
The present section provides a remedy which is completely alternative to cover under the preceding section and applies only when
and to the extent that the buyer has not covered.
 5. Damages for Breach of Warranty: Market Value as Warranted Minus Actual Market Value
o If the buyer accepts and keeps non-conforming goods, then the buyer is entitled to recover for breach of warranty. Under
Section 2-714 the measure of damages for breach of warranty is the difference between the value that the goods would have
had at the time and place of acceptance if they had been as warranted by the seller, and the actual value of the goods.
o This measure of damages gives the buyer who retains non-conforming goods the “benefit of the bargain” – the value of the
goods that the buyer bargained for.
 Section 2-714: Market Value as Warranted Minus Actual Market Value
(1) Where the buyer has accepted goods and given notification (subsection (3) of Section 2-607) he may recover as damages for any
non-conformity of tender the loss resulting in the ordinary course of events from the seller's breach as determined in any
manner which is reasonable.
(2) The measure of damages for breach of warranty is the difference at the time and place of acceptance between the value of
the goods accepted and the value they would have had if they had been as warranted, unless special circumstances show
proximate damages of a different amount.
(3) In a proper case any incidental and consequential damages under the next section may also be recovered.
 6. Buyer’s Right to Incidental Damages
o Under Section 2-715(1), in all cases the buyer is entitled to recover any expenses reasonably incurred as a result of the seller’s
breach for shipping and receipt, inspection, storage, reshipping, and resale of the goods.
 Section 2-715(1): Buyer’s Right to Incidental Damages
(1) Incidental damages resulting from the seller's breach include expenses reasonably incurred in inspection, receipt, transportation
and care and custody of goods rightfully rejected, any commercially reasonable charges, expenses or commissions in connection
with effecting cover and any other reasonable expense incident to the delay or other breach.
 7. and 8. Buyer’s Right to Consequential Damages
o Section 2-715(2) identifies two remedies for consequential damages to the buyer:
(a) any type of loss that the seller had reason to know would result and that could not reasonably have been prevented; and
(b) injury to person and property, which is recoverable if those damages are proximately caused by the seller’s breach.
 Section 2-715(2): Buyer’s Right to Consequential Damages
(2) Consequential damages resulting from the seller's breach include
(a) any loss resulting from general or particular requirements and needs of which the seller at the time of contracting had
reason to know and which could not reasonably be prevented by cover or otherwise; and
(b) injury to person or property proximately resulting from any breach of warranty.
 Setoff
o Under Section 2-717, if the buyer is entitled to monetary damages on account of the breach of the seller, after notifying the
seller the buyer may apply that amount to reduce any unpaid portion of the purchase price owed to the seller.
 Section 2-717: Setoff

145
The buyer on notifying the seller of his intention to do so may deduct all or any part of the damages resulting from any breach of the
contract from any part of the price still due under the same contract.
 Incidental and Consequential Damages
o Incidental damages are always recoverable.
o Consequential damages are recoverable if the seller’s breach proximately caused injury to person or property or if the seller had
reason to know that its breach would cause a particular type of loss.
o Here is an extra case on buyer’s damages, relating to the concept of recovery of consequential damages. Water Works &
Industrial Supply Company v. Wilburn. It’s a case where the court says that “Consequential damages are recoverable for a
breach of implied warranty of fitness.” I think this is significant because the warranty of fitness (§2-315) requires the seller to
have “reason to know” of the particular purpose of the buyer.
o This is also one of the elements for the recovery of consequential damages under §2-715 and the doctrine of Hadley v.
Baxendale.
 Consequential Damages
o Hadley v. Baxendale (1854) 9 Exch. 345
o “Where two parties have made a contract which one of them has broken, the damages which the other party ought to receive
in respect of such breach of contract should be such as may fairly and reasonably be considered either arising naturally, i.e.,
according to the usual course of things, from such breach of contract itself, or such as may reasonably be supposed to have
been in the contemplation of both parties, at the time they made the contract, as the probable result of the breach of it.” Baron
Sir Edward Hall Alderson.
o Accord Section 351 of the Restatement (Second) of Contracts (foreseeability requirement).
A. Accepted Goods
 Accepted Goods
o If the buyer has accepted the goods and has not rightfully revoked acceptance, then the buyer’s remedy for a nonconformity is
for breach of warranty under §2-714.
o Buyer must give timely notice under §2-607(3)(a).
 Casebook Problem
o The world-famous pianist Bart Cristofori made $50,000 a year giving concerts. Recently he decided to experiment with some
new sounds. He purchased an electric piano for $3,000 from the Silbermann Electronic Music Company. The purchase was
negotiated orally; there was no written contract. Cristofori practiced day and night to master the new instrument. After three
months of arduous practice, he noticed a strange ringing in his ears. Subsequent medical examination revealed that Cristofori
was going deaf. The cause was a high-pitched whine (above the level of human perception) emanating from the electric piano.
On learning that the piano had done this to him, Cristofori took an axe and chopped the piano into unrecognizable bits. (This
action ended his ability to revoke his acceptance; §§2-608(3), 2-602(2)(b).) When he calmed down, he brought suit against the
piano company for breach of warranty. His damages were claimed as $1,755,505, based on the following elements: $3,000 was
the cost of the piano, $2,000 was doctor’s fees, $500 was paid to experts to examine the piano and determine if it was the cause
of the ear problem, $750,000 was lost income for the next 15 years, $1,000,000 was the value of Cristofori’s hearing, and $5 was
for the axe. Silbermann Electronic Music defended by (1) denying that it had warranted the piano in any way, and (2) proving
that the whine was harmless to everybody in the world except Cristofori. (The company proved that the accident occurred to
him only because of the bone structure of his skull coupled with the fact that he had a metal plate installed in his head as a
result of an auto accident in his youth.) Answer these questions:
(a) What warranty, if any, did the Silbermann Company breach?
 The warranty is merchantability, §2-314. The issue will be whether the piano was fit for its ordinary purpose. I think it was,
because he was the only one in the world affected because of the metal plate. Not foreseeable.
 If not, it will not be a defense that the injury was freakish, nor that the manufacturer exercised care in its production.
 Note: this is similar to allergy cases, where the courts sometimes hold that the freakishness was so unforeseen that no
breach of warranty occurred.
(b) Which, if any, of Cristofori’s damages are recoverable under §2-714?
 The cost of the piano, if there was a breach of warranty. The piano was worth nothing to him.
(c) Which, if any, of the items claimed are incidental damages under §2-715(1)?
 The $500 paid the piano experts to determine the existence of the breach is the only obvious incidental expense.
(d) The §2-715(2)(a) test of consequential damages with its “reason to know” language is a restatement of our old friend Hadley
v. Baxendale. Is it relevant here?
 No. Under §2-715(2)(a), the buyer may only recover for economic losses arising from needs or requirements that the seller
had “reason to know” of at the time of contracting.
 Under 2(b) the buyer may only recover for personal injury or injury to property that was “proximately” caused by the
breach. There was no breach so no recovery in this case.
(e) If you were the judge of both the facts and the law, what amount would you award Cristofori, and why?
 Certainly not the axe, an unnecessary (though compelling) expense!

146
 If there was a breach of warranty, make sure to identify lost income and pain and suffering, but do not make a double
recovery.
 Query whether there is a double recovery, in whole or in part, in giving him both lost income and damages for lost hearing?
Cristofori will argue they are different, that even apart from future income, the loss of the ability to hear is worth
$1,000,000. Also, the piano should not have been a total loss; he could have mitigated by selling it
 Casebook Problem
o Sheila Spin made it to the finals of the USA Yo-Yo Championship, where she was widely thought to be a cinch to win the $10,000
first prize. The day of the competition she went into the Smalltime Drug Store owned by her Uncle Mort and told him that she
wanted to buy a four-foot nylon yo-yo cord to use in the competition. Mort sold her one for $1.50 (he put it on her bill) and
wished her luck. That she did not have. The cord was defective and broke during her first trick, thus eliminating her from the
competition. When the bill came from the drug store, Sheila refused to pay it. In fact, she filed suit against Mort, asking for
$50,000 consequential damages. Every expert witness who testified stated that Sheila’s ability with the yo-yo was the greatest
in the world. Mort defended on two grounds: (1) merely knowing about the intended use of the yo-yo in the competition was
not enough to impose liability on him unless the parties had agreed to put this risk on him, and (2) her damages were too
speculative. Answer these questions:
(a) Does the UCC permit Sheila to refuse to pay the bill? See §2-717. If a buyer feels obligated to pay only part of the bill, what
procedure should be followed? See §3-311 (dealing with payment-in-full checks).
 Yes, under §2-717, on notice of her intention to do so, she may set-off her claim against the bill. This is a valuable consumer
remedy!
 Practice Pointer: She might be well advised to mark her check as “payment in full.” The payee’s options as to a payment-in-
full check are described in §3-311.
 Generally, any cashing of such a check works an accord and satisfaction (settlement) of the debt, unless the payee returns
the money within 90 days.
(b) Are the consequential damages for which Sheila asked too speculative? See Official Comment 4 to §2-715; White & Summers
§11-4. Cf. Wachtel v. National Alfalfa Journal, (leading contender in canceled contest entitled to value of the chance of winning).
 Maybe. More and more courts are willing to value the chance to profit in a speculative situation; cf. Locke v. United States
I.e., some courts will allow recover for “lost chance” based on her odds of winning.
(c) Is knowledge of the possible consequential damages alone sufficient to impose liability on a seller? Or is Mort right in saying
that the liability for consequential damages attaches only if the seller has agreed (expressly or impliedly) to assume the risk?
See Official Comment 2 to §2-715; White & Summers §11-4; Beal v. General Motors Corp.
 No, knowledge alone is enough. See R.L. Lampus Co. V. Nevile Cement Prods, 474 Pa. 199 (1977), where the court ruled that
knowledge is enough.
 The UCC rejects the “tacit agreement” test, so that knowledge alone is sufficient to charge the seller with responsibility for
the foreseeable consequential losses, as the Beal case so held.
 Actually, the statute says that “reason to know” is enough.
(d) May Sheila recover her attorney fees as consequential damages? See Equitable Lumber Corp. v. IPA Land Dev. Corp. (may be
recovered if the contract so provides, but any specified amount must be valid as a liquidated damages provision); Too Marker
Products, Inc. v. Creation Supply, Inc. (no fees awarded under Illinois law without statutory or contractual provision). Cady v.
Dick Loehr’s, Inc. (trial court has discretion to award attorney fees as consequential damages); Modine Mfg. Co. v. North East
Indep. (Tex. Civ. App. 1974) (no, because such fees are penal in nature and the Code does not authorize punitive damages).
 Probably not. Some statutes allow recovery of attorney fees, but the common law did not.
 See §1-305, stating that consequential damages may only be had as authorized by the UCC. Some state or federal consumer
laws may allow the recovery of attorneys’ fees, but in this country the common law does not.
 Casebook Problem
o Rambo Trucks, Inc., sold Hercules Moving Company a large moving van. The contract of sale limited the buyer’s remedy for
breach of warranty to replacement or repair only and clearly disclaimed liability for consequential damages. The first day on the
job, the truck proved incapable of climbing even small hills, so Hercules Moving Company revoked its acceptance of the truck. It
claimed a security interest in the truck pursuant to §2-711(3) and pending sale stored it at a truck depot, which charged it $50 a
day for storage.
o Must Rambo Trucks pay the storage charges, or is the company protected by the disclaimer of consequential damages? See
§§2-719(3), 2-715(1); Commonwealth Edison Co. v. Allied Chem. Nuclear Prods., Inc., (storage charges of $293 million!); Dietec
Co., Ltd. v. Osirius Group, LLC. (seller’s costs caused by buyer’s breach, in securing additional raw materials and financing, held
to be incidental damages).
 There is a difference between incidental and consequential damages. The courts probably would not favor an exclusion of
incidental damages because they are necessary for the non-breaching party to avoid even larger damages. So no, the
exclusion of recovery for consequential damages should not bar recovery of the storage fees.
B. Unaccepted Goods
 Mirion Tech., Inc. v. Sunpower, Inc.

147
o In this case there was an agreement for the seller to construct and then sell cryocoolers to the buyer. Seller asked the court to
order the seller to make these objects, but the court held that specific performance of specially manufactured goods is only
available where the goods are unique, and the buyer is not able to cover elsewhere. The reluctance to grant specific
performance to make goods stems from the historical unwillingness of the courts to force labor and supervise the result of that
forced labor; forced labor sounds like slavery and the supervision would compel the court to judge the quality of the work, a
task for which your typical judge is unqualified.
o Here the court simply found that the cryocoolers were not sufficiently unique since there was an alternative source of supply
(granting that it was not a perfect substitute).
 Casebook Problem
o Mr. and Mrs. Transient ordered a 2020 Blocklong model mobile home for $8,000 from the Home on Wheels Sales Corporation,
delivery to be made on May 20. The Transients planned on spending an additional $500 to build a foundation that the Blocklong
trailer had to have for maximum utility. Due to widespread industry strikes, the price of trailers rose dramatically in the early
spring, and on May 10 Home on Wheels informed the Transients that the deal was off. The Transients shopped around and on
September 25 bought a 2020 Behemoth model mobile home for $15,000 from another dealer. The Behemoth was larger than
the Blocklong model (it had a basement and a laundry room), but it did not require a foundation. The Transients then brought
suit. Home on Wheels defended by offering to show that (a) the Behemoth was selling for $10,000 up to September 5 when the
price rose to $15,000, and (b) the Behemoth always sells for $2,000 more than the Blocklong since the former is a snazzier
trailer. What damages can the Transients get under §2-712? See Official Comment 2 of that section; White & Summers §7-3.
 Was cover “commercially reasonable” within the meaning of 2-712?
 No, they waited too long in a rising market.
 Under §2-713 they are entitled only to the difference between the contract price and the market value of the Blocklong
within a reasonable time after May 20.
 The $500 foundation cost is an “expense saved.” If the seller (who has the burden of proof on this) can prove that the
$2000 extra value of the Behemoth inures to the Transients’ benefit, that amount should be subtracted out of their
recovery as should the $5,000 price rise if they are found guilty of an unreasonable delay in covering.
 Hughes Communications Galaxy, Inc. v. United States
o FACTS: In 1985 Hughes entered into a contract (the Launch Services Agreement, or LSA) with NASA for NASA to use its “best
efforts” to deploy ten HS-393 satellites for Hughes by 1994. In 1986 the Challenger exploded, and President Reagan suspended
the shuttle program for a while. Thereafter NASA announced that it would not be launching satellites using the shuttle.
o Because the government failed to use best efforts to launch the satellites, it was in breach of the contract. The contract (the
LSA) contained a clause prohibiting Hughes from recovering consequential damages in the event of a breach.
o Hughes Communications Galaxy, Inc. v. United States (Fed. Circuit, 2001)
o Hughes made alternative arrangements for the launch of three more HS-393’s at a higher cost. Then, in part because of higher
launch costs, Hughes developed a larger and more powerful satellite bus, the HS-601, and it had several of those satellites
launched into orbit, charging its customers a higher price than it had charged for launching the HS-393’s. [today these satellites
are manufactured by Boeing, and the current model is the 702.]
o The government’s expert, Mr. Kiraly, estimated that because of technical difficulties and the demands of national security and
defense, that NASA could have launched between one and six HS-393’s before 1994. The Barrington Consulting Group estimated
that using its best-efforts NASA would have launched five HS-393’s by 1994.
o Hughes Communications Galaxy, Inc. v. United States (Fed. Circuit, 2001)
o Based on the Barrington report, the trial court – the Federal Court of Claims -- therefore concluded that Hughes was entitled to
recover the cost of launching five HS-393’s into orbit. It estimated this cost by taking the average cost of the three actual
launches of HS-393’s into orbit that Hughes obtained, and projecting the cost of two additional launches of that type of satellite.
o The $102 million recovery represents the difference between the contract price under the LSA and what the actual cost of
launching those five satellites would have been. [this is about $20 million more per satellite than what it would have cost under
the LSA using the shuttle.]
o Hughes Communications Galaxy, Inc. v. United States (Fed. Circuit, 2001)
o Both Hughes and the government contend that the trial court erred in its calculations.
o Hughes contends that it should have recovered the cost of launching ten satellites, because the government could have made it
a priority to launch commercial satellites. There was even a Congressional recommendation to this effect.
o ANSWER: The court finds that the government was not obligated to give priority to launching commercial satellites, and that
technical and national security concerns would have prevented there from being more than five launches under the contract.
“The Court of Federal Claims did not clearly err in finding that technical obstacles, rather than budget choices, prevented NASA
from launching more shuttles during the LSA contract period.”
o Hughes Communications Galaxy, Inc. v. United States (Fed. Circuit, 2001)
o The government makes three arguments:
1. The government contends that Hughes is not entitled to any recovery; that all of its costs are “consequential damages” –
damages as a consequence of the breach – and that these damages are excluded by the contract.

148
 ANSWER: This is one place where the court applies Article 2 by analogy. This was a services contract, not a sales
contract, but the court used Article 2 by analogy. “the Uniform Commercial Code provides useful guidance in applying
general contract principles.” The court describes Section 2-712, granting a buyer the difference between the contract
price and the cost of cover against a breaching seller. Cover need not be identical but must be “commercially usable as
reasonable substitutes under the circumstances.” Quoting Comment 2 to 2-712
 Later, the court also points out that this remedy for cover is IN ADDITION TO incidental and consequential damages.
 Section 2-712 and the UCC in general distinguishes between expectancy and consequential damages. Both are
“caused” by the other party’s breach; but the expectancy damages are damages that reflect the direct cost of a
substituted performance of the other party’s duty under the contract. “The Uniform Commercial Code is instructive on
this point.”
2. The government contends that at most Hughes should recover only the cost of launching three satellites, the three that it
actually did launch.
 ANSWER: The court accepts the trial court’s finding that Hughes would have launched at least five HS-393’s but for the
government’s breach of its duty under the LSA to use its best efforts to do so. The court found that the use of the HS-
601’s was “reasonable substitutes under the circumstances of this breach. The court states that this was a credibility
issue, and “such determinations are virtually never clear error.”
3. The government contends that the damage award should be reduced by the extra amount that Hughes charged its
customers for the HS-601’s in place of the HS-393’s.
 ANSWER: The court finds this kind of “pass through” of increased costs does not diminish the liability of a breaching
party – that a party’s liability for breach of contract is only taken “one step”. To do otherwise would “destroy the
symmetry between reduction and escalation of damages.”
 Note - In other words, the plaintiff’s expectancy damages include only the cost of substituted performance of the other
party – other damages flowing from the breach are “consequential” and may not be recoverable. The same should be
true for the breaching party – it may not claim that any profits that consequentially result from its breach should
reduce its liability.
 Holding: The Federal Circuit Court affirms the decision of the federal Court of Claims and finds that the Court of Claims
did not abuse its discretion in calculating damages.
 Satellite maker Hughes won the astronomical sum of $102,680,625 in damages, for NASA’s failure to use promised best
efforts to launch Hughes’ satellites.
 Casebook Problem
o The Student Bar Association (SBA) of the Gilberts Law School decided to hold a mammoth wine-and-cheese-tasting party for the
students, faculty, staff, and alumni. The SBA ordered the wine from Classy Caterers. They agreed to pay $1,000 for it; the wine
to be delivered on March 30, the day of the party. Classy Caterers ordered the wine from Grapes Vineyards in California, “F.O.B.
San Francisco” for $750, but Grapes Vineyards went bankrupt on March 25. Classy Caterers was able to find identical wine in its
own city for $750, and it bought the wine on March 25 for that amount. On March 25, the price of similar wine in San Francisco
was $900. The cost of transporting the wine from San Francisco to the site of the party would have been $100. The SBA paid
Classy Caterers $1,000 for the wine. Classy Caterers filed claims for damages in the bankruptcy proceeding of its defaulting
supplier.
(a) Compute the damages due Classy for the failure to deliver the wine under §2-712. Now do it under §2-713. See Official
Comment 5 to §2-713; White & Summers §7-4.
 The damages for failure to deliver the wine would be computed using §2-712(2) since Classy covered, i.e., the difference
between the cost of cover ($750) and the contract price ($750) less expenses saved ($100 transportation); actually, Classy
comes out $100 ahead.
 If Classy had not and could not have covered on the wine, the damages computed under §2-713 would be the difference
between the market price at the time when buyer learned of the breach ($900) and the contract price ($750).
(b) Plus, consequential damages ($150)? No. Less expenses saved ($100)? No, this amount should not be subtracted; see (b)
below. The answer then is $150.
 What role does the $100 transportation cost play in computing damages? Note the definition of “marketplace” in §2-723.
 We believe in computing the §2-712 and §2-713 damages for the wine, prices should be compared at one location and the
$100 ignored or treated as a “wash.”
 Tongish v. Thomas
o This case illustrates the difference between expectancy damages and consequential damages. If the seller breaches the contract
by failing to deliver goods, is the buyer entitled to recover expectancy damages even though the buyer did not experience any
consequential damages?
o This is a fairly common occurrence. If the buyer enters into a contract to purchase goods from the seller, the buyer is purchasing
the goods for resale, the market price of the goods increases, the seller decides to sell the goods to another customer at the
higher price and breaches the contract with the buyer, the buyer decides not to cover, and the buyer’s customer agrees to
rescind their agreement with the buyer, is the seller still liable to the buyer for the difference between the contract price and

149
the market value of the goods? In other words, is a buyer entitled to recover expectancy damages on account of a seller’s
breach even though the buyer did not suffer a loss on resale of the goods?
o FACTS: That is what happened in this case. When the price of soybeans rose dramatically Tongish breached its fixed-price
contract with Decatur Coop Association and sold the sunflower seeds to another customer, Danny Thomas. The Coop was going
to resell the sunflower seeds to Bambino Bean and Seed, Inc., Bambino agreed to cancel their contract, so the Coop was not
burdened with a lawsuit for breach. Tongish contended that because the Coop did not suffer any consequential damages on
account of the breach therefore the Coop was not entitled to any expectancy damages. Tongish contends that because
Bambino did not force the issue and sue the Coop for breach, therefore the Coop should not be allowed to sue Tongish for
breach.
o The courts ruled in favor of the Coop and held that the Coop was entitled to recover direct expectancy damages from Tongish
measured by the difference between the contract price and the market value of soybeans as of the time of the breach.
o Were the courts were right to rule in favor of the Coop?
o Wasn’t this a “windfall” for the Coop?
o What factors lead the court to allow a §2-713 recovery?
o The Supreme Court of Kansas has a wonderful analysis of the question whether the buyer who has actually resold the goods can
ignore what happened in that sale (where the resale was so successful no damages occurred) and instead claim damages under
the artificial market price/contract price comparison test in §2-713, which would give the buyer an arguable windfall.
o It would seem that basic justice requires the buyer to be held to what actually happened in the resale, but other factors lead the
court to allowing the §2-713 recovery.
o These are to prevent sellers from breaching in bad faith and getting away with it, and to protect the buyer’s expectation that the
market may improve and not deprive the buyer of the reward when that occurs.
o Another consideration is that there are lots of damages in a lawsuit, among them attorney fees, that are not recompensed, and
which the §2-713 formula (as sort of a statutory liquidated damages attempt) in part remedies.
o The court makes much of the academic literature on point, and good for us academics!
 Canvas Quiz Questions
o There are four types of money damages recoverable by buyers for breach of contract: restitution, expectancy damages,
incidental damages, and consequential damages. In an appropriate case it would be possible for a buyer to recover all four
types of damages from a breaching seller.
 True - In an appropriate case it is possible that a buyer could recover all four types of monetary damages from a breaching
seller.
o Please identify whether the particular remedy for a buyer against a breaching seller for monetary damages under Article 2 is
a form of restitution, expectancy damages, incidental damages, or consequential damages:
 Damages for injury to person or property of the buyer caused by a - Consequential damages
defective product under Section 2-715(2)(b)
 Buyer's expenses for receipt, storage, reshipment, or resale as a result of - Incidental damages
the breach
 The difference between the cover price for substitute goods and the - Expectancy damages
contract price of the goods
 Return of any portion of the purchase price that the buyer paid to the - Restitution
breaching seller
 The difference between the market value of the goods had they been as - Expectancy damages
warranted and the actual value of the goods
 If the buyer does not cover, the difference between the market value of - Expectancy damages
the goods and the contract price of the goods
 Attorney fees incurred by the buyer arising from negotiation with or - Not recoverable under
litigation against the breaching seller Article 2
 Punitive damages on account of seller's breach of contract - Not recoverable under
Article 2
o Pascal ordered 20 printed tee shirts from Happy Tees for a softball team his hardware store is sponsoring this summer. The price
was $25 for each shirt due on delivery, and Happy Tees promised that the shirts would be "letter perfect." When the uniforms
arrived the name of his store was misspelled "Pascall's." Pascal rejected the uniform shirts, refused to pay for them, and
demanded that Happy Tees reprint the shirts. Happy Tees said it would be glad to print new uniforms -- for an extra $20 per
shirt ($45 per shirt total)! Pascal declined Happy Tee's kind offer, and instead he ordered replacement shirts from Gene's Sports
Shop. As a rush order, Gene's charged Pascal $30 per printed shirt. What measure of damages, if any, is Pascal entitled to from
Happy Tees?
 Answer: Pascal covered, so Pascal is entitled to recover the difference between the cover price ($30) and the contract price
($25) -- $5 per shirt, for a total of $20.
 Pascal is not entitled to anything because Happy Tees offered to cure the defect. - Happy Tees did not offer to "cure" the
defect; instead, it would have charged Pascal to fix the error in the printing.

150
 Because the shirts were not conforming to the contract Pascal is entitled to the difference between the market value that
the tee shirts would have had if they had been as warranted and the market value that they actually had. - Pascal did not
keep the shirts, so he is not entitled to damages for breach of warranty.
 Pascal is entitled to the return of his purchase price. - Pascal did not pay any portion of the contract price, so he is not
entitled to restitution.
o Darin owns an origami shop. He ordered 20 boxes of origami paper from Offshore Supplies: each carton contains 100 packages
of origami paper. The total price of all the paper was $4,000, which Darin paid upon delivery. However, within a few weeks
Darin discovered that some of the paper wasn't perfect. Some of the packages of paper weren't perfectly square, which makes
it useless for origami. About 5% of the packages were non-conforming. The rest of the paper is perfectly square, and the paper
itself is attractive to customers, so Darin decided to keep it. What measure of damages is Darin entitled to, if any?
 Darin is entitled to restitution of the amount he paid for the paper. - Darin not entitled to restitution because Darin
accepted and retained the goods; if he wanted his money back, he should have revoked his acceptance and tendered the
paper back to the seller.
 Answer: Offshore Supplies violated the implied warranty of merchantability. Darin is entitled to damages for breach of
warranty, that is, the difference between the value that the goods would have had had they been merchantable, and their
actual value.
 Darin is entitled to the difference between the market price of the paper and the contract price. - Darin is not entitled to
the difference between the market price of the goods and the contract price because Darin accepted and retained the
goods.
 Darin is entitled to the difference between the cover price of the goods and the contract price of the goods. - And Darin is
not entitled to damages for "cover" because he retained the goods and did not cover.
Comparison of Sellers’ and Buyers’ Remedies Under Article 2 of the Uniform Commercial Code
Sellers’ Remedies Buyers’ Remedies
Situation A: Seller Has Goods. Buyer Repudiates or Situation A. Seller Has Goods. Seller Fails to Deliver the Goods, Buyer
Wrongfully Rejects the Goods (Goods remain with or are Rightfully Rejects or Rightfully Revokes Acceptance of the Goods
returned to seller). (Goods remain with or are returned to seller).
Expectancy Damages Expectancy Damages
Full enforcement: Contract price. If the goods cannot be Full enforcement: Delivery of the goods. If the goods are unique or in
resold, then seller is entitled to recover the entire purchase other proper circumstances the buyer is entitled to specific
price. performance. If the buyer is unable to obtain an adequate substitute
for the goods, then the buyer is entitled to replevin of identified
goods. In addition, in certain circumstances the buyer is entitled to
recover identified goods from an insolvent seller.
OR OR
Resale remedy. If the goods are resold in a commercially Cover remedy. If the buyer purchases goods that are a commercially
reasonable manner the seller is entitled to recover the reasonable substitute, then the buyer is entitled to recover the
difference between the contract price and the resale price. difference between the contract price and the cost of cover.
OR OR
Market price remedy. Whether or not the goods are Market price remedy. If the buyer does not cover then the buyer is
resold the seller is entitled to recover the difference entitled to recover the difference between the contract price and the
between the contract price and the market price at the market price of the goods at the place specified for delivery in the
time of delivery and place of delivery specified in the contract within a reasonable time after the buyer learned of the
contract. breach.

OR OR
Lost profits on sale. If the seller is a “lost volume” seller; Lost profits on resale. If the buyer is unable to cover and the buyer
that is, if the seller in effect “lost a sale” because the lost an opportunity to resell the goods because the seller breached
seller’s supply outstrips demand for the goods, then the and if the buyer can prove that the seller had reason to know that the
seller is entitled to recover its “lost profits” measured by buyer intended to resell the goods then the buyer is entitled to
the difference between the contract price and the variable recover its “lost profits” that it would have earned; that is, the
costs of manufacturing or acquiring the goods. difference between the contract price and the amount of the resale
price (a form of consequential damages).
Mutual Restitution Mutual Restitution
The aggrieved seller is entitled to return of the goods. The aggrieved buyer is entitled to the return of any portion of the
The breaching buyer is entitled to the return of any portion price that has been paid.
of the price that has been paid, however ... The breaching seller is entitled to return of the goods, however ...
The aggrieved seller is entitled to retain any portion of the The aggrieved buyer is entitled to retain the goods as security for
purchase price to the extent of any damages to which the return of any portion of the purchase price and any incidental
seller is entitled. damages to which the buyer is entitled.

151
Incidental Damages Incidental Damages
The aggrieved seller is entitled to recover for any out-of- The aggrieved buyer is entitled to recover for any out-of-pocket
pocket expenses incurred after the buyer’s breach in order expenses incurred after the seller’s breach in order to mitigate
to mitigate damages, including expenses incurred for damages, including expenses incurred for storing, reshipping, or
storing, reshipping, or reselling the collateral. reselling the collateral.
Consequential Damages Consequential Damages
There is no provision in Article 2 for an aggrieved seller to The aggrieved buyer is entailed to recover damages or personal injury
recover consequential damages on account of a buyer’s or damage to property proximately caused by the seller’s breach.
breach. The aggrieved buyer is entitled to recover any lost profits proximately
caused by the seller’s breach if the seller had reason to know of the
buyer’s particular needs or requirements. Lost profits may result from
any delay, business slowdown, or customers lost as a result of the
breach.
Situation B. Buyer Has Goods. Buyer has accepted the Situation B. Buyer Has Goods. Buyer has accepted the goods and has
goods and has failed to rightfully revoke acceptance. The failed to rightfully revoke acceptance. The buyer has paid for the
buyer has not fully paid for the goods despite the fact that goods, but the goods are non-conforming to the contract.
the goods are conforming to the contract.
Expectancy Damages Expectancy Damages
Aggrieved seller is entitled to the entire purchase price; Aggrieved buyer is entitled to damages for breach of warranty; that is,
that is, the difference between the amount that the buyer the difference between the value that the goods would have had if
promised to pay in the contract and the amount that the they had been as warranted and the value that the goods actually had
buyer has already paid. as delivered.
Restitutionary Damages Restitutionary Damages
If the buyer is able to pay for the goods an aggrieved seller An aggrieved buyer would not be entitled to restitution in this
would likely pursue that remedy; the seller would likely not situation; the buyer would not be entitled to the return of the
demand or agree to the return of the goods. However, if purchase price. However, if the seller reclaims the goods because the
the buyer has not fully paid for the goods from the buyer if buyer was insolvent when the buyer would be entitled to the return
the buyer received the goods while the buyer was of any portion of the purchase price that it had paid.
insolvent and if the seller demanded the return of the
goods within ten days of the buyer’s receipt.
Incidental Damages Incidental Damages
An aggrieved seller is not likely to have any incidental The aggrieved buyer is entitled to recover for any out-of-pocket
damages in this situation. expenses incurred after the seller’s breach in order to mitigate
damages, including expenses incurred for storing, reshipping, or
reselling the collateral.
Consequential Damages Consequential Damages
There is no provision in Article 2 for an aggrieved seller to The aggrieved buyer is entailed to recover damages or personal injury
recover consequential damages on account of a buyer’s or damage to property proximately caused by the seller’s breach.
breach. The aggrieved buyer is entitled to recover any lost profits proximately
caused by the seller’s breach if the seller had reason to know of the
buyer’s particular needs or requirements. Lost profits may result from
any delay, business slowdown, or customers lost as a result of the
breach.
IV. Anticipatory Repudiation
 Anticipatory Repudiation is a Breach
o Anticipatory repudiation is an announcement by a party that it does not intend to perform the contract. It is a breach of
contract by a party that occurs before that party performs.
 Examples of Anticipatory Breach by a Seller and a Buyer
o Before the time for delivery of the goods, seller notifies buyer that it will not deliver the goods. This is an anticipatory breach of
the contract.
o Before the time for payment, buyer notifies seller that it will not accept or pay for the goods. This is an anticipatory breach of
the contract.
 Rights of the Non-Breaching Party in Response to an Anticipatory Repudiation
o The non-breaching party may:
 Await performance of the repudiating party for a commercially reasonable time;
 Resort to any remedy for breach; and
 Suspend its own performance, or in the case of a seller, identify goods to the contract or salvage unfinished goods.
 Anticipatory Repudiation §2-610

152
o When one party notifies the other that it does not intend to honor the contract before the time for its performance has come, it
is called “anticipatory repudiation.”
o This is a breach of contract.
o E.g., Seller informs Buyer it will not deliver goods.
 Consider the example of a rapidly rising market price and Seller can get more for the goods by selling to another customer,
not to Buyer. (Diamond Importers example in Lesson 37 on Blackboard)
o E.g., Buyer informs Seller it will not accept or pay for the goods.
 Section 2-610: Rights of the Non-Breaching Party in Response to an Anticipatory Repudiation
When either party repudiates the contract with respect to a performance not yet due the loss of which will substantially impair the
value of the contract to the other, the aggrieved party may
(a) for a commercially reasonable time await performance by the repudiating party; or
(b) resort to any remedy for breach (Section 2-703 or Section 2-711), even though he has notified the repudiating party that he
would await the latter's performance and has urged retraction; and
(c) in either case suspend his own performance or proceed in accordance with the provisions of this Article on the seller's right to
identify goods to the contract notwithstanding breach or to salvage unfinished goods (Section 2-704).
 Retraction of Anticipatory Repudiation
o A party may retract its anticipatory repudiation if it acts before the non-breaching party has cancelled the contract or materially
changed its position in reliance on the repudiation.
o To be effective the retraction must include any assurances that the non-breaching party has justifiably demanded.
o A repudiating party who retracts is liable for damages resulting from any delay caused by the repudiation
 What can the Non-breaching Party Do?
o The non-breaching party may:
 await performance of the repudiating party for a commercially reasonable time;
 resort to any remedy for breach (Section 2-703 or Section 2-711); and
 suspend its own performance.
o A seller may also identify goods to the contract; and may either salvage or complete the manufacture of unfinished goods.
 Is Retraction Possible? §2-611
o Yes! A party may retract its anticipatory repudiation if it acts before the non-breaching party has cancelled the contract or
materially changed its position in reliance on the repudiation.
o To be effective the retraction must include any assurances that the non-breaching party has justifiably demanded.
o If a party’s statement is not clearly a repudiation the other party has the right to demand assurances. UCC §2-609.
o What happens if one party’s statements are equivocating?
 E.g., “I’m not sure that we can honor our contractual commitments.”
o What would you advise your non-breaching client to do?
 Can client treat that statement as an anticipatory breach and cancel?
 Or is cancelation itself be an anticipatory breach?
 Section 2-611: Retraction of Anticipatory Repudiation
(1) Until the repudiating party's next performance is due, he can retract his repudiation unless the aggrieved party has since the
repudiation cancelled or materially changed his position or otherwise indicated that he considers the repudiation final.
(2) Retraction may be by any method which clearly indicates to the aggrieved party that the repudiating party intends to perform
but must include any assurance justifiably demanded under the provisions of this Article (Section 2-609).
(3) Retraction reinstates the repudiating party's rights under the contract with due excuse and allowance to the aggrieved party for
any delay occasioned by the repudiation.
 Seller’s Right to Identify Goods to the Contract or Complete/Salvage Unfinished Goods
o If the buyer commits an anticipatory breach by repudiating the contract before the seller has identified goods to the contract,
Section 2-704(1) gives the seller the right to identify goods to the contract. The seller can then resell these goods and recover
the difference between the contract price and the resale price.
o Similarly, under Section 2-704(2) if the buyer repudiates before the seller has completed manufacturing the goods, the seller
may exercise reasonable commercial judgment to either complete manufacture of the goods or resell the unfinished goods for
salvage.
 Section 2-704: Seller’s Right to Identify Goods to the Contract or Complete/Salvage Unfinished Goods
(1) An aggrieved seller under the preceding section may
(a) identify to the contract conforming goods not already identified if at the time he learned of the breach they are in his
possession or control;
(b) treat as the subject of resale goods which have demonstrably been intended for the particular contract even though those
goods are unfinished.
(2) Where the goods are unfinished an aggrieved seller may in the exercise of reasonable commercial judgment for the purposes of
avoiding loss and of effective realization either complete the manufacture and wholly identify the goods to the contract or cease
manufacture and resell for scrap or salvage value or proceed in any other reasonable manner.
153
 Example: Anticipatory Repudiation by a Seller: 15 Gems
o On August 1 Diamond Importers agreed to sell 15 uncut fancy-green irregular crystal diamonds, average size 1.14 carat, to Rick’s
Jewelry for a total price of $39,000. The delivery date was November 1, which would give Rick time to cut and set the diamonds
for the holiday season. On September 1 Diamond Importers notified Rick that it would not honor the contract. (The market price
was rapidly rising and by September 1 Diamond Importers could get $50,000 for the stones from other buyers.)
o Can Rick simply wait until November 1 and then sue Diamond Importers for breach of contract? Or does he have a duty to try to
“cover” and obtain diamonds from another source? This problem is on the quiz.
 Example: Anticipatory Repudiation by a Buyer: Solar Panels
o Green Builders ordered 2000 270-watt monocrystalline silicon solar panels from Sol Unlimited, a solar panel manufacturer, for a
price of $500,000. The contract calls for the panels to be delivered to Green Builders on March 15. On February 1 Green Builders
notifies Sol that it is cancelling its order. Assume that on February 1 Sol was in the process of manufacturing these panels; none
of the panels was completed, but they were all about 70% completed.
o May Sol complete manufacturing the panels and try to resell them? Or is Sol required to stop the manufacturing process as soon
as Green repudiates? This problem is on the quiz.
 If a Party’s Statement Is Not Clearly a Repudiation, the Other Party May Demand Assurances
o What if a seller or a buyer makes a statement that may or may not be a repudiation? For example, the buyer contacts the seller
and says, “I’m not sure that I still need all that aluminum siding I ordered.” What should the other party do? May the other party
treat this as an anticipatory breach and cancel the contract? Or would that in itself be an anticipatory breach?
o In this situation the safe thing to do is to “demand assurances.” Under Section 2-609(1), “When reasonable grounds for
insecurity arise … the other party may in writing demand adequate assurance of due performance ….” The other party must
respond to this demand for adequate assurance within a reasonable time not exceeding 30 days.
 Section 2-609: If a Party’s Statement Is Not Clearly a Repudiation, the Other Party May Demand Assurances
(1) A contract for sale imposes an obligation on each party that the other's expectation of receiving due performance will not be
impaired. When reasonable grounds for insecurity arise with respect to the performance of either party the other may in
writing demand adequate assurance of due performance and until he receives such assurance may if commercially reasonable
suspend any performance for which he has not already received the agreed return.
(2) Between merchants the reasonableness of grounds for insecurity and the adequacy of any assurance offered shall be
determined according to commercial standards.
(3) Acceptance of any improper delivery or payment does not prejudice the aggrieved party's right to demand adequate assurance
of future performance.
(4) After receipt of a justified demand failure to provide within a reasonable time not exceeding thirty days such assurance of due
performance as is adequate under the circumstances of the particular case is a repudiation of the contract.
 Casebook Problem
o The United States Army contracted with the Hawaiian Cattle Company for the purchase of 1,000 pounds of beef. Delivery was set
six months later, on October 8, the agreed price to be $5,000. Shortly thereafter, the price of beef rose sharply, and Hawaiian
Cattle repudiated the contract on July 10, when the price was $6,000. The Army’s procurement officer scrambled around and on
July 15 discovered it was possible to cover by buying similar cattle from Texas at a cost of $7,000. Instead, the Army sent
Hawaiian Cattle a telegram stating that it did not accept or recognize the repudiation and expected performance on October 8.
By October 8 the price had risen to $8,000. The Army decided not to cover at all and instead served the troops beans. As general
counsel for the Army, advise the Army of the amount it can recover from Hawaiian Cattle. Read §§2-610, 2-713, and 2-723(1).
Does it help reconcile these sections to know that the drafters of §2-713 were thinking of a buyer who learns of the
repudiation after the date set for the original performance, not (as in this Problem) before the due date?
 The courts have generally applied the rule that the non-breaching party must cover “within a reasonable time” after a
repudiation – the other party can’t just wait for performance as if the breach did not occur.
 §2-610 says that the innocent party may “for a commercially reasonable time await performance”.
 That reasonable time had expired in this case well before October 8.
 The price was rising, and the Army had to cover more promptly than it did. The army may not recover its cover price.
 Most courts have preferred the interpretation that measures damages at the end of a commercially reasonable time for
covering.
 The problem with the Code language is that the relevant sections simply do not fit together as a result of sloppy drafting. The
“learned of the breach” language in §2-713 was meant to apply only in the situation in which the buyer first discovered the
breach after the date set for performance.
 In the end, the courts must forge their own way here, and using the date of a commercially reasonable time for covering
most agrees with the language of §2-610 and common sense.
 Summary: Anticipatory Repudiation
o When one party notifies the other that does not intend to honor the contract before the time for its performance has come, it
is called “anticipatory repudiation.” This is a breach of contract.

154
o The non-breaching party may await performance of the repudiating party for a commercially reasonable time; may resort to
any remedy for breach; and may suspend its own performance. A seller may also identify goods to the contract and may either
salvage or complete the manufacture of unfinished goods.
o A party may retract its anticipatory repudiation if it acts before the non-breaching party has cancelled the contract or materially
changed its position in reliance on the repudiation. To be effective the retraction must include any assurances that the non-
breaching party has justifiably demanded.
o If a party’s statement is not clearly a repudiation the other party has the right to demand assurances.
 Canvas Quiz Questions
o Anticipatory repudiation by a party
 Answer: is a breach of contract that occurs before the time for that party's performance has arrived; that is, it is an
announcement by the seller that it intends to breach before the time for delivery, or an announcement by the buyer that
occurs before the time for payment.
 is not a breach of contract; it merely triggers the right of the other party to demand assurances of performance.
 is another term for revocation of acceptance.
 cancels all of the repudiating party's legal obligations under the contract.
o After a party makes an anticipatory repudiation the non-breaching has the following rights:
 await performance of the repudiating party for a commercially reasonable time.
 resort to any remedy for breach.
 suspend its own performance.
 if the non-breaching party is a seller, may identify goods to the contract and resell the goods and recover the difference
between the contract price and the resale price.
 if the non-breaching party is a seller and the goods are unfinished, the seller may either salvage the goods or complete their
manufacture.
o If a party makes an anticipatory repudiation of a contract, it has an absolute right to retract its repudiation up until the time
for its performance.
 False - A party may retract its repudiation of the contract up until the time for its next performance only if the non-breaching
party hasn't cancelled or materially changed its position in response to the repudiation.
o Diamond Importers agreed to sell 15 uncut fancy-green irregular crystal diamonds, average size 1.14 carat, to Rick’s Jewelry for
$39,000. The delivery date was November 1, which would give Rick just enough time to cut and set the diamonds for the holiday
season. On September 1 Diamond Importers notified Rick that it would not honor the contract. (The market price was rapidly
rising and by September 1 Diamond Importers could get $50,000 for the stones from other buyers.) Can Rick simply wait until
November 1 and then sue Diamond Importers for breach of contract, including any lost profits from Christmas sales? Or does
he have a duty to try to “cover” and obtain diamonds from another source?
 Answer: Rick can await performance from Diamond Importers for a "reasonable time," but under the circumstances that
probably isn't two months! Rick had better cover before then in order to mitigate his damages - and so he has something to
sell over the holidays!
 The law gives Rick the absolute right to wait until November 1 to to see whether Diamond Importers will perform the
contract.
o Green Builders ordered 2000 270-watt monocrystalline silicon solar panels from Sol Unlimited, a solar panel manufacturer, for a
price of $50,000. The contract calls for the panels to be delivered to Green Builders on March 15. On February 1 Green Builders
notifies Sol that it is cancelling its order. Assume that on February 1 Sol was in the process of manufacturing these panels; none
of the panels was completed, but they were all about 70% completed. May Sol complete manufacturing the panels and try to
resell them? Or is Sol required to stop the manufacturing process as soon as Green repudiates?
 Answer: Under the law Sol may either complete the manufacture of the panels or sell the unfinished goods for salvage, so
long as it exercises reasonable commercial judgment and acts in good faith.
 Under the law Sol must complete manufacturing the panels and resell them.
 Under the law Sol must immediate stop manufacturing the panels and salvage the unfinished goods.
 Under the law it doesn't matter what Sol does. Sol has no rights as against Green because the panels weren't finished.
o Your client Charlene runs a nightclub. She has an agreement with Nova, a food and beverage supplier, to supply all her needs for
the club for a period of one year. Nova makes deliveries on Saturday morning every week. On a Monday six months into the
contract Fred Joslin, the owner of Nova, sent Charlene an urgent email stating, "Due to inflation I am not sure I can fill the rest of
your orders that we contracted for this year. We may have to increase the contract price." Charlene has drafted a number of
replies to Fred. Which one should she send?
 Answer: "Fred, I am demanding to know whether you intend to honor the contract as written. If you don't respond to this
email by Wednesday at this time then I will consider that you have breached our agreement, and I will seek another supplier
and sue you for damages. You have 48 hours to make up your mind."
 "Fred, I can't do business with someone who won't honor a contract. You're fired!"
 No message. Just wait and see what happens Saturday morning.
 No message. Instead, file suit for specific performance.

155
– Charlene should demand assurances, and under the circumstances two days is a reasonable period of time for Fred
to reply. If she waits any longer, she might not be able to obtain alternative supplies.
V. Statute of Limitations
 Four-Year Statute of Limitations, Reducible to One Year
o §2-725 contains four (4) sections. We will focus on §§(1), (2), and (4).
o Section 2-725(1) establishes a four-year statute of limitations. A lawsuit for breach must be filed within four years of the time
that the cause of action accrues.
o The parties may by agreement reduce this period to as little as one year but may not extend it.
o Note: The four-year period of limitations is within the time that businesses retain records. Only one jurisdiction has stated that a
clause limiting the period of limitations does not have to be conspicuous. (South Dakota)
 Section 2-725(1): Four-Year Statute of Limitations, Reducible to One Year
An action for breach of any contract for sale must be commenced within four years after the cause of action has accrued. By the
original agreement the parties may reduce the period of limitation to not less than one year but may not extend it.
 A Cause of Action Accrues at the Time of Breach
o Under Section 2-725(2), a cause of action accrues, and the period of limitations starts to run when the breach occurs.
o A breach of warranty normally occurs when the seller tender’s delivery of the goods, whether or not the buyer has knowledge of
the defect.
o If the warranty “explicitly extends to future performance of the goods and discovery of the breach must await the time of such
performance,” the period of limitations begins to run when the breach is or should have been discovered.
 Section 2-725(2): A Cause of Action Accrues at the Time of Breach
A cause of action accrues when the breach occurs, regardless of the aggrieved party's lack of knowledge of the breach. A breach of
warranty occurs when tender of delivery is made, except that where a warranty explicitly extends to future performance of the
goods and discovery of the breach must await the time of such performance the cause of action accrues when the breach is or
should have been discovered.
 When Does a Warranty “Explicitly Extend to Future Performance of the Goods”?
o Only warranties that “explicitly extend to future performance of the goods” can trigger the “discovery rule” that delays accrual of
the cause of action and extends the running of the statute of limitations.
o Accordingly, only express warranties can have this effect. The implied warranty of merchantability always accrues when the
goods are delivered.
 A Promise to Repair or Replace the Goods
o A seller’s promise to repair or replace the goods is a warranty within the meaning of the Magnuson Moss Act (if it is in writing).
However, most courts have held that it is not a express warranty within the meaning of Section 2-313 because it does not “relate
to the goods” but rather to the performance of the seller. For the same reason, the “discovery rule” of Section 2-725 does not
apply because a promise to repair is not a warranty that relates to the performance of the goods.
o Accordingly, the courts have developed a specific rule regarding promises to repair. The period of limitations begins to run for
breach of the promise to repair when the seller either attempts or refuses to repair the goods.
o For the same reason, the “discovery rule” of Section 2-725 does not apply because a promise to repair is not a warranty that
relates to the performance of the goods.
o Accordingly, the courts have developed a specific rule regarding promises to repair. The period of limitations begins to run for
breach of the promise to repair when the seller either attempts or refuses to repair the goods.
 Extension of the Statute of Limitations by Fraud, Active Concealment, or Estoppel
o Section 2-725(4) provides: “This section does not alter the law on tolling of the statute of limitations nor does it apply to causes
of action which have accrued before this Act becomes effective.”
o For example, under other provisions of state law the statute of limitations may be “tolled” if the seller’s repeated
misrepresentations prevented the buyer from discovering the defect within the period of limitations.
 Query: under what circumstances would the statute of limitations be different from four years beginning on the day of delivery?
In three cases …
1. When the parties reduce the period in the contract – to as little as one year. Almost all courts require this reduction to be
“conspicuous.”
2. When a warranty explicitly extends to future performance the cause of action accrues when the defect is discovered! Implied
warranties by definition cannot “explicitly” extend to future performance; only express warranties can. For an example see
Perry v. Augustine a Pennsylvania case involving a heating system where the seller promised that the heating system would be
able to keep the home at 75 degrees when it was minus 20 degrees outside. This cause of action did not accrue until winter!
3. Tolling laws are not affected by the U.C.C. §2-725(4). Absence from the jurisdiction, fraud, active concealment, and estoppel can
all serve to toll the statute of limitations.
 Casebook Problem
o Four years and two days after delivery of his new car, Joe Suburb came out of his house to discover he had a flat tire. He
changed it and, for the first time, put on the spare tire. That afternoon as he was driving on the interstate, the tire burst due to a
manufacturing flaw. Joe was killed. Has the statute of limitations run under these situations?
156
(a) The tire warranty read: “These tires are merchantable.” If this were a lease of goods, would Article 2A reach a different
result? See §2A-506.
 The statute has run. There is no warranty as to future performance, so the four-year period is all that is available. Four years
starting with delivery.
 If this were a lease of goods under Article 2A, then four years from the time when the lessee should have discovered the
problem, under 2A-506(2).
(b) The tire warranty read: “These tires have a lifetime guarantee.” See Kelleher v. Lumber, Rawls v. Associated Materials.
 “The statute of limitations has not run. This warranty is “explicitly” extended.
 Query: exactly whose lifetime was specified, the buyer’s or the product’s?
(c) The sales material said with regard to the tires: “Many tires are still on the road after five years.” See Jones & Laughlin Steel
Corp. v. Johns-Manville Sales Corp. v. Brown & Kerr, Inc., (a statement of future performance is “explicit” if it does not arise by
implication but is instead “distinctly stated; plain in language; clear; not ambiguous; express; unequivocal”).
 This is a warranty, but it does not “explicitly extend” the statute of limitations.
 Query: Really? … the obvious implication is that the tires will be good for five years.
(d) The tire warranty read: “These tires are warranted for seven years from date of purchase.” If a tire first malfunctioned at the
beginning of the third year, when would the statute of limitations expire? See Joswick v. Chesapeake Mobile Homes, Inc.
 The statute of limitations starts when the breach should have been discovered, at the beginning of year 3. The four-year clock starts
ticking at the moment within that period when the breach should have been discovered; here, the beginning of year three
(e) There were no express warranties given with regard to the tires; there were also no warranty disclaimers. See Note, When
Does the Statute of Limitations Begin to Run on Breaches of Implied Warranties?
 Four years after delivery. There is no way that an implied warranty can “explicitly” extend into the future, so the courts agree that all
implied warranties die at the end of a four-year period following delivery.
 Mydlach v. Daimler-Chrysler Corp.
o DaimlerChrysler promised to repair any item in a Dodge Neon for 36 months or 36,000 miles. Toward the end of that time
period, after Mydlach had bought the car used, the car had many problems which the dealer was unable to fix. Mydlach finally
filed suit, more than four years after the car had been delivered to its original owner.
o Daimlerchrysler argued that the four year statute of limitations had run, because a cause of action for breach of warranty
accrues when the goods are delivered, under §2-725.
o The court, after surveying the views of courts and commentators, held that this case involved not a breach of warranty but
breach of a promise to repair. The cause of that action accrued when the promise to repair was not kept. So Mydlach had four
years from the failure to repair, meaning that Mydlach filed suit well before the statute of limitations had expired.
o Buyer purchased a used Dodge Neon from McGrath Buick-Nissan on June 20, 1998. The car was first put in service on June 24,
1996. The car had a three-year warranty, a promise to repair or replace defective parts. Buyer took the car in for repairs on July
7, 1998, and dealer was never able to repair the car successfully. The buyer sued under Magnuson-Moss (for attorney fees) and
the UCC for breach.
o The court holds that this is a written warranty under Magnuson-Moss – thus entitling the buyer to attorney fees.
o However, the court also holds that this is not a lawsuit for breach of express warranty under §2-313. The promise to repair does
not “relate to the goods” within the meaning of §2-313. Cites cases and law review articles.
o Section 2-725(2) provides that a cause of action for breach of warranty accrues when the goods are delivered, but the UCC is
silent about when a cause of action for breach of a repair warranty accrues.
o Accordingly, the court looks outside the UCC (citing 1-103). Under the Restatement of Contracts, a breach occurs when
performance is due. Accordingly, if the repairs are unsuccessful, that is when the breach occurs. Again, the court cites cases, and
a commentator Professor Lawrence.
o Policy argument: If the warranty of repair was for five years, and if the cause of action accrued upon delivery, then the statute of
limitations might expire before the repairs even occurred! Policy: the court states, “Statutes of limitation are intended to
prevent stale claims, not to preclude claims before they are ripe for adjudication.”
o Defendant argued that this would make liability on repair warranties “limitless” and “uncertain.” The court replies that there
may have to be some factfinding about when the warranty was breached, but that’s true in every case.
o Repairs commenced July 7, 1998, and the suit was brought in May, 2001 – within the four-year period of limitations.
 Casebook Problem
o If the manufacturer of a vehicle sold it to the dealership, and the dealership then resold the vehicle to a consumer, would the
four-year period on the manufacturer’s implied warranty start running on the date of delivery to the dealership or on the date
of the sale to the ultimate consumer? See Wilson v. Class. Would we reach the same result on an express warranty given by
the manufacturer?
 the majority of courts say that the statute of limitations starts to run upon delivery to the retailer (otherwise the
manufacturer would have no way of knowing how long it was exposed to liability).
 But Whaley disagrees – that the warranties are intended to run to the ultimate consumers, and should run from the time of
purchase by that consumer.
 This is the so-called “shelf life” problem. We can classify warranties according to whom they are made.

157
 If the manufacturer is making its warranty only to the retailer, then the ultimate consumer is at best a third party beneficiary
of that warranty, and is protected or not according to the usual rules of §2-318. In such a situation the statute of limitations
properly ought to expire four years after delivery of the product to the retailer; after that period there is no warranty from
which to acquire a third party benefit.
 On the other hand, if the warranty between the manufacturer and the retailer is clearly intended to be passed on to the
consumer (or is made directly from the manufacturer to the consumer), as many express warranties are, then the retailer, in
effect becomes the agent for the manufacturer in passing on its warranty. In that case the manufacturer should not be free
from liability until four years after the retailer’s sale to the consumer.
 Example of the Last Point
o Wilson v. Class (Superior Court of Delaware 1992).
o Manufacturer sold mobile home to dealer in 1983. Dealer sold it to consumer in 1985. Consumer sued manufacturer from
breach of warranty in 1988.
o Held: the four-year statute of limitations had expired. It would be unfair to manufacturers to make the warranty extend beyond
the date of delivery to dealers because the product might have a long “shelf life.”
 Casebook Problem
o Painter Sally Smock had no luck selling her own works but made good money by creating phony works supposedly by Salvador
Dali, signing Dali’s name to them, and selling them to DaDa Gallery. In 2020, DaDa Gallery unwittingly sold two of Smock’s
forgeries to Mr. and Mrs. America, tourists who paid $150,000. The Americas proudly exhibited the supposed Dalis in their
Cincinnati living room, until 2025 when Smock made the national news as she went to jail for art forgery. The Americas then
sued DaDa Gallery for breach of express warranty, only to be met with the argument that the statute of limitations under §2-
725 had run out. How should this come out? See Balog v. Center Art Gallery; Doss, Inc. v. Christie’s, Inc., (holding that a cause of
action for selling a painting, which had been stolen by the Nazis from a Paris art dealer, accrued in 1991 when the painting was
sold by Christie’s, not in 2006 when there was a challenge to the buyer’s ownership); Equinox Gallery Ltd. v. Dorfman.
 Subsection (4) of §2-725 preserves the non-UCC rules of tolling, and that might help the Americas in this Problem. The Balog
case came out in favor of the buyer using three theories:
1. The sale of a painting with a warranty of authenticity necessarily awaits “future performance,” as that term is used in
§2-725, so the statute does not begin to run until discovery of the true artist. There is no support for this in the case law,
but the Balog court makes quite an argument for it.
2. In Balog, the seller yearly sent out certificates of authenticity, so the court held this renewed the warranty.
3. The best argument is a common law concept (codified in some states) that the statute is tolled during fraudulent
concealment and doesn’t start running until the buyer should have discovered the deception.
 In the Equinox Gallery case the court held that the breach of warranty of title claim for the sale of a forgery was barred by
the statute of limitations in §2-725, but that common law fraud was still possible since the statute of limitations for fraud is
tolled until the fraud should have been discovered. The Doss case used the date of the original sale to start the running of
the statute of limitations. This was hard on the ultimate purchaser who discovered the forgery of the painting many years
after that; worse: there was no fraudulent concealment in this case, so the buyer had to eat the loss of the painting and the
civil suit failed.
 Casebook Problem
o In 2015, Notheby’s Auction Company sold Max Collector a painting expressly warranted to be by Degas for $1 million. Experts
hired by Notheby’s had all agreed it was created by Degas in 1886. In 2032, Max Collector sold the painting to Grandiose
Museum for $5 million, making an express warranty that it was by Degas. Two years later, the museum learned conclusively that
the painting was in fact a forgery created in 1923. It sued Max Collector for return of the purchase price. Does Max Collector
have an indemnification action against Notheby’s, or has the statute of limitations expired on that claim? See §2-725 and the
following case.
 In 2015 Notheby’s Auction Company sold Max Collector a painting expressly warranted to be a Degas for $1 million. In 2032
Max sold it to a museum for $5 million. Two years later the museum discovers that it is a forgery. Does Max have an
indemnification action against Notheby’s?
 Answer – under the following case, a lawsuit for indemnification is an equitable action, separate from the warranty under
the law of contract, though doing so wreaks havoc with the whole idea of a statute of limitations.
 A lawsuit for indemnification can “revive” a warranty, in the sense that the injured party may sue the original seller for
indemnification!
 Central Washington Refrigeration, Inc. v. Barbee
o The court holds that a buyer may bring an indemnity action against the seller, for liability incurred to a third party for a defect in
the goods. The court further holds that the statute of limitations begins to run when the buyer pays damages to the third party
or the third party obtains judgment against the buyer—meaning the buyer may be able to bring an indemnity action long more
than four years following the original delivery.
o The dissent (and some courts agree) differed on both points. An implied indemnity action, in the dissent’s view, would give the
buyer more than the buyer had bargained for, where sales law should simply enforce the parties’ deal. Extending the statute of

158
limitations, again in the dissent’s view, is contrary to the policy of barring claims for breach of warranty after four years,
whether buyer knows of the breach or not.
 Summary: Statute of Limitations
o Article 2 establishes a four-year statute of limitations. A lawsuit must be filed within four years of the date that the cause of
action accrues.
o In the normal case, an action for breach of an implied or express warranty accrues when the goods are tendered for delivery,
whether or not the buyer has knowledge of the breach. However, if the warranty explicitly extends to future performance of the
goods and discovery of the breach must await the time of such performance, the period of limitations begins to run when the
breach is or should have been discovered.
o Article 2 does not alter the law governing the tolling of the statute of limitations. Fraud, active concealment, or estoppel may
extend the period of time that is provided for the buyer to file an action against the seller for breach.
 Canvas Quiz Questions
o How long will the period of limitations be under each of the following contracts for the sale of goods?
 A contract that does not mention the period of limitations - Four years
 A contract that sets the period of limitations at two years - Two years
 A contract that sets the period of limitations at six years - Four years
 The original agreement did not contain any provision relating to - Four years
the period of limitations. Six months later the parties agree to
modify the agreement to reduce the period of limitations to one
year.
o John purchased a computer from BitWorld that was delivered on January 1, 2015.
1. The specifications displayed on the on the store counter in front of the computer stated that its hard drive has 1 terabyte of
disk storage space. It actually has only 100 gigabytes of storage space.
2. In the purchase contract Bitworld promised to repair or replace any defect in the computer for one year after purchase.
John brought the computer in for repair on June 30, 2015 stating that he thought that the computer was a little slow.
Bitworld did nothing and told him that the computer was operating as it should.
3. In the purchase contract Bitworld guaranteed that the computer would operate within normal parameters for a computer
of this type for a period of one year.
John discovered the fact that the hard drive had only 100 gigabytes of storage space on June 15, 2019, and sued Bitworld for
breach of warranty the same day. Is his lawsuit timely with respect to breach of any of these warranties? Which of the
following is the best answer?
 The action is timely with respect to the warranty that the hard drive had 1 terabyte of storage space because John did not
discover the breach until June 15, 2019.
 The action is timely with respect to the warranty to repair and replace any defect for one year, because Bitworld breached
this warranty on June 30, 2015, which is within the four-year period of limitations.
 The action is timely with respect to the warranty that the computer would operate within normal parameters for one year
because this was a warranty as to future performance, and the lawsuit was brought within four years of the end of that
period.
 Answer: It is possible that none of these breaches occurred within the four-year period of limitations, and that John's claims
for breach of warranty are all barred by the statute of limitations.
– Possibly. The first warranty (that the hard drive had 1 terabyte of storage space) is clearly barred by the statute of
limitations. This warranty was breached when the seller tendered delivery of the computer on January 1, 2015. The
second warranty seems like it was breached on June 30, 2015, and there is case law to that effect. However, there
is also case law for the proposition that a promise to repair or replace the goods is not a warranty that extends to
"future performance of the goods," but rather an express warranty by the seller to repair or replace; as such it is a
warranty whose period of limitations accrues when the seller tendered the goods, not when the failure to repair
occurred. The third warranty is also a close call. This is clearly a warranty (will operate within normal parameters
for a period of one year) that extends to future performance of the goods. The problem is that it would have been
possible to discover the breach at any time with one click simply by checking the "devices and drives"; the buyer did
not have to await "future performance" to discover the breach. Accordingly, it is possible that all three claims for
breach of warranty are barred by the statute of limitations. However, John has one other argument: was the
statute of limitations "tolled" by Bitworld's reassurances that the computer was operating normally?
 Review/Assessment Questions
 When is a seller entitled to reclaim the goods?
o If buyer does not pay for the goods, seller does not automatically have the right to reclaim them (unless the parties have so
agreed). A seller has the right to reclaim, under §2-702, where a seller has delivered goods to an insolvent buyer and gives
notice within 10 days. That would be unusual.

159
o More likely, seller will not find out that buyer is insolvent until the 10 days have passed (if it was easily known, seller would not
have delivered). However, a misrepresentation of solvency within three months before delivery makes the 10 day limit
inapplicable, so sellers might might consider requiring buyers to represent that they are solvent.
 Is a penalty clause for nonperformance enforceable?
o No, a penalty clause (one that sets unreasonable liquidated damages) is not enforceable. But parties do have flexibility to set
liquidated damages that are reasonable in light of anticipated or actual harm. §2-718.
 When is a breaching buyer entitled to restitution?
o Where buyer has made payments but breaches, such as by making a wrongful rejection, buyer is entitled to some restitution, as
set out in §2-718(2) and (3).
 Why is the measure of remedies different depending on whether the goods were accepted?
o If buyer accepts but does not pay, seller is entitled to the contract price. If buyer wrongfully rejects, seller still has the goods to
sell to someone else, so Seller is entitled to damages based on the loss of the bargain, such as the difference between the
contract price and the market price, or lost profits. If seller breaches, the rules likewise take into consideration the question,
who has the goods?
 Can buyer be liable for the price if seller breached?
o Yes, if buyer accepts, buyer is liable for the price (unless buyer can revoke acceptance). If seller delivered non-conforming goods
(or delivered late or otherwise breached), buyer has a counterclaim for damages.
 Can seller be liable if buyer has effectively rejected the goods?
o Yes, seller may be liable for buyer loss of bargain damages, such as the difference between the sale price and market price, and
consequential damages.
 Can buyer be liable if buyer has effectively rejected the goods?
o Yes, buyer may be liable for wrongful rejection.
 What is the difference, in seller’s remedies, between market price damages, resale damages, and lost profits damages?
o Market price damages are the loss of the difference between the contract price and the market price, whereas lost profits are
the difference between the contract price and seller’s cost.
 When is a party liable for consequential damages?
o Seller may be liable for consequential damages under §2- 715(2)(a), where seller had reason to know of buyer’s requirements
and the breach caused the loss. There is no provision in Article 2 specifically providing for consequential damages for a seller
flowing from a buyer’s breach.
 How might buyer’s damages differ for breach of warranty and for failure to deliver?
o If seller delivers nonconforming goods, the remedy is likely the difference between the goods as warranted and the goods as
delivered. If seller fails to deliver, the remedy is more likely to be market price differential or cover. Both might be
supplemented by consequential damages.
 Cold Feet Labs entered into a contract to purchase a popular genetic sequencing machine from Precision Devices. The machine was
delivered on time and met all the contract specifications. The manager of Cold Feet Labs inspected the machine and found no
problems but refused to take delivery. The manager cited budgetary concerns. The market price for the machine had become lower
than the contract price, so it would be cheaper to buy the machine elsewhere. How much is the liability of Cold Feet Labs to
Precision Devices?
o Cold Feet Labs is liable for the contract price, because the seller tendered conforming goods in timely fashion.
o Cold Feet Labs is liable for the market price of the machine. - Precision Devices is entitled to damages even though it still has the
machine, because it lost the benefit of the bargain.
o Cold Feet Labs is not liable for anything, because Precision Devices still has the machine.
o Answer: Cold Feet Labs could be liable for the difference between the contract price and the market price.
– Cold Feet Labs has breached the contract. But Cold Feet Labs is not liable for the price of the machine. The buyer
becomes liable for the price under §2-709 if the buyer accepts the goods, but Cold Feet Labs did not accept the
goods. The buyer can be liable for the price if the goods cannot reasonably be resold, §2-709(1)(b), but the machine
is popular, not a unique product for the buyer. The buyer is also liable for the price of goods destroyed while the risk
of loss is on the buyer, §2-709(1)(a), but that likewise does not apply here. Rather, Cold Feet is liable for
nonacceptance under §2-708, which could be market differential damages under §2-708(1) or lost profits under §2-
708(2).
 Pursuant to contract, Gdansk Boats delivered a wooden sailing ship to Historic Cruises. The purchasing agent for Historic Cruises
inspected the ship, and quickly noted that many of the fixtures were mahogany rather than the promised teak, that various repairs
and adjustments would be required in some of the cabins, and that several sails were missing. Historic Cruises indicated they would
take the ship but sue for breach of contract, listing the defects. Is Gdansk Boats liable to Historic Cruises?
o No. Historic Cruises accepted the goods and must pay the contract price.
o Answer: Yes. Historic Cruises is liable for the price but has a counterclaim for damages for breach of warranty, which could be
the difference between the value of the boat promised and the boat delivered, and possible consequential damages. – If the
buyer accepts the goods, the buyer is liable for the price under §2-709, even if the goods do not conform to the contract. But

160
such a buyer may have a claim for breach of warranty under §2-714, provided the buyer gives notice under §2-607(3)(a), which
the buyer did here.
o Yes. Gdansk Boats did not deliver the boat promised, so Historic Cruises is not liable for the price unless all the shortcomings are
cured.
o No. Gdansk Boats is not liable for defects of which Historic Cruises was aware.
 Gdansk Boats delivered a defective boat to another customer, Tolerant Harbor. The buyer noted quite a list of defects in the boat
but took it and paid without complaint. After a year, however, Tolerant Harbor realized the defects substantially impaired the value
of the boat and resulted in higher repair and service costs. Can Tolerant Harbor recover for breach of contract?
o No, because it accepted the boat with defects.
o No, because the statute of limitations has run.
o Yes, because the statute of limitations has not yet run. - The statute of limitations is not the key limit on time in such sales cases.
Rather, buyers may lose their right to sue much more quickly if they fail to give timely notice of breach.
o Answer: No, because it did not give timely notice of breach. - A buyer who accepts the goods may still recover for breach of
warranty, but only if the buyer gives notice of breach to the seller within a reasonable time. §2-607(3)(a). Tolerant Harbor knew
of the breaches for a year before giving notice, which would not be a reasonable time (absent unusual circumstances).
 Pemulis Auto had a contract to sell a 2016 Fireworks Rambler to Suzie Sousa for $35,000. On the agreed day of delivery, Pemulis
Auto notified Suzie Sousa that they would not be able to perform their obligations. Suzie Sousa shopped around and found the
market price for the car had risen to $40,000. After some thought, she decided to stick with her current automobile. Is Pemulis Auto
liable to Suzie Sousa?
o Yes, for the $35,000 contract price.
o Answer: Yes, for the $5,000 difference between the contract and market price.
o No, because she did not purchase a substitute.
o No, because they gave timely notice of nondelivery.
– Where a seller fails to deliver, a buyer may recover for the loss of the benefit of the bargain. If the buyer purchases
the goods elsewhere, the buyer may seek damages calculated by the cover price under §2-712. But the buyer is not
required to cover and may also seek damages for loss of the bargain under §2-713, measured by the difference
between the contract price and the market price. Notice of nondelivery is not required, as opposed to where the
buyer has accepted the goods and must give notice of any breach. The seller is the one who did not deliver so seller
would not need notice.
 Gdansk Boats sold a fishing boat to Olivier Robeson, an actor. Not long afterwards, the boat’s defective engine left Robeson floating
for several hours. As a result, he missed a call offering a juicy part, which was quickly offered to a rival. Is Gdansk Boats liable for
damages for Robeson’s lost role?
o Yes, the damages were caused by their breach and Gdansk Boats knew that the boat needed an engine to travel. – Mere but-for
causation is insufficient.
o No, a breach of sales contract results in damages only for a lost bargain or diminished value. - A buyer may recover
consequential damages, including “any loss resulting from general or particular requirements and needs of which the seller at
the time of contracting had reason to know and which could not reasonably be prevented by cover or otherwise.” §2-715.
o Answer: No, because the damages were not caused by a requirement known to the seller. - The damages here do not fall within
that description. Where seller delivers a machine that seller knows buyer needs to operate a factory, seller could be liable if the
defective machine shuts down the factory. But the seller of a boat would not reasonably expect that failure of an engine could
result in missing a telephone call that in turn would result in loss of an acting role.
o Yes, because the breach was a but-for cause of the lost role. - Mere but-for causation is insufficient.
Assignment of Rights and Delegation of Duties
 Rights and Duties
o “Rights” under a contract are those things that a person is entitled to from the other party. “Duties” are things that a person is
required to do for the other party.
o Every “right” has a corresponding “duty.”
o The seller has a right to payment from the buyer. The buyer has a duty to pay the seller. The buyer has a right to delivery of the
goods by the seller. The seller has a duty to deliver the goods to the buyer.
o Assignment of Rights and Delegation of Duties are governed by Section 2-210.
o The other party may treat any assignment which delegates performance as creating reasonable grounds for insecurity
 Section 2-210: Assignment of Rights and Delegation of Duties
(1) A party may perform his duty through a delegate unless otherwise agreed or unless the other party has a substantial interest in
having his original promisor perform or control the acts required by the contract. No delegation of performance relieves the
party delegating of any duty to perform or any liability for breach.
(2) Except as otherwise provided in Section 9-406, unless otherwise agreed, all rights of either seller or buyer can be assigned
except where the assignment would materially change the duty of the other party or increase materially the burden or risk
imposed on him by his contract or impair materially his chance of obtaining return performance. A right to damages for breach

161
of the whole contract or a right arising out of the assignor's due performance of his entire obligation can be assigned despite
agreement otherwise.
(3) The creation, attachment, perfection, or enforcement of a security interest in the seller's interest under a contract is not a
transfer that materially changes the duty of or increases materially the burden or risk imposed on the buyer or impairs
materially the buyer's chance of obtaining return performance within the purview of subsection (2) unless, and then only to the
extent that, enforcement actually results in a delegation of material performance of the seller. Even in that event, the creation,
attachment, perfection, and enforcement of the security interest remain effective, but (i) the seller is liable to the buyer for
damages caused by the delegation to the extent that the damages could not reasonably be prevented by the buyer, and (ii) a
court having jurisdiction may grant other appropriate relief, including cancellation of the contract for sale or an injunction
against enforcement of the security interest or consummation of the enforcement.
(4) Unless the circumstances indicate the contrary a prohibition of assignment of “the contract” is to be construed as barring only
the delegation to the assignee of the assignor's performance.
(5) An assignment of “the contract” or of “all my rights under the contract” or an assignment in similar general terms is an
assignment of rights and unless the language or the circumstances (as in an assignment for security) indicate the contrary, it is a
delegation of performance of the duties of the assignor and its acceptance by the assignee constitutes a promise by him to
perform those duties. This promise is enforceable by either the assignor or the other party to the original contract.
 Assignment and Delegation
o Rights may be “assigned” to other parties.
o Duties may be “delegated” to other parties.
o Under the terminology of the U.C.C., rights are not “delegated” and duties are not “assigned.”
o Unfortunately, in drafting contracts people are not always careful to use the correct terminology. While the intent of the parties
should control the interpretation of the contract, this drafting mistake often causes needless confusion.
 In General Rights Are Freely Assignable
o Section 2-210(2) provides that rights may be assigned to another party unless the assignment would materially change the duty
of the other party, materially increase the burden or risk, or materially impair the chance of return performance.
o Also, under Section 2-210(2) the original contract may bar any assignment of rights, but a right to damages for breach of the
whole contract may be freely assigned despite language in the contract barring an assignment of rights.
 Section 2-210(2): In General Rights Are Freely Assignable
Except as otherwise provided in Section 9-406, unless otherwise agreed, all rights of either seller or buyer can be assigned except
where the assignment would materially change the duty of the other party or increase materially the burden or risk imposed on
him by his contract or impair materially his chance of obtaining return performance. A right to damages for breach of the whole
contract or a right arising out of the assignor's due performance of his entire obligation can be assigned despite agreement
otherwise.
 Duties, However, Are Not as Freely Delegable
o Under Section 2-210 it is easier to assign rights than it is to delegate duties.
o Section 2-210(1) provides that duties may not be delegated if the other party has a substantial interest in having the original
promisor perform those duties. Furthermore, even if the duties are delegated, the original promisor remains liable under the
contract.
 Section 2-210(1): Duties, However, Are Not as Freely Delegable
A party may perform his duty through a delegate unless otherwise agreed or unless the other party has a substantial interest in
having his original promisor perform or control the acts required by the contract. No delegation of performance relieves the party
delegating of any duty to perform or any liability for breach.
 Right to Demand Assurances
o Under Section 2-210(6) if a seller attempts to delegate its duty to deliver the goods or a buyer attempts to delegate its duty to
pay for the goods, the other party has the right to demand assurances that the contract will be performed.
o The right to demand assurance of performance is governed by Section 2-609.
o Practice Pointer: Section 2-609 is a great tool to have in your legal toolbox!
 Section 2-210(6): Right to Demand Assurances
The other party may treat any assignment which delegates performance as creating reasonable grounds for insecurity and may
without prejudice to his rights against the assignor demand assurances from the assignee (Section 2-609).
 Section 2-609: Right to Demand Assurances of Performance
(1) A contract for sale imposes an obligation on each party that the other's expectation of receiving due performance will not be
impaired. When reasonable grounds for insecurity arise with respect to the performance of either party the other may in
writing demand adequate assurance of due performance and until he receives such assurance may if commercially reasonable
suspend any performance for which he has not already received the agreed return.
(2) Between merchants the reasonableness of grounds for insecurity and the adequacy of any assurance offered shall be
determined according to commercial standards.
(3) Acceptance of any improper delivery or payment does not prejudice the aggrieved party's right to demand adequate assurance
of future performance.

162
(4) After receipt of a justified demand failure to provide within a reasonable time not exceeding thirty days such assurance of due
performance as is adequate under the circumstances of the particular case is a repudiation of the contract.
Construction in Favor of Assignment and Against Delegation
o If a contract prohibits assignment of “the contract,” the presumption is that it permits the assignment of rights but prohibits the
delegation of duties.
 Section 2-210(4): Construction in Favor of Assignment and Against Delegation
Unless the circumstances indicate the contrary a prohibition of assignment of “the contract” is to be construed as barring only the
delegation to the assignee of the assignor's performance.
 Hypothetical 1 – Sample Contract clause: Assignment
o Neither Party shall assign any of its rights or delegate any of its obligations hereunder without the prior written consent of the
other Party, which consent shall not be unreasonably withheld, conditioned or delayed; provided, however, that either Party
may assign its rights or delegate its obligations, in whole or in part, without such consent and upon ten days prior written notice
to the other Party, to (a) one of its Subsidiaries or Affiliates, or (b) an entity that acquires all or substantially all of the business or
assets of such Party to which this Agreement pertains, whether by merger, reorganization, acquisition, sale, or otherwise. Any
purported assignment or delegation in violation of this Section shall be null and void. No assignment or delegation shall relieve
the assigning or delegating Party of any of its obligations hereunder unless the non-assigning or non-delegating Party enters into
a novation releasing the assigning or delegating Party of its obligation under the Agreement. Will the charity prevail in its
action against the mayor?
 But what is the legal issue presented?
• Whether the painter validly assigned his right to payment where the contract for the sale of a $5,000 painting “prohibited
the assignment of any rights or duties arising under the contract without the permission of the other party.”
 What rule of law controls the issue?
• This is a contract for the sale of goods. Therefore, the UCC applies.
• Assignment and delegation are covered by UCC §2-210.
 Rule of Law
• Under UCC §2-210, assignments are not allowed when they materially increase the duty or risk of the obligor or
materially reduce the obligor’s chance of obtaining performance. Generally, prohibitions against assignment are strictly
construed, and assignment is permissible.
• Even if validly prohibited by the contract, the power to assign is retained by the parties, and the only consequence is that
an assignment operates as a breach of the contract.
• Bar Exam Note: Although the two terms (assignment and delegation) are clearly distinct in concept, the term
“assignment” is often used to refer to either assignment or delegation on the MBE.
• Assignment” is the transfer of rights under a contract, and “delegation” is the transfer of duties and obligations under a
contract.
 Answer: Yes, the charity prevails in its action against the mayor because the charity was an assignee of the son’s
contractual rights.
 Under UCC §2-210, contract rights are assignable unless the assignment materially increases the duty or risk of the obligor
or materially reduces the obligor’s chance of obtaining performance. Here, because the right assigned is the right to
receive payment and the painter has performed his obligation under the contract by delivering the portrait, this right may
be assigned and enforced by the charity through a breach of contract action.
 Hypothetical 2 – The Pine Boards
o A lumberyard contracted with a retail home improvement company to provide the company with pine boards of various lengths
and amounts. The contract was silent as to delegation, and the lumberyard, acting in good faith, delegated its duties under the
contract to a third party. After asking for, and receiving, assurances from the third party that the boards would conform to
contract specifications, the company agreed to the delegation. The third party delivered the boards, which the company
rejected because the shipment did not conform to its specifications. The company sued both the lumberyard and the third
party. The lumberyard moved to dismiss the claims against it. Should the court dismiss the suit against the lumberyard?
 But what is the legal issue presented?
• Whether a valid delegation of duties absolves the lumberyard of its contractual liability to deliver boards that conform to
contract specifications.
 What rule of law controls the issue?
• This is a contract for the sale of goods. Therefore, the UCC applies.
• Assignment and delegation are covered by UCC §2-210.
 Rule of Law
• UCC §2-210(1) provides, “No delegation of performance relieves the party delegating of any duty to perform or any
liability for breach.”
• Thus, when obligations are delegated, the delegator is not released from liability, and recovery can be had against the
delegator if the delegator if the delegatee does not perform.

163
• A delegation does not relieve the delegating party of her obligations under the contract unless the other party to the
contract agrees to release that party and substitute a new one (a novation).
• Merely consenting to a delegation does not create a novation. UCC §2-210.
 Answer: No, the court should not dismiss the suit because a delegation of duties does not absolve the lumberyard of its
contractual liability.
 Under UCC §2-210, when contractual obligations are delegated, the delegator is not released from liability, and recovery
can be had against the delegator if the delegatee does not perform. Here, the lumberyard merely delegated its duties to a
third party. The home improvement company did not agree to release the lumberyard or create a novation. Therefore, the
lumberyard remained liable on the contract.
 Canvas Quiz Questions
o The assignment of rights differs from the delegation of duties in that:
 the assignment of rights includes privileges such as the right to receive payment or the the right to receive delivery of goods
under a contract, while a delegation of duties would include the obligation to deliver the goods or the obligation to pay for
goods.
 the U.C.C. favors the delegation of duties but makes assignment of rights more difficult.
 a delegation of duty is not permissible if the other party has a substantial interest in having the original promisor perform;
whereas an assignment of rights is permitted unless the assignment would materially change the duty of the other party,
materially increase the burden or risk, or materially impair the chance of return performance.
 if an agreement prohibits assignment of "the contract," it is presumed that the contract permits assignment of rights but
prohibits delegation of duties.
 even if a party delegates its duty to another person, the original promisor remains obligated under the contract to perform
that duty and remains liable for breach.
o If a seller attempts to delegate its duty to deliver the goods or a buyer attempts to delegate its duty to pay for the goods, the
other party has the right to demand assurances that the contract will be performed.
 True

164

You might also like